You are on page 1of 230

SCHAUM'S

OUTLINE
SERIES

THEORY aiMl>ROBLEMS

of

MATRICES
by

FRANK AYRES,

JR.

including

Completely Solved

in

Detail

SCHAUM PUBLISHING
NEW YORK

CO.

SCHAVM'S OUTLINE OF

THEORY

AI\[D
OF

PROBLEMi;

MATRICES

BY

FRANK AYRES,

JR., Ph.D.

Formerly Professor and Head,

Department of Mathematics
Dickinson College

^C^O
-^

SCHAIJM'S OUTLINE SERIES


McGRAW-HILL BOOK COMPANY
New
York, St. Louis, San Francisco, Toronto, Sydney

('opyright

1962 by McGraw-Hill, Inc.


of America.

AH

Rights Reserved.

Printed in the
be reproduced,

United

States

No part

of this publication

may

stored in a retrieval system, or transmitted, in any form or by any means,


electronic, mechanical, photocopying, recording, or otherwise,

without the prior

written permission of the publisher.

02656

78910 SHSH 7543210

Preface
Elementary matrix algebra has now become an integral part of the mathematical background necessary for such diverse fields as electrical engineering and education, chemistry and sociology, as well as for statistics and pure mathematics. This book, in presenting the more
rial,
is

essential matedesigned primarily to serve as a useful supplement to current texts and as a handy reference book for those working in the several fields which require some knowledge of matrix theory. Moreover, the statements of theory and principle are sufficiently complete that the book could be used as a text by itself.

The material has been divided into twenty-six chapters, since the logical arrangement is thereby not disturbed while the usefulness as a reference book is increased. This also permits a separation of the treatment of real matrices, with which the majority of readers will be concerned, from that of matrices with complex elements. Each chapter contains a statement of pertinent definitions, principles, and theorems, fully illustrated by examples. These, in turn, are followed by a carefully selected set of solved problems and a considerable number of supplementary exercises.

The beginning student in matrix algebra soon finds that the solutions of numerical exercises are disarmingly simple. Difficulties are likely to arise from the constant round of definition, theorem, proof. The trouble here is essentially a matter of lack of mathematical maturity,'
and
normally to be expected, since usually the student's previous work in mathematics has been concerned with the solution of numerical problems while precise statements of principles and proofs of theorems have in large part been deferred for later courses. The aim of the present

book
lems

is

to enable the reader,

in

if he persists through the introductory paragraphs and solved probany chapter, to develop a reasonable degree of self-assurance about the material.

The solved problems, in addition to giving more variety to the examples illustrating the theorems, contain most of the proofs of any considerable length together with representative shorter proofs. The supplementary problems call both for the solution of numerical exercises
and
for proofs.

Some

of the latter require only proper modifications of proofs given earlier;

more important, however, are the many theorems whose proofs require but a few lines. Some are of the type frequently misnamed "obvious" while others will be found to call for considerable

None should be treated lightly, however, for it is due precisely to the abundance of such theorems that elementary matrix algebra becomes a natural first course for those seeking to attain a degree of mathematical maturity. While the large number of these problems in any chapter makes it impractical to solve all of them before moving to the next, special attention
ingenuity.
is

much

directed to the supplementary problems of the first two chapters. mastery of these will do to give the reader confidence to stand on his own feet thereafter.

The author wishes


Publishing

to take this opportunity to express his gratitude to the staff of the

Schaum

Company

for their splendid cooperation.

Frank Ayres,
CarHsle, Pa.

Jr.

October, 1962

CONTENTS
Page
Chapter

MATRICES
Equal matrices. Products by partitioning.
Matrices.

Sums

of matrices.

Products of matrices.

Chapter

SOME TYPES OF MATRICES


Scalar matrices. Diagonal matrices. The identity matrix. Inverse of a matrix. Transpose of a matrix. Symmetric matrices. Skew-symmetric matrices. Conjugate of a matrix. Hermitian matrices. Skew-Hermitian matrices. Direct sums.

10

Triangular matrices.

Chapter

DETERMINANT OF A SQUARE MATRIX.


Determinants of orders 2 and 3. Properties of determinants, and cofactors. Algebraic complements.

20
Minors

Chapter

EVALUATION OF DETERMINANTS
Expansion along a row or column. The Laplace expansion. Expansion along the first row and column. Determinant of a product. Derivative
of a determinant.

32

Chapter

EQUIVALENCE
Non-singular and singular matrices. Elementary transformations. Inverse of an elementary transformation. Equivalent matrices. Row canonical form. Normal form. Elementary matrices. Canonical sets under equivalence. Rank of a product.

39

Rank

of a matrix.

Chapter

THE ADJOINT OF A SQUARE MATRIX


The
adjoint.

49
adjoint.

The adjoint

of a product.

Minor of an

Chapter

THE INVERSE OF A MATRIX.


Inverse of a diagonal matrix. Inverse from the adjoint. Inverse from elementary matrices. Inverse by partitioning. Inverse of symmetric X w matrices. matrices. Right and left inverses of

55

Chapter

FIELDS
Number
fields.

64
General
fields.

Sub-fields.

Matrices over a

field.

CONTENTS
Page
Chapter

LINEAR DEPENDENCE OF VECTORS AND FORMS


Vectors. Linear dependence of vectors, linear forms, polynomials, and matrices.

67

Chapter

10

LINEAR EQUATIONS
rule.

75

System of non-homogeneous equations. Solution using matrices. Cramer's Systems of homogeneous equations.

Chapter

11

VECTOR SPACES
Vector spaces.
section

85

space.

Sub-spaces. Basis and dimension. Sum space. InterNull space of a matrix. Sylvester's laws of nullity.

Bases and coordinates.

Chapter

12

LINEAR TRANSFORMATIONS
Singular and non-singular transformations. space. Permutation matrix.

94

Change of

basis.

Invariant

Chapter

13

VECTORS OVER THE REAL FIELD


Inner product. Length. Triangle inequality. Schwarz inequality. Orthogonal vectors and spaces. Orthonormal basis. Gram-Schmidt orthogonalization process. The Gramian. Orthogonal matrices. Orthogonal transformations. Vector product.

100

Chapter

14

VECTORS OVER THE COMPLEX FIELD


Inner product. Length. Schwarz inequality. Triangle inequality. Orthogonal vectors and spaces. Orthonormal basis. Gram-Schmidt orthogonalization process. The Gramian. Unitary matrices. Unitary transformations.

110

Complex numbers.

Chapter

15

CONGRUENCE
Congruent symmetric matrices. Canonical forms of real symmetric, skew-symmetric, Hermitian, skew-Hermitian matrices under congruence.
Congruent matrices.

115

Chapter

16

BILINEAR FORMS
Matrix form.
formations.

125

Transformations. Canonical forms. Cogredient transContragredient transformations. Factorable forms.

Chapter

17

QUADRATIC FORMS
Matrix form. Transformations. Canonical forms. Lagrange reduction. Sylvester's law of inertia. Definite and semi-definite forms. Principal minors. Regular form. Kronecker's reduction. Factorable forms.

131

CONTENTS
Page
Chapter

lo

HERMITIAN FORMS
Matrix form.
definite forms.

146
Canonical forms.
Definite

Transformations.

and semi-

Chapter lif

THE CHARACTERISTIC EQUATION OF A MATRIX


Characteristic equation and roots.

149

Invariant vectors and spaces.

Chapter

20

SIMILARITY
Similar matrices.

156
Reduction to triangular form.

Diagonable matrices.

Chapter

21

SIMILARITY TO A DIAGONAL MATRIX


Real symmetric matrices. Orthogonal similarity. Pairs of real quadratic forms. Hermitian matrices. Unitary similarity. Normal matrices.
Spectral decomposition.
Field of values.

163

Chapter

22

POLYNOMIALS OVER A FIELD


Sum, product, quotient

172

common

divisor.

Least

common

of polynomials. multiple.

Remainder theorem. Greatest Relatively prime polynomials.

Unique factorization.

Chapter

2o

LAMBDA MATRICES
The X-matrix or matrix polynomial. Sums, products, and quotients. Remainder theorem. Cayley-Hamilton theorem. Derivative of a matrix.

179

Chapter

24

SMITH NORMAL FORM


Smith normal form.
Invariant factors.

188

Elementary

divisors.

Chapter

25

THE MINIMUM POLYNOMIAL OF A MATRIX


Similarity
invariants.

196
and
non-

derogatory matrices.

polynomial. Companion matrix.

Minimum

Derogatory

Chapter

26

CANONICAL FORMS UNDER SIMILARITY


Rational canonical form. second canonical form. Hypercompanion matrix. Jacobson canonical form. Classical canonical form. A reduction to rational canonical form.

203

INDEX

215

INDEX OF SYMBOLS

219

'

chapter
Matrices

A RECTANGULAR ARRAY OF NUMBERS


"2

enclosed by a pair of brackets, such as


1

l'

3
1

(a)
1

and

(b)

-1

5 is

and subject to certain rules of operations given below

called a matrix.

The matrix

(a) could

be

considered as the coefficient matrix of the system of homogeneous

(2x + 3y + 7z = linear equations \ X- y + 5z =


[

or

as the augmented matrix of the system of non-homogeneous linear equations

i2x + 3y = 7
-

\x-

y = 5

Later,
trix (b)

we

shall see

how

the matrix

may be used
or

to obtain solutions of these

systems.

The ma-

could be given a similar interpretation


1),

we might consider

its

rows as simply the coor-

dinates of the points (1,3,


later to settle

(2, 1,4),

and (4,7, 6) in ordinary space.

origin or

such questions as whether or not the three points on the same line through the origin.

lie in the

The matrix will be used same plane with the

In the matrix

^11 ^12

'^IS

'

(1.1)

*mi "m2 ""m3

the numbers or functions a^- are called its elements. In the double subscript notation, the first

subscript indicates the row and the second subscript indicates the column in which the element

stands.

Thus,
ra"

all

elements in the second row have

as

first

subscript and all the elements in


is

the fifth column have 5 as second subscript. order

matrix of

rows and n columns

said to be of

"m by
We

or

mxra.
(

(In

indicating a matrix pairs of parentheses,

),

and double bars,

||

||,

are

sometimes

used.

shall use the double bracket notation throughout.)


[a^
]

At times the matrix (1.1) will be called "the mxra matrix


[a^-]".

"

or

"the

mxn
4".

matrix A =

When

the order has been established,

we

shall write simply "the matrix

SQUARE MATRICES. When m


re-square matrix.

n, (1.1) is

square and will be called a square matrix of order n or an

In a square matrix, the

elements

a^,

022.

" are called

its

diagonal elements.

The sum

of the diagonal

elements of a square matrix A

is called the trace of A.

MATRICES

[CHAP.

EQUAL MATRICES. Two


other, that is,
if

matrices A = [a^] and B =

[bij]

are said to be equal (A =

B)

if

and only

if

they have the same order and each element of one is equal to the corresponding element of the

and only
^^J

if

'V
if

a
and only

= 1,2,
if

ro;

1,

n)

Thus, two matrices are equal

one

is a

duplicate of the other.

ZERO MATRIX. A

matrix, every element of

which

is zero, is

called a zero matrix.

When ^

is a zero

matrix and there can be no confusion as to its order,

we

shall write A = Q instead of the

mxn

array of zero elements.

SUMS OF MATRICES.
is

If

4 = [a^A and S =
matrix

[fe.^-]

are two

mxn

matrices, their
is the

sum sum

(difference),

A B,

defined as the

mxn
A

corresponding elements of

where each element of C A and B. Thus, AB [Oy bij]


[c^A
,

C =

(difference) of the

'\

31
1

Example

1.

It

14

and =

2
I

0'

-12
2
1)
1

then
5

A
and

+B
Lo+(-

+3 +2
+ 5j

1-2
O-(-l)

2-3
1-2

3-0 4-5
or

Two

matrices of the same order are said to be conformable for addition

subtraction.
(a)

Two
and

matrices of different orders cannot be added or subtracted. (b) above are non-conformable for addition and subtraction.

For example, the matrices

The sum

of k matrices

is

a matrix of the same order as

A and each

of its elements is k

times the corresponding element of A.

We

define:

If

k is any scalar (we call k a scalar to disis

tinguish it from [k] which is a 1x1 matrix) then by kA = Ak A by multiplying each of its elements by k.

meant the matrix obtained from

Example

2.

If

[:

'11
I

then

A+A + A
and

-2 3A
A-3
3

-5A

r-5(l)

-5(-2)"|

r-5
L-10

10-1

L-5(2)
In particular,

-5(3) J

-15j

by -A, called the negative of /4, is meant the matrix obtained from A by mulby -1 or by simply changing the sign of all of its elements. For every A, we have A +(-A) = 0, where indicates the zero matrix of the same order as A.
tiplying each of its elements

Assuming
(a)
(b) (c)

that the matrices

A,B,C are conformable

for addition,

we

state:

(d)

A + B = B + A (commutative law) A + (B+C) = (A + B)+C (associative law) k(A + B) = kA + kB = (A + B)k, A- a scalar There exists a matrix D such that A + D = B.

These laws are a result of the laws of elementary algebra governing the addition of numbers and polynomials. They show, moreover,
1.

Conformable matrices obey the same laws

of addition

as the elements of these matrices.

CHAP.

1]

MATRICES

MULTIPLICATION. By

the product
fell

AB

in that order of the

Ixm

matrix A = [a^i a^g ais

a^m]

and

the

mxl

^31

matrix

fi

is

meant the 1x1 matrix C = [on fen + 012

fesi

+ aimfemi]

fell
fe^i

That

is,

[an

Oi

'^imj

L^ii

fell

"*"

^12

feji

imi]

-|^^2

a^^fe^^J

femi

Note that the operation is row by column; each element of the row is multiplied into the corresponding element of the column and then the products are summed.
1'

Example

3.

(a)

[2

4]

[2(l) + 3(-l) + 4(2)]

[7]

L
(b)

2.

-2 [3

-1

4]

[-6 - 6+ 12]

3J

By
is

the product

AB

in that order of the

mxp
p

matrix A = [a^-] and the p

xn matrix B =
=

[bij]

meant the

mxn

matrix

C =

\c;;'\
<-

where
"-ip i>,pj

-'

Hj

ii^ij + "t2 ^2; +

^^^"ikbkj

(f

1, 2,

,m-;

1,

re).

Think of A as consisting of m rows and B as consisting of n columns. In forming C = AB each row of A is multiplied once and only once into each column of B. The element c^ of C is then the product of the ith row of A and the /th column of B.

Example

4.

^11

*^ 1

'011611 + 012621

11 ^12 + "12 622


0^21

B
."31

[611
O32
621

612I 622J

021611+022^21
."31611+032621

^12 +

'^22

^22

"31612+002600

The product ^S is defined or A is conformable to B for multiplication only when the number columns of A is equal to the number of rows of S. If ^ is conformable to B for multiplication {AB is defined), B is not necessarily conformable to A for multiplication (BA may or may not be
of

^^^ined)-

See Problems
that

3-4.

Assuming
(e)
(/")

A,B,C
=

are conformable for the indicated +

sums and products, we have

A(B + C) = AB
(A +

B)C

AC

AC BC

(first distributive law) (second distributive law)

(g)

A(BC) = (AB)C

(associative law)

However,
(A) (i) (/)

AB AB AB

i=

BA, generally,

= =

AC

does not necessarily imply i = or S = does not necessarily imply B = C.

0,

See Problems

3-8.

MATRICES

[CHAP.

PRODUCTS BY PARTITIONING.
into n matrices of order pxl.

Let

A=

[a^J]

be of order

mxp

and

8=
m

[b^j]

be of order pxn.

In

forming the product AB, the matrix A is in effect partitioned into

matrices of order Ixp and B


let

Other partitions may be used.

For example,

A and 6 be

parti-

tioned into matrices of indicated orders by drawing in the dotted lines as


(piXTii)
I

(pixnj)
(p2Xra2)
(p3Xre2)

(mixpi)

(m]^xp2) (m,2Xp2)

(m-Lxps)

(P2X%)
Jmgxpi)
I

(m2Xp3)_

(psxni)

Am
A^i

A^2
I

Aig "21
A,
I

O22
"32

"31
In

any such partitioning,

in exactly the

same way; however

such that mi+ m2 =

necessary that the columns of A and the rows of 6 be partitioned m^, mg, re^, rig may be any non-negative (including 0) integers and rei+ ^2 = n. Then
it

is

^8

^llOll + '4i2021

""

'4l3"31
A-2_iiB^^

A-^-^B-12

+ A-^qBqq + '^13832

Cii

C12

c
A21B11 + 422^21 +

A^^B^^ + ^22^22 + ^423632


2
1

_C21

1^22.

1110
and B
1

Examples.

Compute

/4S, given

/I

3
1

110

12
fill

Partitioning so that

11

^12

2
3

'0
11

A21 A22

and

B
B21

^12

111
2 2
1
1

10

S22
3
1
I

we have

AB

[A^-iB-i^i

+ A-12B21

A-ij^B'12 + A-i,

=
1

AqiB^^ + ^22^21
1 1 1 1 1 1
1"

^21^12 *

-^2^

B,

2
1

i4'
+ [l][2

'
1]

[2]

[3
[1

3]
0]

[?
+ [l][2]

[1

0]

2
3-1

[4

To

o]

To]

Tol

4
7

4 7
3

3 5

5 4

5JL0J
2] [2]

5 2
2

_[l

1]

[2

1]

[0]+[2]

[3

See also Problem

9.

Let A, B.C... be re-square matrices.


orders

Let A be partitioned into matrices of the indicated


(piX Ps)"

(pixpi)
(P2X pi)

(piX P2)
^" ^ (P2X Ps)

All
A21

A-^2

'

""

' j

(P2XPS)
I

A22

T
...
I

.(PsXpi)

(psXp2)

(PsXPs)

/loo

and

let

8, C,

...

may be formed using

be partitioned in exactly the same manner. Then sums, differences, and products the matrices A^i, A^2< : Sii. 5i2. - ^n, C12

CHAP.

1]

MATRICES

SOLVED PROBLEMS
1

2-10
2
1 2_

1.

(a)

4
2
'l

-5
2

2-2 3-1

15

-4

2I
3
=

ri + 3

2+(-4)

-1 +
1

4+1
2

+2

0+5 -5 + (-2)

2+0
+3

0+2 1+3
2 + (-l)

4-202 5 2 4 5 4-741
-2

-1
2

O'
1

3-412
1

(b)

2-5
1

12
2

2-2 3-1
3
1

1-3 4-1 2-2

2+4 -1-1 2-0 0-5 -5+2 1-3

0-2
1-3 2+1

-2
2

3-5

-3 -2

-1

6-3
6
3
6,

(c)

4
2
"l

12
.

-5
2

2.

-15

-1
2

o"
1

(d)

4
2

-5

-1 -2 1 -4 -2 -1 -2 5 -1 -2

-3

2.

If

3 5

4 6

and 6

-2 -5
3

P
find

1
s

D =

r
t

such that A + B -

T)

0.

1-3-p
If

A-\-B-D

3+1-r
_5+4-

2-2- ?1 4-5-s 6+3- u.


-2
0"

2-p
-

-9

"0

0"

4-r 9-

-1-s
9-u_

=
.0

-2-p

and p = -2,

4-r

and

= 4,

Then D

=
-

4
9

-1
9.

A +

3. (a)

[4

6]

[4(2) +5(3) + 6(-l)]

[17]

2(4)
(b)

2(5)

2(6)-

8 =

10
15

12 18

[4

e]

3(4)

3(5)

3(6)

12

[J
4
(c)

-1(4)

-1(5)

-1(6).

.-4 -5 -6

[1

3]
5
[

-6 -7
8

6"

10

-11 -8
1

1(4) + 2(0) + 3 (5)

(-6) + 2 (-7) + 3(8)

1(9) + 2(10) + 3(-ll)

1 (6)

+ 2(7) + 3(-8)]

[19 4

-4 -4]
r2(l) + 3(2)+4(3)1
_
'

poT
L29J
l(-4) + 2(5) + 1(2)]

(rf)

C
l]

Ll(l) + 5(2) + 6(3)J

(e)

"gl
2J

ri(3) + 2(l) + l(-2)


[4(3) + 0(1) + 2 (-2)

2J

_2

4(-4) + 0(5) + 2(2)J

8] [s -I2J

{2
.

-1
1

1"

Let

<4

=
.1

2
1.

Then

'2
A""

-1
1

\2

-1
1

r
2
1-

"5

-3
1

r
4
2.

5
and
2 3

-3 f
1

-1
1

1'

11
=

=
1

2
1

2 _3

4
2
.1

2
1.

8-18

-8

b
show

-1

-1

.8-43

The reader

will

that

A.a'^ and ^^./l^ = /4^.^^.

MATRICES

[CHAP.

5.

Show
2

that:
? 2

(b)

1=17=1
2

a-J

j=i 1=1
b^hCj^j)
^

22
3

a,-,-,

(c)

2:

k=i
2 (")

aife(

S
h=t

h=i
=

2 (2

k=i

aif,bkh)chj
^

J,

ife(*fei +'^fej)

"il^^i +
2

'^lj)

+ '^t2(*2i+'^2i)
2

(il^i+''i2*2i)

(il<^17+'^i2'^2i)

2 (6)

2 2
t=i j=i

a-''

2
i=i

(a^j^+ a-^ + a -p)

(an + 0^2 +
(Oil + O21)
2 2

a^g)

+ (a^t + 092 +

<223)

+ (12 + "22)
2

+ (ai3 + "23)

2
i=l

a-

"

2
i

a + a. = l ^2 i = i 13

=22
3

o--

j = ii = i

This is simply the statement that in summing all of the elements of a matrix, one may sum elements of each row or the elements of each column.
2
3

first

the

2 =

(c)

^_j

a., ( b,,c,-) tfe^^^j^ kh hy

^-^

0-il(^ll'^lj + ftl2C2J+ ^IsCsj)

(oil =

k=l
3

,^

222
a-,(bi, c
tfe^
fei

ij

6,

&,

fee

2JI

fe3

O 37'

+ 012(621^17 + b2QC2J + 623C3J)

fell

+ ai2*2i)cij + (0^1612 + aiQh.22)cQj + (aiifeis + ai2b23)cgj +


(

oifefefei)cii
^

a{fefefe2)c2f
-^

(2
fe=i

fe=i

ai<^b^^)c3j **
-^

/.l/li^^^'fe'^^'^'^r

6.

Prove: It A = [a^-] = AB + AC.

is of order

mxn
of

and

if

S =

[fe^-] ^

and C =

[c--] are of order


'

nxp, then

/}(B +

C)

The elements
feijj+cij,

of the

j'th

row

are a^^

a^^,

...

a^^ and the elements of the /th column of S

+C
sum

are
is Of

62J+

c'2j

fenj

+%j- Then

the element standing in the ith row and /th column of

A(B + C)
the

oil(feij + Cij) + afe(fe2j + C2j) +... +o.i(feni +

%i)

the elements standing in the ith row and /th column of

^^^"ikHj+^kj) AB and ^C.

S^^iifefefej +^2^aifeC;^j,

7.

Prove: If i = [aij] is of order then ^(6C) = (iS)C.

mxn,

if

S =

[6^-] is of order rexp,

and

if

C =

[c.-A is of order
-^

pxo

The elements
p

P
of the
j

throw

of /4 areai^.a^g, ...,a-

and the elements ofthe/th column


in the ith

of

BC

are

2
h=
i

6ih c^

P
b^f^Cf^-

'iJ'

2
P

b^j^c^j-,

hence the element standing


P +
...

row and /th column of A (BC) is


P n ^^"ik<-j^,^hh<^hj^

P + tn

ait^^b^chj +ai2^^b2h%j
=

J^

*n/i<;/ij

P n ^^^^j_"ik''kh)<^hj

n
=

n
+ (X^aif^bf^2)<:QJ +

n
+ (^^"ikbkp)<^pj

(2^^0ifefefei)cij

This

is the

element standing in the

j'th

row and /th column of (AB)C; hence, A(BC)

(AB)C.

8.

Assuming A, B ,C,D conformable, show


Using
(e)
(/")

in
= =

two ways that (A +B)(C +D) =


(A

and then

(/), (e),

Using

and then

(A+B)(C+D) (A+B)(C+D)

+B)C

+ (A

+B)D

AC + AD AC +BC +AD +BD.

BC

BD.

A(C +D) + B(C +D) = AC + AD ^ BC + BD = AC +BC +A.D +BD.

CHAP.

1]

MATRICES

"l

o'
1
1

l'

"l

10
1 1 1 1

'l

o'
1 1

"4

2"

9. (a)

1 1

2 3
_3

[3

2]

2 3

4
6

6 9

3
3

4
7

2_

'1

10
2
'

10
1

0'

'1

10
1
1

10
1

o"

[0]
2

:]

10
4
i

oil
1

(b)
!

10 3 10
5 6

:o]
[0

1
[0:

[0]
3]
]

j2
1

10

10

[0]
3
1

[0

"][o

2 3

12

10
18
"1

0'

4
5 6 7 6 5

5 6

2 3
4'

4
5

5
6_

1 1

-4- i

0^0
1

2 3

3|4
4 15
5
1

4 3
1

2 3
1 1

3
1

2
1

7 8
5

3
1

2
1

3 4
9

2
1
1

5 6

6 7 6

7"

2
1 1

(c)
1

2
1

4
9 8

6
7

2
1

5
8.

2
1

8 5

2
1

_0

+i

-^1

8 7

[l]-[8 7]

[l]-[6 5 4]
9
11'
"13"

[1]-[1]

'7
10 31

11

13

13

16
39"

19
41"

20
13

33 22
13

"35

24
.13

37 26
13
5

31

28
13
4]

30
.13.

20
13
8

33 22
13
7

10 35 24
13
6

13

16

19

37 26
13
5

39 41 28 30
13 13
1

[6

[l]

%i
10.

Hiyi+
os yi +

"1272
11^1
.J1
0]^2^2

Let

%2 %3

= a^iy-i + 0^272 be three linear forms in y^ and yj ^-nd let


1

be a
O21 Zi +
h-2

% 2 72
new coordinates

linear transformation of the coordinates (yi, j^) into

(z^, zg).

The

result of applying

the transformation to the given forms is the set of forms


%i
%<2

=
~

(o-Li

6^j +
"*"

aj^2

621) Zi
1) ^1

+
"*"

(on ii2 +
(^2 ("3
1

'^12^22) ^2
^2 2 ^22) ^2
Z2
-.

((^2 1 C^l 1

^2 2 ^2

^12

"*"

X3

("31^11 + ^32^21) Zl

1^12 + "32 ^22)

*i

%1
"21
^3
1

012
"22

r-

Using matrix notation, we have the three forms


611
O21 612

Vl
Vr,

and the transformation

'^3

The

result of applying the transformation is the set of three forms


'xi

022

"11
"2 Os
1

%2
"^2 ^^3

pri ii2
^2
1

-1

/,
^',

2 2

622

Thus, when a set of m linear forms


formation of the variables with matrix B

in n variables
,

there results a set of

with matrix A is subjected to a linear transm linear forms with matrix C = AB

'

MATRICES

[CHAP.

SUPPLEMENTARY PROBLEMS
"l

-3'
2
,

-1
2

2I

4
,

2I 2
3_

Given A =

5
.1

4
_2

5
3_

and C =

-1

1_

J -2
-3
I

4
(a)

-f
7
4j
,

-51

Compute: A + B =

9
_3

A-C

=
_

5-3
t-2j

-1

-2 -4
(6)

Compute:

-2-4

-10

-2
(c)
(d)

-4 -2

0-S

Verify:

A + {B~C)

(A+B)-C.
that

Find the matrix

D such
1

A+D^B.
1

Verify that

=B-A

-(A-B).
-11

-1
2
1

6 12 6

12.

Given A =
generally.

-3

-1

and B

2
1

4
2

6
3

compute

AB

= Q

and

BA

-22 -11

-2

-1 -2 -1

Hence, ABi^BA

\ -3
13.

2
,

Given A =

-3 -3 -1
1

111 1-212
C
1

14

10
and C

2 3

1-1-2
-2 -1 -1
show
that

AB

AC. Thus, AB

AC

2-5-1

does not necessarily imply B

-1
3
,

3 2
,

14.

Given A =

2 3

and C

^["12
[2

3 -4"!
ij

-1

-1

0-2
=

show

that

(AB)C

A(BC).

15.

Using the matrices of Problem

11,

show

that

A(B + C)
+ B^

AB

AC

and

(A+B)C

AC +BC.

16. Explain why, in general,

(ABf ^ A^

2AB

and A^

- B^ ^ (A-B)(A+B).

-3 -5
5

17.

Given A =

-14
1

-13
B
= 0,
1

-2 -4
4

-3 -5
5

and

-3 -4
=

-13
AC
= A, that

-13
1

-2 -3

(a) (b)

show

that

AB

BA

CA

= C. =

use the results of

(a) to

show

ACB

CS^,

A'^

B"^ =

(A

-B)(A +

B),

(A

Bf

= A'^ + b'

18.

Given
Ans.

where

2
i

= -1, derive a formula for the positive integral

powers of A

I,

A, -I,

-A according

as n

=4p,

4p-l-l,

ip +

2,

4p-^3, where

/=

n
[0

ij-

19.

Show
[

that the product of

any two

or

more

-'"'[;;][-:;] [rM'i

-"]

[0 -]- ["0 3-

is a matrix of the set.

[? 0]

20.

Given the matrices A of order mx, B of order nxp, and C of order rx^, under what conditions on p, 9, and r would the matrices be conformable for finding the products and what is the order of each" (a) ABC (b)ACB, (c)A(B + C)?
Ans.
(a)

p =

r;

mx-q

(b) r = n = g;

x p

(c)

= n, p = q;

x g

CHAP.

1]

MATRICES

21.

Compute AB, given:

10
(a)

11
1
j

10
and

10
j

B
1
"

1 1 1

Ans.

1 1

2
1

0^
1

o"
1

(b)

and
1

B
--1

Ans.

m
4
1

12
1

10
'

10
'2

(c)

A
1

and
1
I

B
1
1
i i

Ans
2

10
2

22. Prove:

(a) trace (A

+ B) = trace A + trace B,

(6) trace (kA) = k trace

A.

h V,
=

-2
1

1 1

2n+r.-3y.
722"]

Y^l

^;^^^l^

y,\

[2

-3

[2

-3

2 2

-1
3

r -zi +

[-221 - 622J'
24. If
-4

= [aiji

and B = [fe^J are of order

m
=

x n

and

if

= [c^j] is of order nx. p,

show
,n).

that

(/4+B)C

=^C
the

+ BC.

25. Let

/4

= [a^-] and

[fej,-;,]

where

(i

1,

m;

1,

p;

A;

1, 2, ...

Denote by

/S-

sum

of

Pi
182

the elements of the /th row of B, that is, let fij =

j)^.

Show

that the element in the ith row of

Pi
is the

sum

of the

elements lying in the sth row of AB.

Use this procedure

to

check the products formed

in

Problems 12 and
26.

13.

A relation (such as parallelism,


(i
)

congruency) between mathematical entities possessing the following properties:


b.

Determinative
Reflexive

Either a is in the relation to 6 or a is not in the relation to


a is in the relation to a, for all
If a.

(II)
(iii)

Symmetric
Transitive

a is In the relation to b then b is in the relation to a.

(iv)

If a is in

the relation to b and b is in the relation to c then a is in the relation to

c.

is called an

equivalence relation.
similarity of triangles,

Show
relations.

that the parallelism of lines,

and equality of matrices are equivalence

Show

that perpendicularity of lines is not an equivalence relation.

27.

Show

that conform ability for addition of matrices is an equivalence relation while con form ability for multi-

plication is not.
28. Prove:
If .4, B,

are matrices such that

AC

CA and BC

CB,

then

(AB

BA)C

C(AB

BA).

chapter 2
Some Types
of Matrices

THE IDENTITY MATRIX. A


lar;

square matrix A whose elements a^: =

for

i>j

is called

upper triangu-

a square matrix A whose elements


"ii

a^=

for

i<j

is called

lower triangular.

Thus

"12
tip o

"13
tZo o

"2n
is

03S

upper triangular and

is

lower triangular.

The matrix D

Ogj

which

is

both upper and lower triangular,

is call-

...

a,

ed a diagonal matrix.

It

will frequently be written as

diag(0]^i, 022, 033,

'

^nn )

See Problem
If

1.

in the diagonal matrix

D above,

Oi^ = Ogs =

ci

k,

D
'1

is

called a scalar matrix;


/.

if.

in addition, k = l, the matrix is called the identity matrix

and

is

denoted by

For example

t"]
When
/+/ +
...

and

1
1

the order is evident or immaterial, an identity matrix will be denoted by /. Clearly, to p terms = p / = diag (p,p,p = I-I ... to p factors = /. Identity map) and

[1 .

2
c

3I
(5
,

then l2-A =

A.L

hAh

A, as the reader

may

readily show.

10

CHAP.

2]

SOME TYPES OF MATRICES

11

SPECIAL SQUARE MATRICES.


matrix,
it

If

A and B

are square matrices such that


It

AB

= BA, then A and B are


if

called commutative or are said to commute.

is a

simple matter to show that

is

any ra-square
2.

commutes with

itself and also with

L
See Problem

If

A and B are such


matrix

that

AB

-BA,
A,
k-t-i

th^ matrices
A:

A and 6

are said to anti-commute.


If

for

which A

,fe+i

where

is a positive integer, is called periodic.

k is

the least positive integer e for which


If

A,

then A is said to be of period

k.

i =

1,

so that A^ = A, then A is callled idempotent

See Problems

3-4.

matrix

for

which

4=0,

aP

where p
0,

is

a positive integer, is called nilpotent.

If

p is the
5-6.

least positive integer for which A'^ =

ther

is said to

be nilpotent of index

See Problems

THE INVERSE OF A MATRIX.

If A and B are square matrices such that AB = BA = I, then B is called the inverse of A and we write B = A'^ (B equals A inverse). The matrix B also has A as its inverse and we may write A = B~^
1

3 3

-2 -3
1
1

o'
/,

Example

1.

Since

-1

=
( 1

each matrix in the product is the inverse of

12
the other.

-1

We

here, however, that if

shall find later (Chapter?) that not every square matrix has an inverse. A has an inverse then that inverse is unique.

We can show
See Problem
7.

If A and B are square matrices of the same order with inverses A^^ and B~^ respectively, then (AB)''^ = B'^-A~^, that is,
1. The inverse of the product of verse order of these inverses.

tijvo

matrices, having inverses, is the product in re-

See Problem

8.

A
utory.

matrix

A such

that A^ = I is called involutory.

An

identity matrix, for example, isinvol-

An

involutory matrix is its

own inve r$e.


See Problem
9.

THE TRANSPOSE OF A MATRIX. The


columns
of

matrix of order

an mxn matrix A

is

called the trahspose of


1

nxm obtained by interchanging the rows and A and is denoted by A' (A transpose). For
4'

example, the transpose of A

^[-e]

IS

i' =

5
6.

Note that the element

aIr

in the ith

row

and /th column of A stands


If

in the /th

row and ith column of A\

A' and ZTare transposes respectively qf


(a)

A and

B,

and
(b)

if

A:

is

a scalar, we have immediately

(A'Y =

and

(kAy = kA'

In

Problems 10 and
II.

11,

we

prove:

The transpose

of the

sum

of two matrices is the

sum

of their transposes, i.e.

(A +

BY

A'+ S'

12

SOME TYPES OF MATRICES

[CHAP. 2

and
in.

The transpose
i.e.,

of the product of

two matrices
=
B'-A'

is the

product in reverse order of their

transposes,

(AB)'

See Problems 10-12.

SYMMETRIC MATRICES. A
matrix
1

A =
2

a^j]

is

square matrix A such that A'= A is called symmetric. symmetric provided o^-- = a,for all values of i and ,^
,

/.

Thus, a square For example,

4-5 3-5 6
2 In

is

symmetric and so also is kA

for

any scalar

k.

Problem
IV.
If

13,

we prove
an ra-square matrix, then A + A'
is

is

symmetric.

A square matrix A such that A'= -A is called skew-symmetric. Thus, a square matrix A is skew-symmetric provided a^.- = -Uj^ for all values of i and /. Clearly, the diagonal elements are
"

-2
2

3"

zeroes.

For example,

is

skew-symmetric and so also

is

kA

for

any scalar

k.

-3 -4
With only minor changes in Problem 13, we can prove
V.
If

is

any re-square matrix, then A -

A' is

skew-symmetric.

From Theorems IV and V follows


VI. Every square matrix A can be written as the sum of a symmetric matrix B = ^(A+A') and a skew-symmetric matrix C = ^(A-A'). See Problems 14-15.

THE CONJUGATE OF A MATRIX.


called a complex number.

being the conjugate of


If

Let a and b be real numbers and let i = V-1 then, z = a+ hi is The complex numbers a+bi and a-bi are called conjugates, each the other. If z = a+ bi, its conjugate is denoted by z = a+ hi.
;

z^

= a + bi

and

Z2

z^

a~

bi,

then

Z2 =

z-^

a-

bi

= a+

bi,

that is, the conjugate of

the conjugate of a complex number z is z itself.


If
(i)

z.i=a+bi and
z-L

Z2

= c+

di,

then
z^

+ Zg = (a+c) + (b+d)i

and
of

z^

= (a+c) - (b+d)i =
is the

(a-bi) + (c-di)
of their conjugates.

that is, the conjugate of the

sum

two complex numbers


and

sum

(ii)

z^- Zg

= (ac-bd) + (ad+bc)i

z^^

= (ac-bd)- (ad+bc)i = (a-bi)(c-di) = F^-i^,

that is, the conjugate of the product of two complex

numbers

is the product of their conjugates.

When i is a matrix having complex numbers as elements, the matrix obtained from A by replacing each element by its conjugate is called the conjugate of /4 and is denoted by A (A conjugate).
l

+
3

2i

-21
3 2

Example 2. When A

then

=
3i

2-3i

If

A and B

are the conjugates of the matrices


(c)
(.4)

A and B and itk


(c?)

is

any scalar, we have readily

= A

and

(X4) = ~k-A

Using

(i)

and

(ii)

above,

we may prove

CHAP.

2]

SOME TYPES OF MATRICES

13

_VII. The conjugate of the sum of two matrices is the sum of their conjugates, (A + B) = A + B.

i.e.,

their conjugates, i.e.,

Vin. The conjugat e of the product of two matrices (AB) = A-B.


of

is the product, in the

same

order, of

The transpose We have


IX.

is

denoted by A^(A conjugate transpose).

It

is

sometimes written as

A*.

A, i.e.,

T^e transpose (Ay = (A').


2 3

of the conjugate of

is

equal to the conjugate of the transpose of

Example

3.

Prom Example
1

- 2i
-i

(AY

=
2+
3i

while

+
i

2i

A'
2

and

[l-2i
(A')

3 2

1
=

(AY

Zi

-i

+ 3iJ

HERMITIAN MATRICES. A
is

square matrix Hermitian provided a^j = uj^ for Hermitian matrix are real numbers.
1

^ =
all

[0^^]

values of

such that i and /.

A'

= A

is called Herraitian.

Thus,

/I

Clearly, the diagonal elements of an

1-i
3

2
i

Example 4. The matrix A

+j
2

is Hermitian.

-I

Is

kA Hermitian
square matrix

if

k is any real number? any complex number?


[0^^]

^ =

such that
for all

1= -A
i

is

called skew-Hermitian.
/.

Thus, ^

is

skew-

Hermitian provided

Clearly, the diagonal elements of a skew-Hermitian matrix are either zeroes or pure imaginaries.
i

a^^ = -o^--

values of

and

l-i
3i
i

2
i

Examples. The matrix A

-1-t
-2

is

skew-Hermitian.

Is

kA skew-Hermitian

it

k is any real

number? any complex number? any pure imaginary?

By making
X.
If

minor changes in Problem 13,


is

we may

prove

an ra-square matrix then 4 +

Z is

Hermitian and

A-T is

skew-Hermitian.

From Theorem X follows


XI. Every square matrix A with complex elements can be written as the sum of an Hermitian matrix B={(A + A') and a skew-Hermitian matrix C=i(A-A').

DIRECT SUM.
ization

Let A^, A^

As be square matrices of respective orders m^,

,ms.

The general-

^1 A^

...

...

diag(^i,^2
...

As)

A,

of the diagonal matrix is called the direct

sum

of the Ai

14

SOME TYPES OF MATRICES

[CHAP. 2

-1
3

Examples. Let

/4i=[2], ^5

B:}

and An

2 4
1

-2
2

12
The
direct

sum

of

A^,A^,Aq

3
is

diag(^3^,

/42, /4g)

12-1
2

1-2

Problem 9(6), Chapter

1,

illustrates
...,Ag)
s),

the

XII. If A = Aia,g {A^,A^, same order for (s = 1, 2

and

S = diag(Si, S2
= diag(^iSj^, A^B^

^s).

where Ai and Behave

then

AB

^s^s)-

SOLVED PROBLEMS
ii
Or,

"12
"22

'm

11^1
22 21

1.

Since

2i

2n

OooOoo

"^22 ''2n

the product

AB

of

''ml

"7)12

"tn

''WB "Ml

''wm"m2

^mm"nn

an m-square diagonal matrix A = diag(oii, a^^ an) and any mxn matrix B is obtained by multiplying the first row of B by %i, the second row of B by a^^- and so on.

2.

Show

that the matrices

and

K
P^+*'^

commute

for all

values of

o, 6, c, J.

This follows from

\"

^1

"^l

'"^

'^l

["

*1

2
3.

-2 -4
4
is idempotent.

Show

that

-13
1

-2 -3_
2

-2 -4'
4

'

-2 -4"
4

-2
3

-4
4

-13
_ 1

-13
.

-2 -3_

-2 -3_

-2

-3

4.

Show

that if

AB

=
=

A and BA =
A-A
=

B,

then i and
= .4(34) =

are idempotent.
=

.45.4 =

(AB)A

A^ and

ABA

AB

then 4^ =

.4

and 4

is idempotent.

Use BAB

to

show

that

is idempotent.

CHAP.

2]

SOME TYPES OF MATRICES

15

1 5.

3 6
is nilpotent of order 3.

Show

that

2
1

-1
3

-3
1 3" 1 1 1

5
-2

3 -1

and

A^.A

-1 -3

-2 -1 -3

-1 -3

-1 -1 -3

-2 -1 -3

6. If

is

nilpotent of index
0,

2,

show
=

that
= 0.

AOAf=

for n
=

any positive integer.


=

Since /? =

A^ =

a''

...

^"

Then A{IAf

A{lnA)

AnA^

= A.

7.

Let
C.

A,B,C
Thus,

5 = C= ^

be square^ matrices such that AB = 1 and CA = 1. Then ^ is the unique inverse of ^. (What is S~^?)

iCA)B =

CUB)

so that B

8.

Prove:

(AB)^ =
definition

B~^-A'^

By

{ABf{AB)
(B' -A

(AB)(ABf
=

/.

Now
B^-I-B
=

)AB

b\a''-A)B
=

B-'.B
=
/

and

AB(B~'^-A~'-)

A(B-B''^)A~^

A-A

By Problem

7,

(AB)'^ is unique; hence,

(AB)^

B-'.A-'

9.

Prove:

matrix

is involutory if

and only

if

(IA)(I+ A) =
I

0.

Suppose (I-A)(l+A) = I-A^ = 0; then A^ =

and

/I

is involutory.

Supposed

is involutory; then

A^ =

and (I-A)(I+A) =

I-A^

I-I

= 0.

10.

Prove:
Let
A'.

(A+B)' =

A' +

B'.

A = [ay] and B

= [6^].

We need
bjj_.

only check that the element in the ith row and /th column of
bj^.

S' and (A+B)' are respectively a^,

and aj^+

11.

Prove:

(AB)' =

S'i'.

Let A = [ay] be of order mxn,

[6y] be

of order
is

nxp
=

then

AB

[cy]

is of order

mxp.

The

element standing in the ith row and /th column of


ing in the /th row and ith column of (AB)'.

AB

cy

J^aife. b^j and this is also the element stand-

The elements of the


"i2

/th row of S'are


in the /th

iy, b^j

bnj

and the elements of the ith column of ^'are a^^,


B'/l'is

Hn- Then the element

row and ith column of

Thus,

(AB)' =

B'A'.

12.

Prove:

(ABC)' = CB'A'.

mite

ABC

(AB)C.

Then by Problem
,

1 1

(ABC)' = \(AB)C\' = C'(AB)' =

C'B'A'.

2g

SOME TYPES OP MATRICES

[CHAP.

13.

Show

that if

A =

[o^j]

is n-square, then

B =

[bij] =

+ A' is symmetric.

First Proof.

The element
bij = aij + a^i.
oj^j]

in the ith

row and /th column of


in the /th

.4

is aij

and the corresponding element of


a^i

/I'

is aji;

hence,
.4'

The element
= a^^ + aij.

row and ith column of A is

and the corresponding element of

is

hence,

bji

Thus,

bij = bji

and B

is

symmetric.

Second Proof.

By Problem

10,

(A+Ay

A' +

(A')'

A'+A

A + A' and

(^ +.4') is symmetric.

14. Prove:

If

A and B

are re-square symmetric matrices then

AB
B'A' =

is

symmetric

if

and only

if

A and B

commute.
Suppose A and B commute so that Suppose
trices

AB

BA.

Then (AB)' =

BA

AB and AB
;

is

symmetric.
=

AB

is

symmetric so that (AB)' = AB.

Now

(AB)' = B'A' =

BA

hence,

AB

BA and

the ma-

A and B commute.

15. Prove:

P'AP

is
.4

the m-square matrix A is symmetric (skew-symmetric) and symmetric (skew-symmetric).


If

if

is of order

mxn then B =

If If

is is

symmetric then (see Problem 12) B'

(P'AP)' = P'A'(P')' = P'A'P

P'AP

and B

is

symmetric.

,4

skew-symmetric then B' = (P'AP)'

-P'AP

and B is skew-symmetric.

16. Prove:

If

A and B are

re-square matrices then


k.

A and 5 commute

if

and only

if

A~

kl and

B-

kl

commute

for

every scalar

Suppose A and B commute; then

AB
=

= B.4

and
+ k^I + k^I
=

{A-kI)(B-kI)

AB - k(A+B)
BA - k(A+B)

(B-kr)(A-kl)

Thus,

Akl

and B kl commute.

Suppose 4 -A;/ and B -kl commute; then

(A-kI)(B-kI)

AB - k(A+B)
BA - k(A+B)

+ k^I + k^I
=

(B

-kr\(A-kI)

AB

BA. and A and B commute.

CHAP.

21

SOME TYPES OF MATRICES

17

SUPPLEMENTARY PROBLEMS
17.

Show

that the product of

two upper (lower) triangular matrices

is

upper (lower) triangular.


=

18. Derive a rule for forming the product

BA

of an

mxn

matrix

S and ^

diag(aii,a22

a).

Hint.

See Problem

1.

19.

Show

that the scalar matrix with diagonal element


of / is the

A:

can be written as

Wand that kA

klA =Aia.g(k,k

k) A,

where the order


20.
If .4 is

row order of A
a'^ a'^ = a'^

re-square,

show

that

A^ where

and q are positive integers.


5

2-3-5
21. (a)

Show

that

-14
1

-13
and

_3 _5
3

are idempotent.

-3

-4J

[-1

(b)

Using A and B, show that the converse of Problem 4 does not hold.
is idempotent,

22.

If

show
2

that

I-A

is

idempotent and that

AB

BA

= 0.

2
1

23. (a)

If

2
1.

show

that

.4

4.A

5! =

0.

2
2 (b) If

2
1

3 2
1
,

1
1

-1
2

show

that

-4

- 2A

- 94

0.

but

-4^

2/1

9/ /

0.

-1 -1 -1"
24.

2
1

o"

3
1

Show

that
_

1 1

-1 -1
1

=
-1

/.

-1 -1

-2 -6
2

25.

Show

that

-3

is periodic, of period 2.

0-3

-3 -4
4
is nilpotent.

26.

Show

that

-13
1

-3 -4

'12
27.

-1 -6
2 9

Show

that

(a)

and

commute.

-1 ~1 -1

_-l -1 -4_
2/3

"112'
(b)

-1/3'

and

-3/5

2/5

1/5

commute.

-12
28.

7/15 -1/5

1/15

Show

that

anti-commute and (A+

Bf

A^ +

B'^

29.

Show

that each of

anti-commutes with the others.

30. Prove:

The only matrices which commute with every n-square matrix


commute with diag(l, 2, 3). which commute with diag(aii,a22
where a,6,
c

are the n-square scalar matrices.

31. (a) Find all matrices which


(b)

Find

all matrices

a^^).

Ans.

(a) diag(a,fe, c)

are arbitrary.

18

SOME TYPES OF MATRICES

[CHAP. 2

3 7
I

"3-2-1
is the inverse of

32.

Show

that

(a)

-4
2
"

-1
1.

-2 -4 -5

3
is the inverse of

(b)

10 2 10 4 2 10

-2100
0-2
_ 8

10
1.

-2311
33. Set

-1 -1

[3:][:^E3

to find the inverse of

[3

4]-

^"'-

[3/2

-1/2]
are different from zero, is a

34.

Show

that the inverse of a diagonal matrix A, all of

whose diagonal elements

diagonal matrix whose diagonal elements are the inverses of those of


inverse of / is /

A and

in the

same

order.

Thus, the

-1
4 4

3"|

35.

Show

that

4-3 3-3

and

-1 0-1 -4 -4 4-3J

are involutory.

10
36. Let

10
-1

I2

/j.

by partitioning.
/I21

Show

that

A
/s

a
c

'2

0-1
(b) (kA)" = kA',

37. Prove:

(a)(A')'=A,

(c) (^^)' = (/l')^ for p

a positive integer.

38. Prove:

(ABCy

C'^B-^A-'^.

Hint. Write

ABC=iAB)C.
(c) (A'^Y^ =

39. Prove:

(a) (A-^)'^ =

A,

(b) (kA)-^ = -j-A'^,

(A'^y forp a positive

integer.

40.

Show

that every real symmetric matrix is Hermitian.

41. Prove:

(a)(A)=A,
1

(b) (A +

B)
I

A + B
i

(c)(kA)=kA,

(d)

{AB) = A B.

1
J

+
2

+ 3

42. Show:

(a)

2- 3
1
i

-i

is

Hermitian,

1
i

+
2i

(b)

-1 +

is

skew-Hermitian,

-2-3J
(c)
(<f )

-1

iB is Hermitian,

is Hermitian

and B is skew-Hermitian.

43.

If

is n-square,

show

that

(a)

AA' and A' A

are symmetric,

(6) A-irA',

AA', and A'A are Hermitian.

44. Prove:

If

is

Hermitian and A is any conformable matrix then (A)'

HA

is Hermitian.

45. Prove:

Every Hermitian matrix A can be written as B + iC where B

is real

and symmetric and C is real and

skew-symmetric.
46. Prove:
(a)

and C
47. Prove: 48.

is real and symmetric.

Every skew-Hermitian matrix A can be written as A = B + iC where B is real and skew-symmetric (6) A' A is real if and only if B and C anti-commute.
A' and

If

A and B commute so also do ^"^ and B'

B\
if

and A' and B"

Show

that for

m and

n positive integers, ^4

and S"^ commute

A and 5 commute.

CHAP.

2]

SOME TYPES OF MATRICES

19

A A
49.
1

A
1

nA
,ra

Show

raA

(a)

(6)

2n('-l)A n-i

nX
A"

A
and

50. Prove:

If

is

symmetric

or

skew-symmetric then AA'= A'A

are symmetric.

51. Prove: If

is

symmetric so also is

a-4

+6/1^

+.-.+/

where

a, 6

are scalars and p is a positive

integer.

52. Prove:

Every square matrix A can


If

bfe

written as

/I

B +C where B

is

Hermitian and C is skew-He rmitian.

53. Prove:

is real

and skew-symmetric or

if

is

complex and skew-Hermitian then iA are Hermitian.

54.

Show

that the theorem of Problem 52 can be stated: Every square matrix A can be written as A =B+iC where B and C are Hermitian.

55. Prove:

If

A and B
(c)

idempotent,

are such that AB = A and ^ = B = / if ^ has an inverse.

BA

then

(a) B'A'= A'

and

A'B"= B\

(b) A"

and B' sue

56.

If^

is involutory.

show

that k(.I+A) and

kO-A)
,

are idempotent and

j(I+A)

^(I-A) =

0.

57. If the
(a)

n-square matrix A has an inverse A

show:

(A

Y=

(A')

(b)

(A)^ = A-\
of

(c)

(r)-^=(^-V
(A'''-)

Hint, (a)

Prom the transpose

AA~^

I.

obtain

as the inverse of

/!'

58.

Find

all

matrices which commute with (a) diag(i.


(b) dia.g(A.B)

i, 2, 3),

(6)

diag(l.

1, 2, 2).

Ans. (a) dia.g(A. b.c).

where A and B are 2-square matrices with arbitrary elements and

b. c

are scalars.

59. If

A^.A^

A^

are scalar matrices of respective orders


'^s)

mj^.m^

m^.

find all matrices

which commute

with

diag(^i, ^2

Ans. dia.g(B^.B^

B^) where Si, S2

-85

are of respective orders m-^.m^,

m^ with arbitrary elements.

60. If

AB

= 0,

that the matrices

where A and B are non-zero n-square matrices, then A and B are called divisors of zero. A and B of Problem 21 are divisors of zero.
A^)

Show

61. If
...,

A
s),

diae(Ai.A2

and

di&giB^.B^

B^)

where ^^ and B^ are of the same order,

(J

1, 2,

show
=

that

(a) (b) (c)

A+B
AB
=
trace

diag(^i+Si,^2 + S2
=

-^s

+ Bs)

diag(^iBi. /I2S2

A^B^)
...

AB

trace /liB^ + trace /I2S2 +

+ trace A^B^.

62. Prove: If

^ and B
A

are n-square skew-symmetric matrices then

AB

is

symmetric

if

and only

if

A and S commute.

63. Prove: If

is n-square

and B = rA+sI, where

and

are scalars, then

A and B commute.
Prove that
Ci =

64.

Let A and B he n-square matrices and let ri, rg, si, S2 be scalars such that ri4+siB, C2 = r^A+SQB commute if and only if A and B commute.

risj

7^

rssi.

65.

Show
(6)
/I

that the n-square matrix A will not have an inverse when (a) A has a row (column) of zero elements or has two identical rows (columns) or (c) ^ has arow(column)whichis the sum of two other rows(columns).

66. If

A and B

are n-square matrices and

A has an inverse, show that


=

(A+B)A~'^(A-B)

(A-B)A''^(A+B)

chapter 3
Determinant of a Square Matrix

PERMUTATIONS. Consider
(3.1)

together the 3! = 6 permutations of the integers 1,2,3 taken

123
4!

132

213

231

312

321

and eight of the


(3.2)

= 24 permutations of the integers 1,2,3,4 taken together

1234 1324
If in

2134 2314

3124 3214

4123 4213

we say that there is an a given permutation a larger integer precedes a smaller one, the permutation is (odd), even is inversion. If in a given permutation the number of inversions there is no inversince even called even (odd). For example, in (3.1) the permutation 123 is permutation the 312 is even since in sion, the permutation 132 is odd since in it 3 precedes 2,
it

2,

and 3 precedes 2. In (3.2) the permutation 4213 4 precedes 1, 4 precedes 3, and 2 precedes 1.
precedes
1

is

even since in

it

4 precedes

THE DETERMINANT OF A SQUARE MATRIX.

Consider the re-square matrix


'11

"12 "IS
*

Uo 1 Oo p ^03
(3.3)

-'ni

"ne "n3

and a product
(3.4)
^j, "'^L ~^3 1 ~-'2 3Jo
'^" "^n

comes from any row and one of n of its elements, selected so that one and only one element of convenience, the factors matter as a column. In from any (3.4), comes one element and only
re; the have been arranged so that the sequence of first subscripts is the natural order 1,2 of the intepermutations re! of the one some then is subscripts second sequence j^, j^ / of (Facility will be gained if the reader will parallel the work of this section bere. gers 1,2

ginning with a product arranged so that the sequence of second subscripts is in natural order.) For a given permutation /i,/2, ...,^ of the second subscripts, define ^j^j^....j^ = +1 or -1

according as the permutation is even or odd and form the signed product
(3.5)

-W-2-

k'^k ^^h

'^Jn

By

ucts of the form (3.5), called terms of


(3.6)

signed prodthe determinant of A, denoted by U|, is meant the sum of all the different Ul, which can be formed from the elements of A; thus.

le Jlj2p=n\
The determinant

M
re

'^h

where the summation extends over

permutations hk---Jn of the integers 1,2,,


is called

of a square matrix of order

a determinant of order

re.

20

CHAP.

3]

DETERMINANT OP A SQUARE MATRIX

21

DETERMINANTS OF ORDER TWO AND THREE. Prom


-"ll

(3.6)

we have

for n

and n = 3,
CE-j^2^21

"12
12

(3.7)

^11^22

"'"

21 '^12^^21

^21

"22

11^22

and
'll

"12

"13
P*^

(3.8)

21 "31

"?

^123

ail0223S +
231

^132 Oll02s'^32

^213 Oi221<%3

"32

"S3

%2 02331
^11023032

312 Ol3'%lC'32

^321

%3 ^^2 %!

flu 022 %3

012^1033

%l(22 033 - 02S32) " "12(021 "ss " ^S^Sl) + ^IS (a2lOS2 - 022^31)
022
"^32

^3 ^3

O21
"31

053

21

<^2
^^32

+
<^33

Oi
'^Sl

Example
1

1.

21
1-4

(a) 3 2
(fc)

2-3

-2

-11
2-0

(-1)3

+ 3

3 2 3 5 10
1 1

(O

11

01

2
11

+
2

11

2(0-0
2
(d)
1

-1-1) - 3(1-0 -1-2) + 5(1-1-0-2)

2(-l)

- 3(-2) +

5(1)

-3 -4

0-2
-5 -6

2{0(-6)-(-2)(-5)! - (-3){l(-6)- (-2)0! + (-4) {l(-5) -20


18

O-o!

20

-18

See Problem

1.

PROPERTIES OF DETERMINANTS.
nant

Throughout this section, A

is the

square matrix whose determi-

Ul

is

given by (3.6).

and we have
I.

Suppose that every element of the sth row (every element of the/th column) is zero. Since every term of (3.6) contains one element from this row (column), every term in the sum is zero
If

is zero, then U| =0. Consider the transpose A' of A. It can be seen readily that every term of (3.6) can be obtained from A' by choosing properly the factors in order from the first, second, ... columns. Thus, II. If 4 is a square matrix then U'l = \A\; that is, for every theorem concerning the rows of a determinant there is a corresponding theorem concerning the columns and vice versa.

every element of a row (column) of a square matrix A

a scalar k.
fth

Denote by B the matrix obtained by multiplying each of the elements of the ith row of A by Since each term in the expansion of |5| contains one and only one element from its row, that is, one and only one element having A; as a factor,
\B\

1
P

\: , , a. , n^n \ *%'2---i'iii2J2----anj^!
.

Jc\A\

Thus,

22

DETERMINANT OF A SQUARE MATRIX

[CHAP.

every element of a row (column) of a determinant U| is multiplied by a scalar k, the determinant is multiplied by k; if every element of a row (column) of a determinant \A\ has k as
III.

If

a factor then k
a 7']^]^
O\
OrQ
1

may be factored from


ltd AQ
/l'

\A\

For example,

CI

t*

QO

f (X'

product in (3.6) of
(3.6) is the

Let S denote the matrix obtained from A by interchanging its ith and (i+l)st rows. Each \A\ is a product of |s|, and vice versa; hence, except possibly for signs,

expansion of
i

\b\.

In

counting the inversions in subscripts of any term of (3.6) as a

term of
its

\b\,

before i+1 in the row subscripts is an inversion; thus, each product of (3.6) with
|s|

sign changed is a term of


IV.
If

and

\e\

= -

\A\.

Hence,

is

obtained from A by interchanging any two adjacent rows (columns), then \b\ =

- \a\.

As a consequence
V.
VI.
|s|
If

of

Theorem

IV,

we have
its

is

obtained from A by interchanging any two of

rows (columns), then

\b\

= -\A\.

If

is

obtained from A by carrying

its ith

row (column) over p rows (columns), then

= (-i)^UI.
VII.
If

two rows (columns) of A are identical, then


of the first

\A\

Suppose that each element


(j

row of A

is

expressed as a binomial

%,
^li

b-^:

c-^j,

= 1,2,

...,n).

Then

^Jd2

in
ii2
O22

^^^ii ^

"^i^ "2 J23 J5

""in
Os-,-

p^kk^ Jn ^^k'^k'^k-h,. 21

"njn^

Os,

"Jn

^13

^m
02n

'In

"23

Oni
In general,
VIII.
If

n2

"na

'^nn

-^n2

'-"ns

every element of the ith row (column) of A is the sum of p terms, then

\A\

can

be expressed as the sum of p determinants. The elements in the ith rows (columns) of these p determinants are respectively the first, second, ..., pth terms of the sums and all other rows

(columns) are those of A.

The most useful theorem


IX.
If

is

is obtained from

A by adding

to the

elements of

its ith

tiple of the corresponding elements of another row (column), then

\b\

row (column), a scalar mul= \a\. For example.

Cfrii

T"/CCEog
{-

^22
Clf=n

Clnr
Clr^r,

Clf,',

Karin.

Cfg ^

H'Qri^

^Q'2

"^

i^Clr

Zgg +

ka^^

See Problems 2-7.

FIRST MINORS AND COFACTORS. Let A


by
(3.6).

be the re-square matrix (3.3) whose determinant \A\ is given

When from A

the elements of its ith row and /th column are removed, the determinant

of the remaining (re- l)-square matrix is called a first minor of

or of \a\

and denoted by

\M^j\.

CHAP.

3]

DETERMINANT OF A SQUARE MATRIX

23

More frequently,
cofactor of
a^

it is

and

is

called the minor of Oy. denoted by a;

The signed

minor,

(-if'*'^ \Mij\

is

called the

"V

11

12
'^SS

''13

Example

2.

If

"21
"'31

"23
''SS

^^32

M 11
and

22

"23

"21 "23

ki2
"32 "33
"31

21

22

kic
"33
"31

"32

1+1
I

1 + 21

(-1)

kll

1^11

ai2
I

(-1)
\m.

1^12!

14-31

(-1)

ki3

Then

(3.8) is

Ml

= =

aiilA/iil

O12IM12I
+

aislMigl

"11*11

ai20ii2

"130^13

In

Problem

9,

we prove

of the determinant U|, where A is the matrix of (3.3), is the sum of the products obtained by multiplying each element of a row (column) of U| by its cofactor, i.e..

X.

The value

n
(3.9)

Ul

aiiflii

a^g a^g

^in * in

^ aik'kk
n
fe?i"fei*j

(3.10)

Ml

"^if^if +

021*2/ +

a^jd^j

(ij, = 1,2

n)

Using Theorem
XI.

VII,

we can prove
of

ra-square

The sum of the products formed by multiplying the elements of a row (column) matrix A by the corresponding cofactors of another row (column) of A is zero.
If

an

Example

3.

is the matrix of

Example

2,

we have
+ +
"320^32

and
while

"310^31

+
+

"330^33

= =

Ul
'4
I
I

"12*12

"22*22

"32*32

and

"31*21 "12*13

+
+

"32*22
"22*23

+
+

"SS*23 "32*33

= =

See Problems 10-11.

MINORS AND ALGEBRAIC COMPLEMENTS. Consider the matrix (3.3). Let i^ arranged in h, i^ order of magnitude, be m, (1 < m < ra), of the row indices 1,2 n and let j^, j^ /^ arranged in order of magnitude, be m of the column indices. Let the remaining row and cofumn indi,

ces, arranged in order of magnitude, be respectively

i^,^,,

i^

a separation of the row and column indices determines uniquely two matrices
a.,
,

and /+;+2

Such

'l'J'2

'^i.

Jm

.Jl.j2.-

im

(3.11)

i2.ii

''2.

72

i2 :h

^n

"^m Ji

%J2

^n

24

DETERMINANT OF A SQUARE MATRIX

[CHAP.

and
''m+i'J'm+i
'-m

+ i'Jffl+s

n.7n
^ra+2>Jn

.7ra

+ i'Jm+2'

Jn

''m

+ S'Jm +

^m+2>7m+2

(3.12)

^^Jm-i-i

^n'j7ii+2

^ri'Jn

called sub-matrices of ^.

The determinant

of each of these sub-matrices is called a minor of


J1.J2.

A and the

pair of minors

Jm

J'm-'-i'Jn^-Q

Jn
''r

and
^m+l'
'-m-^2

are called

complementary minors
5 -square matrix

of A,

each being the complement of the other.

Examples. For the

[iij.
"I'Z

%4
^^34

'^15
'^SS

I
i

1,3 '^2,5

'^21
I

'^23

2,4,5

~
I I

and
f^Sl

'

'^l, 3.4-

"32 "42

63

"44

^^45

are a pair of complementary minors.

Let
(3.13)

U +
q

In

+ in +

h
+
i

and
(3.14)

i-n

+1 +

fm+2 +

+ *n +

^ra

+ /m+2 + " + In

Ji, is.

J,

The signed minor

(-1)

A,-

is called the algebraic Jn


^n

complement

of

Jm-'-i'Jn-i-2
7(4-1.

'to+2

J+i'Jm,+2

7n is called

and

(-l)'^

/I.
''m,+

the algebraic complement of


J1.72

l>^m+2> ''-

Jm
^n

H>h
Exainple4. For the minors of Examples.
I

2+-5 + 14-31

13
Ap' g

(-1)
I

=
I I

,1.3
/i^
I 1

is

the algebraic complement

Of

,2 4 5 A.{3\i

and

l(-3 + 4-^2l-446

(-1)

.2,4,5 A-^ 34

-I

2 4 5 ^ijg',4

is

the algebraic complement of


is

1.3

^^is

Note that the sign given to the two complementary minors

the same.

Is this

always true?
.Ji

Ji

When m =

l,

(3.11)

becomes
J2.J3.

A
Jn
is
OL-Lj.

[%ii]
Ml
i
I

and

"HJi

an element of A.

The

complementary minor
algebraic complement is the cofactor

in the notation of the section

above, and the

minor of A, whose diagonal elements are also diagonal elements of A,

is

called a principal
of A; the alge-

minor of A.

The complement of a principal minor of A is also a principal minor braic complement of a principal minor is its complement.

CHAP.

3]

DETERMINANT OP A SQUARE MATRIX

25

Examples. For the 5-square matrix A

H'
"?2
"11

"24
044

(^25

"15

^1.3

and
31 S3

A,

,t

0^2

045

52
are a pair of complementary principal minors of
.

"54

055

A What is the algebraic complement of each ? The terms minor, complementary minor, algebraic complement, and principal minor as defined above for a square matrix A will also be used without change in connection with U
|

See Problems 12-13.

SOLVED PROBLEMS
1- (")

1-1
1

!l 4|
2

2-4

- 3(-l)

11

4
(b)

5l

3
5

4 6

5 7 6

(1)
6

5 7

3 5

4
(l)(4-7

5-6)
=

+ 2(3-6

4-5)

71

-2-4
-18

-6

(c)

3
5
1

4
6

15
21

1(4-21

11

id)

3
1

5 3

1(3-3

5-1)

2.

Adding to the elements of the


-4
1 1 1 1 1

first

column the corresponding elements


1 1 1 1 1 1 1 1 1

of the other

columns,

1
1 1 1

-4
1

1
1 1

-4
1
1

-4

-4
1 1

1 1

-4
1

-4
1

-4

-4

by Theorem

I.

3.

Adding the second column


using Theorem

to the third,

Vn
1

removing the common factor from this third column and


a+b+ c a
b
c

a
b c

h
c

1 1 1

a
b
c

1 1 1

1 1

+a

a+h

a+b+c a+b+c

(a + b + c)

1 1

4.

Adding to the third row the first and second rows, then removing the common factor 2; subtracting the second row from the third; subtracting the third row from the first; subtracting 'the first row from the second; finally carrying the third row over the other rows

26

DETERMINANT OF A SQUARE MATRIX

[CHAP.

Oj^+fej

a^+b^
62 + ^2

ag+63
&3+^3
"s+^s+'^s
b-i

6^+C-L

62+ ^2
C2 + a2

^3+^3

i^+Ci
a5^+5]^+Cj^

bi + c^

b^+c^

b^+Cr

Cj+a^

C3+a3

a2+fe2+"^2
62

bi

63

62 C2

^3 Cg

a^^

02
^2 C2

flg

6^ +

C-L

62+^2
02

^3+^3
'^3

Cl
a-,

bi
C]^

63 Cg

a^

Oo 03

Oi
5.

%
^2

1
1 1

Without expanding, show that

Ul

02

-(Oj^

- 02) (02 ~

'^s) ('^s

%)

Og

Og

Subtract the second row from the

first;

then
Oi + a2
1 1

l-2
2

fl-j^

^2
ar,

02
2

(a^

2 02)

a2
2

O2

by Theorem

III

Og

Og
Similarly,

Og

ag

and

oj^-oj is a factor of U|.

a^-a^ and ag-a^

are factors.

Now M|

is of order three in the

letters; hence.
(i) \a\

k(a^^-a^)(a^-as)(as-a^)

A:

The product of the diagonal elements. 0^02. is a term of \a\ and, from (i), the term is -ka^a^. Thus, -l and \a\ = -{a^-a^){a^-a2){as-a^). Note that U vanishes if and only if two of the a^, og. os are
|

equal.

6.

Prove:

If

is
is

skew-symmetric and of odd order 2pskew-symmetric, -4'= -A; then Ut = -U! and Ul = 0.

1,

then Ml =

Since A

\-A

(-if^-^-UI

- Ul- But, by Theorem

II,

Ul

Ul; hence,

7.

Prove:

If

is
is

Hermitian, then \A\ is a real number.


Hermitian. A = A'
,

Since A

and
=

\a\ =

U'| =

U| by Theorem

II.

But
'^

if

Ul
then

^iij2...j"yi%

%;

+ *'

Ul
=

|eii;2---Jn^iii"2i?--^"in

"

"

^'

Now Ul

Ul

requires 6 = 0; hence.

U|

is a real

number.

2 3

8.

For the matrix A

2 2

12
a,,
=

(-ir'\l 12
(-ly
,2+1

?1
2 3

2,

ai2 =

(-1) 1+2

2
2 3 2

-2,

Mi3 =

1+3 (-ly

|i

11

2I
2 2

2 2 2 3

a22 = (-1)

2+2I

-1,

a,23

(-1)

2+3

1 1

2 3 2

(-ly

3+

age

(-1) 3+2I

3 2I

a 33

(-1)

3+3I

31

CHAP.

3]

DETERMINANT OF A SQUARE MATRIX

27

Note that the signs given

to the minors of the

elements

in

forming the cofactors follow the pattern

where each sign occupies the same position


cupies in ^,

in the display as the element, Write the display of signs for a 5-square matrix.

whose cofactor

is required, oc-

9.

Prove: The value of the determinant U| of an re-square matrix A is the sum of the products obtained by multiplying each element of a row (column) of A by its cofactor.

We

shall prove this for a row.

The terms

of (3.6) having a^^ as a factor are

Now

e.

kh---Jn~

^jbjb- -in

^i"''^ i"

^ P"'^tin

lJi..fe.

-in.

the

is in natural order.

Then

(a)

may

be written as
(6)

where the summation extends over the cr= (n-i)! permutations of the integers 2,3
022
(c)

n,

and hence, as

2S

<2n

"an
"ii

"11

'twill

^n2

"ns

Consider the matrix B obtained from A by moving its sth column over the first s-l columns. By Theorem \B\ = (-1) U|. Moreover, the element standing in the first row and first column of B is a^s and the minor of a^^ in B is precisely the minor \M^\ of a^s in A. By the argument leading to (c), the terms of ais mis\ are all the terms of \b\ having a^s as a factor and, thus, all the terms of (-1)^"^ having a.^
VI.

a factor.

Then

U|

as

the terms of

ais!(-if

M^/isli

are all the terms of \a\ having a^s as a factor

Thus

(3.15)

+
"11*11
since (-1)
,s+i = (-1)

+ ai5!(-lf||/,3|! + +

...

+ ai !(-!)'*" M,

+ ai2i2

+ ain*in

We have

(3.9) with

We

shall call (3.15) the expansion of

U| along

its first

ments. Let

m.r,JWlTr\T B be the
the first

umn over

s-l columns. Then

"' ''^ '^ ^^^^^ '^' ^^-^^ '' ' = '^ '^ '''^i"^'J by f^P^^ting the above arguJ'*' ^iT^ matrix obtained from A by moving its rth row over the first r-1 rows and then its .th col^

T~l
(-1)

s-l

(-!)

\a

(-1)

u
the minor of a^^ in ^^

the minor of a.rs in

The element standing in the ^ Thus,


.

first

row and the

first

column of 5

is

a and

the terms of

is yreciseiy i precisely

are all the terms of

having a^^ as a factor.

Then
r+fel

,l,-rkU-l)
and we have (3.9)
for
j

M.rk\

2
k=i

a^j^a. rk^rk

r.

28

DETERMINANT OF A SQUARE MATRIX

[CHAP.

10.

When

oLij

is the cofactor of aij in the ra-square matrix


'll

A =
"12

[a^j]

show

that

%,f-i
(h,j-i

"'1

%,j4-i
02
i +
^

^'i.n

*2n

(i)

fC^GC^-j

"T

fCi^OL^j

"T

itj^dj^j

'^i

^2

"W,j-i

<%,

'^n

In making these with k^ 0^7 with Atj, This relation follows from (3.10) by replacing a^j with k^, 027 ^2J appearing is affected since none contains an element (t^j replacements none of the cofactors OLij.QL^j.
, ,

from the

th

column of A
VII, the determinant in {i) is
(i) is
I

By Theorem

when A^= a^^,


+ fea^g,
\7
(r

(r

= 1,2 n

and
s

^
/).

j).

By Theorems Vin,
A
are replaced

and VII, the determinant in

when

Ay

= 1,2

and

Write the eauality similar to (0 obtained from (3.9)

when the elements

of the ith row of

by

/ci,/c2.

3 4

5 3
(e)

28

25

38 65
83

11. Evaluate:

(a)

04
-5
1 1

(c)

2
5

42 38
56 47

-2
2
((f)

-4

4 8

(b)

-2 1 5

-3 2 4

3-4 5-6 3 2-3 4

(a)

Expanding along the second column (see Theorem X)


1

3 2

Cl-^^fX^Q

"f"

022^22 ^ ^32 32

ai2 +

a22 +

(-5)o;g2

-5

34.2I -5(-l)-'

1 2
I

5(4-6)

-10

3 4l

(b) Subtracting

twice the second column from the third (see Theorem IX)
1

4 8

4
1

-2 15
-3 2 4

2
3

8-2-4 5-2-1 4-2-2


-42

4
1

-2 -3

4
2

1 3

3(-l)''

-3

-3(14)

(c) Subtracting three times the


3 4
1

second row from the

first

and adding twice the second row

to the third

5 3

3-3(1) 4-3(2)
1

5-3(3)
3 =
1

-2 -4
2
9 3

-2 -4
9
2

-2 5

-4

-2 +

2(1)

+ 2(2)

-4+

2(3)

(-4 + 36)

-32
in (c)

(d) Subtracting the first

column from the second and then proceeding as


2
1

3-4 5-6 3 2-3 4

-4
3

5-11
4

-2 -3
1

-4 + 4(1) 2-2(1) 1 5-2(-ll) -11 3 + 4(-ll) 4-2(-2) -2 -3 + 4(-2)

27
8

-11 -41 -2 - 11

27

-41 -31 -11

CHAP.

3]

DETERMINANT OF A SQUARE MATRIX

29

(e)

Factoring 14 from the

first

column, then using TheoremIX to reduce the elements in the remaining columns
2

28

25

38
14

25

38
65
14

2
3

42 38 65
56

38

25-12(2) 38-20(2) 38-12(3) 65-20(3) 47-12(4) 83-20(4)

47 83

47 83

2
4 3

-2
5

14

-12
6

9
1

-14

-1
6

4-13
12.

-14(-l-54)
1

770

-1

Show

that p and q, given by (3.13) and (3.14), are either both even or both odd.
is

Since each row (column) index


P + q
=
(1

found in either p or 9 but never in both,


(l

+ 2+-+7J) +

2+-"+n)

2-kn{n + l)

n(n +1)
or both odd.

Now
(-1)

p+q

= (-1)^

even (either n or n + 1 is even); hence, p and q are either both even and only one need be computed.
is

Thus,

12
6
13.

3 8

4
9

For the matrix

[..]

11

12 17 22

13

14 19

10 15

the algebraic

complement

of

Ao's

is

16
.21

18

23

24

20 25

,^2+3+2+4| .1,3,51 , (-1) Ml,4,5l

13
16
21 18

20 25

(see Problem 12)

23

and the algebraic complement of

'4^'4^'5|

is

I12 141

SUPPLEMENTARY PROBLEMS
14.

Show

that the permutation 12534 of the integers

1, 2, 3. 4,

is even,

24135

is odd,

41532

is even,

odd, and 52314 is even.

53142

is

15.

List the complete set of permutations of

1,

2,3,4, taken together; show that half are even and half are odd.

16.

Let the elements of the diagonal of a 5-square matrix A be a.b.cd.e. diagonal, upper triangular, or lower triangular then \a\ = abcde.

Show, using

(3.6), that

when ^

is

17.

Given

-4

[j J] each product is 4.

and B = [^
6J

^^^^

^^^^

AB^BA^ A'b

4 AB' ^

a'b'

^ B'a'

but that the determinant of

18.

Evaluate, as in Problem
2
(a)

1,

-1

2-2
3 =

2 3

2 4
3

27

(b)

12
2 3

(c)

-1

-2 -3 -4

30

DETERMINANT OF A SQUARE MATRIX

[CHAP.

1 2

10
9

19.

(a)

Evaluate

\A

2 3

4 5 11
(b)

Denote by B Evaluate \B
| \

the determinant obtained from


to verify

-4

by multiplying the elements of

its

second column by

5.

Theorem

III.

(c)

Denote by
I

the determinant obtained from

.4

by interchanging

its first

and third columns.

Evaluate

to verify

Theorem V.
1

2 7

2 3

(d)

Show

that

2 3 5

2 3 4 4 5 3

thus verifying Theorem VIII.

4 5 8

7
3

(e)

Obtain from \A

the determinant

|o| =

2 3

by subtracting three times the elements of the

first

5-1
Evaluate
j

column from the corresponding elements of the third column.


(/) In

to verify

Theorem

IX.

subtract twice the first row from the second and four times the first row from the third. the resulting determinant.
I

U
I

Evaluate

(g) In

/I
I

multiply the first column by three and from

it

subtract the third column.


(g).

Evaluate to show that

\A
I

has been tripled.

Compare with

(e).

Do

not confuse (e) and

20.

If -4 is

an n-square matrix and 4

is a scalar,

use (3.6) to show that

U^

/t^M

|.

21.

Prove: (a)

If

U
A

= k.

then

\a\ = k =

\a'\.
|

(b) If

is skew-Hermitian, then

-4

is either real or is a pure imaginary

number.

22.

(a)

Count the number of interchanges and thus prove the theorem.


Same,
for

of adjacent

rows (columns) necessary to obtain 6 from A

in

Theorem V

(b)

Theorem

VI.

23.

Prove Theorem VII. Hint: Interchange the identical rows and use Theorem V.
Prove:
If

24.

any two rows (columns) of a square matrix A are proportional, then


VIII, III,

,4

o.

25.

Use Theorems

and VII to prove Theorem IX.

26.

Evaluate the determinants of Problem 18 as

in

Problem

11.

a b
27.

Use

(3.6) to evaluate

\A\

d
e
;

then check that \a\ =

".P/.

Thus,

if

= diag(A-^. A^).

where

g h
A^,

A^ are 2-square matrices,

U4i|

-1/3
28.

Show

that the cofactor of each element of

2/3 2/3

-2/3 -2/3 1/3 -2/3 -2/3


-4

is that element.

1/3

-3 -3
1

29.

Show

that the cofactor of an element of any row of

is the

corresponding element of the same

numbered column.
30.

Prove: (a)

If

A A

is is

symmetric then

<Xij=

0^j^

when

j
Ot--^

(b) If

n-square and skew-symmetric then

aij = (-1)

"P when

CHAP.

3]

DETERMINANT OF A SQUARE MATRIX

31

31.

For the matrix A of Problem


(a)

show

that

(b)

form the matrix

12
_'^13

(^^22

32
Otgg

and show that

AC = [.
known.

^23

(c) explain

why

the result in (6) is

known as soon as
a
a^

(a) is

be
32.

Multiply the columns of

6^
c^
be

ca
c2

52

respectively by a,b.c

remove the common factor from each of

ab

ab
ca be

ca
be

the rows to

show

that

ab ca

ab

a^
33.

a
6 e

1 1 I 1

bed

a-^

a^ a

Without evaluating show that

6^

aed abd
abc
(a

e^

b)(a

c^

c)(a

d)(b

c)(i

c'^

d)(e

- d).

1 1

d^ d

d d^ d

1 1

1 ... 1

1 1
1

1 1

... ... ...

1 1

1 1 1

1...1
1

34.

Show

that the ra-square determinant

a\

0...1

("-!)
1
1 1 1

n-1
(-1)

(n~l).

1 1

1 1

...
...

1
1 1

...0

re-i

n-2
a^
1

ra-i

n-2
ar, 1

35.

Prove:

S(i

2)(i

as).

(ai-a^H(a2- 03X02-04)...

(02

-a^j

7i-i

- "ni

n-i

ra-2

na^+b-^
36.

na^+b^
nb^ + c^

na^+bg nb^+Cg
ncQ + a^
(n

O]^

02
62

O3 is
Cg

Without expanding, show that

nb-j^+e^
nc-i^+a.^

+ l)(rf--n +

1)

61 Ci

nCQ+a^

Cg

X
37. Without expanding,

xb x-c
has
as a root.

show

that the equation

x-i-a

x+b +6
a
38.

x+c

a
a

a+ b

Prove

,n-i
b

{na + b).

+6

chapter 4
Evaluation of Determinants

PROCEDURES FOR EVALUATING

determinants of orders two and three are found in Chapters. In Problem 11 of that chapter, two uses of Theorem IX were illustrated: (a) to obtain an element 1 or 1 if the given determinant contains no such element, (b) to replace an element of a given

determinant with

0.

For determinants of higher orders, the general procedure is to replace, by repeated use of Theorem IX, Chapters, the given determinant U! by another |b| -= \bij\ having the property that all elements, except one, in some row (column) are zero. If &>, is this non-zero element
'Pq

and ^p^

is its cofactor.

bpq

dp^

i-ir

^Pq

minor of

b.

'pq

Then

the minor of bp

is treated in similar

fashion and the process is continued until a determi-

nant of order two or three is obtained.

Example

1.

3-2 4 3-2 12
2 3 2

2 + 2(3)

+ 2(-2)

-2

+ 2(1)
1

4 + 2(2) 2

8-10
3-2
-6
8
1

8 2

-2
2-3(-2)
4

3 4

3-3(3)

3-3(1)

4-3(2)
5

0-2

-2405
=

-2

-2405
8

8-1
(-1)"
-6 -2
8

+ 8(-l) -1
8(8) 8(4) 8 4

+ 8(-l)

0-10
58
8

-2
5

-6 +
-2 +
'^
37

-2 + 8(8)
5

62
37

+ 8(4)

30

-(-l)- = (-l)P'

-286
See Problems 1-3

30

may be used:

For determinants having elements of the type in Example 2 below, the following variation divide the first row by one of its non-zero elements and proceed to obtain zero
or

elements in a row
Example
0.921
2.

column.

0.185

0.476
0.527 0.637
0.841

0.614
0.138
0.921

0.201

0.517

0.667
0.138
0.921

0.201

0.517

0.667

0.782

0.157 0.484
0.555

0.782

0.157
0.484 0.555

0.527
0.637
0.841

0.123 -0.384

0.872
0.312

0.799
0.448

0.872
0.312

0.799 0.448

0.309
0.492

0.196

0.217
0.240
1

0.680

0.123 -0.384
0.921

-0.320
0.921(-0.384) 0.309
0.492
1

0.309
0.492

0.196

0.217

0.196

0.217

0.680

0.240

0.680

0.240

0.309

0.265
0.757

0.921(-0.384)

0.309
0.492

0.265 0.757
=

0.217 0.240

0.921(-0.384)

0.492

0.921(-0.384)(0.104)

-0.037

32

CHAP.

4]

EVALUATION OF DETERMINANTS

33

THE LAPLACE EXPANSION.


bered
I'l,

The expansion

of a determinant

of order n along a row (column) Is

a special case of the Laplace expansion.


J2

i^

Instead of selecting one row of \a\, let when arranged in order of magnitude, be selected. Prom these

m rows numm rows

n(n-

l)...(ra

1-2

....

m + l) m

minors

can be formed by making all possible selections of m columns from the n columns. minors and their algebraic complements, we have the Laplace expansion
>

Using these

Jrr.

(4.1)

|(-1)^

Jn+i' jm+2
.

Jn

i.

i.

^m4-i'% + 2

^n

where s = i^ + i^+... + i^ +A + /2 + column indices taken m at a time.


Example
3.

+h

and the summation extends over the/? selections of the

Evaluate

3-2 4 2 3-2 12
,

using minors of the

first

two rows.

3 4

-2405
Prom
(4.1),

1+2+1+21
(-1)

1 + 2+1 + SI

U1.2
+

U-5aI ^3,4
1

(-1)"

,1,31

2,41

"Mi',|-U 1,2! 3,4!


1 + 2 + 2 + 31
.2,31

_l + 2 + l + 4|

1,4

(-1)

+
,,1+2 + 2+4. .2,4 .13 , (-1) Mi'jl-Us'.

(-1)

^1,:

(-ly

1 + 2+3+4

.34
*l,2r 1^3, 4I

2 3

4
5

2 3

-2
1

2 4

-2
+
3

+
5 3 4 2

2 3

4 2 3
3

-2

4
5

-2

_2

_2

-2

+
D

-2
1

-2 4

(-13)(15)

- (8)(-6) + (-8)(-12) + (-1)(23) -

(14)(6)

+ (-8)(16)

-286
See Problems 4-6

DETERMINANT OF A PRODUCT.
(4.2)

If

A and B

are n-square matrices, then

Us

Ul-lsl
See Problem 7

EXPANSION ALONG THE FIRST ROW AND COLUMN.


(4.3)

If

^ = [aid
n
n

is

n-square, then
ii

^11 ^1
i=2
j,-=2

IJ

IJ

Where

a^

is the

cofactor of

o^ and

a^^-is the

algebraic complement of the minor

"ii^iil
ii Otj
I

of^.

DERIVATIVE OF A DETERMINANT. Let


tiable functions of a variable x.

the ra-square matrix

A =

[a^,-] ^tj.

have as elements differen-

Then

34

EVALUATION OF DETERMINANTS

[CHAP. 4

\A\, of \A\ with respect to x is the sum of n determinants obtained dx by replacing in all possible ways the elements of one row (column) of by their derivatives with respect to x.
I.

The

derivative,

Example

4.

x^

x^\

3
x""

2x =
1

x2 x +
x^

x+ +
1

3
x-"

dx

2x-\
X

2x-\
X

3^^

2x-i

-2
5

-2

X 6x

-2

4x

12x^

See Problem 8

SOLVED PROBLEMS
2 3

7
1.

4 2 4

-2 4 -3 10
3

-2
3

2 3 3

3-2
-2
2
1

4
2

2(2)
3

4-2(3) -3-2(-2) 10-2(4)


2

4
5

-286
4
5 3

4
5

(See Example 1)

-2

-2

There are, of course, many other ways of obtaining an element +1 or -1; for example, subtract the column from the second, the fourth column from the second, the first row from the second, etc.

first

-1
3 2 2 5

-1 +
3

-2
1

2+2
2 5

2-2(1) -2-2(2)
1-2(2)

10
2 2 3 3

2
3

4
1

2
3 3

4
1

+2

4-6 4-3
-6-2(-3)
-5
4

-3

+ 3 -3-2(3)

18-9
-4

-6
=

3-2(4)
4

4-2(4)
8-3(4)

4
1

4-3
8

4-3

4-3

-9 -4
=

1-3(4)
-72

-9-3(-3)

-11 -4

-5
3

-11 -4

1+

2j

3.

Evaluate

\A\

2-3i
2 +
: i

1-

2i

Multiply the second row by

and the third row by


1

2J;

then
+j

+i

1+2j

1+2;

+J
14j

2J 5j

(l+J)(l+2j)Ml

(-1+3j)|-^|

2
5

5-J
-4 + 7i
1+i
l

2
1

5-J
-4 + 7j -10 +
+
18
2J
1

8-

25 -

-4 + 7J -10 + 2i

2j
I

-6

and
,4

14i

25 -

5j

CHAP.

4]

EVALUATION OF DETERMINANTS

35

4.

Derive the Laplace expansion of

\A\

\aij\

of order n, using minors of order

m<n
in

ji'h

Jrn

Consider the m-square minor


order of magnitude.
into the first

A;
H-''^
1

%\

of 1^1 in

which the row and column indices are arranged


,
|

row
after

by

interchanges of adjacent rows of |^ the row numbered ii can be brought row. by i^- 2 interchanges of adjacent rows the tow numbered is can be brought into the second -m interchanges of adjacent rows the row numbered % can be brought into the mth row. Thus.
i;

Now

by

(I'l- 1)
I'l,

+ (igi^

2)

numbered

+ (ijj -m) 11 + ^2+ occupy the position of the

first

+ 'm-2'"('"+l) interchanges of adjacent rows the rows m rows. Similarly, after /i + j^ + + /^ - \m(m+l)

interchanges of adjacent columns, the columns numbered /i,/2 occupy the position of the first m col/jj umns. As a result of the interchanges of adjacent rows and adjacent columns, the minor selected above occupies the upper left corner and its complement occupies the lower right comer of the determinant; moreover.
I

A has changed sign


I

cr

[1

+ ^2+ +

in

/i

= j- + i + + + /i + + ii + + in changes. Thus
Jm+i'Jn-i-2
'm-H'''m +2-

/2

+ /-"('" +1)

times which

is

equivalent to

Ji.J2'

Jm
'-m

Jn yields
' Jn
m!(ra

AH'''^

-m)! terms of (-1) \a\

or

Ji~h(a)

Jn

Jn-H' Jm+2'
"TOJ-l'

(-ir
n+2'

yields

w!(n-

m)! terms of \a\.

n(n~l)...(n-m + l) different m- square l'2....m m\{n m)\ minors may be selected. Each of these minors when multiplied by its algebraic complement yields m!(/j-m)' terms of U|. Since, by their formation, there are no duplicate terms of U| among these products.

Let

I'l,

12

in be held fixed.

Prom these rows

S(-i/
where
/i, 72

Jn
I'm

Jm-1' Jvn

Jn

I
,

i^

+ i^+

+ i^ +j^ + j^ +

+ in

and the summation extends over the

different selections

in of the column indices.

5.

Evaluate

12 3 4 12 1 11 3 4 12
2

using minors of the first two columns.

(-1)^

21
1
I

|1
ll

+
2 2

(-ir

21 41

2
1

ll

+
3 ll

(-If

2
3

11

13
ll

4
1

41

(-3)(1)

(-2)(1)

(5)(-l)

6. li

A,B, and C are re-square matrices, prove

C B
Prom the
plement
is
first n

B
Its

|s|.

rows of |P| only one non-zero n-square minor, U|, can be formed. Hence, by the Laplace expansion, |p| = |.4|.|b|.

algebraic com-

7.

Prove

\AB\ =
A

| |

S
and

Suppose Problem 6

= [a^j]

= [bij]

are n -square.

Let

= [c^j] =

AB

so that

^Hkhj-

i^rom

36

EVALUATION OF DETERMINANTS

[CHAP. 4

ail

012

"m
2n

021

022

"ni

n2

"nn
6ii

-1 -1

612

..

hi
?,2

621

^22

..

-1

To

the (n+l)st column of \P\ add fen times the first column, 621 times the second column

6i times

the nth column;

we have

On
"21

012

"m
2ra

Cii

"22

C21

"ni

"n2

"nn

Cm
612

-1 -1

h,
62

622

..

Next, to the

(n

+ 2)nd column of |P| add

fei2

times the

first

column, 622 times the second column,

times the nth column.

We have
in 2n
Cii

C12

C21

C22

"nn

Cm

Cn2
^13

^m
b^n

*23

Continuing this process, we obtain finally

A
\P\

C
.

Prom

the last n rows of

only one non-

zero n-square minor, 1-/| = (-l)'^ can be formed.


= (-lf'2"^^'|c|.

Its

algebraic complement is (_i)i+24--"+ra+(n+ii+-"+2n|(-|


=
\c\

Hence,

\p\

(-i)Vlf" ^"^"lc|

and

\c\

\ab\ = U1.|b|.

Oil
8.

%2 %S
where
Oif

Let A =

021 '%2 '^a

aij(x),

(i, j

= 1,2,3),

are differentiable functions of x.

Then

031 032 033

%l"22*33 +
and, denoting -ra;:

122Sa31

%s"S2"21

"

(^ix"^"^ "

0:l202lOs3

"

(hs^^O'Sl

by

a,',-,

dx

^^

^J

CHAP.

4]

EVALUATION OF DETERMINANTS

37

dx

'^ll"22''S3

+ 22ll"33 + ''33"ll"22 + O32OISO2I


f

+ "l2"23"31 + 023012131

+ O31O12O23

+ %3032021
f

021013032
f

Oj^jCggOgg
f

>i23'^n''S2

OgjOj^jOgg

a^OQiOg^

0210-12033

Og^a^a^^

air^a^^a^.^

02205^303^

Ogj^o^gOjg

Oil

0^11

o-L20^i2

"*"

OisOi^g + a^-^dj^x

022(^22

+ 023(^23

"^

Og-LCXgi

+ 032(^32 + 033(^33

oil 012 Ol3


021 031

11

O12 Ol3
O22
O32

'11

"12

"13

022

023

21

O23

i2i

"22 "23

032 033

31

O33

'31

"32 "33

by Problem 10, Chapter

3.

SUPPLEMENTARY PROBLEMS
9.

Evaluate:
3

2
(o)

11

-2 -1
3

-2000 113 4 1116


2 4

156

ic)

2 3
1

-4 4 -3 -4 -5
3
5 3
1

-304

-4 -2
-1
1

-2 -2
3
1

{b)

4
2

16 12 9
2 7

2
1

41

(d)

1
1

118

-4 -3 -2 -5 -2 2 2 -2

10. If

Is n-square,

show

that \A^A

is real

and non-negative.

11.

Evaluate the determinant of Problem 9(o) using minors from the first two columns.

first

two rows; also using minors from the

12. (o)

Let

aJ"'
4B| Use |^B|
=

"^

and B =

''

'^
bjj
(< + 02)(6i+62) (oi

[^-02 ojj
|.4|"|B| i.4|-|B| to

1^-62

show

that

0000
61 + 163

(0161-0262)

+ (0261+0163)

(6)

Let

+ t03

a2+iaA
I

tOj^

and

62 + 164

-Og + ja^

Oi-iogJ
inii. \ts\
to

Use \AB

express (01 + 02+03 + 04X61+62+63+64) as a sum of four squares.

222222,2,2

[-62 + 864

6i-j6gJ

2
13.

1
1

Evaluate
4
5 6 5 4

3
3

2
2 2
2

using minors from the

first three

rows.

Ans. -720

1 1
1

3
3

38

EVALUATION OF DETERMINANTS

[CHAP. 4

112 12
14.

Evaluate

111 110 112


12
2

using minors from the

first

two columns.

.4ns. 2

11
square matrices, use the Laplace expansion to prove
|diag(4i,.42
As)\
=

15. If

^1,^2

^^s

^"^^

Uil-U,!

....

a^ a^ Og a^
*1
16.

Expand
a-^

*2 ^3 *4

using minors of the


a^
^2 a^
flg

first

two rows and show that

a^
*4

*1

^3

a2 62 60
6..

K
17.

62

63

Use the Laplace expansion


A

to

show

that the n-square determinant

A B C

where

is fe-square, is

zero when

> 2"\A\
to = aiiOiii + ai2ai2 + OisO^is

18. In

+ ai4i4.

expand each of the cofactors a^g.


4
4
.^

OL^a. tti*

along its

first col-

umn

show
^11^11

ti

.-^

1=2 J=2

11 "IJ l^lj ^il^lj^lj

where

ffi,-

is the

algebraic complement of the minor


"ii "ij

Of

U
show
that the bordered determinant

19. If a^j

denotes the cofactor of a^j in the n-square matrix A =


"11 "21

[a^,-],

"12

ire

Pi

?i

92
'^12

?n

22

"2ra

P2

Pi "11

"

"m
Pilj .^ 'J t=i j=i

Cti

^V

"ni n2
li

"nra Pre

Pn "ni

"712

"nn

92

In

Hint.

Use

(4 .3).

20.

For each of the determinants

|.4|

find the derivative.

X
()
(b)

I
2*:

2
1
;<:

a:

-1
4
a;

x-l
3
a:

1
2a:

x^

(c)

+5

2x

Zx +

3-2
-4ns.

x^+l
12a;^

x+l
15a:*,

x^

(a)

2a:

+ 9a:^-

8a;=^
,

(6)

6a:

+ 21a:^ +

(c) 6a:^

5*"^

- 28x^ +
=

9a:^

20a;

21.

Prove

If

A and B are

real n-square matrices with

A non-singular and

if

ff

4 + iS

is Hermitian, then

chapter 5
Equivalence

THE RANK OF A MATRIX. A


said to have rank
0.

non-zero matrix A is said to have rank


if

r if

at least one of its r-square

minors is different from zero while every (r+l)-square minor,


3"
1

any, is zero.

zero matrix is

'l

2 3 5

2 3

Example

1.

The rank

of

2 3

4
7

is

r=

since
2

-1^0

while

= 0.

See Problem

1.

An
A
is

re-square matrix

called singular.

A is called non-singular if its rank r=n, The matrix of Example 1 is singular.

that is,

if

A ^

0.

Otherwise,

Prom
I.

AB\

A\-\B\ follows
of

The product

two

or

more non-singular re-square matrices

is non-singular;

the prod-

uct of two or more re-square matrices is singular if at least one of the matrices is singular.

ELEMENTARY TRANSFORMATIONS.
on a matrix do not change either
(1)

The following operations, called elementary transformations,


its

order or its rank:

The interchange The interchange

of the ith and /th rows, denoted


of the ith

by Hij;

and /th columns, denoted by K^j

(2)

The multiplication

of every element of the ith row

by a non-zero scalar

k,

denoted by H^(k);

The
(3)

multiplication of every element of the ith column by a non-zero scalar k, denoted by Ki(k).
to the

The addition The addition

of the /th row,

elements of the sth row of denoted by Hij(k)


;

k,

a scalar, times the corresponding elements

to the elements of the ith column of ments of the /th column, denoted by K^j(k)

k,

a scalar, times the corresponding ele-

The transformations H are called elementary row transfonnations;


called elementary column transformations.

the transformations

are

The elementary transformations, being precisely those performed on


determinant, need no elaboration.
order of a matrix.
rank.
In
It

is clear that

Problem

2, it is

the rows (columns) of a an elementary transformation cannot alter the shown that an elementary transformation does not alter its

THE INVERSE OF AN ELEMENTARY TRANSFORMATION. The


tion is an operation

inverse of an elementary transformawhich undoes the effect of the elementary transformation; that is, after A has been subjected to one of the elementary transformations and then the resulting matrix has been subjected to the inverse of that elementary transformation, the final result is the matrix A.

39

40

EQUIVALENCE

[CHAP.

Example

2.

Let A

4 5 6

7 8 9
"l

3"

The The

effect of the elementary row transformation

H2i(-2)

is to

produce B

2
.7

10
8 9

effect of the elementary row transformation ff2i(+ 2) on B is to produce A again Thus, ff2i(-2) and H2x(+2) are inverse elementary row transformations.

The inverse elementary transformations


"ij
(2')

are

-1

^ij

Hi\k) = H^(i/k)
H--(k)

(3')

= H^A-k)

K^jik)

= Kij(-k)

We have
II.

The inverse

of an elementary transformation is an elementary transformation of the

same

type.

EQUIVALENT MATRICES. Two

matrices A and B are called equivalent, A'^B, from the other by a sequence of elementary transformations.

if

one can be obtained

Equivalent matrices have the same order and the same rank.
Examples. Applying
in turn the elementary transformations
1

-1
3

4 5

12-14
5-3
-1 -2
6

W2i(-2), ^siCD. Ws2(-1).


1

-1
5 5

12-1
'^

2 -1

-2

-7

-7

-3 -3

5-3
S

Since all 3-square minors of B are zero while


the rank of

I
I

t^

0,

the rank of

is 2

hence,

^ is 2. This procedure of obtaining from A an eauivalent matrix B from which the rank is evident by inspection is to be compared with that of computing the various minors of -4.
See Problem
3.

ROW EQUIVALENCE.
lone,

If

is said to

a matrix A is reduced to B by the use of elementary row transformations abe row equivalent to A and conversely. The matrices A and B of Example 3

are row equivalent.

Any non-zero
(a)

matrix

of rank

r is

row equivalent to a canonical matrix C in which


first r

one or more elements of each of the


only zero elements.

rows are non-zero while

all other

rows have

(b) in the ith row, (i

this
(C) ]\

=1,2, ...,r), the first non-zero element element stands be numbered 't
;'.

is 1;

let the

column

in

which

<

/2

<

<

j^.

(d) the only

non-zero element in the column numbered

j^, (i

=1,2

r),

is the

element

of

the ith row.

CHAP.

5]

EQUIVALENCE

41

To reduce A
(ii) (is) If

to C,
7^

suppose

/i is

the
to

number of the
it

first

non-zero column of A.

aij 171

0,

use
but

//i(l/oi,iji

reduce
ffij,

to

1,

when necessary.
(i^).

If

a; J

Vi

o^ ^^7

0,

use

and proceed as in

(ii)

Use row transformations

of type (3) with appropriate multiples of the first row to obtain

zeroes elsewhere in the /^st column.


If

non-zero elements of the resulting matrix B occur only in the


72 is the

first row,

B =

C.

Other-

wise, suppose

number of the
if

first

column
bqj^

in

use ^2(1/^2^2) as in
in
(il),

(ii);

&2J2=

but

0,

which this does not occur. If &2j ^ 0, use H^^ and proceed as in (ii). Then, as

clear the /gnd column of all other non-zero elements.

If

non-zero elements of the resulting matrix occur only in the

first

two rows, we have C.

Otherwise, the procedure is repeated until

is reached.
ff2i(-2), ffgiCD

Example

4.

The sequence of row transformations to A of Example 3 yields


1

2(l/5)

//i2(l). //ssC-S)

applied

2 4

-1
3

4
5
^\j

-1
5

1
'\^

-1
1

1
'%^

2
1

17/5

2
1

-3

-2

-7

-3

-3/5 -3

-3/5

having the properties

(a)-(rf).

See Problem

4.

THE NORMAL FORM OF A MATRIX. By means


r

of elementary transformations any matrix A of rank

>

can be reduced

to

one of the forms


/.

(5.1)

\l%

"'"'
own normal

M
form. of the first row
first
1

called its normal form.

zero matrix is its

Since both row and column transformations may be used here, the element
obtained in the section above can be moved into the
first

first

column.

Then both the


1

row and

column can be cleared of other non-zero elements. row can be brought into the second column, and so on.
For example, the sequence
/^32(-l), ^42(3)
ff2i(-2),

Similarly, the element

of the

second

^31(1).

^2i(-2),
I

Ksi(l),

X4i(-4).

K23, K^{\/%),

applied to 4 of Example 3 yields

the normal form.

See Problem

5.

ELEMENTARY MATRICES.

The matrix which results when an elementary row (column) transforma-

row (column) matrix. Here, an elementary matrix will be denoted by the symbol introduced to denote the elementary transformation which produces the matrix.
0' 1

tion is applied to the identity matrix /^ is called an elementary

Example

5.

Examples

of elementary matrices obtained from

/g

1 1_

"0
1
1

0'

'1

0"

'l

0' 1

= K,
1_

Ha(k)

K-sik).

H^g(k)

k
1_

K^^yk)

_0

k_

42

EQUIVALENCE

[CHAP.

Every elementary matrix

is non-singular.

(Why?)

The

effect of applying an elementary transformation to an

mxn

matrix

A can be produced by

multiplying

A by an elementary

matrix.

To
to
Ijn

effect a given elementary row transformation on

of order

to form the corresponding elementary matrix

and multiply A on the

mxn, apply the transformation left by H.


/ to

To

effect a given elementary

form the corresponding elementary matrix


~1

column transformation on A, apply the transformation to K and multiply A on the right by K.


"O

3~

f
1

'l

3~

"7

9"

Example

6.

When A

4 5 6
_7

H^^-A
3'

=
1

4 5 6
0_

4 5 6
_1

interchanges the

first

and third

9_

7 8 9_

3_

1
rows of A
;

"l

o"

"723'
=

4/^13(2) =

4 5 6

10
_2
1_

16 5 6
_25

adds to t( the

first

column of A two times

J
the third column.

8 9

39J

LET

AND B BE EQUIVALENT MATRICES. Let the elementary row and column matrices corresponding to the elementary row and column transformations which reduce /I to 5 be designated as //i./Zg where //^ is the first row transformation, //g is the second, ...; ^s< J^\,T^Q. K^ is the first column transformation, K^ is the second Then

'^t

(5.2)

//.

IU-H,.A K-i-K^
H^-H^

K,

PAQ

where
(5.3)

Ih

and

We have
III.

Two

matrices A and B are equivalent

if

and only

if

there exist non-singular matrices

P and Q
Example

defined in (5.3) such that


"1

PAQ

= B.

-1

2~|

7.

When

2
_1

5-23,
2
1

^3i(-l) //2i(-2) -^ ^2i(-2) -Ksid) .K4i(-2) -K^sd) .Ks(i)

2J

["100"! r
1

1-200
1

~1

0~

2"

"1000"

o"j
1

0-2
ij

10 10
1

10 10
_0
1

10
1

10 1 10
1_ _0 1_

"1000" 10
5
_0
1_

[j^i

iJ

1-25-4
10
o"l 1

[:;=}
Since any matrix
is

=
1

PAQ

[1

10
1

oj

equivalent to

its

normal form, we have

IV. If ^ is an re-square non-singular matrix, there exist non -singular matrices as defined in (5.3) such that PAQ = 1^
.

P and Q

See Problem

6.

CHAP.

5]

EQUIVALENCE

43

INVERSE OF A PRODUCT OF ELEMENTARY MATRICES.


P
as in (5.3). Since each

Let

H^...H^-H^

and

K-^-K^-.-Kt

and

has an inverse and since the inverse of a product is the product

in reverse order of the inverses of the factors

(5.4)

P~^

H;\hI\..H^^

and
let

Q''

Kt...Kt-Ktthat

Let A be an re-square non-singular matrix and


=
/
.

P and Q defined above be such


=

PAQ

Then A
=

(5.5)

P'\PAQ)Q^

P'^-k-Q^

P'^-Q'^

We have proved
V.

Every non-singular matrix can be expressed as a product

of

elementary matrices.

See Problem

7.

From

this follow
If If

VI.
VII.

is non-singular, the

rank of

AB

(also of

BA)

is that of B.

P and Q

are non-singular, the rank of

PAQ

is that of A.

CANONICAL SETS UNDER EQUIVALENCE.


VIII.

In

Problem

8,

we prove
if

Two mxn

matrices A and B are equivalent

and only

if

they have the same rank.

A set of my.n matrices is called a canonical set under equivalence if every mx-n matrix is equivalent to one and only one matrix of the set. Such a canonical set is given by (5.1) as r ranges over the values 1,2 m or 1,2. ...,re whichever is the smaller.
See Problem
9.

RANK OF A PRODUCT.
matrices

Let A be an mxp matrix of rank


that

r.

By Theorem

III there exist

non-singular

P and Q such

PAQ
Then 4 = P
(5.6)

P''

NQ

Let S be a

pxre matrix

and consider the rank of


=

AB
VI, the rank of

P~'NQ'^B

AB is that of NQ'^B. Now the rows of NQ~'b consist of the firstr m-r rows of zeroes. Hence, the rank of AB cannot exceed r the rank of A Similarly, the rank of AB cannot exceed that of S. We have proved
rows of

By Theorem

Q B and
IX.

The rank

of the product of two matrices cannot exceed the rank of either factor.

suppose iS = then from (5.6). NQ-'b = 0. This requires that the first r rows of Q'^B be zeroes while the remaining rows may be arbitrary. Thus, the rank of Q-'b and, hence the rank of B cannot exceed p-r. We have proved
:

X. If the mxp matrix A rank of B cannot exceed p-r.

is

of rank r

and

if

the pxn matrix

is

such that

AB

the

44

EQUIVALENCE

[CHAP.

SOLVED PROBLEMS
1.

(a)

The rank

of

-4
1

2 3] sj 2 3 5

is 2

since

-4

and there are no minors of order three.

(b)

The rank

of

12
"0

is 2

since

and

^0.

2 2 4 8 2
3"

(c)

The rank

of

A
_0

4 6
6 9_

is 1

since

0,

each of the nine 2-square minors

is 0, but nov

every element

is

Show

that the elementary transformations do not alter the rank of a matrix.

shall consider only row transformations here and leave consideration of the column transformations Let the rank of the mxn matrix ,4 be r so that every (r+l)-square minor of A, it any, is zero. Let B be the matrix obtained from .4 by a row transformation. Denote by \R\ any (r+l)-square minor of A and by Is] the (r+l)-squaie minor of B having the same position as \R\

We

as an exercise.

Let the row transformation be H^j


two of
in the
its rows, or (lii) to

Its effect on |/?| is either (i) to leave interchange one of its rows with a row not of \R\
. ;

it
.

unchanged,
In the

(ii) to
(i),

interchange
= \r\

case

\S\

=0;

case

(ii),

\S\

= -\r\ =

in the

case

(iii),

\s\ is,

except possibly for sign, another (r+l)-square minor

of l^l and, hence, is 0.

ply one of its rows by

Let the row transformation be Hi(k). Its effect on \R\ is either (1) to leave A:. Then, respectively, |S| = |/?| = o or |S| = ;i:|/?| = o.

it

unchanged

or (ii) to multi-

Let the row transformation be Hij(k). Its effect on |/?| is either (i) to leave it unchanged, (ii) to increase one of its rows by k times another of its rows, or (iii) to increase one of its rows by k times a row not of S|. In the cases (i) and (ii), |S|=|ft| = 0; in the case (iii), \s\ = = /?| + A: (another (r+i)-square minor of /I)
| |

k-0

0.

Thus, an elementary row transformation cannot raise the rank of a matrix.


lower the rank
tor,
if it

On
it.

did, the inverse transformation

would have to raise

the other hand, it cannot Hence, an elementary row

transformation does not alter the rank of a matrix.

For each of the matrices A obtain an equivalent matrix B and from


rank of A.
"1

it,

by inspection, determine the

2
1

3"
1
'-^-/

3
"^

"1

2
1

3~
1
-""J

'1

2
1

3"
1

(a)

2
_3

3
1_

-3 -3

-4 -8

_0

2_

_0

1_

The transformations used were


"1

//2i(-2). ffsi(-3);
"1

H^(-l/3), HQ(-l/i); Hg^i-l).


2
3
0"

The rank

is 3.

2 3

0"

1
''V.

0"

0'

(b)

2 4 3 2
3
_6

-3
4

2
3
5_

''\j

-4

-8

-8 -3

'~^

-8 -3

3 2 2

r^

-4

-8 3

S.
-3
2

2
5_

The rank

is 3.

8 7 5_

-4 4 -11 i
+
'~^

p -4 -11
1

-3

1+i
(c)

1
i

A
1

2j
_1

2j
2J_

'-Vy

2J

B.

The rank

is 2.

2j

l+i_

Note.

The equivalent matrices 5 obtained here are not unique. In particular, since in (o) and (i) only row transformations were used, the reader may obtain others by using only column transformations. When the elements are rational numbers, there generally is no gain in mixing row and column
transformations.

CHAP.

5]

EQUIVALENCE

45

4.

Obtain the canonical matrix

C row equivalent

to

each
2
'->-'

of the given matrices A.

(a)

A =

13 12 6
2 3 9

113
12
6 2 3 9

113
13
3
l"

13-2 13-2
-2_
1

10 4 13-2

113
1

13
3
3
--v^

GOOD
3"

2
3

(b)

2 4
.1

-2 -2 -3 -1

2
1

-2

-2 3
1

'10
^\y

7~

-1
1

'^

-1
1

6 4

2
1

2
1
4_

10 10

-1
2
2_

--O

10 1 10 0-1 10 2
1

6_

p -1

5_

pool

5.

Reduce each of
1

the following to normal form.

-1
1

1
^\y

-1
1

"l

o"|

fl

o"

'l

o'

'l
'V.

o'

(a)

3 2

4
3

2 5

-2

5 3

0-2

p-

1-2
Lo 2

1-2
11

10
P

10
p
1
0_

7 2

7 2 sj

7 3_

-7_

Us o]
The elementary transformations
are:

^2i(-3). 3i{2); K2i(-2), K4i(l); Kgg; Hg^-^y. ^32(2). /f42(~5); /fsd/ll), ^43(7)

(6)

[0234"! P 23 5 4^-0
4 8 13 I2J
|_4

354
2
i

1
'-^

3
2

5
3 2 3 4
'Xy

"1000"

"1000'
^\j

"1000"
<^

8 13 12

2 8 13

13 13

4
4.

10 p
1

10

0_

0_

The elementary transformations

are:

^12: Ki(2); H3i(-2); KsiC-S), X3i(-5), K4i(-4); KsCi); /fs2(-3), K42(-4); ftjgC- 1)

6.

Reduce A

1 2 3-2 2-2 1 3 3 04 1

to

normal form

A'

and compute the matrices

P^

and

(^^

such that

P-i_AQ^

A^.

Since A is 3x4,

we

shall work with the array


is

Each row transformation

is

performed on a row of

seven elements and each column transformation


1

performed on a column of seven elements.

-3

-2 -3
1

10
1

1 1 1

12
2-2
3

3
1

3-2
-5
7 7
1 1 1

10
1 1

-6
-6
2

-5

-6 -5 7 -2 -6 -5 7 -3

-5

-2

0-1-1

1/3 -3

1/3 -4/3 -1/3

-1/6

0-1/6 -5/6

10
1 1

10
1

7/6

1-57-210
0-1-11

10
1

or

Pi

0-210
0-1-1
1

46

EQUIVALENCE

[CHAP.

1
1

1/3 -4/3 -1/3

Thus,

Pi

-1/6 -5/6
-2
-1
1

-1

10
1

7/6

and

PiAQ-i =

10 10

N.

7.

Express A =

1
.1

4
3

3
4_

as a product of elementary matrices.

The elementary transformations


/

//2i(-l).
=

^siC-D; ^2i(-3). 'f3i(-3) reduce A

to

that is,

[see (5.2)]

H^-H^-A-K^-K^

ff3i(-l)-^2i(-l)-'4-X2i(-3)-?f3:i.(-3)

fl

0"1

fl

0"j

fl

3"|

fl 3

O'

Prom

(5.5),

Hl-H'^-K^-Kt

110
_0

010
[l

010
[p

010
_0
1_

ij

ij

ij

8.

Prove:
If

Two mxn

matrices A and B are equivalent


the

if

and only

if

they have the same rank.


to =

A and B have
Conversely,
VII,

other.

By Theorem

same rank, both are equivalent to the same matrix (5.1) and are equivalent ^ and B are equivalent, there exist non-singular matrices P and Q such that B A and B have the same rank.
if

each

PAQ.

9.

canonical set for non-zero matrices of order 3 is

[i:l-[:i:]
A
canonical set
tor

[!:][::]

non-zero

3x4

matrices is

Vh o]

[:=:]

nm
r^,

&:]
B consisting
n.

10. If from a

square matrix

of order n and rank

a submatrix
r^

of s rows (columns) of

is selected, the rank r^ of

is

equal to or greater than

+ s -

The normal form of A has n-rj^ rows whose elements whose elements are zeroes. Clearly
'A

are zeroes and the normal form of

6 has s-rg rows

from which follows

>

- n as required.

CHAP.

5]

EQUIVALENCE

47

SUPPLEMENTARY PROBLEMS
2
1

2 2
(c)

12-23
2

4
5 (d)

5 6

6 7 8

7
8 9

2
11.

3 5

2
3

Find the rank of

5-46
2

(a)

3 3

(b)

4 3 4
4 5 7 4 6

-1 -3
2

-2

4-16

10 11 12 13 14

15 16 17 18 19

Ans.

(a) 2,

(b) 3,

(c) 4.

(d) 2

12.

Show by considering minors

that

A,

A'.

A. and

.4'

have the same rank.

13.

Show

that the canonical matrix C, row equivalent to a given matrix A, is uniquely determined by A.

14.

Find the canonical matrix row equivalent


Tl
(a)

to

each of the following:


1
'X^

2-3"]^ri 0-7]
-4j
2 2

12
(b)

3 4

1/9"

3 4
_4

[2 5

[o

12
1

10
1

2j
3
2

1/9

(c)

2 3

11/9

-3

10 10 12

3 (d) 2

10 1 10-1
1 1

-1
-1

-1
6

2
1 1

1
^V/

(e)
2

10 0-12 10
1

-2
1

5
1

-3

15. Write the

normal form of each of the matrices of Problem


[I, 0],

14.

Ans.

(a)

(b).(c)

[/g

o]

(d)

P^
j]

(e)

P'

jl

12
16.

4
1

Let A

2
3

12

(a)

(6)

From Prom

/a

form Z/^^. /^O). //i3(-4) and check that each HA effects the corresponding row transformation. form K^^. Ks(-l). K^^O) and show that each AK effects the corresponding column transformation.

(c) Write the inverses H^l,

H^

{?,),

H^lc-i) of the elementary matrices of

(a).
.

(d) Write the inverses K^l. ifg^C-l).

K^lo)
4)
1

of the elementary matricesof (6)


3

Check Check

that for each

that for each K.

H,H-H~^^l KK~^ = I

"0

0'

"0
and C
=

(e)

Compute B

4"|

^12
^

ft,(3) -//isC-

-4
1

H^^(-4:)-H^(3)-Hi

1/3

0.
ij

(/)

Show Show Show

that

BC

CB
H-.

17. (a) (b)

that

/?',-=
/?

K-(k) = H^(k).

and K^,(/t)

= H^-(k)

that if

is a product of

elementary column matrices. R'is the product in reverse order of the same

elementary row matrices.


Prove:

18.

(a)

(b)

AB AB

and
and

BA BA

are non-singular if are singular


if at

.4

and B are non-singularra -square matrices.

least one of the n-square matrices

A and B

is singular.

19.

UP

Hint. Express

and Q are non-singular, show that A,PA,AQ, and PAQ have the same rank. P and Q as products of elementary matrices.

20.

Reduce B

13 6-1 14 5 1 15 4 3

to

normal form

/V

and compute the matrices

P^

and

Qr,

such that P^BQ^ = N

48

EQUIVALENCE

[CHAP.

21. (a)
(6)

Show Show

that the that the

number of matrices

in a canonical set of n-square matrices under equivalence is

n+l.

m+l

number and n+1.

of matrices in a canonical set of

mxn

matrices under equivalence is the smaller of

22.

Given A

12 13

4
2

4 6

of rank 2.

Find a 4-square matrix S

7^

such that

AB

= 0.

2 5 6

10

Hint.

Follow the proof of Theorem

and take

Q-'b

abed
_e

A_

where a.b

are arbitrary.

23.

The matrix A

of Problem 6 and the matrix

of Problem 20 are equivalent.

Find P and Q such that B - PAQ.

24. If the

mxn

matrices A and B are of rank

rj

and

rg

respectively,

show

that the rank of

A+B cannot exceed

25.

Let ^ be an arbitrary n-square matrix and S be an n-square elementary matrix.


six different types of matrix S,

By considering each

of the

show

that

\AB\ = |^|

|fi|

26.

Let A and B be n-square matrices,

(a) If at least

one is singular show that


\AB\ = \a\-\B\
.

\AB\ =

|/4|-|s|

(6) If

both are

non-singular, use (5.5) and Problem 25 to show that

27.

Show

that equivalence of matrices is an equivalence relation.

28. Prove:

The row equivalent canonical form


Not every matrix A can be reduced

of a non-singular matrix

is I

and conversely.

29. Prove:

to

normal form by row transformations alone.

Hint.

Exhibit a matrix which cannot be so reduced.

30.

Show how

to effect
(3).

on any matrix A the transformation H^: by using a succession of row transformations of

types (2) and

31. Prove: If

.4

is an

mxn

matrix, (m ^n), of rank

m,

then AA' is a non-singular symmetric matrix.

State the

theorem when the rank of A is < m.

chapter 6
The Adjoint of a Square Matrix

THE ADJOINT.

Let A =

[a^ ]

be an n-square matrix and

oij-

be the cofactor of a-; then by definition

(6.1)

adjoint

adj

12

^22

'^ire

'^sn

Note carefully that the cofactors of the elements of the ith row (column) of A are the elements of the jth column (row) of adj A.
1

2 3 3

3
2

Example

i.

For the matrix A

2 3

4
=

11=

6,

ai2 = -2.

Cl-lS

-3.

fflsi

= 1,

0122 =

-5,
6

a^g

= 3,

ffai

= -5,

Otgg

4,

agg = -1

1-5 1
4
3

and

adj

-2 -5 -3

-1
See Problems
1-2.

Using Theorems

and XI of Chapter

3,

we

find

%1 %2
(6.2)

"In
0271

i(adj A)
'^

^^2

ni

CE 7

diag(M),

1^1

1^1)

A-L
-7 and
6 1 -5I -2 -5 5 4 -3 33 -ij

(adj A)

Examples. For the matrix ^

of

Example

1,

U|
2
3 3 3 2

1
/I

f-T
=
[_

0'

(adj /I)

2
3

0-7
-7

-7/

By
(6.3)

taking determinants in (6.2),

we have
=
I

U|.

adj

/I

|,

adj

.
I

There follow
I.

If

is re-square

and non-singular, then


adj
I

(6.4)

4
I

49

50

THE ADJOINT OF A SQUARE MATRIX

[CHAP.

II.

If

is re-square

and singular, then


A(a,diA)

(ad}A)A
If

=
adj

If

is of

rank < ra-l, then adj A =

0.

is of rank ra-1, then

is of rank 1.

See Problem

3-

THE ADJOINT OF A PRODUCT.


III.

In

Problem

4,

we

prove

If

A and B are

re-square matrices, adj

(6.5)

AB

adj

adj

MINOR OF AN ADJOINT.
^ii.i2

In

Problem
i

6,

we prove
-square minor of the
re-square matrix

IV.

Let
^l'''-2

be

A =

[o,-,-].

tjj

7m + i' 7m+2'
let

Jn
be
its

complement

in A,

and

^m+i' ^ra+2' Ji'J2


Jn

let

%.

^2

%
in adj

the m-square minor of adj A dei denote


Ji' J2

whose elements occupy the

Jm

same position

A as those

of

occupy

in A.

Then
Ji' J2

Jm

Jvi + i'

Jm + 2

Jn

(6.6)

M:
s

(-D^'i^l'

where

j\

S2

+ +

%+

/i

72

+ +

Jt,

If in (6.6)

is non-singular,
,?i.

then

J2<

(6.7)

M
When m =
2, (6.7)

' Jm

H, %,

(-1)

aH-i u
I

Jm+i' Jm + 2
i-

Jn

^M+1' 'm+2. . ^n

becomes
Jg>J4-

Jn

(6.8)

(_l)V*2+Jl-'j2m
H'J2
^2'J2
.J1.J2

\A\

algebraic complement of

When m = n-l,
(6.9)

(6.7)

becomes
Jl'j2
Jre-1

M,

(-1)

Ul

a.

When m =

n, (6.7)

becomes

(6.4).

CHAP.

6]

THE ADJOINT OF A SQUARE MATRIX

51

SOLVED PROBLEMS
1.

The

adjoint of

i4

a h
IS c

'^ll

0^21
I

d -b
I

3 4

4 3
2.

4 3l
1
1

3 4
-7

The

adjoint of

A =

[;]

IS

4
3 3

1 1 1
1

3
1

-1 -1
1

1 1 1

3
2

4
1

-2

2
3

3.

Prove:

If

A
we

is of order n

and rank n-\, then adj A

is of

rank

1.

First

IS at least one.

exactly one.

note that, since A is of rank n-\. there is at least one non-zero cofactor and the rank of adj A By Theorem X, Chapter 5, the rank of adj^ is at most n-{n-l) = i. Hence, the rank is

4.

Prove:

adj

^S

= adjS

adj

y4

^y(6.2)
Since

^Badj^S
^S-adjS-adj^
=
.4

\ab\-I
/I

{a.aiAB)AB
= =

(S adj

S ) adj

^(l^l-Oadj/i

jslc^adj/^) =

|b|.U|./
=
\

\AB\-I

and

(adjB-adj^)4fi
that

adjS {(adj^)^lfi =
adj

adjS-U|./.B
=

U|{(adjB)Si

Ab\

we conclude

^S

adj

adj

/I

5.

Show

that
(6.2)

adj(adji) = U|
and
(6.4),

-A,

if

\a\

?^

0.

By

adj

adj (adj

^)

= =

diag(| adj ^|,

adj/l|

|adj^|)
1

diagd^l

Ul

1^-1
)

Ul

Then

adj (adj ^)
adj (adj

= = =

UT'^-^
\aT' -A

^) ^)

and

adj (adj

UT'^-^

6.

Prove:

Ji'

Let

A;

h
;

in
'"n

^^ ^" m-square minor of the re-square matrix


I

A =

[a:-] tj-

Jm+i. Jw4-2
let

in
^n

^m+i. ^m+2

be its complement in A, and

^^^

nH.i2

in\

^*^e the m-square minor of adj


J1.J2

/I

whose elements occupy the same

position in adjv4 as those of

is

^H,i^, ...,i

occupy in A.

Then

52

THE ADJOINT OF A SQUARE MATRIX

[CHAP.

u
where
s =
Ji

Ji.

J/2'

H'

^2-

'

>

Jm
^m

Jra + i' Jii

'

Jn

(-ifur
7m

^i

+ f2+ +

+/1 + 72+

Prom
a,-

a,-

1-

a,-

a,-

h-j9

a,,

^^2

>

'00

,/2

"^ra.ii

"^mJ's

"im.im

"^raJm+i

"

"^m^n

tti,

*i

ai

Wl'A

a,-

a,-

a,-

''m+iJs

'

''m+iJm

'^m+iJm+i

''m+i-Jn

"ht.ji

"W2

in.i

'

"V.im4.i

"traJn

a,-

a,-

a,-

Ul
\a\

h.Jm^i.

''i2Jn

Ul

"^M'Jn^i

" "^-Jn

"%-i-i'Jn-^i

"V+i'in

^ri'^m+i

"in.in

by taking determinants of both sides, we have


Ji. Js'

(-ifUI- M %
where
s is

H\
''w

Jm+1' Jm-t2
4.

Jn

^2

^m+l' ''m+2

%
immediately.

as defined in the theorem.

Prom

this, the required form follows

7.

Prove:
of i.

If

is a

skew-symmetric of order

2ra,

then \A\ is the square of a polynomial in the elements

By

its definition.

a\ is a polynomial in its elements;

we

are to

show

that under the conditions given

above this polynomial

is a perfect square.
r = when A
\

oi
\,

The theorem

is true for n = l

since,

\A\ = a

,1

2
.

\-a Oj
and consider the skew-symmetric matrix A = [a^j] of
'
-zri^i.-zii^-zy '^2k^..2n^\

Assume now
order2A: +

that the theorem is true

when

n = k

2.

By

partitioning, write

P^l
\ \

= E= where

r
\

Then S

is

skew-sym-

"

CHAP.

6]

THE ADJOINT OP A SQUARE MATRIX

53

metric of order 2k and, by assumption,


If

\b\ =
/I,

where /is a polynomial


6,

in the
3,

elements of B.

a^j denotes the cofactor of a^j in

we have by Problem

Chapter

and (6.8)

*2fe+l, 2fe+l
f''2fe+l,

0^2^+2. 2fe+l
0'2fe4-2,

0^2^+2, 2fe+l

2fe+2

2fe+2

^2^+1, 2/!e+2

Moreover,

Otsfe+s.sfe+i =

-fflgfe+i^sfe+s

hence,

1^1/
a perfect square.

Ll2/fe+l.

2fe+2

and

SUPPLEMENTARY PROBLEMS
8.

Compute the adjoint


"l

of:
~1

2
1

3~

2
1

S'

5
'l
2*

2"

(o)
_0

2
0_

(b)
_0

2
1_

(c)

2
_3

110
(rf)

2
1
1

1_

10
1

-2
1

-2
4
1
(rf)

2 2

-4
6

Ans.

(a)
_0

0-2
1_

(i)

-2
1

(c)

-2
1

-5 -2

-16
10

10 3-5
-2

9.

Verify:
(a)
(b)

(c)

The The The

adjoint of a scalar matrix is a scalar matrix. adjoint of a diagonal matrix is a diagonal matrix.
adjoint of a triangular matrix is a triangular matrix.

10. Write a matrix

/I

7^

of order 3 such that

adj^ =0.

11. If

is a

2-square matrix, show that adj(adj/4) = A.

-1 -2 -2
12.

Show

-4 -3 -3
is 3^' and the adjoint of

that the adjoint of

-2
1

2-2
13. Prove:
If

is

itself.

an n-square matrix A is of rank

<n-\.

then

adj^

= 0.

14. Prove: If

is

symmetric, so also is adj-4.

15. Prove: If

^ A

is Hermitian,

so also

is adj/1.

16. Prove: If

is

skew-symmetric of order

, then

adj^

is

symmetric or skew-symmetric according as . is odd

54

THE ADJOINT OF A SQUARE MATRIX

[CHAP. 6

17. Is there a

theorem similar to that of Problem 16

for

skew-Hermitian matrices?

18.

For the elementary matrices, show that


-1 (a) adj Hij =

-Hij
=

-1

(6) adj

H^ (k)

diag(l/A:. l/k
Hij(k), HJ

1/k,

1,

l/k

l/k),

where the element

stands in the ith row

(c) adj Hij (k) Hjk

with similar results for the K's.

19. Prove: If

is

an n-squaie matrix of rank n or


then
-1 -1

n-l

and

if

H^...H^

-H-^

-A K-^-K,2---^t

where \

is

or

-1

-1

-1

-1

adj

adj

Xi

adj

K^

adj

K^

adj

adj H^

adj

H^

adj H^

20.

Use

the method of Problem 19 to compute the adjoint of

1110
(o)

of

Problem 7 Chapter 5
,

2 (b)

3 3

2 2 4
2

12
4 6

-14
7

-3 -3
1
1

2-2
-2
2

14
(b)

-2
1

Ans. (a)

-1 -1

-7
21. Let

1-1

= [a^--]

and B = [^k-a^A be 3-square matrices.


S(adj-4)
=

If

S{C) = sum of elements of matrix C, show that


= k S(adj,4)

S(adjB)

and

\b\

Ui

22. Prove: If

is n-square then

adj (adj

^)

UI

23.

Let A^ =\.%j\ example,

('/ = 1. 2, ...,n)

be the lower triangular matrix whose triangle is the Pascal triangle; for

10

12 10 13
3
1

110

Define
(i)

bij

"V

= (-1)^ ^ai, Hj

and verify

for n

=2,3,4
=

that
[6.^,-]

adJ-4

tjj

24.

Let B be obtained from A by deleting


^ij
iq

its ith

and pth rows and /th and gth columns.

Show

that

^pj

pq

where

o!^-

is the cofactor of a^j in \A\

chapter 7
The Inverse of a Matrix

IF A

AND B
A
is

are n-square matrices such that


B''^).

AB

BA

=1,

is called the inverse of A,

(B=A~^) and

called the inverse of B, (A =


In

Problem
I.

l,

we prove
A has an Inverse
if

An

ra-square matrix

and only

if it is

non-singular.

The inverse
II.

of a non-singular n-square matrix is unique.


is non-singular,

(See Problem?, Chapter2.)


C.

If

then

AB

AC

implies

B =

THE INVERSE

of a non-singular diagonal matrix

diag (i k,
...,

A)

is the

diagonal matrix

diag(i/k^,l/kr,.

1A)

^^

:^i' -^2

are non-singular matrices, then the inverse of the direct

sum diag(/}

diag(Al^.

A^

A~s)

Procedures for computing the inverse of a general non-singular matrix are given below.

INVERSE FROM THE ADJOINT. From

(6.2)

adj

i = U|

/.

if

is non-singular

n/M|
i2/U|
(7.1)

a^^/\A\
a22/!.4|

...

ai/U|
a2/UI

...

adj^

am/Ml

a^n/\A\

...

ann/\A\

2 3

Example

-7
is
1 1

1.

From Problem

-1 -1

2,

Chapters, the adjoint of A

1 1

4 3

-2

"7/2

-3 -

Since

Ml

= -2,

A~^ =

^'^^ -^

f
X

-k
2
1

^J
See Problem
2.

55

56

THE INVERSE OF A MATRIX

[CHAP.

INVERSE FROM ELEMENTARY MATRICES.


by elementary transformations so that

Let the non-singular n-square matrix A be reduced to

H^.-.H^.H^. A-K^-K2...Kt

PAQ

Then A ^ P
(7.2)

Q^
=

by (5.5) and, since

(S~V^ = S,
=

A'^

(P'^-Q'Y
7,

Q-P

K^-K^... KfH^... H^-H^

Example

2.

From Problem

Chapters,
1 1
"l

-3
1

-3
1

HqH^ AK^Kq

-1
1_

1 1

A
_0

=
1

-1
1

1__

-3
1

o"

'l

-3"
1

"106" '100"
-1
1
1

7 =

-3 -3
1

Then

K^K2H2'i

-1

-1

-10
to

In Chapter 5

it

was shown

transformations alone.
(7.3)

that a non-singular matrix can be reduced Then, from (7.2) with Q =1, we have

normal form by row

H,...H^.H,
is,
III.

That

If

is

reduced to

by a sequence of row transformations alone, then A~

is

equal

to the product in reverse order of the corresponding elementary matrices.

Examples. Find the inverse

of

.4

4 3

of

Example

using only row transformations to reduce

/I

to/.

13
Write the matrix [A
Ir,

Q
1

and perform the sequence of row transformations which carry A into

on the rows of six elements.


1

We have
1

3 3

1
"X^

3
1

1
'\j

3
1

4
1

-3
-"o

1 1 1

-3 -3
1

[AI^]

1 1

4 3
3 4

-1 -1

-1
-li

[/3^"']
7

-3 -3
1
1

by

(7,3).

Thus, as A

is

reduced to

/g, /g is

carried into

-1 -1

See Problem

3.

INVERSE BY PARTITIONING.

Let the matrix A = [a^j] of order n and

its

inverse

B =

[6.^,-]

be par-

titioned into submatrices of indicated orders:

41
(pxp)
^21

A12

^11

"12

(pxq)
and

(pxp)
"21

(px?)
where p + q = n
S22

(qxp)

A22 (qxq)

(qxp)

(qxq)

CHAP.

7]

THE INVERSE OF A MATRIX

57

Since

AB

BA
(i)

I^,

we have
=
=
Ip
(iii)

/4iiSii + /lisBgi

Bgi'^ii + B^z'^si

=
=
Iq

(7.4)
(ii)

4iiSi2 +

A-^^B^-z

(iv)

Bjiiia + Bss^^as

Then, provided

/4ij^

is non-singular,

!Bxi
where

'4ij^

(A.^^^

"-i-zK

(^^si '^n)

^21

~C

(^21 '^u)

.422

- A^i^Ali A^^).

See Problem
In practice, A.^^ is usually

4.

taken of order ra-l.

To

obtain A^^, the following procedure is

used.

Let
"11
'

[Oil
2i

%2l
^^aj

%2
022

11

13 O23
33_
,

%2 %3 %4
022

Osi 31

G^ =

021
Ogj

023
033
'''43

'^24 6(34

Ogj

32

41

%2

''44

After computing C^^, partition G3 so that /422 = [o33] and use (7.5) to obtain C^^. Repeat the proc

ess on

G4. after

partitioning

it

so that ^22 = [044], and so on.

Example 4. Find the inverse of A

1 1

4 3
3 4

using partitioning.

Take ^11 11

[!'].

^12

^421 =

[13],

and ^22= [4]-

Now

^=
Then

^22

'42i(^;' ^12)

[4]-[l3]M

[1],

and

f^

[l]

Sii

4\

('4i\/ii2)rw^ii)

[j

i]^[o]fii-'^^^

S12

-('4i\^i2)r^ "
I

oj

S21

-^
r'

(^2i'4la)

[-1,0]

522

[1]
"
fill

-3 -3
1 1

fil2

and
S21 B22
_

-1 -1

See Problems

5-6.

58

THE INVERSE OF A MATRIX

[CHAP. 7

THE INVERSE OF A SYMMETRIC MATRIX. When A


tors

is

symmetric, aij =

ciji

and only ^(n+1) cofac-

need be computed instead of the usual

n^ in obtaining A~^ from adj A.

If there is to be any gain in computing A~^ as the product of elementary matrices, the elementary transformations must be performed so that the property of being symmetric is preserved. This requires that the transformations occur in pairs, a row transformation followed immediately

by the same column transformation. For example,


'0

c
...

a
b
.

...

...

Ht

^l

a
b c

^i(- ^)

^^i(- |)

However, when the element a in the dia^gonal


H^(l/\Ja)
is not

is

replaced by

1,

the pair of transformations are

and K^{l/\Ja).

In general,

^Ja is either irrational or imaginary; hence, this

procedure

recommended.
the method of partitioning is used since then (7.5) reduces to

The maximum gain occurs when

(7.6)

where

A^^ - A.^^(Af-^A^^).

See Problem

7.

When A
is

is

not symmetric, the above procedure


is

may be used

to find the inverse of A'A,

which

symmetric, and then the inverse of A

found by =
(A'Ai^A-

(7.7)

A'^

SOLVED PROBLEMS
1.

Prove:

An

re-square matrix
is

A has an inverse
By Theorem
A''^

if

and only
5,

if it is

non- singular.

Suppose A
that

non-singular.

IV,

Chapter
exists.
If

there exist non-singular matrices

P and Q such

PAQ=I. Then A=P~^-Q'^ and


-1

Q-P

Supposed
A

-1

exists.

The A- A

=1

is of rank n.

/4

were singular,

_i
/l/l

would be of rank <

n; hence,

is non-singular.

CHAP.

7]

THE INVERSE OF A MATRIX

59

2.

(a)

When A
[?

!]
1

/I
I

=5, adji

r4-3l,andi"=r4/5-3/5-]
[-1
2j

[_-l/5

2/5J
1

2 3

-5
1

-5
1

(b)

When A =

2 3
1

then U! = 18, adj A

7
-5

and A

7
-5

2 4
3.

3
5 2

Find the inverse of


i

4 =

3 6 2 5
_4 5

-3
14_
0'
1
"K^

14

2 4

1
1

3/2
5 2

1/2
1
^\j

3/2

'

1/2

[AIJ

3 2

6
5

5
2

10
1 1

3 6 2 5

1/2 -1
1

-3/2
'

1
1

- 3 10
1 1
1

1 1

-3
14

10
1

-1 -5
8

-1

4 5

14

_4

14
0'

-3
5/2

10

-2

1
'Xy

2
1

3/2

1
]

1/2

"1

7/2

11

-2
1

0'

-1 -5
]

-1
;

1 1

'V^

1-1-5
1-2 5-5

-1

1/2 -1

-3/2

-3

2 3
1

-3

10

-2
13
-

h
-7 -2
2 2
--10
1
'-\J

-5
13

10
1
'X'

18

0'

10
1 1

18

-7-2
2
1

'

-7| -4 -2 -3
j

-7 -2

-4
1

-3
[

10

1_

11

-2 3/5 1/5

1
'^-/

-23
10
1

29

-64/5 -18/5
26/5
6/5
3/5

100]
1
i

-12 -2
-2

7/5
2/5
1/5

1
1

Ih

A-']

-23

29

-64/5 -18/5
26/5
6/5 3/5 7/5
2/5
1/5

The inverse

is

10
1

-12 -2

-2

(i)

iiiSn

4.

Solve

(.., /fiiS,2
(11)

+ iisSsi

= =

(iii)

S^i^ii + B^^A^^ =
for

+ A^^B^^

Bii,Si2,S2i, and S

(iV)

^21^12 + 52,42
from
(iii),

= /

Set

S22 = f"^

Prom

(ii),

B^^ = -(-4^1-412)^"^;

B21 =

- ^~\A^^At^y,

and, from

(i),

B-^^

^ti - A'l^A^^B^^ =

4\

+ (^lVi2)f~\'42iA\).

Finally, substituting in (iv),

-f

(-42i'4ii)^i2 + ^'^422

*.

and

'^'2

('^21'*lll)'^12

60

THE INVERSE OP A MATRIX

[CHAP.

5.

Find the inverse of A

12 13
2

3
3

by partitioning.

1111
1

2 3

(a)

Take Gg

3 3

and partition so that

[24],
Now
-1

and

[3]
r 3

11

r 3 -2l jj. ^_^


=

, ^11-41. -

,-1

[3
|^_^

-2
'421-4 11

-2
[2 0],

[24]

,f

^22 -

'42i('4ii^i2)

[3] - [2

4]r

[-3],

and

f
3

=[-l/3]

Then

Sii = 4l\ +

(.4lVi2)rV2i^i\)

[j

"^l +

Hr-i] [2

0]

-2] _ [2 ol
ij

-1

[o oJ

3L-3

3J
-'^

B12 = ('4ii'4i2)C

1
3

f (/I2A1)

i[2

0],

4-fl

3-6
and
-3
3

D01

Oo
2

0-1

3
,

(6) Partition

A so

that

A^^

3 3

A^Q

2 3

-421

=[

1 ]

and

/I22 =

[ I ]

4 3
3

3-6
Now

4i

-3
2

A~'^

A^iA-^-^

"X"L2

3 2J,

0-1
1

.f

[l]-[l

l](i-)

-1

&
3 = 3 2

and

C'^=[3]

-6
3

3'

-6
3

3'
1

-2
1

Then

B,

^1
-1
"

3 [3]-|[2

-3 2]

-1
0'

-1

4
=

6-9
-2
3

1-2

-2

-1

B12 =

-3
_
1_

S21 =

[-2

-2].

B22

[3]

1-210
and
[Sii S21

S12

1-2
-2

2-3
3

B22J

1-11
3-2

CHAP.

7]

THE INVERSE OP A MATRIX

61

3 3

6.

Find the inverse of A =

1 1

by partitioning.

4 3

We cannot

take

-4,

since this is singular.

1 1
1

3 3
.

7 -

3
1

-3
Then
1

By Example 3,

the inverse of

//

1 1

4 3 3 4
"1

is

-1 -1
7

-3 -3'
1 1

0'
1 1

-3
1

-3'
1

B^Ho

1
1

1 1

Thus, if the (ra-i)-square minor A^.^_ of the n-square non-singular matrix A is singular, we first bring a non-singular (n-l)-square matrix into the upper left corner to obtain S, find the inverse of B, and by the proper transformation on B~^ obtain A''^

2
7.

Compute

the inverse of the symmetric matrix

13
-1

1-12
2-3 1-1
4

2-3-1
2
1

-1
2
1

Consider

first the

submatrix G^

3 2

partitioned so that

-1

-421

[-1 2],

^22=

[1]

Now

^22 - ^2i(Ai^i2) =

[l]-[-l

2]rM

[-2]

and

f^=

[-i]

Then

B,

B12

["jj.

B21

[-k

k].

B22 = [-i]

-5
5

and

-1
5

10

-5

-5

Consider now the matrix A partitioned so that


2
'^11
1

-1
2
1

3 2

[2 -3 -1],

[4]

-1

-5
5

-1/5
2/5
.

Now

A<

_1_

10

3-1
-5
5

<f= [18/5

],

-5

-2

[5/I8].

62

THE INVERSE OF A MATRIX

[CHAP.

5-7
521 =

Then

JS^

5-15
-7
5

^[1

-2 10],

B22 =

[5/I8]

11

5-71
5-2
11
5

and

5-1
-7
1

10
5

-2 10

SUPPLEMENTARY PROBLEMS
8.

Find the adjoint and inverse of each of the following:


"

2-1"
1

"2

3 3 2

4"

"1

3"

2 3

(a)

-1
_

2
1_

(b)

4
_1

(O

2 3

4
5

(d)

-1

n U

n w

n ^

1
J.

6
3

3-15
Inverses
(a)

-10

9
,

-3
3

-2/3
1/3

3-1 ^ -15
5
-

^'^i
-

15

-4 -14
1

(c)

-3
-

-1
-

(d)

3
-J

-5

2-1

1/2 -1/6 1/3

9.

Find the inverse of the matrix of Problem

8((?)

as a direct sum.

10.

Obtain the inverses of the matrices of Problems using the method of Problems.

13
1 1 1 1

3
3

3 2
.

4
3

2 3 3 2

7 2 (c)

5 3 6

2
3 3

3
1

2
3

11.

Same,

for the

matrices

(a)

-4 -5
8

3-1

2
3

4
(d)

(b)

5 2

7
3

9
3

13
1
1

11

3
2 (a)
1

-4 -5
4

12

11-1 1-2-12 2

16

-6
- 30

-144
48
(c)

36

60

21

22

41

-1

18

-10 -44 4 -13


-1
11 7

30
6

-2 -1

48

-5 -20 -12 -4 -12 -13 48 12 -12 3


30 -20 -15 30 -11
25
7

-26
16

(b)

-1 -7 -3
1

-18
21

-5 -8
6

(^)i

-1
2

-30 -15
15

12

-9

-1 -1

12

6-9
-6 -1

-7

-1

12.

Use the result

of

Example 4

to obtain the inverse of the matrix of

Problem 11(d) by partitioning.

13.

Obtain by partitioning the inverses of the matrices of Problems 8(a), 8(6), 11(a) - 11(c).

CHAP.

7]

THE INVERSE OF A MATRIX

63

12-12
14.

12
(b)

2
3

Obtain by partitioning the inverses of the symmetric matrices (a)

2-11
1

112
2

-1 -1
2

-1

2
3

2
3

3 3

1-12

-1

-1 -1 -1 -1 -1 1 -1 -5 -1
1

Ans. (a)
-1

(b)

-1

-1 -1

3-3 2 3-4 4-2 -3 4-5 3 2-2 3-2


-3
=

15. Prove: If

.4

is non-singular,

then

AB

AC

implies

= C.

16.

Show
(a)

that if the non-singular matrices

andS,
that
if

(b)

A und B

(c)

a.nd

A and B commute, so also do B Hint, (a) A (AB)A


is

(BA)A

17.

Show
Hint:

the non-singular matrix


:r

A
.

symmetric, so also is

/I

^.

A^A

(AA"^)' =

(A~^)'A

18.

Show

that if the non-singular symmetric matrices

A and B commute, then

(a) A~'^B.

(b) AB"'^.

and

(c) A~^B~^

are symmetric.

Hint: (a)

{A'^ B)' = {BA^'^)' = (A'^)'B' = A~^B.

19.

An mxn matrix A
a right inverse
if

is said to

and only

if

have a right inverse B if AB = I and a left inverse C it CA = I. Show that A has A is of rank m and has a left inverse if and only if the rank of A is n

20.

Find a

right inverse of

13 2 3 14 13 13 5 4

if

one exists.

2
1

Hint.

The rank

of A is 3 and the submatrix S

1
1

4
3

is

non-singular with inverse S

right inverse of

17

-9 -5
3
1

is the

4x3 matrix

-4 -1

L J
3 3

-3 -3
1

21,

Show

that the submatrix

1 1

4 3
3 4

of A of Problem 20 is non-singular and obtain

-1
-1

as another
1

right inverse of A.

22. Obtain

-310b
-3
1

-1 -1

as a

left

inverse of

3
3

4
3

3
,

where a,b. and

c are arbitrary.

23.

Show

that

13 14
2
3

4
5 5

7
9

has neither a right nor a

left inverse.

24. Prove: If

|.4ii|

0,

then

/111

A21
then
(/

^12 Aq2

|.4ii|

-1^22 - ^^si^ii/ligl

25. If

|/+

l|

;^

0,

.4)-^

and

(I

- A) commute.

26. Prove:(i) of

Problem 23, Chapter

6.

chapter 8
Fields

NUMBER

FIELDS. A

collection or set S of real or complex numbers, consisting of more than the ele-

ment 0, is called a number field provided the operations of addition, subtraction, multiplication, and division (except by 0) on any two of the numbers yield a number of S.

Examples

of

number fields
numbers,

are:

(a) the set of all rational

numbers,

(b) the set of all real (c) the set of all (d) the set of all

numbers of the form a+b\f3


complex numbers a+
bi,

where a and b are rational numbers, where a and b are real numbers.
,

The

set of all integers and the set of all numbers of the form

bvs

where

i is

a rational

number, are not number fields.

GENERAL FIELDS. A
F, i.e. scalars,

collection or set S of two or more elements, together with two operations called
().

addition (+) and multiplication

is called a field

provided that

a, b, c,

...

being elements of

a + b is a unique element of

a+b

= b+a

a + (b + c)

= (a +

b) + c

For every element a in

F F

there exists an element

in

F such
-a
in

that

a+

+a =

a.

As

For each element a in


ab =

there exists a unique element

F such

that

a + (-a) =

0.

a-b

is a

unique element of

ab = ba
(ab)c = a(bc)

For every element a in


For each element a ^
a~

there exists an element

?^

such that

1-a =

o.

in

there exists a unique element a'^ in

F such

that

a-

a'''^

-0=1.
c)

Di

a(b +

= ab+ ac

D^'-

(a+b)c = ac + bc

In addition to the number fields listed above, other examples of fields are:
(e) the set of all quotients

^^^
Q{x)

of polynomials in

x with real coefficients.

(/) the set of all

2x2

matrices of the form

ti

where a and

b are real

numbers.

(g) the set in

In this field, for example, the


is is

This field, called of characteristic 2, will be excluded hereafter. customary proof that a determinant having two rows identical not valid. By interchanging the two identical rows, we are led to D = -D or 2D =

which

0+0=0.

but

is not

necessarily

0.

64

CHAP.

8]

FIELDS

65

SUBFIELDS.
If

If

S and T are two sets and

if

every member of S is also a member of T, then S is called

a subset of T. S and T are fields and


if

S is a subset of T, then S is called a subfield of T.

For exam-

ple,

the field of all real numbers is a subfield of the field of all complex numbers; the field of

all rational

numbers

is a subfield of the field of all real

numbers and the

field of all

complex

numbers.

MATRICES OVER A FIELD, When 'Mis over F". For example.


A =
1

all of the

elements of a matrix A are in a field F, we say that

1/4

1/2 2/3

is

over the rational field

and

11
2
1

- 3i

is over the

complex

field

Here, A is also over the real field while B is not; also A is over the complex field.

Let

A.B, C,

...

be matrices over the same field


is, if all

F and

let

F be

the smallest field

which
is the

contains all the elements; that


fined

the elements are rational numbers, the field

rational field and not the real or complex field.

An examination

of the various operations de-

elements other than those in

on these matrices, individually or collectively, in the previous chapters shows that no F are ever required. For example:
difference, and product are matrices over F.

The sum,
If
If

is non-singular, its inverse is

over F.
that

A ^l then A

there exist matrices

P and Q over F such


is of

PAQ

and

is over F.

If

is over the rational field

and

rank

r,

its

rank is unchanged when considered over

the real or the complex field.

Hereafter when A is said to be over taining all of its elements.

it

will be

assumed

that

is the

smallest field con-

In later chapters it will at times be necessary to restrict the field, say, to the real field. At other times, the field of the elements will be extended, say, from the rational field to the real field. Otherwise, the statement "A over F" implies no restriction on the field, except for the excluded field of characteristic two.

SOLVED PROBLEM
1.

Verify that the set of all complex numbers constitutes a field.

To do
product

this

we simply check
1.

the properties

A^-A^, Mi-Mg, and D^-D^. The zero element


are

(/I4) is

and
the

the unit element (M^) is


(A/ 1)

If

a + bi

and

di

two elements, the negative (A^) of a +


;

bi

is

-a-bi.

is

(a+bi){c +

di) = (ac
1

-bd) + (ad + bc)i


_ g

the inverse (M5) of a +


a

bi^o

is

bi

_
b^

hi

a + bi

a^

+62

a^+

a^ + b^

Verification of the remaining properties is left as an exercise for the reader.

66

FIELDS

[CHAP.

SUPPLEMENTARY PROBLEMS
2.

Verify (a) the set of all real numbers of the form a + b\r5
(6) the set of all quotients

where a and

b are ra:ional

numbers and

^
Q(x)

of polynomials in x with real coefficients constitute fields.

3.

Verify (a) the set of all rational numbers,


(b) the set of all

(c) the set of all

numbers a + bvS, where a and b are rational numbers, and numbers a+bi, where a and b are rational numbers are subfields of the complex
a
b

field.

4.

Verify that the set of all

2x2 matrices of the form

b
,

where a and
a
b

b are rational

numbers, forms a

field.

Show

that this is a subfield of the field of all 2x2 matrices of the form

h
a

w]\eK a and

h are real

numbers.

5.

Why does not

the set of all

2x2

matrices with real elements form a field?


D^, D^) of

6.

set

of elements a,b.c.... satisfying the conditions {Ai, A^. A^. A^, A^; Mi.

M.y,

Page 64

is called

a ring.
ring.

To emphasize the fact that multiplication is not commutative, R may be called a non- commutative When a ring R satisfies Mg, it is called commutative. When a ring R satisfies M^. it is spoken of as

a ring with unit element.


Verify:
(a) the set of

even integers 0,2,4,

...

is an

example of a commutative ring without unit element.

(b) the set of all integers


(c) the set of all (d) the

0,+l,2,+3,

...

is an example of a commutative ring with unit element.

n-square matrices over

is an

example of a non-commutative
,

ring with unit element.

set of all

2x2

matrices of the form

where a and

are real numbers, is an example of a

commutative ring with unit element.


7.

Can the set


ments 1

(a) of

Problem 6 be turned into a commutative ring with unit element by simply adjoining the ele-

to the set?

8.

By Problem

4,

the set (d) of Problem 6 is a field.

Is every field a ring?

Is every
i

commutative ring with unit

element a field?
To
ol
^
,

9.

Describe the ring of

all

2x2

matrices
Call

where a and

b are in

F.

If

A.

is

any matrix of the ring and

=
.

show

that

LA

= A.

L a

left unit element.

Is there a right unit element?

10.

Let C be the field of


u, V are real

all

complex numbers p + qi and

K
bi

be the field of

all

2x2
-b^

matrices

where

p, q,

^
as corresponding elements of

numbers.

Take the complex number a +


-3l
2j

and the matrix

the two sets and call each the image of the other.

To

[2
3

-4l
;

L4

Oj

2i, 3+ ^^

5.

(b) (c) (d) (e)

Show
Show
What

that the image of the


that the
is the

sum (product) of two elements of


of

is the

sum (product)

of their

images

in C.

image of the identity element

is the identity

element of C.

image

of the conjugate of a + bi?

What is the image of the inverse of


[:

"!]

This

is

an example of an isomorphism between two sets.

chapter 9
Linear

Dependence

of Vectors

and Forms

THE ORDERED PAIR

pair of real numbers (%, x^) is used to denote a point Z in a plane. The same 2-vector or vector two-dimensional the to denote used here will be as x^], written [x^, of numbers,

OX

(see Fig. 9-1).


X2
''^3(*11

^21..

^12 +

^2!

X.2(X^1. X22)

X(Xi. Xq)

Fig. 9-1

If

.Yi

= [%i,xi2]

and

X^=

[x^^.x^q]

are distinct 2-vectors, the parallelogram law for

their

sum (see Fig.

9-2) yields

A3

A^ + A2

^11

"^

-^21

-^12

"^

-'^221

Treating X^ and Xg as 1x2 matrices, we see that this en in Chapter 1. Moreover, if k is any scalar,
kX-i
L rC%'\
-1

is

merely the rule

for

adding matrices giv-

fv

X-y

is the familiar multiplication of a vector

by a real number of physics.

VECTORS. By
of F, as

an n -dimensional vector or re-vector

over

is

meant an ordered set

of n

elements x^

(9.1)

X
x^,X2

[a;i,x2, ...,%]
...,

The elements

%
it

are called respectively the first, second,

reth

components of X.

Later we shall find

more convenient to write the components of a vector in a column, as

(91)

I X-i

X2,

Xi

Now
and

(9.1) and (9.1') denote the


(9.1')

as a column vector.

same vector; however, we shall speak of (9.1) as a row vector We may, then, consider the pxq matrix A as defining p row vectors
components of a
17-vector) or

(the elements of a row being the

as defining g column vectors.

67

68

LINEAR DEPENDENCE OF VECTORS AND FORMS

[CHAP.

The
tor are

vector, all of

whose components are


of

zero, is called the zero vector and is denoted by 0.

The sum and difference

two row (column) vectors and the product of a scalar and a vec-

formed by the rules governing matrices.


1.

Example

Consider the 3-vectors


A:i= [3,1,-4],
(a)

-^2= [2.2,-3],

^3= [0,-4,1],
= [6. 2,

and
10,

^4= [-4,-4,6]
-I5]
=

2X-^-iX^
IX^+X^.
2X^
=

2[3,l,-4] - 5[2,2,-3]

-S] - [lO,

[-4,-8,7]

(6)
(c)

2[2,2,-3]+ [-4,-4.6]
=

[o,0.o] =

-3X2-^3
X^- Xq+

{d)

2X^ -

X^

The vectors used here are row vectors. Note umn vectors, the results remain correct.

that if

each bracket

is

primed to denote col-

LINEAR DEPENDENCE OF VECTORS. The m


Ai
=

re-vectors over

|.%]^,%2'

^mJ

are said to be linearly dependent over


all zero,

provided there exist

m
=

elements

h-^^.k^,

,k^

of F, not

such that
k.^X-1

(9.3)

k^X^^

1.

k^X-^

Otherwise, the

vectors are said to be linearly independent.

Example 2. Consider the

four vectors of

Example
(c)

By

(6) the vectors


{d).

^2 ^"d

Xj^_

are linearly dependent;

so also are X^, X^, and X^ by

and the entire set by

The vectors X^ and


fel^l

Xq^.

however, are linearly independent. For. assume the contrary so that


=

+
0,

;c2A'2

[Zk^ + 2k^,
0,

k^+

2k^, -'iJc.i_~ Zk^l


lik^

[o,

0,0]

Then

3fti + 2k^ = and then ^2 = 0.

ft^

+ 2^2 =

and

-ik^ -

= 0.

Prom

the first two relations A^ =

Any

n-vector

and the n-zero vector

are linearly dependent.

A
if

vector ^,B+i is said to be expressible as a linear combination of the vectors Ai, X^


k^, k^, ...,k^

X^

there exist elements

of

F such
%A]^ +

that
:2A2

Xfn+i

k^Xf^

BASIC THEOREMS.

If

in (9.3),

k^

?^

0,

we may solve

for

^i
(9.4)

= Xi

- T"!^!'^! + =
SiJVi

+ ^i-i^t-i + -^i+i^i+i +

+ ^m^m!

or

+ s^_iZi_i + s^j-i^i+i +

+ s^J^

Thus,
I. If m vectors are linearly dependent, some one of them a linear combination of the others.

may always be expressed as

CHAP.

9]

LINEAR DEPENDENCE OF VECTORS AND FORMS

69

II.

If

vectors X^, X^

ing another vector


bination of Xi, X^,

, X^
2,

X^j,-^

is

X^ are linearly independent while the set obtained by addlinearly dependent, then Jt^+i can be expressed as a linear com-

Examples. Prom Example

the vectors X^, and X^ are linearly independent while

linearly dependent, satisfying the relations 2X.i^-2X^-

Xg=

0.

Clearly,

X^.X^.^niXg Zg=2A^i-3^2-

are

III.

If

among the m vectors

X^, X^

X^ there

is

a subset of

r<m

vectors which are

linearly dependent, the vectors of the entire set are linearly dependent.

Example

4.

By

(b) of

Example

1,

the vectors

X^ and X^ are linearly dependent; by

(d),

the set of four

vectors is linearly dependent.

See Problem

1.

IV. If the rank of the matrix


Xi 1

%2

(9.5)

m<n
*TOl

^n2

%
r

associated with the

vectors (9.2) is r<m, there are exactly

vectors of the set which

are linearly independent while each of the remaining m-r vectors can be expressed as a linear combination of these r vectors. See Problems 2-3.

V.
is

necessary and sufficient condition that the vectors (9.2) be linearly dependent

that the matrix (9.5) of the vectors be of rank

r<m.

If

the rank is m, the vectors are

linearly independent.

The
If

set of vectors (9.2) is necessarily linearly dependent

if

m>n.

the set of vectors (9.2) is linearly independent so also is every subset of them.

A LINEAR FORM
(9-6)

over

in

ra

variables n

x^, x^

a^x^

is a

polynomial of the type


+

2
i=i

OiXi

a^x^

a^A:^

where the coefficients are


Consider a system of

in

F
linear forms in n variables
/l

CIu% + %2^2 +
+
'^2n*re

(9.7)

/m

1^1

0~foXo

%n^n

and the associated matrix

-'ml

"m2

If

there exist elements k^.k^

k^ +

not all zero, in +


...

F such
=

that

Kk

^^^2/2

A;4

70

LINEAR DEPENDENCE OF VECTORS AND FORMS

[CHAP. 9

the forms (9.7) are said to be linearly dependent;

independent.
to the linear

Thus, the linear dependence

dependence
/i =

or

otherwise the forms are said to be linearly independence of the forms of (9.7) is equivalent independence of the row vectors of A.
or

Example

5.

The forms

2xi

- 2 +

3*g,

/2 =

x.^+

2%

4^=3.

/g =

ix^

Tx^ + Xg

are linearly depend-

2-13
ent since

A =

4
1

is of rank 2.

Here,

3/^

"if^

fs =

-7

The system

(9.7) is necessarily

dependent

if

m>n. Why?

SOLVED PROBLEMS
1.

Prove:

If

among

the

vectors X^,X^, ...,X^ there is a subset, say, X^,X^

X^,

r<m,

which is

linearly dependent, so also are the


Since, by hypothesis,
k^X-^ +

vectors.

k^X^ +

+ k^X^ = + k^X^ +

with not all of the k's equal to zero, then


0-.Y^+i +

k^X^

+ k^X^ +

0-.Yot

with not all of the k'a equal to zero and the entire set of vectors is linearly dependent.

2.

Prove:

If

the rank of the matrix associated with a set of

ra-vectors is

r<m, there are exactly

vectors which are linearly independent while each of the remaining m-r vectors can be expressed

as a linear combination of these

vectors.
.

is

Let (9.5) be the matrix and suppose first that m<n If the r-rowed minor in the upper left hand comer equal to zero, we interchange rows and columns as are necessary to bring a non-vanishing r-rowed minor

into this position and then renumber all rows and columns in natural order.

Thus, we have

11

11?

21

25-'

Consider now an (r+l)-rowed minor

*11

12

%-r Xiq
.

%1 %2
*rl X^2

Xqt x^q

^rr Xrq
.

xp^

Xpr,

xpf xpq

where the elements xp; and


...,A;^+i =

xj^q

are respectively from any row and any column not included in A.
x^q. xpq,

Let h^,k^,
Then,

be the respective cofactors of the elements x^g. x^q

of the last column of V.

by (3.10)

CHAP.

9]

LINEAR DEPENDENCE OF VECTORS AND FORMS

71

fci^ii

k2X2i +

+ krXri + ^r+i*^i

= =

(i

= 1,2 =

r)

and by hypothesis

k^x^q + k^xQq +

+ krx^q + krA-ixpq

Now
u.

let the last

column of

not appearing in A.

The

V be replaced by another of the remaining columns, say the column numbered cofactors of the elements of this column are precisely the k's obtained above
k^x^n + ^2*2W +

so that

+ ^rXru + ^r-n^pu

Thus,
k^x^j;

k^2t +
t,

"

f'r'^rt

f'r-n'^pt

(t

= 1,2

n)

and,

summing over

all

values of

k^X^ + k^X^ +

k^X^ +
r

k^^-^Xp

Since

/i:,^+i

ji^

0,

Xp
X^.

is

a.

linear combination of the


-V^+i,

linearly independent vectors

X-^^.

X^

X^.

But

^^j

was any one


binatlon of

of the m-r vectors

^r+2

^I

hence, each of these

may be expressed as a linearcom-

^j^,

X^

consider the matrix when to each of the given m vectors m-n additional zero compoThis matrix is [^ o]. Clearly the linear dependence or independence of the vectors and also the rank of A have not been changed.

For the case

m>n.

nents are added.

Thus, in either case, the vectors


tors
X-^.X^.
...,

Xr+^

X^

are linear combinations of the linearly Independent vec-

X^

as

was

to be proved.

3.

Show, using a matrix, that each triple of vectors


X^ = [1,2,-3.4]
(a)
X-L

= [2,3,1,-1]
[2, 3,

^2 = [3,-1,2,1]
^^3= [1,-5,8,-7]

and

(b)

^2=

1,-2]

^3= [4,6,2,-3]

is linearly dependent.

In

each determine a maximum subset of linearly independent vectors and

express the others as linear combinations of these.

(a)

Here,

12-34 3-121
1-5

is of

rank

2;

there are two linearly independent vectors, say

X.^

and X^

The minor

8-7
1

-3
The cofactors
of the elements of the third column are

2
j^
.

Consider then the minor

-1
and -7.

3-12
1-5
8

respectively -14,

7,

Then -1^X^ +

1X2-1X2=

and Xg

-2X^ + X^.

2
(b)

3 3

Here

4
2 2

6
3

1-1 1-2 2-3


;

is of rank 2;

there are two linearly independent vectors, say

X^ and Xg.

Now
-1

the

we interchange

the 2nd and 4th columns to obtain

3 2

2-113 2-213 4-326

2 for

which
2

-2

5^0.

-1

1 1

The cofactors

of the elements of the last

column of

-2

are

2,2,-2 respectively. Then

4-32.
2X^ + 2X2 - 2Xs =
and

Xg

Xi +

X,

72

LINEAR DEPENDENCE OF VECTORS AND FORMS

[CHAP. 9

4.

Let Pi(l, 1, 1), PsCl, 2, 3), PsiZ, 1, 2), and P^(2, 3, 4) be points in ordinary space. and the origin of coordinates determine a plane tt of equation

The

points

Pi,

P^

X y z
(i)

1111
12
3
1 1

2y +

Substituting the coordinates of P^ into the left


2
3

member
4

of

(i).

we have
2
3
1

1111
12
3
1
1

1110
12
3
1

4
1

3
1

4
1

Thus, P4 lies in

tt.

The

significant fact here is that

[P^.,

Px.Pq

]'

is of rank 2.

We have
Show

verified:

Any

three points of ordinary space lie in a plane through the origin provided the matrix
2.

of their coordinates is of rank

that Pg does not lie in v.

SUPPLEMENTARY PROBLEMS
5.

Prove:

If

vectors

X,^.

X^

X-^

are linearly independent while the set obtained by adding another vector
X^^.

-^m+i is linearly

dependent, then ^m+i can be expressed as a linear combination of X^.X^

6.

Show

that the representation of /^^+i in

Problems
siXi

is unique.

m
Hint: Suppose
^7^+1 =

n
kiXi
=
1,

m and consider

X
i=i

2
i=i

(A:^.

- s^ ) X^

i=i

7.

Prove:

necessary and sufficient condition that the vectors (9.2) be linearly dependent

is that the matrix

(9.5) of the vectors be of rank

r<m.
In (9.3) subtract
S2.

Hint: Suppose the

vectors are linearly dependent so that (9.4) holds.


s^,

from the ith row the

product of the first row by

the product of the

second row by

^s Indicated in (9.4).

For the

converse, see Problem

2.

8.

Examine each of the following sets of vectors over the real field for linear dependence or independence. In each dependent set select a maximum linearly independent subset and express each of the remaining vectors
as a linear combination of these.

^1
Xj_

[1,2,1]
(c)

^1

[2,1, 3,2, -1]

= =

[2,-1,3,2]
X2 = [2,1.4]
^2 =
[4,2 1.-2 .3]

(a)

X^

[1,3,4.2]

(6)

Xa

= =

[4,5,6]

Xs X^
Xs
(c)

[0,0 5.6, -5]


[6,3 -1,--6,7]

X3 = [3,-5,2,2] ^4
Ans.
(a)

[1.8.-3]
A.Q

=
= =

Xq

2X^ - X^

A3 = 2a^ +
(b)
A. A

5a 1 2a o

x^

2^1--X^ 2X2--^1

CHAP.

9]

LINEAR DEPENDENCE OF VECTORS AND FORMS

73

9.

Why can
Show

there be no more than n linearly independent -vectors over F'>

10.

that if in (9.2) either Xi = Xj

or

X^ = aXj, a

in F. the set of vectors is linearly

dependent.

Is the

converse true?

11.

Hint: Consider k^^X +

Showthatanyn-vectorZandthen-zero vector are linearly dependent; hence, k^-O = where fc^ = o and ftj ^ 0.

A"

ando are considered

proportional.

12. (a)

Show Show

that

X._ =

[l.l+i,i], X^ = [j,_j,i_i] and X^ = [l-i-2i,l-i, 2-j


[l.l+i.i], X^ = [i.-i.l-i], and Xq = [o.l-2i.2-i] complex field.

are linearly dependent over

the rational field and, hence, over the complex field.


(b)

that

Zi

are linearly independent over

the real field but are linearly dependent over the

13. Investigate the linear


/i

dependence or independence of the linear forms:


fx

3% 2::i

Xg + 2Xg + x^
(b)

2Xx
3%-L 5Xj^

3Xg +

4A!g

- 2*4

()

fz /a

= =

+ 3x2 - Xg+ 2x^

f^ = fg =

+ 2^2 -

2x3 + 5*4
X4.

5x^
3/i

95C2

+ 8xq /g

x^.

- X2+ 2Xq +

Arts, (a)

2/2

14.

Consider the linear dependence

or

independence of a system of polynomials


+ ai_^x + a;
(i =

Hox
cient matrix

+ aij^x

1,2

m)

and show that the system is linearly dependent or independent according as the row vectors of the coeffi"10 20
"11 21

"^0

"ni

nn
r

are linearly dependent or independent, that is, according as the rank

of

is less than or

equal to

15. If the

polynomials of either system are linearly dependent, find a linear combination which is identically
x

zero.

Pi =
(a)

~ 3x2 + 4^ _
2x2

Pj_

2x* +

3:c^

-4x^+5x
x^ +

+ 3
\

P2 = Ps = x

- 6 +
X
2P3 =

(b)

P2 = Pg =

x +

2x2- Zx +

2*2 +

X* +
=

2x-

X + 2

Ans.

(a)

2Pi + Pg

(6)

P^ +

P^-

2Pg

16.

Consider the linear dependence or independence of a set of 2x2 matrices M.


over F.

[::]-Ci]-[::]

Show

that

fe^il/i

A:2^^2

+ ^3^3 =

when

not all the k's (in F) are zero, requires that the rank of the

abed
matrix
e

f
q

g h
s
t

be <

3.

(Note that the matrices

M-^.Mz.Mq

are considered as defining vectors of four

components,)

Extend the result

to a set of

mxn matrices

74

LINEAR DEPENDENCE OF VECTORS AND FORMS


-

[CHAP.

2
2

2
3

3 2
1
,

3 6

17.

Show

that

3
1

4
2
_

4 2

and

are linearly dependent.

2
_

3
-

18.

Show

that

any 2x2 matrix can be written as a linear combination

of the matrices

n
n

[o

oj'

[o oj

and
[i
oj'

Generalize to nxn matrices.

19. If the ra-vectors

X^^.X^

X^

are linearly independent,


if

show

that the vectors

Y-^.Yr,

1^

where

7^

2
A

aijXj.

are linearly independent

and only

if

\_aij'\

is non-singular.

20. If

is of rank
C^,

r,

where

C^

show how to construct a non-singular matrix B such that AB C^ are a given set of linearly independent columns of A.

= [C^,

C2

C7-,

o]

21.

Given the points Pi(l,


(a)

1, 1, 1),

Pjd,

(6) (c)

Show Show Does

that the rank of [Pi, P3]' is that [P^, P^. P3,

PaY

is

Ps(2, 2, 2, 2), and /VO, 4. 5, 6) of four-dimensional space, so that the points lie on a line through the origin. of rank 2 so that these points lie in a plane through the origin,
2, 3. 4), 1

P5(2, 3. 2. 5) lie in the plane of (6)?

22.

Show

that every n-square matrix

A over F satisfies an equation of the form


k-^A^
^

A^ +
where the k^ are scalars of F
Hint: Consider
/,

+ kr,A^

...

+ kp^^A + kpl

4,^^,/!^

in the light of

Problem

16.

23.

Find the equation of minimum degree (see Problem 22) which


(a)

is satisfied

by

L J, [:;]
4^-24

A '-[:-:]
(b)

\_

\.

(c)

[;:]
=

Ans.

(a)

4^-24=0,
(c),

(b)

+ 27 =

0,

(c)

A^ - 2A +1

24. In

Problem 23(b) and

multiply each equation by 4"^ to obtain


is non-singular, then

(b)

A'''^

I-^A.

(c)

A~'^=2l-A,

and

thus verify: If A over cients are scalars of F.

A'

can be expressed as a polynomial in A whose coeffi-

chapter 10
Linear Equations

DEFINITIONS. Consider a system

of

linear equations in the n

unknowns

xi.a;.

>

*r?

OqiX-i +

022X2 +

(10.1)
"Wl*!"'" Ob2*^2 +
\

+ c!2n*-n

^2

OIj,

Aa

in

which the coefficients

(o's)

and the constant terms (A's) are in

F
x^
in

By

a solution in

of the system is meant any set of values of %,%2.

F which

sat-

isfy simultaneously the

equations.

When

the system has a solution,

it

is

said to be consistent;

otherwise, the system is said to be inconsistent.


tion or infinitely

consistent system has either just one solu-

many

solutions.

of linear equations over F in the same number of unknowns are called equivevery solution of either system is a solution of the other. A system of equations equivalent to (10.1) may be obtained from it by applying one or more of the transformations: (o) interchanging any two of the equations, (b) multiplying any equation by any non-zero constant in

Two systems
if

alent

F, or (c) adding to any equation a constant multiple of another equation. Solving a system of consistent equations consists in replacing the given system by an equivalent system of prescribed form.

SOLUTION USING A MATRIX.


as

In matrix notation the

system of linear equations (10.1) may be written


1

^11

\'
=

(10.2)

02

02 2

^2n

*2

K
hm

r".
or,

more compactly, as

(10.3)
?here

AX
A =
[o^^-] is

H
=
[x^.Xr,

the coefficient matrix,

xj\ and H

[h^h

A^]'

Consider now

for the

system (10.1) the augmented matrix


Oil

ai2
02 2

"i?i

K
[A

(10.4)

021

^271^2

H]

ml

0^2

<^nn

(Each row of (10.4) is simply an abbreviation of a corresponding equation of (10.1); to read the equation from the row, we simply supply the unknowns and the + and = signs properly.)

75

76

LINEAR EQUATIONS

[CHAP. 10

To solve
to replace

the entire

the system (10.1) by means of (10.4), we proceed by elementary row transformations A by the row equivalent canonical matrix of Chapter 5. In doing this, we operate on rows of (10.4).

Example

1.

Solve the system

3xi +
^.X-^
2x-j^

x^
^Xq^

2X2
Xg

=
=

+ ^Xq + 2xq
2
1

4
1 1

2'

"1

2
1

1 1

2
1

-2

The augmented matrix VA H\

=
-3

-1

-]

-5 -5 -5 -5
0.

V
.0
1

-11 -5 -5

'1

-1
1

1'
1 1 1

1
1

.0

OJ
x-i

0.

Thus, the solution

is the

equivalent system of equations:

=1, 2

= 0,

xq =

1.

Ex-

pressed in vector form, we have

= [l,

0, l]

FUNDAMENTAL THEOREMS.

When the coefficient matrix A of the system (10.1) is reduced to the A:]'. row equivalent canonical form C, suppose {A H] is reduced to [C K], where K= ^1,^5 If A is of rank r, the first r rows of C contain one or more non-zero elements. The first non-zero element in each of these rows is 1 and the column in which that 1 sta,nds has zeroes elsewhere.
of zeroes.

The remaining rows consist


the variables x:
,

Prom

the first

x:
,

...

bleS X:
If

Jr+1
If

X:

Jr+2
= k

...

Jr X; and one of the i^, Uq Jn

,xj

(the notation is that of

r rows of [C K], we may obtain each of Chapter 5) in terms of the remaining varia-

k^.

k^
ac,-

0,

then (10.1) is consistent and an arbitrarily selected set of

values for

X,-

X.

together with the resulting values of


if at

% Ji
,,

a solution.
kj; 7^

On

the other hand,

least one of

V+i'

"r+s

'

J2 is different

x ,

...

X- constitute Jr

from zero, say

0,

the corresponding equation reads


Qx.^

0%

+ 0*71

7^

and (10.1)

is inconsistent.

In the consistent case,

A and

[A

H]

have the same rank; in the inconsistent case, they

have different ranks.


I.

Thus
=

A system AX

of

linear equations in n

the coefficient matrix and the augmented matrix


II.

of the

unknowns is consistent if and only system have the same rank.

if

r<n, re-r of the unknowns may be chosen When these n-r r unknowns is of rank r. are uniquely determined. unknowns other r values, the whatever assigned any unknowns are
In a consistent

system (10.1)

of rank

so that the coefficient matrix of the remaining

Xi +

2*2

3*^3

~ 4^4

2x4.

= = =

6 4

Example

2.

For the system

aci

+ 3^2 +
5a:2

xg

2^1 +

2%3

5x^1

10

CHAP.

10]

LINEAR EQUATIONS

77

12-3-4
[A

12-3-4
">-/

6
'Xy

1 1

-11 -8 10
4

H]

13 1-2 2 5-2-5

4 10_

1 1

4 4

2-2 3-2
lO"
4

2-2

_0

10

"l

-11
1

-V

0-2

[C K]

^0

10
-2-4o,

Since A and [A H] are each of rank r = 3, the given system is consistent; moreover, the general solution contains n-r = 4-3 = 1 arbitrary constant. Prom the last row of [C K], x^= 0. Let xs = a, where a is arbitrary; then aj-l = 10 + 11a and xq = -2-4o.

The solution

of the

system

is given

by x^ = 10 + lla, xj =

xg = a,

x^,

X
If

or

= [lO

+ llo,

-2-4a,

a, o]'

a consistent system of equations over

has a unique solution (Example


2)
it

1) that

solution

is over

F.

If

the system has infinitely

many solutions (Example

has infinitely many solu-

F when the arbitrary values to be assigned are over F. However, the system has many solutions over any field 2F of which F is a subfield. For example, the system of Example 2 has infinitely many solutions over F (the rational field) if o is restricted to rational numbers, it has infinitely many real solutions if a is restricted to real numbers, it has infinitely many complex solutions if a is any whatever complex number.
tions

over

infinitely

See Problems

1-2.

NON-HOMOGENEOUS EQUATIONS. A
a-^

linear equation

Xi + 0^X2 +

+ n*n

is called

non-homogeneous

if

equations provided

is not a

A 7^ 0. A system AX = H is called a system of non-homogeneous zero vector. The systems of Examples 1 and 2 are non-homogeneous

systems.
In

Problem 3 we prove
ni.

A system

of

re

non-homogeneous equations

in n

unknowns has a unique solution


provided \a\ ^
0.

provided the rank of its coefficient matrix


In addition to the

is n, that is.

method above, two additional procedures for solving a consistent system non-homogeneous equations in as many unknowns AX = H are given below. The first of these is the familiar solution by determinants.
of n
(a) Solution

by Cramer's Rule. Denote by 4, (i = 1,2 n) the matrix obtained from A by replacing Its Jth column with the column of constants (the h's). Then, if \A\ y^ 0, the system AX = H has the unique solution

<1"'5)

777

X2

'

"'

See Problem
2xi

4.

X2 + Sxg + X2

*4

Example

Xl +
3.

Solve the system


3x:i 2%-^

- 3x3 - 4x4 6x2 - 2x3 + X4. - 3x4

using Cramer's Rule.

+ 2*2 + 2x3

We

find
1 1

6 2

-3 -4 -2 1

-120.

-3

-4
1

-2
2

-240

2-3

-3

78

LINEAR EQUATIONS

[CHAP. 10

2
1

15
6 2 8

-1 -3 -4

3 2

8-21
2

-24,

1-1-4
1

2-3
2
1 1 1

2
5 5

2-3

and
3 2

6
2

-3 -1 -2 8
2
2_

-96

Then

x-i

-240
-120

A^

-24
-120

= 0,

and

Ml

-120

-96
^4

-120

(b) Solution using

^.

If

^ #
|

0,

A~^ exists and the solution of the system


=

AX

is

given

by
(10.6)

A-^-AX

A-^H

or

A-^H
Xg
is 2 3 2
1 1

2xi + 3X2 +

Example

4.

The

coefficient matrix of the system

x^ +

2%
a;2

+ 3xg +
1

3 2

3^1 +

2a:3

-5
1

From Problem

2(6), Chapter 7,

A
18

-5
1 ["9"

Then
"35'
1

-5
1

-5
1

AX

A-^H

J_
18

-5
1

29
18
.

-5

L8_

5_

The

solution of the system is x^ = 35/18, x^ = 29/18, x^ - 5/18.

See Problem

5.

HOMOGENEOUS EQUATIONS. A
(10.7)
is called

linear equation
'^1*1 +

"2*2 +

+ ''n'^n

homogeneous.

A system

of linear equations

(10.8)
in n

AX
is called a

unknowns

of the coefficient matrix A

always consistent.

system of homogeneous equations. For the system (10.8) the rank and the augmented matrix [A 0] are the same; thus, the system is = is always a solution; it is Note that X = 0, that is, %i = xs = =

called the trivial solution.


If

the rank ot
xj^

A
=
II

is n,

unique solution

X2=

...

then n of the equations of (10.8) can be solved by Cramer's rule for the = x^= and the system has only the trivial solution. If the rank of

is

r<n. Theorem
IV.
trivial

assures the existence of non-trivial solutions.


for (10.8) to

Thus,

A necessary and sufficient condition solution is that the rank of A be r < n. A

have a solution other than the

V.

necessary and sufficient condition that a system of n homogeneous equations in


|/4

n unknowns has a solution other than the trivial solution is

=
|

0.

VI. If the rank of (10.8) is r < n, the system has exactly n-r linearly independent solutions such that every solution is a linear combination of these n-r and every such linear See Problem 6. combination is a solution.

CHAP.

10]

LINEAR EQUATIONS

79

LET Iiand X^he


Thus,

two distinct solutions of AX = H. Then AX^ = H, AX^ = H, and A (Xx- X^) = Y = X^~ X^ is a non-trivial solution of AX = 0.

AY

0.

then Zy, +
'P

Conversely, if Z is any non-trivial solution of AX = and if X^ is any solution of AX = H, X = Xl+ Z is also a solution of AX = H. As Z ranges over the complete solution of AX = 0, Z ranges over the complete solution of AX = H. Thus,
VII.
If

the system of non-homogeneous equations

AX

solution of the system is given by the complete solution of


lution of

E AX

is

consistent, a complete

plus any particular so-

AX

= H.
system
i

Example

5.

In the

Xi

2x2

+ 3x3 =
set x^ = 0; then xg = 2 and x^ =
5
1.

Xj +

^2 + 2 Xg

particular

solution is

A:^,

= [o,

1, 2]'.

The complete

solution of < *^
\^Xi

~
+

*^ Xq + 2*3 =
-^

IS

[-7a,a,3oJ

where a

is arbitrary.

Then the complete solution of the given system


=

is

[-7a,a,3o]' +

[O, 1,2]'

[-7a,

+a, 2+3a]'

Note. The above procedure may be extended to larger systems. However, it is first necessary to show that the system is consistent. This is a long step in solving the system by the augmented matrix method given earlier.

SOLVED PROBLEMS
xi +
e

X2 ~ 2xs + ^2 +

X4 + 3 K5

=
= =

2i 3
ail

2%

+ 2x4 + 6*5

+ 2 X2 - 4 Xg - 3 X4 - 9 xg

tion

The augmented matrix


'l

[A H]

1-2 2-1 2
2

l'
2
-v-

"1

r
'\J

1 1

-4 -3 -9

3_
1

-3 -1

-2 -2
2
1

-6
1

8
3
1

0_

0-1
-2

-18
3

1-2
-6
1 1

-18
1

-2000
13

Then

x^ -

solution

1, x^- 2xs = 0, and may be given as xx=

^4
1.

+ 33=5 =

0.

Take

x^^

2a,

x^^a,

xg = a and x^ = b. where a and b are arbitrary; the complete x^ = -3b. x^ = b or as AT = [l, 2a, a, -3 6, 6]'.

x^ +

X2 + 2Xg +

X4

= = =

5 2 7
.

Solve

2%i + 3x2 -

4xi +
Snliitinn*

5%
'11
2

s - 2x4 + 3xg
2
5"'
"V-

'112

1 1

[A H]

3-1-22
5

13

7
O-^tj +

1-5-4 1-5-4

-8
-13

-5 -4 -8 -5
is inconsistent

The

last

row reads 0-xi +

O-^g + 0-4 = -5; thus the given system

and has no solution.

80

LINEAR EQUATIONS

[CHAP. 10

3.

Prove:

A system AX
7^

of n

non-homogeneous equations

in n

unknowns has a unique solution

provided \A\
If

0.

is non-singular,
[/

it

is

equivalent to
=

/.

When A

is

reduced by row transformations only to


system.

/,

suppose

[A H] is reduced to

K].

Then X

is a solution of the

Suppose next that


Since A is

X = L is a second solution non-singular, K = L, and the solution is

of the system; then

AK

and

AL

H, and

AK

AL.

unique.

4.

Derive Cramer's Rule.


Let the system of non-homogeneous equations be il*l + a^jAiQ +
(1)
]

+ ain*n
+ 2n*n

h^
^z

uqiXi + 022*2 + "ni*i + an2*2 +

+ "nn^n

^n

Denote by A the coefficient matrix

[a.

and

let a^,-be the cofactor of

in

Multiply the first equa-

tion of (J) by ttn, the second equation by Ksi

the last equation

by

ttni.

n
ai^di-^x-^ +

n ai20iii% +
10,

and add. n
1

We have

S
which by Theorems

2
i=i

.S

i=l

1=1
3,

ain^ilXn
=1

and XI, and Problem


hx ai2
^^2

Chapter

reduces to

"m
'^2n

22

^n

""no.

so that

x-i

^11 j-

^1

"n
n2 and sum to obtain

Next, multiply the equations of (i) respectively by a^i, ^ii


"11 ^1 Oqi
/12

''is '^23

\A\.x^

"m
"2n
so that
*<,

Continuing in this manner, we finally multiply the equations of

(i)

respectively by a^n. 2n

(^,71

and sum to obtain


Oil "21
"l,n-i "1

"2,n-i ^2

so that

x^.

'ni

2Xi +
5.

X2 + 5 *3 +

*^4

~ = =

Solve the system

X2 ~ 3x3 - 4*4 Xi + :4 3 Xi + 6 Xj - 2 aig + + + 3x4 2a;3 2x1 2x:2

-1
using the inverse of the coefficient matrix.

=
120 120

Solution:
2

15

-120'
80

The inverse

of

1-3-4
6-2 1 2-3 2

3 2

-69 -73 17 120 -15 -35 -5

Then

40

.24

-40.

CHAP.

10]

LINEAR EQUATIONS

81

"120
1

120

-120"
80

5"

"

120

-69 -73 17 -15 -35 -5

-1
8
.

1/5

40

.24
(See Example
3.)

-40.

2.

4/5

Xl +
6.

^2 +

s +

*4
4a;4
Xj^

= =

Solve

i + 3*2 + 2xq +

2Xi
Solution:

Xg -

o"
'X/

1
1
-

11110
2

\A

U\

2
1

13

-1
1
''\J

-2 -1 -3
i
1

11110
1

-i
3 2

o"

i 1

is 2,

The complete solution of the system is we may obtain exactly n-r = 4-2
taking a =
1,

x^ =
= 2
1

-^a + 16, ^^

linearly independent solutions.


is

-! - |6, xs=a, x^ = h. Since the rank of One such pair, obtained by

first

6 =

and then a =
X2 = -2, *3 =

3,

6 =

x^ =

0.

1,

X4 =

and

x^ = -1, x^ = -3, ^3 =
b =

3,

:x:4

What can be said

of the pair of solutions obtained

by taking a =

and a =

b =

37

7.

Prove: In a square matrix A of order n and rank n-1, the cofactors of the elements of any two rows

(columns) are proportional.


Since
1^1

=0,
0).

the cofactors of the elements of any row (column) of

are a solution X-^ of the

AX

system

(A'X =

Now the system has but one linearly independent solution since A (A^) is of rank n-l. Hence, for the cofactors of another row (column) of A (another solution X^ of the system), we have X^ = kX^.

8.

Prove:

If

/"i,

f^

f^ are

m<n

linearly independent linear forms over

in n variables, then the

linear forms

^j
are linearly dependent if and only if the

^^'ijfi-

0-=1.2

p)

mxp

matrix [5^] is of rank r<p.

The g's
that

are linearly dependent

if

and only

if

there exist scalars a^,a^

a.

in

not all zero, such

lgl + "2g2 +

+apgp

ai

^^Hxfi
P

.|

si^fi

...

ap

.2

sj^fi

.2

2
J=l

1=1

a;:S--)f.
J ^J

82

LINEAR EQUATIONS

[CHAP. 10

Since the /'s are linearly independent, this requires

j?/i^v
X

^i^-ii

+ "p'iP
in p unknovras

(i

1,

m)

Now, by Theorem IV, the system of m homogeneous equations


trivial solution

s^j xj -

has a non-

= [a^.a^,

apV

if

and

only if [^^j] is of rank

r<p.

9.

Suppose A =

[a-

] of order n

is singular.

Show

that there

always exists a matrix R =

[h^j]

?^

of

order n such that

AB

0.

Let Bi.Bg

B be the column vectors


say AB^
= 0, or

of

B.

Then, by hypothesis,

AB^

^65

..

AB^

= 0.

Con-

sider any one of these,

ii&it + "iQ^'st +

"m ^nt

ni*it + n2^2t+

"nn^nt
i>nt

Since the coefficient matrix ^ is singular, the system in the unknowns h^t,^'2t

has solutions other

than the trivial solution.

Similarly,

AB^

= 0,

AB^

= 0,

...

have solutions, each being a column of S.

SUPPLEMENTARY PROBLEMS
10.

Find

all solutions of:


x-^

x^ +

= = =
=

4
3 5

{a)

Xj_

2^:5

+ *3

3*'4

(c)

2%i + 5%2
x-^
-^

1 x^

~ 2%3 7^3
->=3

Xj^

+ %2 +

+ %4

X^ +

Xq +
3C2

Xg
2 Xg

= =

4
(d)

^1 + %2 +

-^S

%
:>:4

= = =

4 -4
2

(b)

\2x^

+ 5

X\
X-i

-^

Xq

x^ +
Xr,

Xq +

+ x^

/4ns.

(a) (b)

! = xi =
=

(d) Xi

1 + 2a -7a/3 + % = 1,

+ 3c,
x:2

ajj

= o, X3 = b, x^ =

17/3,
:;3

4a/3 = 2

5/3, Xg

11.

Find

all non-trivial solutions of:


i

x-i

2x^

*1 + 2*2

"*'

3*3

+ 3x3
(c)

(a)

<

2*1 +
3*i

X2 + 3*3
*3

12*1

+ 5*2 + 6*3

+ 2*2 +

2x-i
(.h)

4*1
X2 + ^x^

+ 2*3 +

2*1

3*1 + 22 +
*i

*3

- 4*2 + 5*3
0,

*3 ^ *47*2 4*3 5*4


3

*2

2*1

11*5

1*3

5*4

/4ns.

(o)

*i = -3o, X2 =

*g = a

(6) *i = (d)

*2
5

*3
J.

= a

3,

*i

*o =

*4

CHAP.

10]

LINEAR EQUATIONS

83

12.

Reconcile the solution of 10(d) with another


2 4
6_

x-i

= c, x^ = d, x^ =

^c -
O

x^ =

c +d.
o
o

13. Given

A =

2 3

2
3

find a matrix

of rank 2

such that

AB

= 0.

Hint.

Select the columns of

B from

the solutions of

AX

= 0.

14.

Show that a square matrix


Let

is singular if

and only

if its

rows (columns) are linearly dependent.

15.

AX

that any non-zero vector of cofactors

be a system of n homogeneous equations in n unknowns and suppose A of rank r = n-1. [a^i, a^j OLinV of a row of ^ is a solution of AX = 0.

Show

16.

Use Problem
/
^1

15 to solve:

- 2%2 +

ixg 3%

= =
(b)

/ 2xj_

+ 2x^

Xg
(c)

\2x-^ + 5%2 + 6*3 Xg


Hint.

y'ix^

4%2

2.^1

+ 3*2 + 4

= =

+ 2%3
=

1 2xi

%2 + 6 Xg

To

the equations of (o) adjoin


"l

Oxi + Ox^ + 0%3

and find the cofactors of the elements of the

-2
5

a""

third

row of

Ans.

(a) xi =

-27a. X2

= 0, X3 =

9a

or

[3a,

0,

-a]',

(6) [2a,

-7a, -17a]',

(c)

[lla, -2a, -4a]'

17.

AX =H

Let the coefficient and the augmented matrix of the system of 3 non-homogeneous equations in 5 unknowns be of rank 2 and assume the canonical form of the augmented matrix to be
1 1

613
623

614 624

fcj^g

ci

625

C2

_0

with not both of


tion of

ci, C2

equal to

0.

First choose
1,

AX

= H.

Then choose

X3 =

tain other solutions X^,Xg, and X^.

and obtain X^ = [c^, c^, 0, 0, o]' as a solualso X3 = x^ = 0, x^ = 1 and X3 = x^ = 0, Xg == 1 to obshow that these 5-2 + 1=4 solutions are linearly independent.
x^ = xg =
0,

X3 = x^ = x^ =

18. Consider

the linear combination

s,X^ + s^X^ + s^X^ + s^X^


if

of the solutions

that

is a solution of
i^

AX

of

Problem 17

Show

if

and only

(i)

s, +.2 +-3 +^4 =

1-

Thus, with s

for (O,

IS a

complete solution of
Hint. Follow

AX

s,. .3,

.4 arbitrary except

= H.

19.

Prove: Theorem VI. Prove:


Hint.

Problem 17 with c^ = c^
r^

= 0.

20.

If ^ is an m xp Use Theorem VI.

matrix of rank

and S

is a p

matrix of rank

r^

such that

AB

= 0, then

r,

r^

< ^-

n f

21.

.1 = [a^^-] of rank 2, verify: In an x matrix A of rank .. the r-square determinants formed from the columns of a submatrix consisting of any r rows of A are proportional to the r-square determinants formed from any other submatrix consisting r

Using the 4x5 matrix

Hint.
(7 =

Suppose the
1.2
5).

of rows of A two rows are linearly independent so that a^j = p^.a^j +Ps^a,.. a^.-p^a^,f^ ij +p42a2,ha2 2j. j Evaluate the 2-square determinants j
first
I

"17

ai5

a^q

025

\'^3q

agg

and
"4(7

"AS

22. Write a proof of the theorem of Problem 21.

^'"
f?ctors''hcJ?'"

'
where (h,i,j,h =

'''''"''

''

*'' ""''"'''' '""'"'

'" ' '^"' ""'


(t)

*^'" "'' '''^'"^ ^^1^""^ ^"-""S


=

"^

(a)
1,

a^^-a^^

a^J^(x^,

a^^aJJ

a^jUji

n).

84

LINEAR EQUATIONS

[CHAP. 10

24.

Show

that

11114 123-4 21126


2

32-1-13

IS

row equivalent to

-,,.

122-2
2 3

01000 00100 00010 00001

10

From B =[A H]

infer that the

-3

ij

[_0

system

of 6 linear equations in 4

m>n

linear equations in n
ra

unknowns has 5 linearly independent equations. Show that a system of unknowns can have at most re +1 linearly independent equations. Show that when

there are

the system is inconsistent.

25. If

AX =H

is consistent

and of rank

r,

for

what set of

variables can one solve?

equations in n unknowns with coeffi26. Generalize the results of Problems 17 and 18 to m non-homogeneous and the augmented matrix of the coefficient If the r to prove: cient and augmented matrix of the same rank Xn-r-n are rank r and if X-^^.X^ have unknowns = in n equations non-homogeneous AX H of m

system

linearly independent solutions of the system, then

X
n-r+i
where

si^i + 51^2 +

^n-r+i^n-r+i

S
i=i

1,

is

a complete solution.

27. In a four-pole electrical network, the imput quantities

i and
a
c

h
h

are given in terms of the output quantities

2 and

Iq

by

E^

oEq +

6/2

"11 _/lJ

'eI

'e2

h
Show
that

cE^ + dlQ

d
'b

h.

>-

and
d
I^,

A
c

Solve also for 2 and

and

I^,

and E^

28.

Let the system of n linear equations in n unknowns 4X = H, H ^ system AX = K has a unique solution for any n-vector K ^ 0.
'l

0,

have a unique solution.

Show that the

-1
1

1"

'xi

"n"
=

29. Solve the set of linear forms

AX
X

2
1

3 3

X2
_s_

72

for the x^ as linear forms in the y's.

p.
= E^, =
n).

Now
30. Let

write

down the solution

of A'

Y
(i

A be n-square and non-singular, and let S^ be the solution of AX n-vector whose ith component is 1 and whose other components are 0.
Let 4 be an m X
matrix with

1,

where ^is the


S^].

Identify the matrix [S^, Sg

31.

71

m<n
1

m-vector whose ith component is

and let S^ be a solution of AX - E^, {i -1,2,. and whose other components are 0. If K = \k^. k^.
k^Si + A:2S2
+ +

m).
,

where E^ is the
,

k^' show

that

^n^%

is a solution of

AX

= K.

Chapter
Vector Spaces

11

UNLESS STATED OTHERWISE,


played,

all

vectors will
a^]'.

we

shall write [xj^.x^

now be column vectors. When components are disThe transpose mark (') indicates that the elements are

to be written in a column.

A
them
if

set of such 7z-vectors over

is a vector of the set.

F is said to be closed under addition if the sum of any two of Similarly, the set is said to be closed under scalar multiplication

every scalar multiple of a vector of the set is a vector of the set.


1. (a)

Example

all vectors [x^. x^. x^Y of ordinary space havinr equal components (x-^ = x^^ x^) closed under both addition and scalar multiplication. For, the sum of any two of the vectors and k times any vector (k real) are again vectors having equal components.
is

The set of

(6)

The set

of all vectors

[x^.x^.x^Y of ordinary space

is

closed under addition and scalar

multiplication.

VECTOR SPACES. Any

set of re-vectors over

F which
if

is

plication is called a vector space. linear combinations

Thus,

X^, X^

closed under both addition and scalar multiX^ are n-vectors over F, the set of all

<"!)
is a vector

K^i
space over F.

KX^

KX^

(kiinF)

For example, both of the sets of vectors (a) and (b) of Example 1 are Clearly, every vector space (ll.l) contains the zero re-vector while the zero re-vector alone is a vector space. (The space (11.1) is also called a linear vector space.)
vector spaces.

The

totality V^iF) of all re-vectors over

is called the re-dimensional vector

space over

F.

SUBSPACES. A

set V of the vectors of V^(F) is called a subspace of V^(F) provided V is closed under addition and scalar multiplication. Thus, the zero re-vector is a subspace of F(F)- so also IS V^(F) Itself. The set (a) of Example 1 is a subspace (a line) of ordinary space. In general If X^, X^, ..., Z^ belong to V^(F), the space of all linear combinations (ll.l) is a subspace of

vector space V
lie in

is

vided (a) the Xi vectors X^, X^

V and

said to be spanned or generated by the re-vectors (b) every vector of F is a linear combination

X^,

X^

(11.1).

X^ proNote that the

X^_

are not restricted to be linearly independent.

Examples. Let F be the field R of real numbers so that the 3-vectors X^ ^3= [1.3,2]' and X^= [3,2,1]' lie in ordinary space S =
s can be expressed as

V^R).

[i.i.lY X^ Any vector


.

\i
.
,

2 i]'

\a b -

cY
j

ot

85

86

VECTOR SPACES

[CHAP. 11

yi +
Ti ^1

y2 +

ys

3y4.
274.

+ 72^2 + ys'^s +

y4-^4

yi

+ 2X2 + Sys +
3y2 +
2y3

yi +

Ja

since the resulting system of equations

yi +
(i)

y2

ys

3x4

yi +
yi

2y2 +
3y2

3ys + 2y4
2y3
y4-

is consistent.

Thus, the vectors

Xj^,

Xj. Xg,

X4 spanS.
tt)

and ^2 are linearly independent. They span a subspace (the plane real numbers. S which contains every vector hX^ + kX.^. where /i and k are

The vectors

X-y

of

The vector X^ spans


A is a real number.

hX^. where a subspace (the line L) of S which contains every vector

See Problem

1.

BASIS

maximum number of linthe dimension of a vector space V is meant the number of linearly inminimum same thing, the early independent vectors in F or, what is the is considered as space ordinary geometry, dependent vectors required to span V. In elementary it as a considering been have we Here (a, 6, c). a 3-space (space of dimension three) of points is of line L the and is of dimension 2 3-space of vectors ia,h,c \. The plane n of Example 2

AND DIMENSION. By

dimension

1.
r

A
we

vector space of dimension

consisting of 7i-vectors will be denoted by F(F).

When

n,

shall agree to write p;(F) for %\F).

basis of the space. Each veclinearly independent vectors of V^(F) is called a of this basis. All bases of vectors the combination of tor of the space is then a unique linear independent vectors of the linearly r V^(F) have exactly the same number of vectors but any

set of

space will serve as a basis.


Example
3.

The vectors

X^.

Xr,.

X^

of

Example

span S since any vector [a,

b. c ]'

of S can be

expressed

yi
X-y 71-^1

+ +

ys
2y2 3X2
a
h c
,

+ + +

ys 3ys
2ys

+ j^Xq + yg^a

yi

yi +

!yi xi
The
resulting system of equations

y2 +

ys =

,^ + 3X3 = yi + 2y2

unlike the system

i),

has a u-

Xi+
nique solution. basis of S
.

3X2 + 2xs =

are not a The vectors X^.X^.X^ are a basis of S. The vectors X^.X^.X^ X^. X^ set the basis is whose tt Example 2, of (Show this.) They span the subspace

Theorems I-V
stated as:
I

of

Chapter 9 apply here, of course.


are a set of re-vectors over

In

particular,

Theorem IV may be

re-

If

X X^

..

X^

of their components,' then from the set r the remaining m-T vectors lie. T vectors span a V^iF) in which

the rank of the raa:ro matrix Unearly independent vectors may be selected. These

F and

if r is

See Problems

2-3.

CHAP.

11]

VECTOR SPACES

87

Of considerable importance are


II.

If

JYi,

^2,

...,

Zm

are

m<n
X^,

linearly independent n-vectors of

V^iF) and

if

J^+j^,

^m4.2. .

^n

are any n-m vectors of V^iF)

which together with

X.^,

X^

X^ form a linearly See Problem


4.

independent set, then the set

X^

is a basis of V^iF).

III.

If

Z^.Zg,...,! are m<7i linearly independent -vectors over F, then the p vectors m
^i

i^'ij^i
if

(/=l-2

P)

are linearly dependent

if

p>m

or,

when p<m,

[s^,] is of rank r<p.

IV. If

Zi,

.2, ...,

are linearly independent re-vectors over F, then the vectors

n
Yi

=
7

1
=
1

a^jXj
"

(i

= 1,2

re)

are linearly independent if and only

if

[a^-] is nonsingular.

IDENTICAL SUBSPACES.
only
is
if

If

,V^(F) and

X(F)

are two

each vector of X(F) a subspace of the other.

is a vector of ^V;[(F)

subspaces of F(F), they are identical if and and conversely, that is, if and only if each
See Problem
5.

SUM AND INTERSECTION OF TWO SPACES.


sum
is

meant the totality of vectors

X+Y

Let V\F) and f3^) be two vector spaces. By their where X is in V^(F) and Y is in V^iF). Clearly, this

is a vector space; we call it the sum space V^^iF). The dimension s of the sum space of two vector spaces does not exceed the sum of their dimensions.

the intersection of the two vector spaces is meant the totality of vectors common to the Now if Z is a vector common to the two spaces, so also is aX; likewise if X and y are common to the two spaces so also is aX^bY. Thus, the intersection of two spaces is a vector space; we call it the intersection space V\F). The dimension of the intersection space of two vector spaces cannot exceed the smaller of the dimensions of the two spaces.

By

two spaces.

V. If two vector spaces FV) and V^(F) have V^\F) as section space, then h + k = s + t.

sum space and V^(F) as

inter-

Example 4. Consider the subspace 7f, spanned by X^ and X^ of Example 2 and the subspace tt^ spanned by Xs and X^. Since rr^ and tt^ are not identical (prove this) and since the four vectors span S, the sum space of tt^ and tt^ is S.

Now 4X1 - X2 = X^: thus, X^ lies in both tt^ and 7t^. The subspace (line L) spanned by X^ IS then the intersection space of 77^ and 77^ Note that 77^ and 77^ are each of dimension 2, S IS of dimension 3, and L is of dimension 1. This agrees with Theorem V.
.

See Problems 6-8.

NUIXITY OF A MATRIX.
,

For a system of homogeneous equations AX = 0, the solution vectors X constitute a vector space called the null space of A. The dimension of this space, denoted by ^ A'^ IS called the nullity of A.
Restating Theorem VI, Chapter
VI.
If

10,

we have

A has

nullity N^

then

AX

has N^ linearly independent solutions X^. X^

88

VECTOR SPACES

[CHAP. 11

Xti A

such that every solution of

AX

is a linear

combination of them and every such

linear combination is a solution.

basis for the null space of A is any set of N^ linearly independent solutions of

AX

0.
9.

See Problem

Vn. For an
(11.2)

mxre matrix

of rank rj
rA

and nullity
Nji

N/^,

SYLVESTER'S LAWS OF NULLITY.


rank and nullity of their product

If

A and B are

of order

ra

and respective ranks q and

rg

the

AB

satisfy the inequalities


''AB

>

'^

'B

- "

(11.3)

Nab > Na

Nab > Nb
Na + Nb

Nab

<

See Problem

10.

BASES AND COORDINATES. The


El = [1,0,0

ra-vectors
0]',

0]',

E^ = [0,1,0

En = [0,0,0

1]'

are called elementaiy or unit vectors over F. The elementary vector Ej, whose /th component The elementary vectors E^, E^ constitute an is 1, is called the /th elementary vector.

important basis for f^^Cf).

Every vector

[%,%
n
1=1

uniquely as the ^nl' of 1^(F) can be expressed

sum

X
Of the elementary vectors.

xiEi

XxE-^

2-^2

+
are

'^nEr,

The components %,

x^

x^ oi

X X

relative to the E-basis. Hereafter, unless otherwise specified,


is

now called the coordinates of we shall assume that a vector


%,
03

given relative to this basis.

Let Zi, Zg in F such that

Zn be another basis of 1^(F).


n

Then there

exist unique scalars

a^

X
i

1
=i

aiZi

a.^Z.^

02

Zg +

dj^Zj^

These scalars
JY^

01,05

= [oi, og

o are called the coordinates of a^Y, we have

relative to the Z-basis.

Writing

(11.4)

X
is the matrix

[Zi.Zg

Zn]Xz

=
Zi,

Z-Xz
Zg
Z.
(F) and

where Z

whose columns are the basis vectors


Z^ = [l,
2,

Examples.

If

Zi =

[2, -1, 3]',

-1

]',

Z3

= [l, -1,

-1

]'

is

a basis of

Fg

Xz

[1.2.3]'

is a vector of Vg(F) relative to that basis, then

r
2
1

"1

[Zi, Zg, Zg]A:^

-1
3

-1

[7,0,-2]'

-1 -1

-2
See Problem 11.

relative to the -basis.

CHAP.

11]

VECTOR SPACES

89

Let
(11.5)

W^,

be yet another basis of f^(F).

Suppose
-r

Xig

= [61,^2

K\' so

that

X
(11.4) and (11.5),

= =

\_\,\

WX^

Xy

From
(11.6)

JV^

IT

Zj^

and

X^
IF"^Z.

^'^.Z-X^

PX,

where P =
Thus,

VIII.

If

a vector of f^(F) has coordinates Z^ and X^ respectively relative to two bases

of P^(F), then there exists a non-singular matrix

P determined
,

solely by the two bases and

given by (11.6) such that

Xf/

= PX^

See Problem

12.

SOLVED PROBLEMS
1.

The

set of all vectors

A'

= [%,

x^, Xg,

x^Y,

where x^ + x^ + x^ + x^ = Q
vectors of the set.

is

a subspace V of V^(F)

since the sum of any two vectors of the set and any scalar multiple of a vector of the set have

components whose sum

is zero, that is, are

2
2.

4
is of rank 2, the vectors
X.^

Since
2
1

= [1,2,2,

1 ]',

X^ = [3,4,4,3]', and X^ = [1,0,0,

1 ]'

4
3
1

are linearly dependent and span a vector space

(F).
X-^

Now any two of these vectors are linearly independent; hence, we may take and Xg as a basis of the V2{F).

and X^, X^ and

A:g.

or

X^

3.

Since

14 13 12 12
2

is of rank 2, the vectors

^"1

= [1,1,1,0]',

A's

= [4,3,2,-1

]',

A'g

= [2,1,0,-1]',

-1 -1 -2
and
^"4

= [4,2,0,-2]' are linearly dependent and span a V^(F).

For a basis, we may take any two of the vectors except the pair Xg, X^.

4.

The vectors

X^, X^, Xg of

Problem

2 lie in V^(F).

Find a basis.

For a basis of this space we may take X^,X^,X^ = [l.O.O.O]', and Xg = [o, 1,0,0]' or X^.X2.Xg = since the matrices [X^, X2, X^.Xg] and [X^. X2.Xg. Xy] are of rank [1.2,3.4]'. and X; = [1,3,6,8]'
4.

90

VECTOR SPACES

[CHAP. 11

5.

Let Zi = [l,2,l]', Z2 = [l,2,3]', A:3 = [3,6,5 ]', Y^ = [0,Q,lY, i'2=[l.2,5]' be vectors of Show that the space spanned by X-^, X^, Xq and the space spanned by Y.^, Y^ are identical.
First,

Vs(F).

we

note that X^ and X^ are linearly independent while


iF|(''').

Xq

2Zj^

X^..

Thus, the X^ span a space

of dimension two. say

Also, the Yi being linearly independent span a space of dimension two. say

2lg^(F)
(c

Thus, any vector iX.^, Y^ = 2^2- ^li ^i = ^i^ - 4^1. X^ = Y^- 271. (50 + 26)^2 - (2a + b)X.^ of iPg^CF) and any vector cXj^ + dX^ of iff(f) + d)Y2 - (4c + 2<i)Ki of QVi(F). Hence, the two spaces are identical.
Next,
Fi =

kXz -

aY-^

fcK,

of

is

a vector

is a vector

6. (a) If

= [%, 2.%]' lies in the Vg(F) spanned by

X^ =

1,-1,1]' and X^ = [3,4,-2]',

then

-2% +
1

5X2 + IXg

(b) If

= [xi,x^,X3,x^Y lies in the Pi(F) spanned by X^ = [1,1,2,3]' and X^ =


1
1

1,0,-2,1]',

then

3Ci

1
1 1

%
is of

%
%4.

2 3
1
1

-2
1
1

rank

2.

Since
1

0,

this requires

= - 2*1 + 4% -

and

xi

%
4.

=
1

%+

2*2 -

s;*

These problems
linearly independent

verify: Every ^(^ ) may be defined as the totality of solutions over F of a system homogeneous linear equations over F in n unknowns.

of n-k

7.

two vector spaces Vn(F) and I^(F) have Vn(F) as sum space and V^iF) as intersection space, then h + k = s + t.
Prove:
If

Suppose
s

= h; then Vr!XF) is a

subspace of (^^F) and their sum space


that the

is 1^' itself.

Thus, s=k.

=h

and

h+k. The reader

will

show

same

is true if

= k.

Suppose next that t<h. t<k and let X^^.X^ Yh so that X1.X2 ^t.i't-n A:i,A:2 ^t.Zt+1 2fe span I^'^F).
^t^-i, yt-+2

X^ span
Yh

V^(F).

span l^(F) and vectors

Then by Theorem H there exist vectors Z^ so that Zj^^. Z^+j

Now suppose
(11-4)

there exist scalars a's and 6's such that


t

X 1=1
t

"iXi

.S
t=t+i

a^Yi

S
i=t+i

biZi

ft

i=i h

i=t+i

i=t+i
k

to

The vector on the V^(F). But X^.X^

left

belongs to P^(F),and from the right member, belongs also to ^(F); thus

it

belongs

X-t

span Vn(Fy, hence,


t

a^+i = at+2 =
k
2;

"f,

= 0.

Now

from (11.4),

2
i=i

"iXi

b^Z^
''t^i =
s t*^"^, *t+2 = = ^fe = as was to be proved.
:

i=t+i

But the X's and Z's are linearly independent so that a^ = 05 = = t = the ^f's.y-s, and Z's are a linearly independent set and span ^(F). Then

=h + k-t

CHAP.

11]

VECTOR SPACES

91

8.

Consider ^FsCF) having X^ = [l,2,2Y and Z2 =

1,1,1

]'

as basis and jFgCF) having 71 = [3,1,2]'

and

Fj

1.0,1

]'

as basis. Since the matrix of the components

113 2 110
1

is of rank 3, the

sum

12

space

is V,(F).

As a basis, we may take


= s

X-^,

X^,

and

Y^

Prom

h + k

t.

the intersection space is a VsiF).


2 2

To

find a basis,

we equate

linear combinations

of the vectors of the bases of iFg(F) and s^'sCF) as

- 3e
c

take d =

for

convenience, and solve

^2a
( 3a

66

= =
1

obtaining

a = 1/3,

6 = -4/3,

c =

-2/3.

Then

- 2e

aX;]^

+ 6^2 = [-1.-2/3.-1/3

]' is

a basis of the intersection space.

The vector [3,2,1]'

is also a basis.

113
9.

Determine a basis

for the null

space of A

2
2

4
1

10

113
Consider the system of equations

AX

x^ =
=

which reduces

to

\x2+
A
basis for the null space of
.4

Xs + 2x^ =

is the pair of linearly

independent solutions [1.2,0,-1]' and [2,l,-l,o]'

of these equations.

10. Prove:

r >
AB

r A

- n.

Suppose

first that

A has the form

Then the
10,

first

r,

rows of

AB

are the first

r.

rows of B while
"

the remaining rows are zeros.

By Problem

Chapters, the rank of

AB

is

^45

>

'k

+ %

PAQ

Suppose next that A is not of the above form. Then there exist nonsingular matrices P and Q such that has that form while the rank of PAQB is exactly that of AB (why?).

The reader may consider

the special

case when

11.

Let X=[l,2.lY relative to the S-basis. Z2 = [1,0,1]', and Zg = [1,1. l]'.


Solution
(a).

Find

its

coordinates relative to a new basis Zi =

[l 1 0]'

Write
1 1

!a
Then
a

+ i + c = +
b
c c

(i)

aZ^ + 6Z2 +

cZr^.

that is.

2
1

+ h
1

= =

2
1

and a =

0,

6 =

1,

c = 2.

Thus

relative to the Z-basis,

we have X^

[0,-1,2]'

92

VECTOR SPACES

[CHAP.

11

Solution (6).

Rewriting

(i)

as

{Z-^.Z^.Z^^X^ = ZXg,
1

we have
1

-1

X^

Z X

1 1

-1
1 1
.

2
1 _ _

[0,-1,2^

12.

Let X^ and

X-^

Z2=[l,0,l]', trix ? such that X^ = PX^

be the coordinates of a vector Z3= [1,1,1]' and f4 = [l,l,2]',


.

X
ff^

with respect to the two bases = [2,2,1


]',
Ifg

7,^

= [1,1,0]'

= [1,2,2

]'.

Determine the ma-

1 1

'l

2 2
1

l'

Here

\_Z^, Z^,

Zg]

2 2

and

2-3 2 0-1 2
-3

-1

4
1

1 1

Then

W'^Z

by (11.6).

0-3

SUPPLEMENTARY PROBLEMS
13.

Let

[x-i^.

x^. x^.

X4,y

be an arbitrary vector of Vi(R). where

R denotes

the field of real numbers.

Which of the

following sets are subspaces of K^(R)'?


(a)

All vectors with Xj_= X2 = X3 = x^.

(d) All vectors with


(e) All vectors with

x-^

(6) All vectors


(c) All

with x^ = x^.
0.

x^=2x^.

x^.Xr,.Xs.xj^

integral.

vectors with %4 =

Ans. All except {d) and

(e).

14.

Show

that

1.1.1.1

]'

and [2,3.3,2]' are a basis of the

fi^(F) of

Problem

2.

15.

Determine the dimension of the vector space spanned by each set of vectors.
[1.1,1.1]'
[1,2,3.4.5]'
(a)

Select a basis for each.

[l.l.O.-l]'
(b)

[3,4,5,6]'
,

[5.4,3,2,1]',

[1,2,3,4]'

^'^

[1.2,3,4]'

[1.1,1.1,1]'

[2.3.3,3]'

[1,0,-1,-2]'

Ans.

(a), (b), {c).

r= 2

16. (a)

(6)

Show that the vectors X-^ = [l,-l,l]' and X^ = [3,4,-2]' span the same space as Y^ = [9,5,-1 ]' and 72= [-17,-11.3]'. Show that the vectors X^ = [ 1,-1,1 ]' and A'2 = [3,4,-2 ]' do not span the same space as Ti = [-2,2,-2]'
and K, = [4,3,1]'.

n. Show

that if the set

X^.X^

Xfe is a basis lor Vn(F). then


X-^,

any other vector Y of the space can be repre-

sented uniquely as a linear combination of


k

X^

X^

Hint.

Assume

51

aiXi

biXi-

CHAP.

11]

VECTOR SPACES

93

18.

Consider the

4x4

the \i(R) of Problem

matrix whose columns are the vectors of a basis of the Vi(R) of Problem 2 and a basis of 3. Show that the rank of this matrix is 4; hence. V^R) is the sum space and l^(R), the

zero space, is the intersection space of the two given spaces.

19.

Follow the proof given

in

Problem

8,

Chapter 10, to prove Theorem HI.

20.

Show

that the space spanned by [l,0,0,0,o]', [0,0,0,0,1 ]', [l.O,l,0,0]', [0,0,1,0,0]' [l,0,0,l,l]' and the space spanned by [l,0.0.0,l]', [0,1,0,1,0 ]', [o,l,-2,l,o]', [l,0,-l,0,l ]', [o,l,l,l,o]' are of dimensions 4 and 3, respectively. Show that [l,0,l,0,l]' and [l,0,2,0,l]' are a basis for the intersection space.

21. Find, relative to the basis


(a)

Z^=
(c)

[l,1.2]', Zg = [2.2,l]', Zg = [l,2,2]' the coordinates of the vectors

[l.l.o]',
(a)

(b)

[1,0, l]',

[l.l.l]'.

Ans.

[-1/3,2/3,0]',

(6) [4/3, 1/3,

-1

]',

(c)

[l/3, 1/3,

]'

22. Find, relative to the basis


(a) [2,-1,0]',

Zi=[o,l,o]', Z2=[i,l,l]', Z3=[3,2,l]' the coordinates of the vectors


(c) [0,0,l]'. (c)

(b) [1,-3,5]', (6)

Ans.

(a) [-2,-1,1]',

[-6,7,-2]',

[-1/2, 3/2, -1/2

]'

23.

Let X^ and
trix

X^^

be the coordinates of a vector

with respect to the given pair of bases.

Determine the

P such

that Xj^ =

PX^

Zi=
ea)
1^1

[1,0,0]',

Z2=[i,o,l]',
[^2= [1,2,3]',
2

Z3=
If'3=

[1,1, il^^^

Zi = [0,1,0]',
!fi

2^

= [1.1,0]',

23

= [1.2.3]'

= [0,1,0]',

[1,-1,1]'

= [l,1.0]'.

1^2= [1,1.1]',

1^3= [1,2.1]'

41
,

r
(6)

-2"1
2

Ans.

(a)

^
2

-1

2j

ij

24. Prove: If Pj
[''1.^2

is a solution of

AX

n
=

Ej

(j = 1,2

n).

then

hjPj

is a

solution of

AX

= H.

where H =

KVH
=

Hint.

h^Ej^ +

h^E^+

+hnE^.

25.

The vector space defined by all linear combinations of the columns of a matrix A is called the column space of A. The vector space defined by all linear combinations of the rows of A is called the row space of ^. Show that the columns of AB are in the column space of A and the rows of AB are in
the row space of
fi.

26.

Show

that

the vector

AX = H a system of m non-homogeneous H belongs to the column space of A


.

equations in n unknowns, is consistent

if

and only

if

the

1111
(6)

27. Determine a basis for the null

space of

(a)
1

1-1,
1

12 12
3 4 3

Ans.

(a)

[1,-1,-1]',

(6)

1,1,-1, -i

]',

[l, 2,-1, -2]'

28. Prove:

(a)

N^,>N^. N^^>N^
= n

(b)N^,<N^^N^
r^

Hint:

(a) /V^g (b)

r^g

r^g

<

and

rg

Consider n~r^g

using the theorem of Problem 10.

29. Derive a procedure for

Problem 16 using only column transformations on A = [X^. X^, y^ Y^].

Then resolve

Problem

5.

chapter 12
Linear Transformations

DEFINITION. Let

[x,., x^, ....

%]' and Y =

lyx. y^.

JnY ^^ ^^ vectors

of l^(F), their co-

ordinates being relative to the same basis of the space.

Suppose that the coordinates of

are related by
yi
'^ll'^l
"^

^12 "^2

"^

^^Yi J^

+
(12.1)

df^Yj.^ri

or, briefly,

AX
[a^.-]

where A =

is over F.

V^(F) into (usually) another vector


If

Then (12.1) is a transformation T which carries any vector X Y of the same space, called its image.

of

(12.1) carries X^ into F^ and X^ into Y^, then

(a)
(fe)

it

carries kX^ into ^y^, for every scalar k, and


carries aX^ + bX^ into aY^ +
feFg. for

it

every pair of scalars a and

b.

For this reason, the

transformation is called linear.

2 5 3
'12"
A"

Example

1.

Consider the linear transformation Y =

AX

in ordinary

space Vq(R).

13
'l
1

2 5 3

(a)

The image

of

A"

= [2,0,5]' is

1 1

2 3

= 5

27 17

[12,27.17]'.

'l

'x{
*2
=

'2'

(6)

The vector X whose image

is

= [2,0.5]' is obtained

by solving

1
.1

2 3

5
3_

3.

112
Since

12 13

5
3 5

10 10
1

13/5
11/5
.

[13/5,11/5,-7/5]'.

-7/5

BASIC THEOREMS.
if

If in (12.1),

= [\,Q
"nf]'-

then
I.

Y =

[a^j.a^j

0]'='i then Hence,

Y =

an,

Ogi,

...,

a^J'

and, in general,

linear transformation (12.1) is uniquely determined

when the images (Y's)

of the

basis vectors are known, the respective columns of A being the coordinates of the images
of these vectors.

See Problem

l.

94

CHAP.

12]

LINEAR TRANSFORMATIONS

95

linear transformation (12-1) is called non-singular

are distinct vectors Y^.


II.

if the images of distinct vectors Xi Otherwise the transformation is called singular.

A A

linear transformation (12.1) is non-singular if and only if A, the matrix of the

transformation, is non-singular.
in.

See Problem

2.

non-singular linear transformation carries linearly independent (dependent) vec-

tors into linearly independent (dependent) vectors.

See Problem

3.

Prom Theorem HI follows


IV. Under a non-singular transformation (12.1) the image of a vector space Vn(F) is a vector space VjJ,F), that is, the dimension of the vector space is preserved. In particular, the transformation is a mapping of ^(F) onto itself.
k

When A

is non-singular, the inverse of (12.1)

A'^y

carries the set of vectors Y^, Y^, ...,\ whose components are the columns of A into the basis vectors of the space. It is also a linear transformation.

V. The elementary vectors ^ of \{F) may be transformed into any set of n linearly independent n-vectors by a non-singular linear transformation and conversely.
VI. If
IT

CZ

carries

Y = AX carries a vector X into a vector F, if Z = BY carries Y into Z, and if Z into W, then Z = BY = {BA)X carries X into Z and IF = (CBA\X carries

into

IF.

VII. When any two sets of re linearly independent re-vectors are given, there exists a non-singular linear transformation which carries the vectors of one set into the vectors of

the other.

CHANGE OF

BASIS. Relative to a Z-basis, let 7^ = AX^, be a linear transformation of ^(F). Suppose changed and let X^ and Y^ be the coordinates of X^, and Y^ respectively relative to the new basis. By Theorem VIH, Chapter 11, there exists a non-singular matrix P such that X-^ = ?X^ and Yy, = PY^ or, setting ?~^ = Q, such that
that the basis is

X^

QX^
=

and

Y^

QY^
=

Then
where
(12.2)

Y^

Q-%

Q-^X,
6
=

Q-^AQX^

BX^

Q-^AQ

Two

are called similar.

matrices A and B such that there exists a non-singular matrix We have proved

for

which B = Q'^AQ

vm. If Y^ = AX^ is a linear transformation of V^(F) relative to a given basis (Z-basis) and Yjf = BX^ is the same linear transformation relative to another basis (IF-basis), then A and B are similar.
Note. Since

P"^, (12.2)
later.

will be
for

made

might have been written as B = PAP-^. A study of similar matrices There we shall agree to write B = R'^AR instead of S = SAS'^ but

no compelling reason.

96

LINEAR TRANSFORMATIONS

[CHAP. 12

3
1

Example

2.

Let

AX

1
1

2 3

be a linear transformation relative to the -basis and

let

W^

2
IFg

[l.2,l]', W^ = [1,-1.2]',

[1,-1,-1]' be a new basis,


to the W-basis.

find the coordinates of its


Yjr

image relative

= BXjf

corresponding to

AX.
1

(c)

Given the vector X = [3,0,2]', Find the linear transformation Use the result of (b) to find the image ijf of Xy =
(a)

(b)

[1.3,3]'.
1 1

Write

[W^.W^.W^] =

2
1

-1 -1
2

then

-1

1
9

-2

-1

-1 -3
Xff =

(a)

Relative to the

If -basis,

the vector

= [3,0,2]' has coordinates

W X
Yf^

= [l,l,l]'. =

The image

of

is

AX

[9,5,7]'

which, relative to the IF-basis is

W Y

[14/3,20/9.19/9]'.
36
(b)

21

-15
-11
-1

w\
36

W^AX

(W~^AW)Xjf = BXj^

21

10

-3 23
21

-15
-11 -1

6
= 2

(c) Yj.

21

10

3
3

[6,2,7]'

-3 23
L.

See Problem

5,

SOLVED PROBLEMS
1. (a)

Set up the linear transformation and 3 into Fg = [2,1,3]'.

Y = AX which

carries E^ into

Y^

= [1,2,3]',

E^ into [3,1,2]',

{h)
(c)

(d)

Find the images of li= [1,1,1]', I2 = [3,-1,4 ]', and ^3 = [4,0,5]'. Show that X^ and Zg ^-^^ linearly independent as also are their images. Show that Xi, X^, and Z3 are linearly dependent as also are their images.
1

3
1

2
1

(a)

By TheoremI, A

= [y^, Fg, K3]

the equation of the linear transformation is

=^

AX

2 3

13
(6)

2
1

The image

of

X^=

[l,l.l]' is

Y-^

[6.4,8]'.

The image

of

-Yg is

Ys = [8,9,19]' and the

image of Xg

is

K3

=[ 14.13,27]'.
1

6
is 2 as also is that of [^i, Kg]

8 9

(c)

The rank

of [A'^.Xg] =

1
_1

-1
4

4
8

Thus, X^ and X^ are linearly

19

independent as also are their images


(rf)

We may compare

the ranks of \_X^. X^. X^] and {Y^.Y^.Yq\; however,

X^

X^ + X^ and Fg = Fi+Zg so that

both sets are linearly dependent.

CHAP.

12]

LINEAR TRANSFORMATIONS

97

2.

Prove:

linear transformation (12.1) is non-singular

if

and only

if

is non-singular.
=

Suppose A is non-singular and the transforms of X^ ^ X^ are Y = AX^ = AX^. Then A{X-i^-Xi) homogeneous linear equations AX = Q has the non-trivial solution X = X-^-X^. This sible if and only if .4| = o, a contradiction of the hypothesis that A is non-singular.
the system of
|

and

is pos-

3.

Prove:

non-singular linear transformation carries linearly independent vectors into linearly in-

dependent vectors.

Assume the contrary, that is, suppose that the images Yi = AXi. (i = 1,2 ent vectors X-^.Xr, Xp are linearly dependent. Then there exist scalars s-^.s^
P

p)
sp

of the linearly independ,

not all zero, such that

^
1=1

H'^i

^lYi +

s^Y2+

s^Yfy ^^
f^

P
'

.|^ ^(-4^1)

A(Sj^X^+ S2X^+

+ spXp)

Since A is non -singular,


linearly independent.

s^X^ + s^X^ + -. + spXp =


Y^ are linearly

o.

But this

is contrary to the

hypothesis that the Xi are

Hence, the

independent.

4.

certain linear transformation

F =

iZ

carries Z^ =

1,0,1

]'

into

[2,3,-1]',
,!
fj^,

1.2,-1]' into [-2,7,-1]'. tion of the transformation.


a + b +
c

[3.0,-2]', and I3

=[

Find the images of

^^s

=[ 1.-1.1

]'

into

and write the equa-

Let

aX.i^

+ bXr,+

cXg= E^;

then

-6
a + b

+ 2c =

and a

-^,

6 = 1,

c =

i.

Thus,

E^= -hX^ + X^

^Xg

c =

and

its

imageis
J

Y,

-^2, 3,-1

]'+ [3,0,-2]' +
is K, = [l,l,l

Y2 = 1-1.3,1

and the image of Eg

]'.

H-2.7.-1 ]' = The equation


1

[l,2,-2]'. Similarly, the image of 5 is of the transformation is


1 1

-1
3
1

[Y^.Y^.Yg]X

-2

2
1

5. If

Yy

= AXf, =

2
3

2
1

X^

is a linear transformation relative to the Z-basis of

Problem

12,

Chap-

ter 11, find the

same transformation

7^ =

BX^

relative to the f'-basis of that problem.


1

4
1

1"

From Problem

12,

Chapter

11,

Xf^r

PX^

^z-

Then

--3

0_

-1
P''X,.,

-1"

-1
2
1
:

^!i

Q^w

d
-2
14 -6 14
9
3

and

PY

Q-^AX,

Q^^Q^r.,

\^

X,

-9

98

LINEAR TRANSFORMATIONS

[CHAP. 12

SUPPLEMENTARY PROBLEMS
6. In

Problem

show:

(a) the transformation is non-singular,

(b)

A Y

carries the column vectors of

into

the elementary vectors.

7.

Using the transformation of Probleml, find (a) the image of [-2.-5.-5]'. -4ns. (a) [8,5.11]', (b) [-3.-1. 2]'
Study the effect of the transformation Y = IX. also Y = klX.

Af

= [1,1,2]',

(h) the vector

X whose

image

is

8.

9.

Set up the linear transformation which carries E^ into [l,2,3]', 5 into [3.1. 2]', and 3 into [2,-1,-1]'. Show that the transformation is singular and carries the linearly independent vectors [ 1,1,1 ]' and [2,0,2]' into the same image vector.

10.

Suppose (12.1) is non-singular and show that ages Y^.Y^ Y^.

if

X-^.X^.

....

X^

are linearly dependent so also are their im-

11.

Use Theorem
changed.

III

to

show

that under a non-singular transformation the dimension of a vector space is un-

Hint. Consider the images of a basis of P^ (F).


1 1

12.

Given the linear transformation Y

X.

show

(a) it is singular,

(6) the

images of the linearly

in-

-2
dependent vectors
of V^{R) is a Vs(R).
1
1

3 5
]',

^i=[l,l,l]',

JVg =

[2.I.2

and A:3=[i.2,3]' are linearly dependent,

(c) the

image

13.

Given the linear transformation Y


2
V,

2 4 X.

show
1

(a) it is singular,

(6) the

image of every vector of the

113
(R) spanned by
[

1.1,1

]'

and [3.2.0

]'

lies in the K,(fl)

spanned by [5,7.5]'.

14.

Prove Theorem Vn.


Hint.

Let Xi and

Yi,

(j

= 1,2

n) be the given sets of vectors. Let


Kj.

AX

carry the set Xi into E{ and

Y
15. Prove:

BZ

carry the E^ into

Similar matrices have equal determinants.

12
16.

3
1
1_

Let

AX
^2

3
_1

2
1

A^

be a linear transformation relative to the -basis and let a

new

basis, say

Z, =

[1,1,0]',
(a)
(6)

= [1.0.1]',

Z3

= [1.1.1]'
A^

be Chosen.

Let

AT

= [1,2,3]' relative to the E-basis.

Show

that

= [14,10,6]' is the

image of

under the transformation.

X. when referred to the

new

basis, has coordinates X^, = [-2,-1.4]' and


1

Y has coordinates

Y^ = [8,4,2]'

0-1"
-1
1

(c)

X^

PX

and Y^ = PY.

where P

iZ^,
1

Zf2,

Z^i

-1
(d)

Yy =

Q ^AQX.

where Q

=P

^.

0"
1 1
1

17.

Given the linear transformation

7^
1

1
1

Xjf

relative to the IF-basis:

W^= [o,-l,2]',

IK,= [4,1,0]'

CHAP.

121

LINEAR TRANSFORMATIONS

99

IK5

[-2.0,-4]'- Find the representation relative to the Z-basis: Z^ = [i,-l,l]', Z2 = [l,0,-l]', Z3=[l,2,l]'.

-10

Am

2-5 -10 2
2 in the linear transformation

18.

If.

AX. A

is singular,

then the null space of A is the vector space each of

whose vectors transforms

into the zero vector.

Determine the null space of the transformation of

123"
(a)

Problem

12.

(6)

Problem

13.

(c)

2 3

4
6

X.

-4ns.

(a)

(R) spanned by [l.-l.l]'

(6) (c)

I^\R) spanned by [2,l,-l]'


I/^()

spanned by [2,-l,o]' and [3,0,-1

]'

19.

If y = AX carries every vector of a vector space I^ into a vector of that same space, v^ is called an Invariant space of the transformation. Show that in the real space V^{R) under the linear transformation
1

-f
\

(a)

12
2
2

X. the

\l^

spanned by [l.-l.o]', the V^ spanned by [2,-1.-2]', and the V^ spanned by

[1.-1,-2]' are invariant vector spaces.


2
(6)

13
1

X. the Vq spanned by [l.l.l]' and the

^ spanned

by [l,0,-l]' and [2,-l,0]' are invariant

spaces.

(Note that every vector of the V^ is carried into itself.)


1

(c)

10
1

X, the li spanned by [l,l,l,l]' is an invariant vector space.

-14-6
20.

Consider the linear transformation


tion of
(a)
1,

PX

yi

(i

n.

^h

1.2

n)

in

which

/1./2

/^ is

a permuta-

Describe the permutation matrix P


Prove: There are n! permutation matrices of order n. Prove:
If If

(b) (c)
(d)

Pj and fg are permutation matrices so also are P3 = P-lP2

and

P^^P^Pt.
number of the elementary
n

Prove:

is a permutation matrix so also are P' and

PP'

/.

(e)

(/)

Show that each permutation matrix P can be expressed as a product umn matrices K^2, ^28 ^n-T-nWrite P = [^^, E^^. ^^] where ii, ij % is a permutation
mentary n-vectors.

of a

col-

of
.

1,2

and ^

are the ele-

= [s, 1, 4,

Find a rule (other than P~ = P') for writing P~ For example, when n = 4 and 2], then P'^ = [2. '4. 1. 3]; when P = [E^ E^. 1, 3], then P~^ = [g, 2, 4, 1].

chapter 13
Vectors Over the Real Field

INNER PRODUCT. In this chapter If Z = [%,%, ..., x^y and y


defined to be the scalar
(13.1)

all

vectors are real and l^(R) is the space of


72, ,

all real re-vectors.

= [ji,

YnY
+

are two vectors of l^(R), their inner product is

X-Y
1.

x^y^

x^j^

XnJn

Example

For the vectors X^=\\.\,\\',


(a) (6) (c)
(rf)

^2=
1-

[2.1,2]',

^3 = [l.-2.l]':

X^-X^
X-^-X^

= = = =

1-2
1-1 1-1

1- 1

= = =
=

+ l(-2) + +
1-1 1-2

+1-1
1-1

X.yX^
^1-2^:2

+ +

1-4

1-4

10

2(^1-^2)

Note. The inner product is frequently defined as


(13.1')

X.Y

X'Y

Y'X

The use of X'Y and Y'X is helpful; however, X'Y and Y'X are 1x1 matrices while X-Y is the element of the matrix. With this understanding, (13.1') will be used In vector analysis, the inner product is callhere. Some authors write Z|y for X-Y
.

ed the dot product.

The following
(a)

rules for inner products are immediate

X^-X^ = X^-X^,

X^-hX^ = HX^-X^)
(X^+Xs)-X^
= =
+

(13.2)

ib)
(c)

X^-(X^ + Xs)
(X^+ X^)
-

X^-X^ + X^-X^

(Xs+ X^)

X^-X^

X^-X^ + X^-Xs + X^-X^

ORTHOGONAL VECTORS. Two


product is
0.

vectors

X and Y
of

of V^iR) are said to be orthogonal if their inner


1

The vectors Z^ and Xg

Example

are orthogonal.

THE LENGTH OF A VECTOR X of thus. ner product of X and X


;

^i(R),

denoted by

\\

X\\

is

defined as the square root of the in-

(13.3)

II

^11

\/

X-X

\/

xl +

xl+

---

X.

Examples. Prom Example

1(c),

\\

X^\\ =

V3

See Problems

1-2.

100

CHAP.

13]

VECTORS OVER THE REAL FIELD

101

Using (13.1) and


(13.4)

(13.3),

it

may be shown
=

that

x.Y
vector

iil|z+y||'
=
1

WxW

- ||y|h
The elementary vectors E^

A
are

X whose

length is

||z||

is called a unit vector.

examples of unit vectors.

THE SCHWARZ INEQUALITY.


(13.5)

If

and Y are vectors of ^(/?), then

\X-Y\

<

\\x\\.\\y\\

that is, the numerical value of the inner product of two real vectors is at most the product of
their lengths.

See Problem

3.

THE TRIANGLE INEQUALITY.


(13-6)

If

and Y are vectors of

)/(/?),

then

l!^+yll

<

IUII + ||y||

ORTHOGONAL VECTORS AND SPACES.


n-vectors and
0.
if

If

X^,
0,

X^

X^
i

are

m<n

CiZi + ^2^2+ ...+ c^^ = Since this requires 0^ = for i = 1,2


I.

then for

= 1,2

m. (c.lX^+ 0^X2+

mutually orthogonal non-zero + c^X^) Xi =

we have

Any

set of

m<

n mutually orthogonal non-zero re-vectors is a linearly independent


Ijf(/?).

set and spans a vector space

vector

is said to be

orthogonal to a vector space Vn(R)

if it is

orthogonal to every

vector of the space.


II.

If

a vector

is

orthogonal to each of the re-vectors X^, X^

X^,

it is

orthogonal

to the space

spanned by them.
See Problem
4.

HI. If Vn(R) is a subspace of I^(/?),

k>h. there exists

at

least one vector

of V^CR)

which

is orthogonal to

V^\R).

See Problem

5.

Since mutually orthogonal vectors are linearly independent, a vector space V'^(R), m>0, can contain no more than m mutually orthogonal vectors. Suppose we have found r<m mutually orthogonal vectors of a V^(R). They span a V^iR), a subspace of V*(R), and by Theorem HI, there exists at least one vector of V^(R) which is orthogonal to the I^(/?). We now have r+l mutually orthogonal vectors of l^(R) and by repeating the argument, we show
IV.

Every vector space V^(R), m>0, contains m but not more than

mutually orthog-

onal vectors.

Two vector spaces are said to be orthogonal if every vector of one is orthogonal to every vector of the other space. For example, the space spanned by X^ = [1,0,0,1]' and X^ = [0,1,1,0]' is orthogonal to the space spanned by X^ = [ 1,0,0,-1]' and X^ = [0,1,-1,0 ]' since (aX^ + bXr,) (0X3+ dX^) = for all a,b,c,d.

102

VECTORS OVER THE REAL FIELD

[CHAP. 13

V.
tor

The set
\

of all vectors orthogonal to every vector of a given Vn,(R) is a unique vecJ

space Vn'^(R). ^ n

ggg Problem

6.

We may associate with any vector ^ 7^ o a unique unit vector U obtained by dividing the components of X by \\X\\ This operation is called normalization. Thus, to normalize the vector X = [2,4,4]', divide each component by ||^|| = V4 + 16 + 16 = 6 and obtain the unit vector
.

[1/3,2/3.2/3]'.

sis of the space;

basis of Vn(R) which consists of mutually orthogonal vectors is called an orthogonal baif the mutually orthogonal vectors are also unit vectors, the basis is called a

normal orthogonal or orthononnal basis.

The elementary vectors

are an orthonormal basis of ^(R).

See Problem

7.

THE GRAM-SCHMIDT ORTHOGONALIZATION PROCESS.


V^(R).

Suppose

X^,

X^

are

a basis of

Define
= X,

y,

^S

Y ^3

^g'^3

^1-^3

Y V

V V

-'w-l
-'m

Xi

'^

%-l

"

ll*^!

y'

"'l

Then the
basis of

unit vectors

Gj =

^
1^11

(i

= l,2,...,m)

are mutually orthogonal and are an orthonormal

F(/?).

Example

3.

Construct, using the Gram-Schmidt process, an orthogonal basis of V2(R). given a basis

A'i= [1,1,1]', a:2= [1,-2,1]',


(i)

Xs=[i.2.zY.

Y^

X^ = [1.1.1]'

(ii)

Y^

X^- ~^Yi
^3 -

[1,-2,1]'

^1-1 =

[1,-2.1]'

(ill)

^3 =

^Y, - ^Y,

[1,2,3]'

- -^y, - ^[1,1.1]'

[-1,0,1]'

The vectors

G^ = -jpj =

[l/\/l. l/\/3, l/^/3l

]',

G2 =

-^,

[l/Ve, -2/V6, l/Ve]'

and

Gg

-ii-

[-I/V2,

0,

1/\A2]'

are an orthonormal basis of ^(fl).


0.

Each vector
X.^

G^ is a unit vector and each product G^

Gj =

Note that Fg =

-^2

here because

and

A^2 a^re

orthogonal vectors.

See Problems 8-9.

CHAP.

13]

VECTORS OVER THE REAL FIELD

103

Let
y^, Yg

Zi,

^2

^m be a basis

of a f^(/?) and suppose that

X^,

X^

Xg,(l< s<

m),

are

mutually orthogonal.
}^

Then, by the Gram-Schmidt process, we may obtain an orthogonal basis of the space of which, it is easy to show, Yj^ = X^, (i = 1,2 Thus, s).
X-i^,

VI. If

X2,

exist unit vectors

, Xs,(l< s<m), X^^^, ^m


X^^.^-

are mutually orthogonal unit vectors of a Vn(R), there


i" the

space such that the set

X^, X^, ...,X^

is an

orthonormal basis.

THE GRAMIAN.

Let X^, X^

Z>,

be a set of real n-vectors and define the Gramian matrix A^ A2


'

A^ A^
' '

A-L

A>) Ajj

A]^

A^ A^ A2

X-^ A,

A2 Aj X2 A2
(13.8)

A2

'

A2 A^ A2 A2

X^Xp XpXp

Xp- X^

Xp- X2

...

Xp- Xp

XpX-i

XpX^

...

Clearly, the vectors are mutually orthogonal if and only if


In

is diagonal.

Problem
VII.

14,

Chapter

17,

we

shall prove
Z^, X^,
....

For a set of real re-vectors

Xp,

G >0. The
|

equality holds

if

and only

if

the vectors are linearly dependent.

ORTHOGONAL MATRICES. A
(13.9) that is, if
(13.9')

square matrix A is called orthogonal

if

AA'

A'A

A'

A'

Prom

(13.9)

it

is clear that the

column vectors (row vectors) of an orthogonal matrix A are

mutually orthogonal unit vectors.

l/\/3

l/x/6

-1/^2
is orthogonal.

Example

4.

By Examples, A

l/\[Z

-2/\/6
\/\[&
l/x/2

l/\/3

There follow readily


VIII.
If

the real re-square matrix

is orthogonal, its

column vectors (row vectors) are

an orthonormal basis of V^(R), and conversely.


IX.

The inverse and The product

the transpose of an orthogonal matrix are orthogonal.

X.
XI.

of two or more orthogonal matrices is orthogonal. of an orthogonal matrix is 1.

The determinant

ORTHOGONAL TRANSFORMATIONS.
(13.10)

Let

AX

104

VECTORS OVER THE REAL FIELD

[CHAP. 13

be a linear transformation in Xi(R) and let the images of the n-vectors I^ and X^ be denoted by Yi and Y^ respectively. Prom (13.4) we have

x^-x,
and

=
=

u\\x,^x,f k\\\Y,^Y,f if

\\X,f
y,
II

\\X,f]

Y,-Y,

Y,f]
it

Comparing

right

and

left

members, we see that


Thus,

(13.10) preserves lengths

preserves inner

products, and conversely.


XII.

linear transformation preserves lengths

if

and only

if it

preserves inner product s.

A
lem
10,

linear transformation

Y=AX

is called

orthogonal

if its

matrix

is orthogonal.

we prove
XIII.

In Prob-

linear transformation preserves lengths

if

and only

if its

matrix is orthogonal.

l/\/2

l/v/6

-I/V2

Examples. The linear transformation Y

AX

l/\/3

-2/\/6
l/\/6

X
l/v^

ii3

orthogonal.

The image of

I/V3

[a,i,c]'is

y
[

"

_^

_1_

_f

26

c "]/

and both vectors are of length yja^ + b^ + c^

XIV. If (13.10) is a transformation of coordinates from the -basis to another, the Zbasis, then the Z-basis is orthonormal if and only if A is orthogonal.

SOLVED PROBLEMS
1.

Given the vectors

Zj.

= [1,2,3]' and X^ = [2,-3,4]',


(b) the

find:

(o) their inner product,

length of each.
2

(a)

X^-X^

XiX^

[1.2.3] -3
4

1(2)

2(-3) +

3(4)

(6)

\\X^f

X^.X^

X[X^

[1,2,3]

14

and

\\Xji

vTi

lA-jf

2(2)

+ (-3)(-3) + 4(4)

29

and

\\X^\\

V29

CHAP.

13]

VECTORS OVER THE REAL FIELD

105

2. (a)

Show

(b)

that 1 = [1/3, -2/3, -2/3 ]' and Y ^ [2/3.-1/3, 2/3]' are orthogonal. Find a vector Z orthogonal to both X and Y.

2/3
(a)

X-Y

Xy

[1/3,-2/3,-2/3] -1/3
2/3
1/3

and the vectors are orthogonal.

2/3

O"

(6) Write

[A:,y,o]

-2/3 -1/3
-2/3

and compute the cofactors -2/3. -2/3, 1/3


0_

of the elements of the

2/3
(3.11)

column of zeros.

Then by

= [-2/3, -2/3, 1/3]'

is

orthogonal to both

A:

and K.

3.

Prove the Schwarz Inequality:


Clearly, the theorem is true
If

If

X
or

and Y are vectors of F


is the zero vector.

Vn(R), then

\X-Y\ <
that

||A'||.||y||

if A"

Assume then

and Y are non-zero vectors.

a is any real number,

llaA^

yf

= =
=

(aX + Y)-(aX + Y)
[ax^ + y^.ax^+y^
(a x^

axn+yn]-[ax^+y^. ax^ + y^
yj^)

axn+jnY

+ 2ax^y^ +

+ (a^x^ + Zax^y^ + y^)

+ (o^^ + 2a%j^ + y^

a^xf

+ 2aX.Y

\\Yf >

Now a quadratic polynomial in a is greater than or equal to zero for all real values of o discriminant is less than or equal to zero. Thus,
i(X.Yf - i\\xf- \\Yf
and \x-y\

if

and only

if its

<

<

m-\\Y\

4.

Prove:

If

a vector

is orthogonal to

each of the n-vectors

X^,

X^

X^.

it

space spanned by these vectors.

is orthogonal to the

Any vector

of the space spanned by the X's can be written as a^X^+a^X^-^ ^-c^Xtji

Then

{aiXi + a^X^+... + Since

a^X^)-Y
Thus, Y

a^X^-Y + a^X^-Y +

a^X^-Y

Xi-Y

= 0, (i = 1,2

m).

is
is

orthogonal to the space. orthogonal to that space.

In particular, if

orthogonal to every vector of the space and by definition is orthogonal to every vector of a basis of a vector space, it is

5.

Prove:

If

a ^() is a subspace of a V^(R), k>h. then there exists at least one vector

A"

of

which

is orthogonal to the f^(fi).


-^I'-^s

v\R) "
and

^^*

^h be a basis

of the

FV).

let X^,^^

be a vector

in the

vM)

but not in the P^(R)

consider the vector

<*)

X
that

a^X^ + a^X^ +

...

aj^X,^

+ a^^^Xf,^^

The condition

be orthogonal to each of X^.X^

Xf,

consists of h homogeneous linear equations

106

VECTORS OVER THE REAL FIELD

[CHAP. 13

a^X^-X^ + a^X^-X^ +

...

+ af,Xh-X-, + a,,^^Xf^^^. X^ + a^Xh-X^ + af^^^Xf,^^- X^

=
=

o o

a^X^.X^ + a^X^.X^ +

...

ft + l unknowns a^.a^ ay,^^. By Theorem IV, Chapter 10, a non-trivial solution exists. When these values are substituted in (i), we have a non-zero (why?) vector X orthogonal to the basis vectors of the kA) and hence to that space.

In the

""^^

Vn-k'J'^^

^ ^^^ ^^''^'''^ orthogonal to every vector of a given V^{R) is a unique vector space

^^* -^i-^s be a basis of the V^{R). A-fe system of homogeneous equations


^'>

The

..-vectors

orthogonal to each of the

Jt,-

satisfy the

X^.X=o.X^.X=Q
X^^

Xk.X=0
system
(i) is

Since the

are linearly independent, the coefficient matrix of the

of rank k

hence, there are

n-k linearly independent solutions (vectors) which span a K"-\/?).

(See Theorem VI, Chapter 10.)


is the zero-

Uniqueness follows from the fact that the intersection space of the V^iR) and the V^'^(R) space so that the sum space is Xi{R).

7.

Find an orthonormal basis of

V^(R),

given

= [1/^6, 2/x/6, l/x/6]'.


0,

Note that A- is a unit vector. Choose Y = [l/Vl, Then, as in Problem 2(a), obtain Z = [l/^Jl. -l/VI,

-IA/2 ]'
to

another unit vector such that


set.

-Y =

1A/3]'

complete the

8.

Derive the Gram-Schmidt equations (13,7).


^^'
^^' -^2
>^n

be a given basis of V^(R) and denote by

Y^.

Yr,

Y^

the set of mutually orthogonal

vectors to be found.
(a)

Take Take

Y-^^

X^.

(b)

Y^ = Xr, + aY-^

Since

Y-^

and Y^ are to be mutually orthogonal.

Y^.Y^
and
o =

Y^-X^ + Y^-aY^

Y^-X^ + aY^-Y^

- -i-2

Thus.

K, =

Y X ^Sill X^ ~ ^1^ Xo-

Y^

(c)

Take

Y^ =

X3 +

aYr,

+ bY^
=

Since

Y^. K,,

Y^ are to be mutually orthogonal,


=

yi-Yj,

Yi-Xg + aY^.Y^ + bY^-Y^


Y^-Xs + aY^-Y^ + bY^.Y^

Y^-

X^ + bY^-Y^

and
Y^.Y^
= =

Y^-X^ + aY^-Y^

Then

- ^ll^

and

7, =

Z3 -

^ki^ K _

2W^ v

(d)

Continue the process until K,

is obtained.

CHAP.

13]

VECTORS OVER THE REAL FIELD

107

9.

Construct an orthonormal basis of

Fg,

given the basis Z^ = [2,1,3]', X^ = [1,2,3]', and Xg = [1,1,1]'.

Take

Y^ =

X^
=

= [2.I.3Y.

Then
=
[1,2.3]'

K,

X^ -

^^i-i
"2

- ii[2,1.3]'

[-6/7.15/14,3/14]'

^3 V

^1 '"^3

[1.1.1]'

- ^
9

14

I4J

[_3

3j

Normalizing the y-s. we have

[2/\/l4. l/i/Ii, 3/v'l4

]',

[-4/\/42, 5/V42. l/\/42]', [l/Vs. 1/V3.

-1/^3

]'

as the required orthonormal basis.

10. Prove:

linear transformation preserves lengths

if

and only

if its

matrix is orthogonal.

Let

y^. Yq

be the respective images of X^,X2 under the linear transformation


is

AX.

Suppose A
(1)

orthogonal so that A/l =

/.

Then
=

Y^-Y^

YiY^

(X'^A')(AX^)

X\X^

X^^-X^

and, by

Theorem XII lengths

are preserved.

Conversely, suppose lengths (also inner products) are preserved.


Y^-Y^
=

Then

X{(A'A)X2

X^X^.

A'A=1

and A

is orthogonal.

SUPPLEIVIENTARY PROBLEMS
H. Given
the vectors A'l = [l,2,l
]'.

.Yg =

[2.I.2]', vYg = [2.1,-4

]'.

find:

product of each pair, (6) the length of each vector.


(a) the inner (c)

a vector orthogonal to the vectors X^, X^


(a) 6, 0,

X.^,

Xq

Ans.

-3

(6)

Vb,

3,

V^

(c)

[l,0,-l

]',

[3.-2.1]'

12.

Using arbitrary vectors of


Prove (13.4).

^3(7?).

verify (13.2).

13.

14.

Let
(a)

a:

= [1.2.3,4]' and

= [2,1,-1.1]' be a basis of a V^(R) and

= [4,2,3,l]' lie in a V^{R)

containing

and y.

Show

that

(b) Write

Z is not in the ^^{R). aX + bY + cZ and find a


I^(ft) is

vector

of the V^{R) orthogonal to both

and Y.

15. (a) Prove: (6) Prove:

vector of

orthogonal to itself

if

and only

if it is

the zero vector.


if

If X-^. X^. Xg are a set of linearly dependent non-zero ^-vectors and X^ and Xq are linearly dependent.

Z^ Xg = Xj_-Xq=

0,

then

108

VECTORS OVER THE REAL FIELD

[CHAP. 13

16.

Prove:

vector

is orthogonal to every vector of

P^"(/?) If

and only

if it

is orthogonal to every vector of

a basis of the space.

17.

Prove:

If

two spaces V^iR) and

t^(fl) are orthogonal, their intersection

space is

I^(if).

18.

Prove: The Triangle Inequality.


Hint.

Show
A"

that

!|Z +

<

(||.Y||

||y||)^,

using the Schwarz Inequality.

19.

Prove:

||

1|

|1a:|1

+ \y\

if

and only

if

and Y are linearly dependent.

20.

Normalize the vectors of Problem


Ans.

11.

[l/Ve, 2/V6. l/\/6]',


that the vectors X.

[2/3, 1/3, 2/3]',

[2/V2i, l/V^I, -4/V2i

]'

21.

Show
(o)

Y .Z
X^

of

Problem

2 are

an orthonormal basis of V^{R).

22.

Show
them.

that if X^.

X^

are linearly independent so also are the unit vectors obtained by normalizing

(6)

Show

that

if

the vectors of (a) are mutually orthogonal non-zero vectors, so also are the unit vectors

obtained by normalizing them.

23.

Prove: (a)

If

A
.4

is is

orthogonal and

.4

1,

each element of A

is

equal to

its

cofactor in

^4

(6) If

orthogonal and \a\ = -i, each element

of^

is equal to the negative of its cofactor in |^|

24.

Prove Theorems

VIII, IX, X, XI.

25.

Prove;

If

.4

and B commute and C

is orthogonal,

then C'AC and

C'BC commute.
if

26.

Prove that AA'


are orthogonal.

(or A'A),

where A

is n-square. is a

diagonal matrix

and only

if

the rows (or columns) of A

27.

Prove:

UX
If

and Y are n-vectors, then XY'-YX' is symmetric, and Y are n-vectors and A

28.

Prove:

is n-square, then

X-(^AY) = {A'X) -Y

n
29.

Prove:

If

A"]^,

Z^

A'

be an orthonormal basis and


+

if

A^

2
^=1

c^X^,

then

(a)

X-X^ =

c^,

(i = 1,2

n);

(b)X.X
30.

cl+4+...

cl
VsiR). given
(a)

Find an orthonormal basis of


Ans.
(a)
(6)

X^ =

3/VT7, -2/\/T7, 2/1/17]';


3/\/34]'

(6) [3,0,2]'

X^.

[0.l/^^2.l/\f2Y,
0,

[-4/\^, -3/^34,
0,

[3/VI3,

2/\/l3]',

[2/VI3.
V^i^R)

-3/VT3]', [0,1,0]'
in order:

31.

Construct orthonormal bases of


{a)

by the Gram-Schmidt process, using the given vectors

[1,-1,0]',
[1.0,1]',

[2,-1,-2]',
[1,3,1]',

[1,-1,-2]'
[4,0,-1]'

(6) (e)

[3.2,1]'

[2,-1,0]',
(a)
(b)

[4,-1,0]',

Ans.

[iV^. -iV^,
[^\/~2, 0,

0]',

[V^/6, V2/6, -2v'V3]', [-2/3,-2/3,-1/3]'


[0,1,0]',

iV2]',

[iV^.

0,

-iV2]'
[o,0,-l]'
]'

(c)

[2V5/5, -V5/5,

0]',

[^/5/5.2^/5/5.oy,
I^(ft),

32.

Obtain an orthonormal basis of


Hint.

given

^"1

=[ 1,1,-1

and

ATg =

[2,1,0

]'.

Take

Y^ = X^,

obtain Y^ by the Gram-Schmidt process, and Y^ by the method of Problem 2(6).


[5\/2. 0, 2\A2]',

Ans.

[\A3/3, V'3/3, -V^/3]',

[\/6/6, -\/6/3, -\/6/6

]'

CHAP.

13]

VECTORS OVER THE REAL FIELD

109

33-

Obtain an orthonormal basis of V^iR). given

X-^ =

[7,_i,_i

]'.

34.

Show

in

two ways that the vectors

[l.2,3,4]',

[l. -1.-2, -3]',

and [5,4,5,6]' are linearly dependent.

35.

Prove:

If

is

skew-symmetric. and

+ A is non-singular, then

= (I

-A)(I + A)~^

is orthogonal.

36.

Use Problem 35
"

to obtain the orthogonal matrix

given

12
s"!

(a)

(b)

-5

10
2-3
(b)

oj'

"5-14
-12
Ans.
(a)

-51

^
.4

-10
10

-5 -10
2

5 -I2J'

-11
where P
non-singular, then

37. Prove:

If

is an orthogonal matrix

and

it

AP

is

PB^

is orthogonal.

38.

In a transformation of coordinates from the

-basis to an orthonormal Z-basis with matrix P. Y =


12).

AX

be-

comes

71 =

P^^APX^

or

7^= BX^ (see Chapter

Show

that if

is

orthogonal so also is

B.

and con-

versely, to prove

Theorem XIV.
and
/

39.

Prove:

If

is orthogonal

+ .4 is non -singular then

= (I

- A) (I + A)'"^

is

skew-symmetric.

40.

Let

X
y

= [xj^.x^.xsY

and

= [yi.yQ.ygY
25, 23]'

be two vectors of VsiR) and define the vector product,


X2

X xY

of

and

as

ZxF

Jl
,

= [21,

where

21 =

3 ys
Z2 =
.

^1 Zg =

yi

^3 ys

%
X^, Y^.
],

After identifying
2 yi

yi
estciblish:

the z^ as cofactors of the elements of the third column mn of


(a)

The vector product


The vector product

of

two linearly dependent vectors

is the zero vector.

(6) (c)

of two linearly independent vectors is orthogonal to each of the two vectors.

XxY

-(YxX-)
=

(d)

(kX)xY

k(XxY)

XxikY),

k a scalar.

41. If W. X. Y.
(a)

are four vectors of V^{R), establish:


=
=

X x{Y+Z)

XxY

XxZ
=

(b)X-(YxZ)
(c)

(d)

Y-(ZxX) W-Y (WxX)-(YxZ) = X-Y X-X (XxY)-(XxY) = Y-X

Z-(XxY) W-Z
X-Z

\XYZ\

X-Y Y-Y

chapter 14
Vectors Over the Complex Field

COMPLEX NUMBERS.

x and j are real numbers and i is defined by the relation j^ = 1, z = x^iy is called a complex number. The real number x is called the real part and the real number y is
If

called the imaginary part of x +

fy.

Two complex numbers

are equal if and only if the real and imaginary parts of one are equal

respectively to the real and imaginary parts of the other.

A complex number
The conjugate

x + iy =

if

and only
z

if

x = y =

0.

of the

complex number
its

= x+iy

is given

by

= x+iy = xiy.

The sum

(product) of any complex number and

conjugate is a real number.


|z|

It

The absolute value |z| of the complex number z = x+iy is given by follows immediately that for any complex number z = x + iy,
|z|

\J

z-z =

\fW+y^

(14.1)

> \A

and

\z\

>

\y\

VECTORS. Let X

be an ra-vector over the complex field C.


Since ^(R) is a subfield,
it

The
is to

totality of

such vectors constitutes

the vector space I^(C).

be expected that each theorem con-

cerning vectors of I^(C) will reduce to a theorem of Chapter 13 when only real vectors are considered.
If

= [%, x^

XnY and y =

[ji,

72

y^Y are two vectors of

P^(C), their inner

product

is

defined as

(14.2)

X-Y

XY

%yi

x^y^

XnJn

The following laws governing inner products


(a) (b)

are readily verified;

I-y =

y^

(/)

X-Y+Y.X

2?.{X.Y)
is the real part of

(cX)-Y = c(X-Y)
X-(cY) = c(X-Y)
(g)

where R(X-Y)

X-Y.

(14.3)

(c) (d)

X-Y-Y-X

X-(Y+Z) = X-Y + X-Z

= 2CiX-Y) where C(Z-y) is the imaginary part of Z-F.

(e)

(Y+Z)-X =

Y-X+Z-X
X
is

See Problem
||Z||

1.

The length

of a vector

given by

\/

X-X =
Q.

\/%%

%^2

+ XnXn-

Two

vectors

and Y are orthogonal


V^{C), the

if

X-Y = Y-X =

For vectors of
(14.4)

Triangle Inequality
\\X+Y\\
2)

<

||Z||

||i'll

and the Schwarz Inequality (see Problem


(14.5)
hold.

\X-Y\

<

\\X\\-\\Y\\

Moreover, we have (see Theorems I-IV of Chapter 13)

110

CHAP.

14]

VECTORS OVER THE COMPLEX FIELD

111

I.

Any

set of

mutually orthogonal non-zero re-vectors over C is linearly independent

and, hence, spans a vector space I^(C).


II. If a vector Y is orthogonal to each of the re-vectors thogonal to the space spanned by these vectors.

X^,

X^

X^,

then

it

is or-

III.

If

V^iC) is a subspace of V^C), k>h, then there exists at least one vector

in

V^(C) which is orthogonal to V^(C).


IV.

Every vector space

J^(C),

m>0, contains m

but not more than

m mutually

orthog-

onal vectors.

basis of I^(C) which consists of mutually orthogonal vectors is called an orthogonal If the mutually orthogonal vectors are also unit vectors, the basis is called a nonnal or orthonormal basis.
basis.

THE GRAM-SCHMIDT PROCESS.


Y,

Let
=

X,. X^

X^

be a basis for F^^CC).

Define

X,

In

Xn

(14.6)

Yd

Xn

^2-^3

Yi-Xs
Y^
Y,.Y,

Y .Y l2-l2

^
y
-'m-i

y n

_ ~

y
"Si

T;

Y y 'a-i'^m y

^m-i'-'m-i

'y^

The

unit vectors

Gi

7.

(i

= 1.2

m)

are an orthonormal basis for ^^(C).

V. If X^, X^ X^, (l<s<m), are mutually orthogonal unit vectors of ^(0) there exist unit vectors (obtained by the Gram-Schmidt Process) Z,,,, X, ^ ;i' in the snaoe such that the set Z Z^ Z, is an orthonormal basis.

THE GRAMIAN.

Let Gramian matrix.

X^

Xp be a

set of

;s-

vectors with complex elements and define the

Xi'Xi Xj^-Xq
X^'X'i

X^-Xp
X^-Xp

Aj

X-y

X-^Xq

X^ Xp X2 xp

X^-X^

(14.7)

Ag X^

XqXq

Xp-X^ Xp-X^

Xp-Xp

XI X-^ XI x^

x^x. p^p

Clearly, the vectors are mutually orthogonal if and only if

is diagonal.

Following Problem

14,

Chapter

17,

we may prove
\G\

VI For a set of re-vectors X^. X^ Xp with complex elements, holds if and only if the vectors are linearly dependent.

>

0.

The equality

112

VECTORS OVER THE COMPLEX FIELD

fCHAP. 14

UNITARY MATRICES. An

n-square matrix A is called unitary

if

(AyA = A(A)'=

I,

that is

if

(i)'=

^.

The column vectors (row vectors)

of a unitary matrix are mutually orthogonal unit vectors.

Paralleling the theorems on orthogonal matrices of Chapter 13,


VII.

we have

The column vectors (row vectors)

of an re-square unitary matrix are an orthonormal

basis of l^(C), and conversely.


VIII.

The inverse and

the transpose of a unitary matrix are unitary.

IX.

The product

of two or more unitary matrices is unitary.

X.

The determinant

of a unitary matrix has absolute value

1.

UNITARY TRANSFORMATIONS. The


(14.8)

linear transformation

AX

where A

is unitary, is called a unitary transformation.

XI.
if its

A
If

linear transformation preserves lengths (and hence, inner products) if and only

matrix is unitary.

XII.

AX

is a transformation of

coordinates from the i'-basis to another the Zif

basis, then the Z-basis is orthonormal if and only

is unitary.

SOLVED PROBLEMS
1.

Given X = [l+j, -J, 1]' and Y = [2 + 3J, 1- 2J, i]', (c) verify X-Y + Y-X = 2R(X-Y) (a) find X-Y and Y-X (d) verify X-Y -Y-X = 2C(X-Y) (b) verify X-Y = Y-X

2+
(a)

3/

X-Y

X'Y

[l-i,f,i;

l-2i
i

(l-i)(2+3!) + i{l-2i) + 1(0

7 4 3j

+i

Y-X

Y'X

[2-3i.l + 2i.-i]

3i

(b)
(c) (d)

From

(a):

Y-X. the conjugate of Y-X. is


= = (7 (7

3i

= X-Y. =

X-Y + Y-X X-Y

+ +

3i) 3j)

+ (7-3i) =

14
6j

= =

2(7)

2R(X-y) 2C(X-Y)

-Y-X

-(7-30

2(30 =

2.

Prove the Schwarz Inequality:

\X-Y\

<

p||

11Y||.

As

in the

case of real vectors, the Inequality

is true if

or

= 0.

When X and Y

are non-zero

vectors and a is real, then


\\aX +

Yf

{aX+Y)-{aX+Y)

a X-X + a{X-Y + Y-X) + Y-Y

a'Wxf

+ 2aR{X-Y) +

\\Y\\

>

0.

CHAP.

14]

VECTORS OVER THE COMPLEX FIELD

113

Since the quadratic function in a is non-negative

if

and only
and

if its

discriminant is non-positive,

R{X-Yf If

\xf\Yf
-F
|

< < ||x|

Ra-Y)
if

<
O.

IkMyll
define
c =

X-Y

0,

then

|;f

fl(;5:-y)

||y

X-Y

,i

X-Y
\x-y\

Then
R[ciX-Y)]

R(cX-Y) <
=

\\cX\\-\\y\\

|c||U||-iyl|

|UM|y||

while, by (14.3(6)),

R(cX-Y)

\X-Y\.

Thus,

U-yl < U||-||y||

forall^andy.

3.

Prove:
(B)

B =
=

(A)'A is Hermitian for any square matrix A.

\(A)'AV

(A'A)

(A)A

and B is Hermitian.

4. If

i = B + iC
Since

is

Hermitian, show that (I)/l is real


is

if

and only

if

B and C anti-commute.

B+iC

Hermitian,
=
if

(B+iC)' = B + iC; thus.


= =

(A)A
This
is real if

(B + iCUB+iC)

(B + iC)(B+iC)

B^ + i(BC + CB) - C^
if

and only

BC + CS

= o or

BC

-CB

thus,

if

and only

B and C anti-commute.

5.

Prove:

If

is skew-Hermitian. then

iA

is Hermitian.
(A)' =

Consider B = ~iA.

Since A is skew-Hermitian,
(S)'

-A. Then
i(-A)
=

(riA)'

t(Z)'

-iA

and S

is Hermitian.

The reader

will consider the case

= iA.

SUPPLEMENTARY PROBLEMS
6.

(a) (b)
(c)

Given the vectors X^=[i.2i,iY. find X^-X^ and X^-X^,


find the length of

A:2 = [l,

1+

/,

o]',

and Xg =

[i, 1

j,

2]'

each vector Xi
is

show

that

[l-i, -1, i-j]'

orthogonal to both X^ and X^.

(d)

find a vector orthogonal to both


(a)

X^ and

Jf g

Ans.

2-3i.-i

(b) ^/e

V^ V^
.

(d)

[-1 -5iJ .3 -i]

7.

Show

that [l + i.i.lV.

[iA-i.oy, and

[l -i.

1. 3j ]'

are both linearly independent and mutually orthogonal.

8.

Prove the relations (14.3).


Prove the Triangle Inequality.

9.

10.

Prove Theorems

I-IV.

11.

Derive the relations (14.6).

114

VECTORS OVER THE COMPLEX FIELD

[CHAP. 14

12.

Using the relations (14-6) and the given vectors


vectors are
(a) (b)
Arts,

in order, construct

an orthonormal basis

for iy.C)

when

the

[0,1,-1]', [l + j,l,l]'. [l-j,l,l]'


[l + i.i.lY.
(a)

[2.1-2i.2 + iY, [l-i.O.-iY.


I. 2
5t
]'.

[O.^V2.-iV2]', [1(1 + 0.
ri

[-^i,
3jy
r

^(1 + 0.

^d

]'

(&)

[2(1+0. 2..2

T J.

3+

[3;^.

4^. 4^ J-

-6 + 3i 7 -5 -5 3j -b 1-i -t y L2V30 'aVso- 2\Am^

13. Prove: If

/I

is

a matrix over the complex field, then A + A has only real elements and

A- A

has only pure

imaginary elements.
14.

Prove Theorem V.

15. If

is n-square,

show
if

(a) A'A is (b)

diagonal
/

and only
if

if

the columns of

A are mutually orthogonal vectors.

A'A =
If

if

and only

the columns of

are mutually orthogonal unit vectors.

16.

Prove:

and Y are n-vectors and A

is re-square,

then

X-AY

A'X-Y

17.

Prove Theorems VII-X.


Prove:
If

18.

is

skew-Hermitian such that

+ A is non-singular, then B = (l-A){I + A)~^ is unitary.


I

+t

r
19.

i+j1
(6)
i i

Use Problem 18

to form a unitary matrix, given

"^

L-1+.-

J-

-1 + j

_9 + 8i
1

-10 -4i
l

-16-18i
-10 -4t -9 + 8j
and

r-l +
|_

2j

-4-2i"|
(b)

Ans.

(a) 5

-2-24J
29

12i

2-4i

-2-jJ'

4-lOj
of the

-2-24J

20. Prove:
21.

If

^ and B are unitary and


in

same

order, then

AB

BA

are unitary.

Follow the proof

Problem

10.

Chapter

13. to prove

Theorem XI.

22. Prove: If

is unitary

and Hermitian. then A


3+

is involutory.

1(1 +

J//3
l/\/3

2^
4 + 3i
is unitary.

23.

Show that

-k

2y/T5

-i/y/3

-^ 2Vl5_
-1

24. Prove: If

A A

is unitary

and

if

= .4P

where P is non-singular, then PB

is unitary. is

25. Prove:

is unitary

and I+A

is non-singular, then

(I

- A)

(,!

+ A)'^

skew-Hermitian.

chapter 15
Congruence
CONGRUENT MATRICES. Two
(15.1)

re-square matrices

there exists a non-singular matrix

A and B over F P over F such that


B
=

are called congruent,

over

if

FAP
same

Clearly, congruence is a special case of equivalence so that congruent matrices have the
rank.

When P

is

verse order of the same elementary row matrices; that

expressed as a product of elementary column matrices, P' is the product in reis, A and B are congruent provided A can

be reduced to B by a sequence of pairs of elementary transformations, each pair consisting of

an elementary row transformation followed by the same elementary column transformation.

SYMMETRIC MATRICES.
I.

In

Problem

1,

we prove

Every symmetric matrix A over F of rank


first r

whose
Example
1.

r is congruent over F to a diagonal matrix diagonal elements are non-zero while all other elements are zero.

Find a non-singular matrix P with rational elements such that

- P'AP

is diagonal, given

12
2
3

3 5

3 5
8

10

10

-8

ff2i(-2) and K2i(-2). then //gjC-S)

reducing A to D, we use [A /] and calculate en route the matrix P' First we use and XgiC-S), then H^-ii-2) and K^^{-2) to obtain zeroes in the first row and in the first column. Considerable time is saved, however, if the three
In
.

row transformations are made first and then the three column transformations. transformed into a symmetric matrix, an error has been made. We have

If

is not then

12
[AH^
2 3

3 5 8

2 8

1 1 1
1

10
c
'V-

10
4 4

3
2

5
8

10

0-1-1 0-1-1
4

10

-8
1

4-12
1

-2100 -3010 -2001


1

0-100
4

-2
-1
10

c
~\^

0-100
4

-2

10
4
1

-1
4

-10

-1-110

[DP']

115

116

CONGRUENCE

[CHAP. 15

-2 -10 -1
1

4-1
1

Then

10
The matrix D
to

which A has been reduced

is not unique.

The additional transformations

10
0-100
ffgCi) and Kg(^).
for

example, will replace

D by

the diagonal meitrix

10

while the

10
transformations

H^d) and K^Ci)

replace

D by

0-900
.

There

is,

however, no pair of

rational or real transformations which will replace


ative elements in the diagonal.

D by

a diagonal matrix having only non-neg-

REAL SYMMETRIC MATRICES.

Let the real symmetric matrix A be reduced by real elementary transformations to a congruent diagonal matrix D, that is, let P'AP = D. While the non-zero diagonal elements of D depend both on A and P. it will be shown in Chapter 17 that the number
of positive non-zero diagonal elements

depends solely on

A.

By
of

a sequence of row and the same column transformations of type 1 the diagonal elements may be rearranged so that the positive elements precede the negative elements. Then a

sequence of real row and the same column transformations of type 2 may be used to reduce the diagonal matrix to one in which the non-zero diagonal elements are either +1 or 1. We have
II.

real symmetric matrix of rank

r is

congruent over the real field to a canonical

matrix
P

(15.2)

'r-p

The

integer p of (15.2) is called the index of the matrix and

p-(r p)

is called the

signature.

Example

2.

Applying the transformations H23. K^a and H^ik), Kr,(k) to the result of Example
1 1

1,

we have

10
1

[A\n

0-100
4

-2
-10

C
1 1

10
0-10

-5
-2

10
s

IC\(/]

-1 -1
and (/AQ = C. Thus, A
is of

-1-110
index p =
2,

rank

= 3,

and signature

= 1.

if

III. Two re-square real symmetric matrices are congruent over the real field if and only they have the same rank and the same index or the same rank and the same signature.

In the real field the set of all re-square matrices of the type (15.2) is a canonical set over

congruence

for real ra-square

symmetric matrices.

CHAP.

15]

CONGRUENCE

117

IN

THE COMPLEX FIELD, we

have
r

IV. Every ra-square complex symmetric matrix of rank complex numbers to a canonical matrix
/^

is

congruent over the field of

(15.3)

Examples. Applying the transformations

H^ii)
1

and

K^f^i)

to the result of

Example
1

2,

we have

10
1

-5 -2

[^1/]

2
1 1

0-10

10 10 10

-5
-2i

2
i

[D

']
1

-1 -1

-1 -1

and

R'AR

^&:]
if

See Problems 2-3.

V.

Two
if

ra-square

numbers

and only

complex symmetric matrices are congruent over the field of complex they have the same rank.

SKEW-SYMMETRIC MATRICES.

If

is

skew-symmetric, then
=

(FAP)'
Thus,
VI.

FAT

r(-A)P

-FAP

Every matrix

B =

FAP

congruent to a skew-symmetric matrix A is also skew-

symmetric.

In

Problem
VII.

4,

we prove
is

Every n-square skew-symmetric matrix A over F

congruent over

to a canoni-

cal matrix (15.4)

B
= r
I

diag(Di, Dj

0^,0,..., 0)

where

D,-

,(f=l,2

t).

The rank

of

/}

is

2t.

See Problems.

There follows
Vin. Two ra-square skew-symmetric matrices over they have the same rank.

are congruent over

if

and only

if

The set

of all matrices of the type (15.4) is a canonical set over congruence for re-square

skew-symmetric matrices.

HERMITIAN MATRICES. Two

n-square Hermitian matrices A and B are called Hermitely congruent,


there exists a non-singular matrix

[^

],

or

conjunctive

if

P such

that

(15.5)

FAP

Thus,

118

CONGRUENCE

[CHAP.

15

IX.

Two

re-square Hermitian matrices are conjunctive

if

and only

if

one can be obtain-

ed from the other by a sequence of pairs of elementary transformations, each pair consisting of a column transformation and the corresponding conjugate row transformation.
X.

An Hermitian

matrix

of rank

r is

conjunctive to a canonical matrix


Ip

(15.6)

-Ir-p

The

integer p of (15.6) is called the index of A and s =


XI.

p-(r-p)

is called the signature.

Two

re-square Hermitian matrices are conjunctive if and only if they

have the same

rank and index or the same rank and the same signature.

The reduction
of

of an Hermitian matrix to the canonical form (15.6) follows the procedures


to the proper pairs of in

Problem 1 with attention troublesome case is covered

elementary transformations.

The extreme

Problem?.
See Problems 6-7.

SKEW-HERMITIAN MATRICES.

If

is

skew-Hermitian, then
=

(FAPy
Thus,
XII.

(PAT)

FAP

Every matrix B =

FAP

conjunctive to a skew-Hermitian matrix A is also skew-

Hermitian.

By Problems, Chapter

14,

= -iA is Hermitian

if

is

skew-Hermitian.

By Theorem

there exists a non-singular matrix

P such

that
Ip

FHP

-Ir-p

Then

iFHP

= iF{-iA)P =

FAP
=

= iC

and

Up
(15.7)

FAP

- ilr~p

Thus,
XIII.

Every re-square skew-Hermitian matrix A

is conjunctive to a matrix (15.7) in

which

is the rank of

A and

p is the

index of iA.
if

XIV. Two re-square skew-Hermitian matrices A and B are conjunctive they have the same rank while -iA and -iB have the same index.

and only

if

See Problem

8.

CHAP.

15]

CONGRUENCE

119

SOLVED PROBLEMS
1.

Prove: Every symmetric matrix over r non-zero elements in the diagonal.

of rank

can be reduced to a diagonal matrix having exactly

transformations of type reduce A to

Suppose the symmetric matrix A = [a^,-] is not diagonal. If a^ / 0. a sequence of pairs of elementary 3, each consisting of a row transformation and the same column transformation, will

"n2

"ns

Now

the continued reduction is routine so long as b^^.c^^.

are different from zero.

Suppose then

that along in the reduction,

we have obtained

the matrix

hss
S+1, s+2

*s+l, n

k^ S+2, s+ 1

'^ri

s+i

"n,

s->-2

in

which the diagonal element k^^^^^^^


k^j
,

= o.

If

every
it

4y

= 0,

we have proved

the theorem with s =

r.

however, some

say

If,

s+^ /

0,

we move

into the (s+l,s+i) position by the proper row and

column

transformation of type 1 when u = v; otherwise, we add the (s+u)th row to the (s+t;)th row and after the corresponding column transformation have a diagonal element different from zero. (When a^, = o, we proceed as in the case Ar^+i ^^.^ = o above.)

Since

we

whose

first t

are led to a sequence of equivalent matrices, A is ultimately reduced to a diagonal matrix diagonal elements are non-zero while all other elements are zero.

'12
2.

2 5 5
to

Reduce

the symmetric matrix

3 5

canonical form (15.2) and to canonical form (15.3).

.2 In

each obtain the matrix P which effects the reduction.


1

2 3 5

2
1

1
1

u\n

1 1 1

2
2

5 5

c
1

-1
1

1 1

-2 -2

C
1

-1 2

c
1

-4
-1

1
1

-2

[0|^']

To

obtain (15.2),
1

we have
1 1 1 1 1 1

[0

Pi']

-4
-1

-2^f2

k\f2
1

i\Pi

\C\P'\

-2

0-1

-2

120

CONGRUENCE

[CHAP. 15

-av^ -2
j\/2
1

and

5a/2

To

obtain (15.3),

we have
1 1 1
1

[D\Pl]

1
1

1 1

-2\/2
2i

2\/2
-i

iV2

Vc\?'\

-1

-2V2
k\f2

2J

and

-i

3.

Find a non-singular matrix ? such that ?'A? is

in
1

canonical form (15.3), given


i

=
1

2-j
i

2-i

10 + 2i

^
1
i

1+J 2-j

1 1
1

1 1

[^1/]
1

c
'-\J

2j

-i

1 1

+i

2-i

10 +

2j

3-2J
_

10 p

-1-j

10
1

1 1

c
2i
1

-i
7+ 13
4i!'

5+12J

2i

-5+12i 3-2t
13

13

[c\n
1

-I

7+

4i

13

-5+12i
Here.
1

13

3-2j
13

4.

Prove: Every ?i-square skew-symmetric matrix A over

of rank 2t is congruent over


7)^,0

to a matrix

B
where
A
D^

=
)

diag(Di, Dg

0)

U :]
then S = ^.

(J

= 1,2,

It

= Q.

If

-4

0,

then some

mj

and /th and second rows; then Interchange the Jth

first

-aji ?^ 0. Interchange the sth and first rows and the columns and the /th and second columns to replace

oy a..
j

^ by the skew-symmetric matrix

-a,;, -Oy

g 2

Next multiply the

first

row and the

first

column by

l/a^^-

'S*
i

CHAP.

15]

CONGRUENCE

121

1
''2

to obtain

-1 4

and from

it,

by elementary row and column transformations of type

3,

obtain

-1

Di
F4

If

/v=

0.

the reduction is complete; otherwise, the process is repeated on F4

until

is obtained.

5.

Find a non-singular matrix P such that P'AP

is in

canonical form (15.4), given


2 4

-2 -1 -4 3
Using
a^s 4 0,

we need

only interchange the third and second rows followed by the interchange of the
2

third

and second columns of

1-3
-2 -1 -2

10 10

10

-2
to obtain
1

-1 -2

10
1

0-3

-4320
Next, multiply the

10 10
1

-4230
first
1

first row and first column by 2; then proceed to clear the two columns of non-zero elements. We have, in turn.

two rows and the

first

10
-1
1

k
1

-1 -2

0-3

10 10
1

and

-5
5

i -2

-2230
1

-1

-2

Finally, multiply the third row and third column by

-1/5

to obtain

1/2

-1
1

10
1/10 -1/5

Di

P'

Do
1

0-10
1/2

-1

0-2

1/10 -1

Thus when

-1/5
=

10-2
1

P'A?

= diag (Di.Dj).

6.

Find a non-singular matrix P such that P'AP

is in
1

canonical form (15.6), given


1

-3 + 2i

-3 - 2j

122

CONGRUENCE

[CHAP. 15

1-j -3 + 2i
j

1
1

[^1/]

1+i

2
i

-i

1
1

c
5

-l-i
3

1 1

-3 -

2j

-13

2j

10

00

25
13

2-3i
1

5_

2-3f
1

13

13
3

13
1

5V13

5\/l3

\/l3
1

-13

2i

0-1

2i

VT.5

Vl3

vc\p'^

2+

3t

3-2i
\fl3

5\/l3
13

and
5\/T3
1

^/I3

\/T3

7.

Find a non-singular matrix P such that P'AP is in canonical form (15.6), given
1 l

+ 2i
5

- 2i
3f

- 3i -4 - 2
2

2+
1

-4 +
1

2f

13

+
5

2i

3j

1 1

[41/]

2i
3i

-4 - 2s
2i

HC
1

5j

-1 +

2j

10
1

2+

-4 +

13

10
1

-5i

-2-3!
1

10
HC
10
5i

10
1
i

HC

10

-5J

-2-Zi

-5/2

-2-2J

5J

HC

10
0-1

'10

10

\/T6

-4 -

4j

\/To

x/Io

x/To

[cin
-4 +
4i

ao
and
10

\/T0

10

vTo

CHAP.

IB]

CONGRUENCE

123

8.

Find a non-singular matrix P such that P'AP

is in
i

canonical form (15.7), given

1 +
l

=
1

+ 2t
2j

-l +
i

2i

Consider the Hermitian matrix

-J

2
2

-i
2

1-i
1

+i

-l-2j
1

-i
1

The non-singular matrix

is

such that P'HP = diag[l,

1,

-l].

10
Then P'AP
= diag[i.
i.

-i

SUPPLEMENTARY PROBLEMS
9.

Find a non-singular matrix P such that P'AP


1

is in O'

canonical form (15.2), given


"O
(c)
1

2"

"
(d-)

-1 o1
2
1

(a)

[-:

-:]

(6)

3-1
-1 -2_
1

4 4
0,

-1
.

.2
2

11.
1 1

-2
1

"^V2 -^v^ -1
.

-1 -1
1

Ans.

(a)

(b)

(c)

[::]
I

kV2

ky/2

-k
i

(d)

10.

Find a non-singular matrix P such that P'AP

is in
2i

canonical form (15.3). given

r
I

H- j
1

4j

i + 2i 1+211

I
'

+j

+i
2i

-l-2i
-3 5i

|_1

2i

+ 4iJ
2

- 4;

-1 -

Ans.

(a)

2(-i-2,)]
(b)

2(1 -J)

i/\/2

(1

+ 0/2

(l-J)/\/2

(-3-2i)/l3
(3

+ 2i)/l3

11.

Find a non-singular matrix P such that P'AP


(a)
1
/I

is in

canonical form (15.4). given


(c) (d)
1

(6)

12-2
:

-1 -2

A
3

A =
-"

-2
3

-2 -3 (b)

Ans.
(a)
1

2
1

-3/2"'
1
1

(c)

0-1 2-3 "0010"

-2103
2-1-3
'1

-1

0-1

(d)

10 10

P
2
1

1
=

-1/3 -2/3 2
1/3
1

124

CONGRUENCE

[CHAP. 15

12.

Find a non-singular matrix P such that P'AP is

in

canonical form (15.6). given


2
1 1

(a)

tl +
l

1 3i

+i
3
i

+
3

2i

3i"|
'

(6)

1-i
2

-i
4

(c)

1-j
3 + 2f

3-4i
18
1

10

3 + 4t

-1-i (-5-i)/\/5
1

-1-j (-2 + 51)


1

Ans.

(a)

[1

-i +
1

sfl
(6)

(2-f)/V5

(c)

(-2-J)
1

i/Vs
canonical form (15.7). given
i

13.

Find a non-singular matrix P such that P'AP is

in

-1-i
-1-j
1

-1

(a)

(c)

1-j
.

1-i
-i.

L-i+j
i

J l+i
i

-1
2i
i

2+j

(6)

(i)

-1
_-2 + i

l-2i

-l+i

6i
,

-1-2J
-l'

i
Ans.
(a)

(l-J)/\/2
l/\/2

P [:
1

-r]
-i
1

(c)

P
,0

-1
1,

-2 + 3j -2 1
(rf)

l/x/2

(l-2i)/VT0
(-2-i)/yJT0
l/\/l0

-l/\/2'
i/\/2

(6)

i/y/2

14

If

^1

show

that a 2-square matrix

satisfies

C'DC

if

and only

if

|Cl=l.

L-i oj
Let A be a non-singular n-square real symmetric matrix of index
is even. p.

15.

Show

that

>

if

and only

if

-p

16- Prove:

non-singular symmetric matrix A is congruent to its inverse.


=

Hint.

Take P

BB' where B'AB

= I

and show that P'AP = A

17.

Rewrite the discussion of symmetric matrices including the proof of Theorem


matrices.

to obtain (15.6) for Hermitian

18.

Prove:

It

then A is symmetric (skew-symmetric)

if

and only

if

is

symmetric (skew-symmetric).
(S

19.

Let S be a non-singular symmetric matrix and T be a skew-symmetric matrix such that (S + singular. Show that P'SP = S when

7")

- T)

is non-

P
Hint.

(S+TT^(S-T)

P'SP =[(S-Tf^(S+T)S~'^(S-T)<.S+T)'^r^.

20.

Let S be a non-singular symmetric matrix and let T be such that (S+T)(S - T) is non-singular. then T is skew-symmetric. if P'SP = S when P = (S + TT^iS- T) and I+P is non-singular,
Hint.

Show

that

S(/-P)(/ + P)-^

S(I + PrHl-P).

21.

Show

that congruence of n-square matrices is an equivalence relation.

chapter 16
Bilinear Forms

AN EXPRESSION
and
(y^, y^

which
Jri)

is linear

is called a bilinear

and homogeneous in each of the sets of variables fonn in these variables. For example,

(x^, x^

m)

XxJi

2x172

ISxija

4x271

15x272

^s/s

is a bilinear form in the variables {x^, Xg)

and

(y.^, y^, y-g).

The most general


written as

bilinear form in the variables (x^, Xg

x)

and {y^.y^

Jn)

maybe

%i%yi
+ 021^271

+ +

oi2%y2 +

+ +

%r!

X i7?i

022^72 +

fl2ra%7ra

+ ami^myi +
or,

Om2%y2 +

""mv^n Jn

more

briefly, as

n
(16.1)
/(^.y)
t

S
=i

S ai-x^y
j=i
-^

'

%1
%
*2> .

012
^22

in

'7i' 72

Ojl

2n

_0'n\

Om2

Omra_

_Yn_

where

[x^, x^

]',

^ =

[0^^],

and

i'

= [yi,72

7]'.

The matrix A

of the coefficients is called the matrix of the bilinear form and the rank of

A
1.

is called the rank of the form.

See Problem
Example
1.

The bilinear form


1 1
1 1

Ti

^iXi + ^irs + *2yi + ^2X2 + ^373

Ui.

%.%

y2

73

X'AY

125

126

BILINEAR FORMS

[CHAP. 16

CANONICAL FORMS.
(16.2)

Let the

x's of (16.1) be replaced by

new variables

u's

by means of the

lin-

ear transformation
Xi

1.

bijUj,

(f=l,2

m)

or

BU

and the n y's be replaced by new variables v's by means of the linear transformation
(16.3)

Ti

2
J=i

c^jVJ,

(f=l,2

n)

or

Y = CV
IX,

We have X'AY = (BU)"A(CV) = U'(BAC)V. Now applying the linear transformations U = V = lY we obtain a new bilinear form in the original variables X\B'AC)Y = X'DY

Two

bilinear forms are called equivalent

if

and only

if

there exist non-singular transfor-

mations-which carry one form into the other.


I.

only
If

if

Two bilinear forms with mxn matrices A and B over they have the same rank.
r,

sire

equivalent over

if

and

the rank of (16.1) is

there exist (see Chapters) non-singular matrices

P and Q such

that

FAQ
Taking B = P'
(16.4)
in (16.2)

Ir

and C =

in (16.3), the bilinear form is Ir

reduced

to

UXPAQ)V

U'

U^V^

^2^2

U^V^

Thus,
II.

Any

bilinear form over

of rank

can be reduced by non-singular linear transfor+ u^v^ +

mations over

to the canonical form

u^^v^

+ u^i'r-

1 1 1

Examples. For the matrix

of the bilinear form

X^AY

X'

ofE^xamplel,

10 10
1

10

-1

1 1

-1
1 1

10
1

10
1

10 110 110 10 10 1

10

10
1

10
1

10
1

10
1
1

10

1-1-110
1

1-1-110

0-110
1

h
Thus,

p'

-1
1
1

-1
1 1

PU

and

QV

reduce X'AY to

1 1 1 1 1

1 1

-l'
1 1

-1

U%V

U-^V-:^

+ U^V^ + Ugfg

CHAP.

16]

BILINEAR FORMS

127

The equations
X-j^

of the transformations are


=

Ur,

Ti

= =

^1
V^

"3
1^3

X2

=
=

U,

and

72 Ts

Xg

^3

See Problem 2

TYPES OF BILINEAR FORMS. A


symmetric
alternate

bilinear form

1 S

a,-,-

ij x '^iJj

X'AY

is called

symmetric
according as A is

skew-symmetric
Hermltlan

Hermltlan
alternate Hermltlan

skew-Hermltlan

COGREDIENT TRANSFORMATIONS.
(i. 2

^) and
=

(yi.ys. .3^)-

Consider a bilinear form X'AY in the two sets of n variables When the ;'s and y's are subjected to the same transforma-

tion

CU
III.

and Y =

CV

the variables are said to be transformed cogredlently.

We have

Under cogredient transformations

CU

and

Y = CV,

the bilinear form X'AY.

where A
If

is re-square, is carried into the bilinear form

U\C'AC)V.

is

symmetric, so also is C'AC; hence,

IV.

symmetric bilinear form remains symmetric under cogredient transformations of

the variables.

V.

Two
if

bilinear forms over

variables

and only

if their

F are equivalent under cogredient transformations of the matrices are congruent over F.

From Theorem

I,

ChapterlS, we have
r

VI. A symmetric bilinear form of rank transformations of the variables to

can be reduced by non-singular cogredient

(16.5)

%%yi
II

02*272

OrXryr

From Theorems

and IV, Chapter

15,

follows

VIl. A real symmetric bilinear form of rank r can be reduced by non-singular cogredient transformations of the variables in the real field to

(16.6)

iyi
in the

%y2 +

xpyp

a;^4.i

7^4.1

x^Jr

and
(16.7)

complex

field to

%yi +

2y2

x^y^

See Problem

3.

CONTRAGREDIENT TRANSFORMATIONS.
formation

the x's are subjected to the transformation

Let the bilinear form be that of the section above. When X = (C~^yU and the y's are subjected to the trans-

Y = CV,

the variables are said to be transformed contragredlently.

We have

128

BILINEAR FORMS

[CHAP. 16

VIII.

Under contragredient transformations

= (C-K^T )'U and

Y = CV,

the bilinear form

X'AY, where A
IX.

is n-square, is carried into the bilinear form

U\C AC)V.
is

The

bilinear form

X'lY =

x^y^ + x^y^ +

%y

transformed into itself

if

and

only

if

the two sets of variables are transformed contragrediently.

FACTORABLE BILINEAR FORMS.


X.

In

Problem 4, we prove
if

non-zero bilinear form is factorable

and only

if its

rank is one.

SOLVED PROBLEMS
1.

%yi

+ 2iy2

13%yg

4a^yi + 15x^y^

-13]

Jx
1

-13l

x^y^

%. 2

4 15

-1

Ji 73

= X'

-4

15

-ij

2.

Reduce %yi + 2x^y^ + 3%y3


ical form.

Ix^y^ + 2%yi

2^y2 + x^y^ + 3x2y4 +

"ix-^y^

+ 4%y3 + x^y^ to canon-

3-2
1

The

matrix of the form is

2-2
3

3
1

By Problem

6.

Chapter

5.

the non-singular matrices

10
-2
1 1

1/3 -4/3 -1/3

-1/6 -5/6
and

-1 -1
mations

10
1

7/6
are such that

/g

PAQ

Thus, the linear transfor-

14
X^
=

Ui

2U2 U2

Uq

y
6
^

P'U

or

:x:2

U3 "3

and

QV

or

xs =

7a reduce X'AY to u^v^ +


U2t)2.

12
3.

3 5 8

Reduce the symmetric bilinear form

X'AY = X'

2 3 2

Y
10

by means of cogredient transfor-

5
8

10

-8

mations
field.

to (16.5) in the rational field,

(b) (16.6) in

the real field, and (c) (16.7) in the complex

(o)

Prom Example

1,

Chapter

15. the linear transformations

-2 -10 -1 4 -1 1
u.
1
1

-2 -10 -1
1

4-1
1

10

CHAP.

16]

BILINEAR FORMS

129

reduce

XAY

to

uj^'i

"2% +

^usfg.

-5 -2 -1
2
1

-5 -2 -1
2

(6)

Prom Example

2,

Chapter

15,

the linear transformations

-1
u.
1

1-1
1

reduce X'AY to u^v^ + ug^s


(c)

- "3%2,

Prom the

result of

Example
1 1

Chapter
1

15.

we may obtain

000'
2
1
1

-1

-5 -2
-1

10 10 10

-5202
-2j
i

-1-110
1

Thus, the linear transformations

-5 -2J -1 i -1 2
u.
1
1

-5 -2J -1 -1 2
t

reduce X'AY to

U^V^ + U2V2 + UgVg.

4.

Prove:

non-zero bilinear form f(x,y) is factorable


is factorable so that

if

and only

if its

rank is

1.

Suppose the form

and, hence, a^j = b^cj.

Clearly, any second order minor of

= [a^-], as

aij

aj5

a^bj
fk^'j

aib^
bib^
fe*s_

H
"k

Lfei "ks_

"k

vanishes.

Thus the rank

of

/4

is 1.

Conversely, suppose that the bilinear form is of rank 1. Then by Theorem I there exist non-singular linear transformations which reduce the form to U'(BAC)V = u^v^. Now the inverses of the transformations
=

jnr

and

'^jyj

carry u^u^ into

(S rijXj)(l s^jyp

f(x.y)

and so f(x.y) is factorable.

130

BILINEAR FORMS

[CHAP. 16

SUPPLEMENTARY PROBLEMS
5.

Obtain linear transformations which reduce each of the following bilinear forms to canonical form (16.4)
(a) x^j^

- 2^73 +

"ix^j^

+ ^272 - 3*273 - ^sTi - ^72


3 -2
(c)

^sTs
7
(rf)

2-510
(6)

10
8
1

4 7
3 5

5 3

8
5 6

-4

-11

7 y.

X'
1

5-510
6.

-2 -2

4
5 8

12
6

10

Obtain cogredient transformations which reduce


1

2 5

(a)

y and

(6)

X'

3
1

10
2

2
5_

to canonical form (16.6).

14
1

-2 -7
1

'l

-3 -4V3/3
1

Ans.

(a)

2
1

(6)

\/3/3
\/3/3

Ir
7. If Bj^,

B2. Ci

C2 are non-singular n-square matrices such that

B-^A^C^ = B^A^C^ =

find the transfor-

mation which carries X'A^Y into UA2V.


Arts.

= (B'^B^iU,

C^C^V
5, in

8. Interpret

Problem

23,

Chapter

terms of a pair of bilinear forms.


-

1 1
1 1
.

1
2
/.

1 22
i
2

9. Write the

transformation contragredient to

.Y

=
1

.4ns.

-2
i 2

2
2

~2

10.

Prove that an orthogonal transformation


Prove: Theorem IX.

is

contragredient to itself, that is

.Y

PV Y
.

PV.

11.

-1
12. If

X'AY

is a real

non-singular bilinear form then

XA Y

is called its reciprocal bilinear form.

Show

that

when

reciprocal bilinear forms are transformed cogrediently by the same orthogonal transformation, reciprocal bilinear forms result.

13.

Use Problem

4, Chapter 15, to show that there exist cogredient transformations duce an alternate bilinear form of rank r = 2t to the canonical form

PU. Y

PV

which

re-

U1V2

W2V-L

+ U3V4.- u^V3 +

+ ugt-i "^t - "sf^et-i

14.

Determine canonical forms for Hermitian and alternate Hermitian bilinear forms.
Hint. See (15.6) and (15.7).

chapter 17
Quadratic Forms

A HOMOGENEOUS POLYNOMIAL
(17.1)

of the type

n
q

X'AX

whose coefficients
^It ^2
^7J'

a^,-

"u

are elements of

is called a quadratic form

over

in the variables

Example

1.

2
as^^

+ 2*2 -

2
"^x^

'ix^x^

+ 8x^x^ is a quadratic form in the variables xi.x^.xg.


012X1X2.021^*1

trix of the form

may be

written in various

The maways according as the cross-product terms -4x1x2


and
013X1X3,031X3X1.

and 8x1x3 are separated to form the terms

We

shall

agree that the matrix of a quadratic form be symmetric and shall always separate the crossproduct terms so that a a^ Thus,

'

"-^s
1

7Xg
4

4Xj^X2

SXj^Xg

-2
2

X'

0-7
and the rank of

The symmetric matrix A = A


otherwise, non-singular.

[o^^- ]

is called the matrix of the quadratic form

is called the rank of the form.

If

the rank is r<n,

the quadratic form is called singular;

TRANSFORMATIONS. The
(17.2)

linear transformation over F,

BY

carries the quadratic form (17.1)

with symmetric matrix A over

into the quadratic form

(.BYyA(BY)

YXB'AB)Y

with symmetric matrix B'AB.

Two

quadratic forms in the same variables %, xz

are called equivalent if and only if

there exists a non-singular linear transformation

X=BY

which, together with Y =IX, carries

one of the forms into the other.


I.

Since B'AB is congruent to A,

we have

The rank

of a quadratic form is invariant under a non-singular transformation of the

variables.
II.

Two

quadratic forms over

are equivalent over

if

and only

if their

matrices are

congruent over F.

Prom Problem
the form
(17.3)

1,

Chapter

15, it

follows that a quadratic form of rank

can be reduced to

K^i

Kr^

+ h^y^,

hi/=o

131

132

QUADRATIC FORMS

[CHAP. 17

in

tion

which only terms in the squares of the variables occur, by a non-singular linear transformaX = BY. ViTe recall that the matrix B is the product of elementary column matrices while 6'

is the product in reverse order of the

same elementary row matrices.


4

-2

Example

2.

Reduce

X'

X
-7
1

of

Example

to the form (17.3).

-2
2

o'
1
1

We have

[A I]

-2
4

0-2
8

1
1

0-7
1

-23

-4

0-2
9

=
1

[D Bl

2
1

4
4
1

Thus.

BY

reduces q to

q'

y=-2v^+9v=.
See Problems
1-2.

LAGRANGE'S REDUCTION. The


pleting of the square.

reduction of a quadratic form to the form (17.3) can be carried out by a procedure, known as Lagrange's Reduction, which consists essentially of repeated com-

Example

3.
1

7%; 3
'

4x^*2

"*"

^*1*3

[x^- 'ix^{x^-2x^)\ + 2x1

7^:

{xl-'^x^{x^-2x^) + i{x^-2x/\ + 2x1 - Ixl -

Ux^-2xJ

(x^-2x^ + ix/ - 2(xl-8x^x^)

2Sx

(x^-2x^ + 'ix^f - 2(.xl-Zx^x^-\-\Qxl, + Qxl

(\-2^2+H)'
yi =

- 2(A:^-4^3f + Qxl
xx

- 2x2 +
xq^

4acs

xi
ot

= = =

yi

2y2 +

4ys

Thus,

ys 73

=
=

- 4x3
Xg

X2 X3

y2 + 4y3
ys

reduces q to

y^ - 2y^ + 9y^.

See Problem

3.

REAL QUADRATIC FORMS.

q = X'AX be reduced by a real non-singular one or more of the h^ are negative, there exists anonsingular transformation X= CZ, where C is obtained from 6 by a sequence of row and column transformations of type 1, which carries q into

Let the

real quadratic form


If

transformation to the form (17.3).

(17.4)

sz^ s,z7

11

s z^ s^z2 2

...

Sfyzj,

s*+iZa+i

...

s^;.

in

which the terms with positive coefficients precede those with negative coefficients.

CHAP.

17]

QUADRATIC FORMS

133

Now

the non- singular transformation

Wi
"'i

= =

VSi Zi
'j

a
(/

= 1,2

r)

=r+l,r+2

71)

or

'''4k-

;-

'

.1.1
J

carries (17.4) into the canonical form

(17.5)

wl + wl +

w^^

;2^^

wl

Thus, since the product of non-singular transformations is a non-singular transformation,

we have
Every real quadratic form can be reduced by a real non-singular transformation to the canonical form (17.5) where p, the number of positive terms, is called the index and r
III.

is the rank of the given quadratic form.

Example

4.

In
I

Example
o

2,

the quadratic form


Z
.

into

y^ q" =

"

The non-singular transformation yi = z^, y^ = 23, ys = 22 carries q' 2y^ + 9yQ ~ + ^^3 ^^^ ^^^ non-singular transformation 21 = w^, 22 = w^/S, zg = wa/v^ 922 zf
i.

^12
o

a^ + 2x^

- Ix^ - ixx
3

8xx 1213
+

was reduced

to

J reduces q

to

"'

2,2 w^+ w^ -

w^.

Combining the transformations, we have that the non-singular linear transformation


Xl =

Wi + ^W2 + V^M>3
1

4/3

v^

X2

|mJ2 + 5V^;g

4/3
1/3

2^

X3

3"^
=

qr

to

W^ +

wl-

w%.

The quadratic q

form is of rank 3 and index

2.

SYLVESTER'S LAW OF INERTIA.

In

Problem

5,

we prove

the law of inertia:

IV. If a real quadratic form is reduced by two real non-singular transformations to canonical forms (17.5), they have the same rank and the same index.

Thus, the index of a real symmetric matrix depends upon the matrix and not upon the mentary transformations which produce (15.2).

ele-

The difference between the number of positive and negative terms, p - {r-p), in (17.5) is called the signature of the quadratic form. As a consequence of Theorem IV, we have
V.

Two
if

and only
nature.

real quadratic forms each in n variables are equivalent over the real field if they have the same rank and the same index or the same rank and the same sig-

COMPLEX QUADRATIC FORMS.

Let the complex quadratic form X'AX be reduced by a non-singular


It is

transformation to the form (17.3).


2i

clear that the non-singular transformation


(i

= =

^iVi.
yj
,

1.

r)

Zj

(7

= r+1, r+2,

...,7i)

134

QUADRATIC FORMS

[CHAP. 17

Ul

Y
carries (17.3) into
(17.6)

1
-'-

''''{/h^'/h;'-

1, 1,

)^

<
VI.

-4--

4
field of rank
r

Thus,

Every quadratic form over the complex

can be reduced by a non-

singular transformation over the complex field to the canonical form (17.6).
VII.

Two complex

quadratic forms each in n variables are equivalent over the complex

field if and only if they

have the same rank.

DEFINITE AND SEMI-DEFINITE FORMS. A


n variables is called positive definite

real non-singular quadratic form


if its

rank and index are equal. j\-^ J^-^


-^

positive definite quadratic form can be reduced to

q = X'AX, \a\ f 0, in Thus, in the real field a y^ and for any non-trivial set

of values of the x's, ^ >

0.

real singular quadratic form


i.e.,
r

=X'AX,
n.

\a\

=0,

is called positive semi-definite if its

rank and index are equal,


ratic form can

= p<
"^

be reduced

to

y^ + yj

Thus, in the real field a positive semi-definite quad+ y^ r< re, and for any non-trivial set of values of
,

the

a;'s,

q>

0.

i.e.,

q = X'AX is called negative definite if its index p = 0, = n,p = 0. Thus, in the real field a negative definite form can be reduced to -y^ - y^ - y^ and for any non-trivial set of values of the x's, q < 0.

real non-singular quadratic form

A
i.e.,
r

real singular quadratic form q = <


re,

X'AX

is called negative semi-definite if its index p = 0,


to

p =

-yl

- y^ Clearly,
if

Thus, in the real field a negative semi-definite form can be reduced ^f ^y non-trivial set of values of the x's, q 5 0. ^"^ y^
0.

q is negative definite (semi-definite), then -q is positive definite(semi-definite).

For positive definite quadratic forms, we have


VIII. If q

=X'AX

is positive definite, then

U| >0.

PRINCIPAL MINORS. A

minor of a matrix A is called principal if it is obtained by deleting certain rows and the same numbered columns of A. Thus, the diagonal elements of a principal minor of A are diagonal elements of A.
In Problem
IX.
6,

we prove
r

Every symmetric matrix of rank

has

at least

one principal minor of order

r differ-

ent from zero.

DEFINITE AND SEMI-DEFINITE MATRICES. The


X.

matrix i of a real quadratic form q = X'AX is called definite or semi-definite according as the quadratic form is definite or semi-definite. We have

real symmetric matrix

is positive definite if and only if there exists a non-

singular matrix
XI.

C such C

that

A =

C'C.

real symmetric matrix

exists a matrix

of rank

such that

A of rank r A = C'C.

is positive semi-definite if

and only

if

there
7.

See Problem

CHAP.

17]

QUADRATIC FORMS

135

XII.

If

A A

is positive definite, every principal minor of

is positive.

See Problem
XIII. If

8.

is positive semi- definite, every principal minor of

is non-negative.

REGULAR QUADRATIC FORMS.


principal minors as

For a symmetric matrix A =

[a^,-]

over F,

we

define the leading

Oil

O12

%1
Pa
Ogi
Oqi

04.2

%3
O23
'^3

(17.7)

PO =

1,

Pl=

Oil.

P2 =
021

052

Pn= Ul

022

O32

In

Problem

9,

we prove

XIV. Any re-square non-singular symmetric matrix A can be rearranged by interchanges


of certain rows and the interchanges of corresponding columns so that not both p_^ and Pn-2 are zero.

XV. If A is asymmetric matrix and have opposite signs.

if

Pn-iPn^^
1

but Pn-i = 0,

then p_2

and p

Example

5.

For the quadratic form

X'AX

112 112
2

X, 2
3

4
1

po =

1,

pi =

1,

p2 =

0,

p3 =

0,

P4 =

m=

1.

12
Here
Oigs
t^

0;

the transformation

Kg^X

yields

'1112 112 2
(i)

X'

12 14
2 2 4
1-

for

which po =

1,

Pi =

1,

P2 =

0.

Ps = -1.

P4=

Thus,

for

i) not both p2 and ps are zero.

in the

symmetric matrix A of rank r is said to be regularly arranged if no two consecutive p's sequence po.pi, -.Pr ^^^ zero. When A is regularly arranged the quadratic form X'AX is said to be regular. In Example 5, the given form is not regular; the quadratic form (i) in the same example is regular.

Let i be a symmetric matrix of rank

r.

By Theorem

IX,

A contains

at

least one non-

vanishing r-square principal minor M whose elements can be brought into the upper left corner of A. Then p^ ^^ while p^^.^ = p^^.^ = ... = p = 0. By Theorem XIV, the first r rows and the first r columns may be rearranged so that at least one of Pr-i and p^_2 is different from zero. If p^_^ j^ and p^_2 =0, we apply the above procedure to the matrix of Pt--i; if Pt--2^0,

we apply

the procedure to the matrix of P7-_2;

^^^^

so on, until

is regularly arranged.

Thus,

XVI. Any symmetric matrix (quadratic form) of rank

can be regularly arranged.

See Problem
XVII. A real quadratic form X'AX is positive definite leading principal minors are positive.
XVIII.
if

10.

and only

if its

rank is

re

and

all

real quadratic form

X'AX of rank
Pr

is positive semi-definite if and only if

each of the principal minors po.Pi

is positive.

136

QUADRATIC FORMS

[CHAP. 17

KRONECKER'S METHOD OF REDUCTION. The


into one in

method of Kronecker which only the squared terms of the variables appear

for

reducing a quadratic form

is

based on
r,

XIX.

q =

X'AX

is a quadratic form over

in n variables of rank

then by a non-

singular linear transformation over F it can be brought to q' = singular r-rowed minor C oi A occupies the upper left corner of B.

X'BX

which a nonMoreover, there exists


in

a non-singular linear transformation over quadratic form in r variables.

F which reduces

q to

q" = X'CX, a non-singular

XX. li q = X'AX is a non-singular quadratic form over F in n variables and a ^ 0, the non-singular transformation

if

Pn-i =

a
^nnyn
or

1, 2.

re-1)

10
1

a^n

BY
1

<^n-i,

carries q into

HiJiYj + Pn-xPnYn

in

which one squared term in the variables

has been isolated.


1

-2
2

4
1
-^i

-2
-2 ^ 0.

Example

6.

For the quadratic form

X'AX

= X'

-2 4

P2 =

O^S

The

0-7
- 8y3

-2

non-singular transformation
Xl

n
= =

a.13

ys

yi

1 1

-8 -8

ys +

a23 ys
ss ys
=

ys -

8y3
2ys

xs

0-2

reduces X'AX to
1
1

-2
2

1 1

-8 -8
Y'

-2
2

Y'

-2
4

-2

8 -8 -2

0-7
in

0-2
squared form

36

in

which the variable yg appears only

XXI.
but

U
'

q =
0,

a, 're,?!-! ^

X'AX is a non-singular quadratic form over the non-singular transformation over F


"

F and

if

o^-i, n-i

= ^nn

xi
*ra-i

yi

CLi,n-iyn-i

'^inyn,

(i

1, 2,

...,n-2)

^n-i.n yn.

Xn

a n n-i Yn-
,

or
1 1
... ...

!, ^.

a2 n-1
1
a.-2,
_i n-1

sy

= a '-'-n.-2, n
'-n-i,
...

,0

...

a__i

CHAP.

17]

QUADRATIC FORMS
n-2

137

n-2

carries 9 into

(Hjyiyj

i-i j-\

1'^n,n-\Pnjn-\yn-

The

further transformation
'

Ji

Zi,

(i

1,

n-2)

Jn-x

= =

Zn-i Zn-i

Zn

2n

Jn
n-2

n-2

yields
V

2
1

.2
J -1

a^jz^zj +

2a_^_j^p(zn-i

- z^

in

which two squared terms with opp-

site signs are isolated.

Example

7.

For the quadratic form


1

X'AX

X'

2
1

4
3

3
1

^22

- O'ss =

but
xi = = =

tXg2 =

-1
+

i^

0.

The non-singular transformation


+

yi

cti272

*i3ys
*23r3
or

X2 xq

-1

a 32 72

0-10
]

reduces X'AX
1 1

to
1

1 1 1

-1
-1

2
1

4
3

3
1

rSY

yl

+ 2y,y3

-]

The transformation
)'l
1

= = =

21

72

22

23 23

1 1

-1
1

73
carries

22 +

Y'BY
1

into
1
1

o'
1 1

1 1 1

Z'

1
1

-1
1

z'

<
-2

-1

2zi

Consider now a quadratic form in n variables of rank r. By Theorem XIX, q can be reduced to 9i = X'AX where A has a non-singular r-square minor in the upper left hand corner and zeros elsewhere. By Theorem XVI, A may be regularly arranged.
If

pr-i

0,

Theorem

XX

can be used to isolate one squared term

(17.8)

Pr-iVrYr

If pr-i = but a.r-1, r-i ^0, interchanges of the last two rows and the last two columns yield a matrix in which the new pr-i = a^_i^^_i f 0. Since pr-2 ^ 0, Theorem XX can be used twice to isolate two squared terms

(17.9)

Pr-2

r-i,

r-iJ

a r-i., r-iPrYr

which have opposite signs since pr-2 and pr have opposite signs by Theorem XV.

138

QUADRATIC FORMS

[CHAP. 17

If

pr-L =

used

to

and a^_i_r-i = 0, isolate two squared terms

then (see Problem 9)

a,-,

r-i ^

and Theorem XXI can be

(17.10)

2ar,r~ipr{yr-i-yr)

having opposite signs.

This process may be repeated

until the given quadratic form is

reduced

to

another con-

taining only squared terms of the variables.

sequence pr-i,Pr presents a permanence or a variation of sign. In (17.9) and (17.10) it is seen that the sequences p^_2, oLr-i. r-v Pr ^"^^ Pr-2' ^r, r-v Pr Present one permanence and one variation Thus, of sign regardless of the sign of dr-i, r-i and a.^, r-i
In (17.8) the isolated term will be positive or negative according as the

XXII. It q = X'AX, a regular quadratic form of rank r, is reduced to canonical form by the method of Kronecker, the number of positive terms is exactly the number of permanences of sign and the number of negative terms is exactly the number of variations of
sign in the sequence
po, pi, P2

P^,

where a zero in the sequence can be counted either


See Problems 11-13.

as positive or negative but must be counted.

FACTORABLE QUADRATIC FORMS.


ratic form.

Let X'AX f

0,

with complex coefficients, be the given quad-

Suppose X'AX factors so that


(1)

X'AX

(oj.%1

+ 02*2 +

+ ;)( iiXi +

b.2_X2

+ + b^x^)
0-1

to.;

hi
J

tj

.^

is non-singular.

Let the

becomes

The non-singular transformation


yx

= = =

OiaCi

+ 02*2 +

+ 0 + ^n^n

y2
ys

^1*1 + ^2^2 +

Xs
(

=
2.

x^

transforms
If

i )

into

y-^y-z

of rank

2.

Thus,

1) is of rank

the factors are linearly dependent, at least one element a^^f 0.


.

Let the variables and

their coefficients be renumbered so that o^ is ci

The non-singular transformation

!Y^
72

=
=

a^Xx + 02*2 + X2

+ (^n^n

Xn
1.

transforms (i) into

y?

of rank

1.

Thus, (1) is of rank


1

Conversely,

if

X'AX has rank

or 2

it

may be reduced respectively by Theorem VI

to y^ or

r^ + y^, each of which may be written in the complex field as the product of two linear factors.

We have proved
XXIII.

quadratic form
if

X'AX ^
rank is

with complex coefficients is the product of two

linear factors

and only

if its

r<2

CHAP.

17]

QUADRATIC FORMS

139

SOLVED PROBLEMS
12
1.

3 5

2
8

Reduce

q = X'AX = X'

2 3 2

3 5 8

10

to the

form (17.3).

10-8

Prom Example

1,

Chapter

15,

12
[^1/]
2 3 2 3 5 8

3 5 8

2 8

1 1 1 1

c
^\J

0-100
4

10 -2100
-10
iD'.P']
4
1

10

10

-8
1

-1-110
Y
reduces q to the required form y^

-2
1

-10 -1 4 -1
1 1

Thus, the transformation

PY

yl^'^yl-

2.

Reduce

q = X'AX = X'

2
2

4 8
8 4

to the

form (17.3).

We

find
1

[^1/] C

[D\P']

2
1

i
2

-4
1
1

Thus, the transformation

PY

-i Y
2

reduces q to y^ + 8y^ - 2y^

3.

Lagrange reduction.
(a)

= =

2 ,...2 2 2x^ + 5^2 + , 19*3


. ,

- 24x^+

8a:^a=2

+ I2x^x^ + Sx^x^ + l%x^x^ - 8x^x^ - IGx^x^

2{xf+ 2x^(2x^ + 3x^+2x^)1 + 5x^+ 19x1 " 24*4 + ^S^j^g - 8x^x^ - IGx^x^ 2{xl+ 2x^(.2x^+Zx^+2x^) + {2x^ + 3x^ + 2x^f\
+ bxl + 19*2 - 24*2 ^ jg^^^^ ~ 8*2^^ g^^^^ _ ^^ "2*3 _

_ 2(2*^ + 3v +2*,)2
^q_^

2 (*i 2 (*i

+ 2*2 + 3*g + 2*^)2 - 3 + 2*2 + 3*g + 2*^)2 _


3

{*^ + 2*2(*g+ 4*4)

+ *3 - 32*2 -

(^ + ^ + 4^^)2
ryi

+ 4 (*g

- 2*4)2
+ 2*4

*i + 2*2 + 3*3

Thus, the transformation

\yi =
Irs
=

*2 +

*s + 4*4

*3-2*t
*4

'Educes,

to

2y2_3y22+4y2.

00^

(i)

For the quadratic form of Problem


9

2,

we have
16*2*3 + **
,

*2 +

4*^*2 +

4*^*3 + 4*2 +
tenii in

(*^

+ 2*2 + 2*3)2 + 8*2*3

Since there is no term in *2 or *2 but a

*2*3

we use

the non-singular transformation

140

QUADRATIC FORMS

[CHAP. 17

i )

Xl

Zl

A;2

22

Xs -

Z2 + ^3

to obtain

(2i

+ 4z2+2z3f'

+82^+82223

(z^ +

422+2z3)2+8(22+2Z3f - 2z2
-

y^

+ ^y^ -

2yi^

4 2
1

1 1

4 2
1

1 1

Now Y

X
2
1

Z and

from

i),

X\ hence,
1 6"

i
1

X
1

ii
-1
1

-1
'1

-1

-4
1
1

Thus, the non-singular transformation

2
I
1

y effects the reduction.

4.

Using the result of Problem

2,

[^1/] C

-4
-2

-2

i
2

and applying the tiansfoimations HgCiV^). K2(i+-\/2) and ^3(5 a/2). ^3(2
-|

V2

),

we have

10
[^!/]

1
1 1

10
0-1

10
[cic']

0-2

-2
1

-iVl

ii\/2

-V2
k^f2 -\\f2 y i\f2
reduces
q = X'

2 4 8

Thus, the transformation

= (^y

2 2

8 4

to the canonical form

2,2

iv^

,2

5.

Prove:

If

a real quadratic form q is carried by two non-singular transformations into two distinct

reduced forms
(i)

y; +

yg

y^

and
<ii)

y' + yl +

yq

yq,i

yj.2

y'

then p = q

Suppose q>p. Let mation which produces ( ii

X
).

FY

be the transformation which produces (1) and

GY

be the transfor-

Then
filial

+ +

bi2X2 +
622*^2

+ binxn + b^nXn

621X1

F~^X

bjiixi

+ bn2X2 +

+ bnnpcn

and

cii% + ci22 +
C21X1 + C222 +

Cin^n

+ C2ren

G'^X
cr!.i;i

cn2*:2

cnnXn

CHAP.

17]

QUADRATIC FORMS

141

respectively carry (i) and (ii) back into

q.

Thus,

(iii)

(''ii*i

*i2*2+- +

*in^n)^ +

...

+ {bp^'^^ + bp^x^-\-- + bp^Xyf

(CiiA:^

+ c^2*2+- + c^^f +
('^g

"*"

+ (cq^x^ +
'^q

x^+- + c

x^f

Consider the
*ii

+ l.l^l'''

"^g

+1,2*2

+ l,/!^^

r-q+p <

equations
^g+i,i*i + '^(7+1,2*2 +
't?+2,l*l

+ 6i2 *2 +

+ *in*n = + 6^

'^g

M*re =

''21*1+^22*2 +

+ '^9+2,2*2 +

bpiH
By Theorem

+ *;^2*2 +

+ 6jbn% =

^ri

*i

+ ^2*2

+ c^^x^

IV,
(

substituted into

Chapter 10, they have a non-trivial solution, say iii ), we have


+ ^^+1,2*2 + i +

(Ot^.ttg

0!^^).

When

this

solutionis

(*;b+i,i'i

+ bp^i,rf-J^

(Vll + V22+--- + V?In)


+ (<^7ii + ^(?22 +

(-^11

%22 +

+ c.^n'^nf +

>an)^

Clearly, this requires that each of the squared terms be zero.

But then neither F nor G is non-singular,

contrary to the hypothesis.

q<p

will also lead to

q<p. A repetition a contradiction. Hence q=p.


Thus,

of the above argument under the

assumption that

6.

Prove: Every symmetric matrix A of rank


from zero.
Since A is of rank
r,

has

at least

one principal minor of order

different

it

has

at least
'rfi,s2

stands in the rows numbered Ji,i2 matrix and let the columns numbered

I^^t

one r-square minor which is different from zero. Suppose that it these rows be moved above to become the first r rows of the i^ be moved in front to be the first r columns.

rows are linearly independent while all other rows are linear combinations of them. By taking proper linear combinations of the first r rows and adding to the other rows, these last n-r rows can be reduced to zero. Since A is symmetric, the same operations on the columns will reduce the last n-r columns to zero. Hence, we now have
the first
r

Now

ili2

H2i2

"Vii

%i2

"vv

in

which a non-vanishing minor stands

in the

upper left hand corner of the matrix.

Clearly, this is a princi-

pal minor of

7.

Prove:
matrix

A
C

real symmetric matrix


r

of rank

is positive semi-definite if

and only

if

there exists a

of rank

such that A = C'C.


r,

Since A is of rank

its canonical form

'--[t:]

Then there exists a non-singular matrix B

142

QUADRATIC FORMS

[CHAP. 17

such that A
of rank
r

B%B.
= C'C

Since

A'i'=

Ai = A^

we have A

= B'NxB = B'N^Ny^B = B'N(-N^B

Set

= A^B; then

is

and A

as required.
r;

Conversely, let C be a real n-square matrix of rank


form be

then A = C'C is of rank s<r.

Let

its

canonical

N^

diag(di,

(^2

^5,0,0

0)

Set

where each di is either +1 or -1. Then there exists a non-singular real matrix E such that E'(C'C)E = CE = B = [bij\. Since B'B = N^, we have
2

A^g

2 (i =
1,

s)

and
*ii

*i2

^jn

0'

ii =

s+l,s+2

n)

Clearly, each

d;>0 and A

is positive

semi -definite.

8.

Prove:

If

is positive definite, then every principal minor of

is positive.

Let q = XAX. The principal minor of A obtained by deleting its ith tow and column is the matrix A^ of the quadratic form q^ obtained from q by setting x^ = 0. Now every value of qj_ for non-trivial sets of values of its variables is also a value of g and, hence, is positive. Thus, Aj_ is positive definite.

three,

This argument may be repeated for the principal minors ... rows and the same columns of A.

A^j,

A^^j-^, ...

obtained from A by deleting two,

By Theorem

VI,

Ai>

0,

A^j>

0,

...

thus, every principal minor is positive.

Prove: Any ra-square non-singular matrix A = [ay] can be rearranged by interchanges of certain rows and the interchanges of corresponding columns so that not both p_^ and p_2 are zero.
Clearly, the theorem is true for A of order
1

and of order

2.
?^

Moreover,

it is

true for

of order

n>2

when

p^_^ =

a^
(a)

j^

0.

Suppose
?^

01^ =

then either (a) some a^^

or (b) all a^^ = 0.

Suppose

some CL^

0.

of the last row and the last column, the

After the ith row and the ith column have been new matrix has p_^ = CC^j^ 0.
at

moved

to

occupy the position

Suppose (6) all a^i = 0, Since \A\ J^ 0. and the ith column into the (n-l)st position.
a a

In the

least one a^i i^ new matrix

0.

Move

the ith row into the (n-l)st position

_i,n =

\n-i

^-

^^

(^-^^^ '^ ^^''^

^n-i,n-i

^n-i,n

Vi,n

-a.n-i,n

PnsPn

a n, n-i
and

Vi,n

Pn^^O.
Note that this also proves Theorem XV.

2
10.

1 1 1

Renumber the variables so

that

q =

X'AX = X'
2

13
3

is regular.

1111
Here

po
2

1,

pi =

0,

ps =

0,

ps = -4, p* = -3.
2

Since

pi = P2 =

0,

ps ^

0,

we examine

the matrix

3 4

of p3

The cofactor B22

=
2

?^

the interchange of the second and third rows and of

CHAP.

17]

QUADRATIC FORMS

143

the second and third columns of

A yields
2
1

r
for

4 3

1111
which p^ 1,

11
;<:g

p^ =

0,

p^ = -4, pg = -4, p^ = -3.

Here, X2 has been renumbered as xq and

as xg

11.

Reduce by Kronecker's method

X'

2 3

12 3 15
5

4
6
3

X.

2 3

Here Pq - 1, p^ - 1, p^ and q is regular. The sequence of p's presents one 3, pg - 20, p^ = -5 permanence and three variations in sign; the reduced form will have one positive and three negative terms.
Since each pj /
0,

repeated use of Theorem

XIX

yields the reduced form


=

PoPiTi + PiPgrl + PsPsrl + PsP^rl

y\

"^yl

- eo^g -

looy^

12
12.

3 6 9 2

Reduce by Kronecker's method

= X'AX = X'

2 3

4
6

3 2 5

X.

13
Here A is
of rank 3 and
fflgg

i 0.

An interchange
1

of the last two rows and the last two columns carries

12 13
A
into

2 4 3 6

12 10
^
0.

13

in
5

which

2
1

4 3
3 5

Since S is of rank

3, it

can be reduced to

2 4 3

13

3 6 2 9
1

2 1 4 3
3 5
1

Now

has been reduced

to

XCX

= X'

2
1

for

which p^ =

1,

p^ =

1,

p^ =0, pg = -1.

The reduced

form will contain two positive and one negative term.


form is by (16.8)

Since p =

but
1

= 4 5

i^

0,

the reduced

PoPiri +

Pxy-i-zJi

y-iiP^yl

yl + 4y| -

4y|

1-2
13.

Reduce by Kronecker's method

= X'

-2

2-12
Here p^
- 1, p^ 1,

1112
1
;

1-1

X.

p^ -

0,

Pg

9,

p^ = 27

the reduced form will have two positive and two nega-

-2
4
I

1 1

tive terms.

Consider the matrix B

2
1

of pg.

Since /^gg =

but /3g2

=-3^0

the reduced for

ij

is by (16.8) and (16.9)

PoPiTi^ + 2/332 Pg (y| - y|) + pgp^y^

yl + 54y|

54^3^

- 243y|

144

QUADRATIC FORMS

[CHAP. 17

14. Prove:

For a set of real ra-vectors Xi, X^,


Ai

Xp,
A]_

Ai

A2

Ag Ai
*

A2 ^2
'

;t2-x >

Z^ Zi

Xp- X2

Xp- Xp

the equality holding


(a)

if

and only

if

the set is linearly dependent.


P
let

Suppose the vectors Xi are linearly independent and


and

X
=

= [i,2

xp]' f 0.
=

Then Z

2
i=i

X^x^

0<Z-Z

1=1

X-x.

)( 2 X:xA
j=i
J
3

X\X':XAX t-j'
|

r(A:^-;f,-)^ t
J'

X'GX

Since this quadratic form is positive definite,

G >
|

0.

(6)

Suppose the vectors X^ are linearly dependent.


P

Then there

exist scalars

k^^^.k^

kp, not all zero,

such that

S f = i=i
X..^

4,,jy.^

and, hence, such that

kiXjj-Xi

+ k^Xj

X2 +

+ kpXj

Xp

= 1.2

p)

Thus the system

of

homogeneous equations

Xj-X^xx + Xa-Xqxq +
'J

+ Xa-X, 'j
'

P ^P
0.

= 1,2

p)

has a non-trivial solution xi=

k;

(i = 1,2

p),

and \G\

We have proved

that

G >
1

0.

To prove the converse of


;?,

(b).

we need only
p
i= 1

to

assume
Thus,
J

=
|

and

reverse the steps of (6) to obtain

= 0, (/ = 1,2

p)

where

f= l^k^Xi.

S kjXy^ =
-

^.

^=

0,

if

= 0,

and the given vectors Xj are linearly dependent.

SUPPLEMENTARY PROBLEMS
15. Write the following quadratic forms in matrix notation
(a)

xl + 4*1^2 +

34
^JX

(6)

2x1 -

6j;i2

^s

<<^)

^1

" 2^| -

3a:

+ 4j:i%2 + 6;c^*g - ^x^x^


3

2-3
Ans.
(a)

12
(c)
1

rP

(b)

X'

-3

2 3

-2 -4 -4 -3
2

-3
2

16. Write out in full the quadratic form in

xi.x^.xs

whose matrix

is

-3
1

-5

Ans.

2x1

- 6x^x^ +

2x.^Xg

+ 2x^ + 8X2^3

5<

17.

Reduce by the method


1

of

Problem

and by Lagrange's Reduction:

4
(6)

1111
X

(a)

X'

2 4

6-2 X
-2 18
yl

1-13-3

012
X
(c)

13
2^2^

r
y^

-1

(d)

^
1

-2
3

1-3 1-3
(6)
y,=

2-10
(c)

-2

Ans.

(a)

+ 2y^ - 48y|

+ 4y^

,,

72 + 8yg

id)

2 Ji

2,2
-y^z^ya

Hint. In (c) and (d) use

a;i

= zg, X2 = zi, xg = Z2.

CHAP.

17]

QUADRATIC FORMS

145

18.

(a)

Show

that

X'\

\x = ^'

\x but the matrices have

different ranks.

(b)

Show

that the symmetric matrix of a quadratic form is unique.

19.

Show

that over the real field

xf+xl- ixl+ix^x^ and 9xf+2x^+2xl + 6x^x^-6x^x^-8x^x^

are equivalent.

20.

Prove:

real symmetric matrix is positive (negative) definite if and only if it is congruent over the real

field to / (-/).

21.

Show that X'AX Theorem XIX.


(a)
(6)

of

Problem 12

is

reduced

to

X'CX hy X

RX.

where

Ks^K^^{-5)K^^a).

Then prove

22.

Show

Show

two real quadratic forms in the same variables are positive definite, so also is their sum. that if q^ is a positive definite fonn in x^.x^ xs and 92 is a positive definite form in xs^i. xs+2. ..x^, then g = ?i + 92 is a positive definite form in xx,x2 x^.
that if

23.

Prove: If

Hint: Consider

is any real non-singular matrix, then C'C is positive definite. XlX = Y'ClCY

24.

Prove: Every positive definite matrix


of

Theorem

X.)

Hint: Consider

A can be D'AD=I.

written as 4 =

CC. (Problems

23 and 24 complete the proof

25.

Prove: If a real symmetric matrix A is positive definite, so also is A^ for p any positive integer.
Prove: If ^ is a real positive definite symmetric matrix and CB is orthogonal.

26.

if

B and C

are such that

B'AB =1 and A

= C'C

then

27.

Prove; Every principal minor of a positive semi-definite matrix A is equal to or greater than zero.

28.

Show

that

ax^ - 2bx^x^ + cx\ is positive definite

if

and only

if

a>0 and \a\

ac-l^ >0.

29.

Verify the stated effect of the transformation in each of Theorems

XX

and XXI.

30.

By Kronecker's

reduction, after renumbering the variables

when necessary, transform each

of the following

into a canonical form.


1

-1
2

(a)

X'

-1

-1
2

(c)

12 12 12 r
2

10-2
(e)

0-1

1112
2 2 2 3
1

X'

(g)

X'
1

-2
3_

-2
1

-2
2

4-4
(b)

X'

-4
2

-3 X
1

(d)

X'

2-462
3

2 -1"
2

-3

9
3

3
1

(/)

X'

4
3

2 3

(h)

X'

12

-12

11

1111
17(rf).

Hint: In (g), renumber the variables to obtain (e) and also as in Problem

Ans.

(a)

p^=p^=p^=p^=l;
iyl - 16y| + 16y|

+ y| + y2 yf

^^j
(/) (g) (A)

Pq = Pi =

22 =

-!

P3

-1

2 Ti

2,2
72
"^

^3

(b) (c)
(d)

Po =

Pi=l. 2S=-4. P3
(e).

-16;

y^^

+ 128y|- 128y=

yl jI
q

^yl
8y,

+ 4y= - 3y2

See

4y2 - 16y| + 16y| + 12y^

31.

Show

that

xf-

Gxl -

6^

- Zxl - x^x^ -

xx

2xx

Uxx

XUx 2

4-

^xx^ can be 3
4-

factored

chapter 18
Hermitian Forms

THE FORM DEFINED


(18.1)

by

_
h

X'HX

S
v=\

S
j=i

__

_
hij

h^jXiXj,

= hj^

where

is Hermitian

and the components of


of

are in the field of

complex numbers, is called an


If

Hermitian form.

The rank

is called the rank of the form.

the rank is Kra, the form is

called singular; otherwise, non-singular.


If

and

are real, (18.1) is a real quadratic form;

hence,

we

shall find that the theorems

here are analogous to those of Chapter 17 and their proofs require only minor

changes from

those of that chapter.


Since H is Hermitian, every h^i is real and every h^^^x;^ is real. cross-products h^j'x^Xj and hj^XjXj^,
h^jx^xj
is real.

Moreover,

for

the pair of

hjiXjXi

hijXiXj

+ hijXiXj

Thus,
1.

The values

of an Hermitian form are real.

The non-singular linear transformation X = BY carries the Hermitian form (18.1)


other Hermitian form
(18.2)

into an-

{mH{BY)

Y(WHB)Y

Hermitian forms in the same variables x^ are called equivalent if and only if there exists a non-singular linear transformation X = BY which, together with Y = IX, carries one of the forms into the other. Since B'HB and H are conjunctive, we have

Two

H. The rank of an Hermitian form is invariant under a non-singular transformation of


the variables,

and
HI.

Two

Hermitian forms are equivalent

if

and only

if their

matrices are conjunctive.

REDUCTION TO CANONICAL FORM. An


form
(18.3)

Hermitian form (18.1) of rank

can be reduced

to

diagonal

Kjxn

k^y-iLji

'crfryr'

^i^^

and real

by a non-singular linear transformation X = BY. From (18.2) the matrix B is a product of elementary column matrices while B' is the product in reverse order of the conjugate elementary row matrices.

By a
(18.4)

further linear transformation, (18.3) can be reduced to the canonical form [see (15.6)]

%Zi + i^22 +

+ 'zpzp

z^+i2^ + i

z-rV

146

CHAP.

18]

HERMITIAN FORMS

147

of index p and signature p-(r-p). Here, also, p depends upon the given form and not upon the transformation which reduces that form to (18.4). IV. Two Hermitian forms each in the same n variables are equivalent if and only they have the same rank and the same index or the same rank and the same signature.
if

DEFINITE AND SEMI-DEFINITE FORMS. A


is called positive definite if its^rank

mitian form can be reduced to


the x's, A>0.

non-singular Hermitian form h = X'HX in a variables and index are equal to n. Thus, a positive definite Hery^y^ + y^y^ + +Jnyn and for any non-trivial set of values of

A
a.:e

singular Hermitian form h =


i.e.,

equal,

rj= p <

n.

yiTi + y2y2 +

+ yrYr

r<

n,

X'HX is called positive semi-definite if its rank and index Thus, a positive semi-definite Hermitian form can be reduced to and for any non-trivial set of values of the x's, h>0.

H of an Hermitian form XliX is called positive definite or positive semidefinite according as the form is positive definite or positive semi-definite.
The matrix
V.

An Hermitian fom

is positive definite if and only if there exists a non-singular

matrix C such that


VI. If
VII.
If

= C'C.

is positive definite, every principal minor of is positive semi- definite,

is positive,

and conversely.
non-negative, and

every principal minor of

is

conversely.

SOLVED PROBLEM
1 1. 1

+ 2i
5

- 3i
to

Reduce

X'

\-2i
2 + 3i

-4-2i X
2f
15,

canonical form (18.4).

-4 +

13

Prom Problem
1

7,

Chapter
2j

+
5

2-Zi
-4 2j

I I

o"

'l

I I

0'
i/^To
j/a/TO

1-2/
2

HC
'^^

2/\/io

j/yTo

3i

-4 +

2i

13

l_

-1

(-4-4j)/v^

1/vTo

Thus, the non-singular linear transformation


1

2/VlO
1/VIO

(-4 +

4i)/V^

BY
,0

-i/yflQ

-j/VTo

1/v^

reduces the given Hermitian form

to

Jiyr + Jsys - ysys

148

HERMITIAN FORMS

[CHAP.

18

SUPPLEMENTARY PROBLEMS
2.

Reduce each

of the following to canonical form.


1

3i

2
2

+
4

3i 3i

[1
l-2i

+ 2il
2

(c)

+ +
I

3j
3j

3t

1-i

2i

(b)

x'\

i"
For

jx :]^
(6), first

(d)

1+j
3

2-J
2

2J

+J

4
first row.

Hint:

multiply the second row of

by

and add to the

Ans.

{a)

(-l-20/\/3l^
=

tl

(.b)

X V2
1

nyi - y-m
(-1 + 30/3
1/3

-l"

(c)

-1
1

nn

- 7272

'l

(-l+f)/A/2

(-l +

30/\/^
y
;

(d)

=
.0

1/^2

(-3-2J)/\/lO

J<iyi

- 7272 " 7373

2/VlO

3.

Obtain the linear transformation

X
I
I

BY

which followed by Y = IX carries

(a) of

Problem 2 into

(6).

Ans.

X= -^\

j_ri
v/2"Lo

Ti (-i-2i)/v3i :^'
~

n
\y
ij'

l/v^

Jb

1+i

-1
-3+J
11

1+j

l+2i
5 10

4.

Show

that X'

1-t

is positive definite

and X'

1-i

3 5

is positive semi-definite.

-1

-3-J

l-2i

5.

Prove Theorems V-VII.

6.

Obtain

for

Hermitian forms theorems analogous to Theorems XIX- XXI, Chapter

17, for

quadratic forms.

xi
X-L

X2

^n
hin

All
/J21

hi2 h22

n
_S Tj^-Xj^Xj

7.

Prove:

^2

h^n

where

77-

is the cofaotor of h^: in

= \h^j\

1=1 j=i

^n
Hint:

K.

K.

^nn

Use

(4.3).

chapter 19
The Characteristic Equation of a Matrix
THE PROBLEM.
Let Y = AX,
where A = [o^],
is

(i,j

= 1,2

n).

be a lineal transformation over F.

In general, the transformation carries a vector

Z=

[%i,%2

x^Y

into a vector

Y =

\.%,J-z

JriY

whose only connection with


possibility of certain

X vectors X

through the transformation.

We

shall investigate

here the

a scalar of

or of

some

field 3F of

being carried by the transformation into KX, where A is either which F is a subfield.

Any vector X which by


(19.1)

the transformation is carried into KX, that is, any vector

for

which

AX

XX

is called an invariant vector under the transformation.

THE CHARACTERISTIC EQUATION.

From

(19.1),

we

obtain

A di^
(19.2)

O^ 2
^ f^2

\X-AX

(XI-A)X

^21

^271

~
The system
of

On2

X a-nn

homogeneous equations (19.2) has

non-trivial solutions if and only if

(19.3)

\XI-A\

A Oil a-zi
c-ni

Oi2 A 022 On2

0-tn

ci^n
A Onn

The expansion of this determinant yields a polynomial 0(A) of degree re in A which is known as the characteristic polynomial of the transformation or of the matrix A. The equation <^(X) = is called the characteristic equation of A and its roots Ai,A2 A are called the characteristic roots of i. If

A = Ai is a characteristic root, then (19.2) has non-trivial solutions which are the components of invariant or characteristic vectors associated with (corresponding to) that root.
Characteristic roots are also known as latent roots and eigenvalues; characteristic vectors

are called latent vectors and eigenvectors. r2

Example

1.

Determine the characteristic roots and associated invariant vectors, given A

A-2
The characteristic equation
is

-2

-1 -1
1,

A-3
-2

-1 -1

A^'

- tA^ + llA

and the

A-2

characteristic roots are

Ai=

5,

A2=

A3=

1.

149

150

THE CHARACTERISTIC EQUATION OF A MATRIX

[CHAP. 19

When A

Ai =

5,

(19.2)

becomes
1

3
-1 -1

-2

-ll Vxi
^2
xs
=
or
1

-1
-1

Xl

2-1
-2
3

X2

_X3

-2 -1
2

-1
-1

since

-1

-1
3

is

row equivalent

to

-1

-2

solution is given by
is the

A=5

hence, associated with the characteristic root x-i = x^ = xq = i one-dimensional vector space spanned by the vector [l,l,l]'. Every vector
\

[k.k.kY of this space is an invariant vector of A.

When A

A2=1,
1

(19.2)

becomes
Xl

-2 -l'
-2 -2
-1
-1

1
1

X2
_X3_

Xl + 2x2

+^3

Two

linearly independent solutions are (2,-1,0) and (1,0,-1).


1

Thus, associated with

the characteristic root A =

and X2 = [1,0,-1]'.

Xi = [2,-l,0]' Every vector hX^+kX^ = [2h+k,-h,-ky is an invariant vector of A.


is the

two-dimensional vector space spanned by

See Problems

1-2.

GENERAL THEOREMS.
I.

In

Problem

3,

we prove a special case

(A:

3)

of

Xj^ As Xk ^f^ distinct characteristic roots of a matrix A and if Xi, X^ the X's are linewith these roots, respectively vectors associated invariant are non-zero If Ai,

arly independent.

In

Problem

4,

we prove a special case

(ra

3) of

k\

where A is re-square, with respect to A is II. The Ath derivative of cfi(X) = \XI A\ times the sum of the principal minors of order n-k of the characteristic matrix when k<n,
,

is

re!

when k = n, and
of

is

when k>n.

As a consequence
III.

Theorem II, we have

If

is not less than

Ai is an r-fold characteristic root of an re-square matrix A, the rank of Xil A nr and the dimension of the associated invariant vector space is not
r.

greater than

See Problem

5.

In particular
Iir.

is

re

Xi is a simple characteristic root of an re-square matrix A, the rank of Xil and the dimension of the associated invariant vector space is 1.
If
2 2

1 1

Example 2. For the matrix A

1 1

3
2

of Ex.

1,

the characteristic equation is

<^(A) =

(A-5)(A-1) =

0. The invariant vector [l,l,l]' associated with the characteristic root A=5 and the linearly independent invariant vectors [2,-l,o]' and [l,0,-l]' associated with the multiple root A=l are a linearly independent set (see Theorem I).

The

invariant vector space associated with the simple characteristic root

A=5

is of

CHAP.

19]

THE CHARACTERISTIC EQUATION OF A MATRIX

151

dimension
multiplicity

1.

2,

is of dimension 2 (see

The invariant vector space associated with Theorems III and III').

the characteristic root

= l, of

See also Problem

6.

Since any principal minor of A" is equal to the corresponding principal minor ot A, we have

by (19.4) of Problem
IV.

1,

The characteristic

roots of

A and

A' are the same.

Since any principal minor of A is the conjugate of the corresponding principal minor of A,

we have
V.

The characteristic roots of A and

of

/I'

are the conjugates of the characteristic

roots of A.

By comparing
VI. If

characteristic equations,

we have
if
A;

Xi,

A2

scalar, then
VII.
If

AAi,

kX^

A are the characteristic roots of an ra-square matrix A and AA are the characteristic roots of kA.

is a

Ai, A2, ...,

scalar, then

A are the characteristic roots of an n-square matrix A and \-Lk,\2k \ik are the characteristic roots of 4 -A/.

if

& is a

In

Problem
VIII.

7,

we prove
is

If

a characteristic root of a non-singular matrix A, then \A\/a is a character-

istic root of adj A.

SOLVED PROBLEMS
1. If

is re-square,

show

that

(19.4)

0(A) (m =
1,

|A/-^|

A" + 51 A"-^ + 52 A"-^ +

...

+ s_,A + (-l)"|i|

where s^

re-1) is (-1) times the

sum

of all the m-square principal minors of A.

We

rewrite \\! - A\ in the form

A-aii
0-021

0-ai2

a-in

A-a22

0-a2n
Xar

O-'^ni

O-Ons

and, each element being a binomial, suppose that the determinant has been expressed as the sum of 2" determinants in accordance with Theorem VHI, Chapter 3. One of these determinants has A as diagonal elements and zeroes elsewhere; its value is A". Another is free of A; its value is (-1)"|^| The remaining determinants have m columns, {m = 1, 2, ....- 1), of -A and n~m columns each of which contains just one non-zero element A.

Consider one of these determinants and suppose that


of

its

-A

columns numbered h,

ia

are columns

terminant becomes

After an even number of interchanges (count them) of adjacent rows and of adjacent columns, the de-

152

THE CHARACTERISTIC EQUATION OF A MATRIX

[CHAP. 19

%.ii
"is.ii

"iiA

"inA
(-1)'"

"imA

"im.-im
I

'i.'2

^n

A
"in-k

..

%.fe

%''''OT
..

A
.

t^.to

im!

where

Aj

i ^'2

i\

is an m-square principal minor of

-4

Now

(-1)"
n (n

S ^'^
P

%
^
n taken

H.^

as

(ii, is

i^

runs through the

1)
...

(n

-m + 1)

different combinations of 1, 2

at a

time.

-4 -1 -4

2.

Use

(19.4) of Problem

to

expand |A/-^|, given

A =

-1 -1

5-4 1-2 3 4-1 6


-4
+
4 6

We have

Sl

1+0-2+6
1

-4
+

-1

-4
+
6
1

-2
+

3 6

S2 2

-1
+

-2

-1

-2

-1

-3
1

2-5+16-9
1

-4 -1
5
1

SS

-4 -4 -4
4 7 6

-1

-4
3 6

5-4
+
1

1
1

-2
-1

-2

3 6

-1
-3

-2
--

-1
=

4-1

+ 16

8 + 2

Ml
Then
\XI

2
A^'

- A\

A'^- 5

+ 9A^ -

7A

2.

3.

Let

Ai, A"!;

As, ^s)

Show

that Xi, X2,

A3, ^3 be distinct characteristic roots and associated invariant vectors of A. X3 are linearly independent.

Assume
(i)

the contrary, that is,

assume

that there exist scalars 0%, 02, 03

not all zero, such that

a-]_Xi

+ 0^X2 + 03X3
;

Multiply
(II)

(1)

by A and recall that AX^ = A^

A^^

we have
=

a^AXi + oqAX^ + agAXg


(11)

aiAi^i + 02^2X2 + osAgXg

Multiply
(III)

by A and obtain

aiAi^i + osAjA^s + a3A3A^s

Now

(i), (11), (111)

may be

written as
1 1

aiXi

(iv)

Ai Ai

Ao As \l

An A3

02X2 ooXq
"3^3

x\

A\

CHAP.

19]

THE CHARACTERISTIC EQUATION OF A MATRIX

153

By Problem

5,

Chapter

3,

Ai
\l

As Al

As x%

0;

hence,

B~

exists.

Multiplying (iv) by

B~

we

have

[aiX-j,

a^X^.asXaY

= 0.

But this requires 01 = 02 = 03=0, contrary to the hypothesis.

Thus. Xi. X^, Xq are linearly independent.

A On
4.

-012

From

(j!)(X)

\\1

A\

-021
-<31

A 022
-032

-ai3 - 123

we obtain

A- 033
A -Oil

O12
1

-Ol3

A -Oil
-02

-ai2 A (322

<^'(A)

-021

A-O22
- O32

-023

Ois 023
1

- Osi

A-O33
A -Oil Os 1

-031
-Ol3

-O32

A-a3g
A -Oil
-O2I -012

A-O22
032
the

-O23I

A Ossl

A
of

Og 3

A 022

sum of the principal minors


1

XI - A

of order

two

<^"(A)

A -022

-O23
+
1

-032
+

A -033
1

OSI c

A -Oil

-oia
1

A 033

A -01 1

-0 12
1

-021

A-o22

2!(A-Oll) + (A-a22) + (A-Ogg)!


2!

times the sum of the principal minors of XI

-A

of order

one

0"'(A)

Also

<j!.^"\A)

= cji^^hX) =

...

5.

Prove: If X^ is an r-fold characteristic root of an re-square matrix A, the rank of X^J -A is not less than n T and the dimension of the associated invariant vector space is not greater than r.
Since Ai is an r-fold root of <;6(A) = 0, ^SCA^) = ct>(X^) = 4' <.\) = = 4>''^~^\x^) = and cfy'^AXi) + 0. Now (^"^HX^ is r! times the sum of the principal minors of order n-r of A^/ -A; hence, not every principal minor can vanish and X^l-A is of rank at least n-r. By (11.2), the associated invariant vector space otXj^l -A,
I.e., its

null-space, is of dimension at most

r.

6.

For the matrix of Problem

2,

find the characteristic roots and the associated invariant vector spaces.
are
1
1, 1, 1, 2.

The characteristic roots

4 4
is of rank 3;

For A =

2:

XI

-A

-2
1 1

2-5
-1

-3

its null-space is of dimen-

-4
is that

sion

1.

The associated invariant vector space

spanned by
5

[2, 3,

-2, -3]'.

For

A=

1:

XI -A

=
1

-1

-4 -2
-1
is of rank 3; its null-space is of

dimen-

1-4
sion
1.

1-5
[3, 6,

The associated

invariant vector space is that spanned by

-4, -5]'

154

THE CHARACTERISTIC EQUATION OF A MATRIX

[CHAP. 19

7.

Prove:

If

a is a non-zero characteristic root of the non- singular

n-

square matrix A, then \A\/a is

a characteristic root of adj A.

By Problem
(i)

1,

a"+ .!"-%
s^.
(i

^n-i^ + (-1)
all

l'^!

where

= 1. 2

n - 1) is (-1)

times the sum of


=

f-square principal minors of A, and

l/xZ-adj/ll

m"

Si/^""^

...

+ S^^^/s +

(-if

ladj^l

where Sj

(/

= 1, 2

1)

is (-1)-^ times the

sum

of the /-square principal minors of adj

A
=

and

By and the definitions of llll-M A\ = \AY' ^ then


(6.4)
l

s,-

and

S,-

Sj = (-1)" |-4|s._ Si= (-l)"s^_.,, -^ To

Jl 1

(-if Ml"-^si,
+

|adj

|;x/-adj A\

(-if !(-if ;x"

+ ._,A^"-' + ._2MlAi"-" +

...

s,|/irV

siMf-V

n-i

and

Mr"|/x/-adj
Now

4|

(-if !i (-if ii

s,(i^) +
\A\

...

._i(f^f-' \A\
+

(-if

(^f Ml
\A\

/(/^)

/(R)
and by
(i)

+ ,,(1) +

...

._,(lf-

(-if (If Ml!

a"/(4^)

(-ifia"+ sia"-'+

...

^_,a

(-i)"Ml

Hence, M|/Ot is a characteristic root of adj A.

8.

Prove: The characteristic equation of an orthoganal matrix

We have
</,(A)

|X/-P|

\\PIP'-P\

|-PA(^/-P')|

AAA
F
is a reciprocal equation.
=

A"|f/-P|

A"c^(i)

SUPPLEMENTARY PROBLEMS
9.

For the following matrices


vector spaces.
1

determine the characteristic roots and a basis of each of the associated invariant

-1
1

2
1

-2 -8 -12
(e)
1

2
(g)
1

1 1
1

-1 -1

-1

(a)

12
2
1

(c)

4
1

(O

2
1

-12
1
(rf)

-1 J
'

-2
1

-3 -9
1

-12
4
1_

2 (A)

2 2
1_

'2

(6)

2
1

(/)

2
_0

(/)

1+t

-1
3 2
(A:)

-3

_i
-1

2-i

2-4 2-1
2-1
2, [2, 2,
2,

6-10
9
6

'-1
1

-6 -3

3'

112-1
2

(0

-5 -4 -3 -2
-3 -3

-6 -4

-2
1

n)

-1

-5

-1

-1

-5

Ans.

(a)

1, [1,

-1,0]';

-1,-2]';

3,

[1,-1,-2]'

(b) -1, [1,0, 1]'; (c)


(rf)

[1,3,1]'; [2,1,0]';

1, [3, 2, 1]'

1,

[1,1,-1]';
[1,1,1]'

1,

CHAP.

19]

THE CHARACTERISTIC EQUATION OF A MATRIX

155

(e)

2,

[2,-1,0]';

0,
1,

[4,-1,0]';
[12,-4, -1]'
3,

1,

[4,0,-1]'

(/) 0,

[3,-1,0]';

(g) 1, [1,0,-1]', [0,1,-1]';


(A) 0, (j)

[1,1,0]'
4,

[1,-1,0]';

1,
i,

[0,0,1]';
[l

[1,1,0]'
-J,

-1, [0,1,-1]';

+ j,l,l]';
1,0]';

[l-i.1,1]'
[0,3, 1,2]'

(/) 2, [1,0,1]';
(/c)

1+t,

[0,

2-2j, [1,0,-1]'
2,

1, 1, 0,

[1,0,-1,0]', [1,-1,0,0]';
[1,2,3,2]';
[2,1,0, 1]';

[-2,4, 1.2]';

3,

(/) (m)

-1, [-3,0,1,4]'
1,

[3,0, 1,4]';

-1, [3,0, 1,2]'


=

10.

Prove:

If A" is

a unit vector and

if

AX

XX

then

X'AX

H.

Prove:

The characteristic roots of a diagonal matrix are the elements

of its diagonal and the associated in-

variant vectors are the elementary vectors E^


12.

Prove Theorems

and

VI.

13.

Prove Theorem
Hint.
If

VII.

|A/-.4|

=(A-Ai)(A-X2)...(A-A) then
roots of the direct

\(X

+ k)I

-A\

= (X + k~ X-O (X + k

-X2)

(X + k -X^^.

14. Prove:

The characteristic

sum

diag(.4i,

A2

A^) are the characteristic roots of A^ A^

13.

Prove:

If

A and

A'

equation.
16.

t:]
A and B

! are n-square and

r<n, show

that

NA

and

AN

have the same characteristic

Prove: Prove:
roots.

If the n-stjuare

matrix A is of rank

r.

then at least n

-r

of its characteristic roots are zero.

17.

If

are n-sauare and

is non-singular, then

A~'^B and BA~'^ have the same characteristic

18.

For A and B of Problem

17,

show

that

B and A'^BA have the same characteristic


-A)\

roots.

X9.

Let ^ be an n-square matrix.


the characteristic roots of ^4"^

Write

\XI -A''^\ = \-XA~'^(^I

and conclude that

iAl IA2

lA

are

20. Prove:

The characteristic
If

roots of an orthogonal matrix

are of absolute value

1.

Hint.

A-, X. are a characteristic root and associated invariant vector of P, then

X'

(PX.Y(PX-)

21.

Prove: If A^ ^ \ is a characteristic root and A^ is the associated invariant vector of an orthogonal matrix P, then XlXi = 0.

22. Prove: 23.

The characteristic
II,

roots of a unitary matrix are of absolute value

1.

Obtain, using Theorem

0(0)
<^ (0)

= =

(-i)"M|
(-1)

times the sum of the principal minors of order n -

of

<P

(0)

(-1)

r!

times the sum of the principal minors of order n -

of

.4

0^">(O)

n!

24. Substitute from

Problem 23 into
c/3(X)

(?i(0)

+ (^'(0)-X +-^(f^'(0)-X^

...

+^0'^'^'(O)-A"

to obtain (19.4).

chapter 20
Similarity

TWO

ra-SQUARE MATRICES A and B over F are called similar over F matrix R over F such that
(20.1)
2 2
3

if

there exists a non-singular

B = R'^-AR
11
1

Example

1.

The matrices A

1
1

of

Example

1,

Chapter

19,

and

2J

B
are similar.

R~'^AR

-110
-1 E7

-3 -3]
ij

li 2
1

2
3

1 1

133"
1

5 =

14
1

13]

[1

12
6
is

13

iJ

The characteristic equation (A 5)(A

1)^ =

of
5

is also the characteristic equationof ^.

An
that
root

invariant vector of

B associated with A=

Y^ = [l,0,0]' and

it

is readily

shown

an invariant vector of A associated with the same X-L A = 5. The reader will show that Fg = [7, -2, O]' and Yg = [n, -3, -2]' are arly independent invariant vectors of B associated with A = 1 while X^ = RY2 are a pair of linearly independent invariant vectors of A associated with the same

= RYi =

[1, 1, 1]' is

characteristic

a pair of lineand Xg = RYg


root A =
1.

Example
I.

illustrates the following theorems:

Two

similar matrices have the

same characteristic

roots.

For a proof, see Problem


II.

1.

If

is an invariant vector of

A^ of B, then
root A^ of A.

RY

is an invariant vector of

B = R~ AR corresponding to the characteristic root A corresponding to the same characteristic


For a proof, see Problem
2.

DIAGONAL MATRICES. The

characteristic roots of a diagonal matrix

D =

diag(ai, 02

a^)

are

simply the diagonal elements.

diagonal matrix always has n linearly independent invariant vectors.

The elementary

vectors E^ are such a set since

DE^ = a^E^,

(i

1, 2, ...,n).

As a consequence, we have (see Problems


III.

and 4

for proofs)

Any
If

re-square matrix A, similar to a diagonal matrix, has n linearly independent

invariant vectors.
IV.

an re-square matrix

A has

re

linearly independent invariant vectors,

it

is similar
5.

to a diagonal matrix.

See Problem
In

Problem
V.

6,

we prove
field

Over a

an re-square matrix A is similar to a diagonal matrix

if

and only

if

\IA

factors completely in
1/

F and

the multiplicity of each A, is equal to the dimension of

the null-space of X.I A.

156

CHAP.

20]

SIMILARITY

157

Not every ra-square matrix is similar to a diagonal matrix. The matrix of Problem 6, Chapexample. There, corresponding to the triple root X = 1, the null-space of A/-i is of dimension 1.
ter 19, is an

We can
VI.

prove, however,

Every ra-square matrix A is similar

to

a triangular matrix

whose diagonal elements


See Problems
7-8.

are the characteristic roots of A.

As special cases, we have


VII.
If

is

any real

n-

orthogonal matrix

P such

that

square matrix with real characteristic roots, there exists an P'^ AP = P'AP is triangular and has as diagonal elements

the characteristic roots of A.

See Problems 9-10.


VIII. If A is any re-square matrix with complex elements or a real re-square matrix with complex characteristic roots, there exists a unitary matrix U such that W'^AU UAU is triangular and has as diagonal elements the characteristic roots of A.
=--

See Problem

11.

The matrices A and P' AP


The matrices A and U'^AU

of

Theorem

VII are called orthogonally similar. are called unitarily similar.

of

Theorem VHI

DIAGONABLE MATRICES. A
Theorem IV
is

matrix A which is similar to a diagonal matrix is called diagonable. basic to the study of certain types of diagonable matrices in the next chapter.

SOLVED PROBLEMS
1.

Prove:

Two

similar matrices have the


=

same characteristic

roots.

Let A and B
(')

R~^AR

be the similar matrices; then


=

A/-B

\I -

R'^AR
=

R-'^UR - R-''aR
=

R'^(XI-A)R

and

\\I-B\

\R-^.\XI-A\.\R\

\XI-A\

Thus, A and S have the same characteristic equation and the same characteristic roots.

2.

Prove:

If

is an invariant vector oi

RY

is an invariant vector of
=

B = R ^AR corresponding to the characteristic root A corresponding to the same characteristic root \,- of A.
=

A,-,

then

By hypothesis, BY

X^Y and RB

AR; then
=

AX
and
A"

ARY

RBY

RX^Y

X-RY

X^X

Is an invariant vector of

A corresponding

to the characteristic root X^.

3.

Prove:
vectors.

Any matrix A which

is similar to a diagonal matrix

has

re

linearly independent invariant

Let R AR = diag(6i, 62. > b^) = B. Now the elementary vectors fii, Eg ^n ^'^ invariant vectors Then, by Theorem n, the vectors Xj = RE. are invariant vectors of A. Since R is non-singular, its column vectors are linearly independent.
of B.

158

SIMILARITY

[CHAP. 20

4.

Prove:

If

an n-square matrix A has n linearly independent invariant vectors,

it

is similar to a

diagonal matrix.
Let the n linearly independent invariant vectors X^. Xq
teristic roots Ai, Xs. /^n ^ '^^"^
,4A'^ = A^A'^,
0'

1, 2

n).

X^ be associated with the respective L,et R = [X^,X2, ....X^]; then


^n^n^

charac-

AR

[AX^.AX^

AX^]
Ai

[Ai^i, ^2^2
...

1X1,

X2

A2
'^n-'

...

diag(Ai, A2

A)

Hence,

R ^AR

= diag(Ai,

A2

A).

5.

set of linearly independent invariant vectors of the matrix

of

Example

1,

Chapter

19, is

Zi = [1,1,1]',
1

Z2 = [2,-1,0]',
2
1
;

^3 = [1,0,-1]'
1

Take

R =

[Jf 1,

Z2, ^3!

-1

then

1-2
1

and

10-1
1

-3
5
1 0"

1 1

2
1
,1

1"

1 1 1

R'UR
a diagonal matrix.

1 1

-2
2

3
2

1
2_

-1 -1
P

-3

6.

Prove:
of

Over a

field

factors completely in

F an n-square matrix A is similar to a diagonal matrix if and only if Xl-A F and the multiplicity of each A.^ is equal to the dimension of the null-space

X^I-A.
First,

and that exactly k of these characteristic B R-'^AR diag(Ai, A2, ^n' Then X^I - B has exactly k zeroes in its diagonal and, hence, is of rankn-fe; its thus, A.^/-^ has the same null-space is then of dimension n-(n -k)=k. But \l-A = R (Xj^I - B) R~^ \^I B rank n -k and nullity k as has Conversely, let Ai,A2 Ag be the distinct characteristic roots of A with respective multiplicities

suppose that

roots are equal to A^.

'i-''2

r,

where

ri+r2+...+rc

Denote by V^^. V^^,

the associated invariant vector spaces.


(i

Take

Xi^.Xi^.
XJ'

.,Xir-

as a basis of the invariant vector space Vf..

= 1,2

s).

Suppose that there

exist scalars a
(i)

not all zero,

such that
+
*
(o2i''''2i
"^

(aiiA^ii + 012-^12 + + "iTi^iri)

+ 022^^22 +
=

''2'r2^2r2)

...

+
Yi =

(osi^si

"^

^s^Xs'z

"^

^'T^ ^STg a XciT^) ' STg )

Now

each vector

ai^Xu

ai^Xi^ +

+ air^Xir^) =

0,

(i

1,

s).

for

otherwise,
(i);

it

is

an

Theorem I their totality is linearly independent. But this contradicts X's constitute a basis of K and A is similar to a diagonal matrix by Theorem IV.
invariant vector and by

thus, the

7.

Prove:

Every ra-square matrix A

is similar to a triangular matrix

whose diagonal elements

are the

characteristic roots of A.

the characteristic root Ai.

A and let X-l be an invariant vector of A corresponding to column of a non-singular matrix Qi whose remaining columns may be any whatever such that \Qi\ ^ 0. The first column of AQi_ is AX^ = X-^X^ and the first column of O]'. Thus, Ql^AQi is Ql^XiXi^. But this, being the first column of Qi'^XiQi. is [Ai,
Let the characteristic roots of A be Ai, Aj

Take

X-i

as the

first

CHAP.

20]

SIMILARITY

159

(i)

Qi^AQ^
A-^

hi
[o

Bil

A,j

where

is of order n |A/

1.

Since

-ft

AQ^\ = (X-Xi)\XI - Ai\,

follows that the characteristic roots of Ai are Xg. Ag with Q = Qi.

and Qi^ AQi_ and A have the same characteristic roots, it A. If n = 2, A-i = [A2] and the theorem is proved

the first

Otherwise, let X^ be an invariant vector of Ai corresponding to the characteristic root As- Take X^ as column of a non-singular matrix Qq whose remaining columns may be any whatever such that IftI /^ 0.

Then
(il)

Q2^A^Q^

[^2
[0

52I
A^j

where A2

is of order n

2.

If

n =

3,

/I2 = [As],

and the theorem is proved with Q = Q^[0

Q2J

Otherwise, we repeat the procedure and, after n /i


(iii)
(?2

steps at most, obtain

I2
<?3

n-2

Qn-i

such that

(J

/1()

is triangular

and has as diagonal elements the characteristic roots of A.

8.

Find a non-singular matrix Q such that

Q AQ

is triangular, given

9-1
6 5

8-9
5

-1
1

-5
5

-4
5

-4

Here |A/-^| = (A^-l)(A^-4) and the characteristic roots are 1,-1,2,-2. Take [5,5,- 1, 3j', an invariant vector corresponding to the characteristic root 1 as the first column of a non-singular matrix Qx whose remaining columns are elementary vectors, say

^5000'
5

-10
3

10 10
1

Then
1 1

5-1
5

8-9
-15 -12
20
16
[O

-5
1

and
5

Qi AQ^

4
3

-3

17
[4, 0,

Aq
4

A
then

characteristic root of Ai is -1 and an associated invariant vector is

-l]'.

Take ft

i
o"!

-20

II

and

ft A1Q2

J_
20

-15 -48
-11

20]

r_i
=

5I

64
48j

L
Take Qs

"d
}
[.:

characteristic root of A2 is 2 and an associated invariant vector is

[s, ll]'.

then

<?3'

H[-11
8

2/5I
"I sj

,!

and

ft'-42ft

[2

-2J

160

SIMILARITY

[CHAP. 20

Now
5

32
4
.

pi
[o

o"|
'

[h
[o

o1
Qsj

5
1

Q''
8

-40 40
4

Q^j

160

20
160_

-1

11
"l

1_

-180 40 -220
-7 -9/5"
5 2
1

-1

and

2/5

-2

9.

If

is

matrix

P such

any real re-square matrix with real characteristic roots then there exists an orthogonal that P''^AP is triangular and has as diagonal elements the characteristic roots of ^.

vectors will also be real.

X be the characteristic roots of A. Since the roots are real the associated invariant As in Problem 7, let Qi be formed having an invariant vector corresponding to Aj as first column. Using the Gram-Schmidt process, obtain from Qt an orthogonal matrix Pi whose first column is proportional to that of Q-^. Then
Let Ai.Xs

P'lAP^
where Ai
is of order n

hi

fill

and has As, A3, ...Ayjas characteristic roots.

Next, form Qq having as first column an invariant vector of Ai corresponding to the root A2 and, using the Gram-Schmidt process, obtain an orthogonal matrix P^- Then

TAs
iji-g

Ss"!

[0
After sufficient repetitions, build the orthogonal matrix

A^j

P
which P~^AP

pi
=

Pi.
|_o

0]

pn-2
"I

pj

P_J

for

is triangular

with the characteristic roots of A as diagonal elements.

10.

Find an orthogonal matrix P such that


"2

1"
1

P-^AP

P""

1
1

3 2

2_

is triangular

and has the characteristic roots of A as diagonal elements.


1,

Prom Example
to

Chapter 19, the characteristic roots are


"

5, 1, 1

and an invariant vector corresponding

is [1,0, -1]'.
1

We take

Qi

10
1

and, using the Gram-Schmidt process, obtain

-1

1 1

/V2
1

l/v^
1/1/2"
[l, 0, -l]'.

Pi

-l/\/2
an orthogonal matrix whose
first

column is proportional to

We

find

1/a/2

-I/V2
1

2
1

1
1

l/>/2
1

I/V2"
=

Pl'^Pi

3
2

3^/2"
\0

A^j

1/1/2"

l/^/T

-1/V2

1/

V2

21/2"

CHAP.

20]

SIMILARITY

161

Now Ax has A
we

a characteristic root and

[l,

-\/2]' as associated invariant vector. From Q2 =

r
obtain by the Gram-Schmidt process the orthogonal matrix

"2=1 1/V3
-1/^^
1/a/3

2/v/6

r-n
I

Lv2

\_

rn

ij

Then

L-2/V6
1/\A2

I/V3J

1/^6

P-,

I
1 1

X]

2/^6
1/V6"

-I/V2"

-I/a/S"

is orthogonal and

P~^ AP =

->/2
5

11.

Find a unitary matrix

f/

such that

V
2

All is triangular and has as diagonal elements the charac-

teristic roots of A, given


5 + 5i

-4 - 6i
+ 3i
A(A^

1 +t 2 - 2J
-1 + i

-6 -

4f

6 + 4f

-3 5

2i

The characteristic equation


1-i, 3 +
2i.

of

is

+(-4-J)A+

i)

and the characteristic roots are


"
1

0,

For A =

0,

take

[l,

-1, l]' as associated invariant vector and form

Qi

-1
_
1

1_

The Gram -Schmidt process produces the

unitary matrix

i/\fz
f/i

i/Ve'

-i/\/2

-l/vA3

2/v^
l/\/6"

1/V3"

1/^2"

Now

-2y/2a-i)
1-i

~(26 + 24:i)/y/6
(2 + 3i)/\A3"
3

+2J

so that, for this choice of Qi, the required matrix

= Ui.

12.

Find an orthogonal matrix

P such

that

P ^AP

is triangular

and has as diagonal elements the char-

acteristic roots of A, given


3
-1
1

-1
5

-1
may be taken as
[l,0, -l]'.

The
[1,1,1]',

characteristic roots are 2,3,6 and the associated invariant vectors

[1,-2,1]' respectively.

Now

these three vectors are both linearly independent and mutually

orthogonal.

Taking

l/V^
P
=

l/y/J
l/y/3

l/y/6

-2/^/6
l/yje

-1/-/2

l/y/3

we

find

P~^ AP

= diag(2,3, 6).

This suggests the more thorough study of the real symmetric matrix made

in the next chapter.

162

SIMILARITY

[CHAP. 20

SUPPLEMENTARY PROBLEMS
13.

roots of

Find an orthogonal matrix P such that f'^AP is triangular and has as diagonal elements the characteristic A for each of the matrices A of Problem 9(a), (6), (c), (d). Chapter 19.
\l\pZ
Ins.
(a)

\/Zs[2

2/3
2/3
1/3
(c)

I/V3"

1/-/2

-1/^/6""

-l/v/2

\/^^pi
-4/3a/2

i/Va"
-1/1/3

2/>/6

1/^

1/^6"
-i/Ve"'

I/V2"
(b)
1

-l/V'2
(rf)

1/V^ -i/'v^
1/V3"

2/V6"
I/a/2"

1/^2"

1/V2"
(6)

I/V3"

-1/\/6"

14.

not.

and (rf) are Chapter 19 and determine those which are similar to a diagonal matrix having the characteristic roots as diagonal elements.

Explain why the matrices (a) and

of Problem 13 are similar to a diagonal matrix while (c)

Examine the matrices {a)-{m)

of

Problem

9,

15.

For each of the matrices A of Problem 9(j). (/), Chapter 19, find a unitary matrix V such that angular and has as diagonal elements the characteristic roots of A.

U'^AU

is tri-

l/v^
Ans.
(i)

-(l+i)/2
2
(/)

\l\p2.
1

-1//!

l/>/2

a-i)/2^/2
(1

\/\[2

-0/2/2"

i 2

\/^[2

\/y[2

16.

Prove:

If

is real

and symmetric and P is orthogonal, then P''^ AP is real and symmetric.


of Problem 9 to prove

17.

Make the necessary modification

Theorem

VIII.

18.

Let B^ and C^ be similar matrices

for

(i

1, 2

m).

Show

that
= diag(Ci, C2

S
are similar.
19.

= diag(5i, B2, ...,S)


-1 i?^
.

and
ij

C)

Hint.

Suppose C^ =
and

B^ fl^ and form Write

= diag(Bi,

Bg

B^).
/i

Let

= diag(Bi, Sj)

= diagCSj, B^).

/ = diagC/i./g),

where the orders of


=

and

/g are

those

of Bi and B^ respectively, and define

B
B

Show
oJ

that

BT^EK

to prove

B and C are

similar.

20.

Extend the result of Problem 19


S^ along the diagonal.

to

= diag(Bi,

Bg

B)

and C any matrix obtained by rearranging the

21. If

A and B are n-square, then


Let P AQ =N; then

AE

and B/1 have the same characteristic roots.


=

Hint.

PABP"^ =NQ''^BP''' and Q~^ BAQ

Q'^BP'^N.

See Problem

15,

Chapter 19

22. If A-i.A^
...

A^ are non-singular and of the same order, show thut A-^A^-.. A^, A2As-.. A^A^, A3... A^A-^^A^. have the same characteristic equation.

23.

Let
of
(a)
.4.

Q'^AQ

B where B

is triangular

and has as diagonal elements the characteristic roots Ai, X2

Show that S
roots of A.

A Q

is triangular

and has as diagonal elements the kth powers of the characteristic

(b)

Show

that

X,-

= trace

24.

Show

that similarity is an equivalence relation.


'2

r
1 2_

-1

25.

Show

that

1
_1

3
2

and
-3

2-1
-2
3

have the same characteristic roots but are not similar.

chapter 21
Similarity to

a Diagonal Matrix

REAL SYMMETRIC MATRICES. The


be combined but
I.

study of real symmetric matrices and Hermitian matrices may

we

shall treat them separately here.

For real symmetric matrices, we have:

The characteristic

roots of a real symmetric matrix are all real.

See Problem
II.

1.

The

invariant vectors associated with distinct characteristic roots of a real sym-

metric matrix are mutually orthogonal.

See Problem

2.

When A
III.

is real
If

and symmetric, each B^ of Problem

9,

Chapter 20, is

0;

hence,
...,

symmetric matrix with characteristic roots Ai, \^, then there exists a real orthogonal matrix P such that P'AP = P'^AP = diag(Ai, Aj

is a real ra-square

A,
A).

Theorem
IV.

III

implies

If

A^ is a characteristic root of multiplicity

r^

of a real
r^.

symmetric matrix, then

there is associated with A^ an invariant space of dimension


In terms of a real quadratic form,

Theorem

III

becomes

BY

V. Every real quadratic form q = X' AX can be reduced by an orthogonal transformation to a canonical form

(21.1)

Aiyi + X^yl +
r

...

+ X^y"^

where

is the rank of

A and

Ai,

Aj

A^ are

its

non-zero characteristic roots.

the

Thus, the rank of q is the number of non-zero characteristic roots of A while the index is number of positive characteristic roots or, by Descartes Rule of signs, the number of varia-

tions of sign in |A/


VI.

^|

0.

real symmetric matrix is positive definite if and only if all of its characteristic

roots are positive.

ORTHOGONAL

SIMILARITY. If P is an orthogonal matrix and 8 = P"^ AP then B is said to be orthogonally similar to A. Since P~^ = P', B is also orthogonally congruent and orthogonally equivalent to A. Theorem III may be restated as
.

VII. Every real symmetric matrix A is orthogonally similar to a diagonal matrix whose diagonal elements are the characteristic roots of A.

See Problem

3.

Let the characteristic roots of the real symmetric matrix A be arranged so that Ai ^ A2 = Then diag(Ai, A2, . A.) is a unique diagonal matrix similar to A. ... ^A. The totality of such diagonal matrices constitutes a canonical set for real symmetric matrices under orthogonal similarity. We have
VIII.

Two

the

same characteristic

real symmetric matrices are orthogonally similar if and only if they roots, that is, if and only if they are similar.

have

163

164

SIMILARITY TO A DIAGONAL MATRIX

[CHAP. 21

PAIRS OF REAL QUADRATIC FORMS.


IX. If X'

In

Problem

4,

we prove

(x-^^, x^ x^) and if tive definite, there exists a real non- singular linear transformation X = CY

AX

and

X'BX

are real quadratic forms in

X'BX

is posi-

which carries

X'AX

into

AiXi + A2Y2 +
and X' BX into

+ Ay

yi

22
+ 72

+ Tn

where A^ are the roots of

\\BA\=0.
See also Problems
4-5.

HERMITIAN MATRICES.
X.

Paralleling the theorems for real symmetric matrices,


of an Hermitian matrix are real.

we have

The characteristic roots

See Problem
XI.

7.

The

invariant vectors associated with distinct characteristic roots of an Hermitian

matrix are mutually orthogonal.


XII.
If

is an re-square Hermitian matrix with characteristic roots Ai, A2.

A, there

exists a unitary matrix

U such

that

U'HU = U~^HU =
.

diag(Ai, A2,

...,

A).

The matrix H

is called unitarily similar to


XIII. If

U'

HU

is a characteristic root of multiplicity


A,-

r-

of the Hermitian matrix H, then


r.

there is associated with

an invariant space of dimension

Let the characteristic roots of the Hermitian matrix

be arranged so that Ai i A2 ^

Then diag(Ai, A2
follows

A.) is

a unique diagonal matrix similar to H.

The

totality of

i Ansuch diago-

nal matrices constitutes a canonical set for Hermitian matrices under unitary similarity.

There

XIV.

Two

Hermitian matrices are unitarily similar


is, if

if

and only

if

they have the same

characteristic roots, that

and only

if

they are similar.

NORMAL MATRICES. An
diagonal,
real

n-square matrix A is called normal


real

if

AA' = A'A. Normal matrices include


skew-Hermitian, and

symmetric,

skew-symmetric, orthogonal, Hermitian,

unitary matrices.

Let .4 be a normal matrix and f/ be a unitary matrix, and write B = U'AU. and B'B = U'A'U -U'AU = U'A'AU = U'AJ'U = U'AU -U'A'U = BB'. Thus,

Then B' = U'A'U

XV.
In

If

^
8,

is a

normal matrix and

is

a.

unitary matrix, then

U'AU

is a normal matrix.

Problem
XVI.
If

we prove
is an invariant vector corresponding to the characteristic root A/ of a northen X^ is also an invariant vector of A' corresponding to the characteristic

X^

mal

rnatrix A,

root A^.
In

Problem
XVII.

9,

we prove
square matrix A is unitarily similar to a diagonal matrix
if

and only

if

is

normal.

As a consequence, we have
XVIII.
If

is normal,

the invariant vectors corresponding to distinct characteristic

roots are orthogonal.

See Problem

10.

CHAP.

211

SIMILARITY TO A DIAGONAL MATRIX

165

MX.

If

A- is a characteristic root of multiplicity


r^.

r^

of a normal matrix A, the associ-

ated invariant vector space has dimension

XX. Two normal matrices are unitarily similar

if

and only

if

they have the same char-

acteristic roots, that is, if and only if they are similar.

SOLVED PROBLEMS
1.

Prove:

The

characteristic roots of an re-square real symmetric matrix


is a

A are

all real.

Suppose that h+ik

complex characteristic root of A.


=
{(h

Consider
=

B
which
is real

+ ik)I-A\[(h-ik)I-A\ + ik)I

(hl-Af+k'^I

and singular since

{h

-A

is singular.

There exists a non-zero real vector

such that

BX

and, hence,

X'BX
The vector

X\h!-AfX
A"

+ k^X'X
{(hi

X'(hl-A)'(hl

-A)X
0.

+ k^X'X
Also,

= =

(/(/- 4)

is real; hence,

- A)X\' {(hi -A)X\ ^

X'X>0. Thus,

A:

and

there are no complex roots.

2.

Prove:

The invariant vectors associated with

distinct characteristic roots of a real symmetric

matrix A are mutually orthogonal.


Let
A'l

and X^ be invariant vectors associated respectively with the distinct characteristic roots Xi and

As of 4.

Then
=

AX^

Ai^i

and

AX^

= Aj'^s.

also

A^2'4^i =

X-iX^X^

and

Xj^AX2 = XqXiX^

Taking transposes
^1 A X^ = A 1 A*! ^2
and
XqAX-j^ =
= 0.

X2X2X1

Then

X^X^X^

A2^iA^2

and, since Ai ^ A2,

X[X2

Thus, X^ and X2 are orthogonal.

3.

Find an orthogonal matrix P such that P'^AP is diagonal and has as diagonal elements the characteristic roots of

A given
,

^7-2
-2
1

10

-2
7

-2

The characteristic equation

is

A-7
2

-1
2

A -10
2

A - 24A^ + I8OA - 432

-1
and the characteristic roots are
1

A-7
- 2

6, 6, 12.

-1
2

Xl

For A =

6,

we have

2
1

-4
2

X2
X3

or

Xi

- 2x2 + xs

and choose as associated

in-

-1

variant vectors the mutually orthogonal pair X^ = [l,


[1, -2,1]' as associated invariant vector.

0,

-l]' and

X2

= [l,

1, l]'.

When A

= 12,

we take X3 =

166

SIMILARITY TO A DIAGONAL MATRIX

[CHAP. 21

Using the normalized foim of these vectors as columns

of

we have

1/VT
P
=

1/x/y

X/\f&

I/VT
-i/x/T

-2/sf6
l/yjl

l/VT

It

is left as an exercise to

show

that

P~^AP

= (iiag(6,

6, 12).

4.

Prove:
nite,

If

X'AX and X'BX

are real quadratic forms in (xi,x2

there exists a real non-singular linear

transformation
...

) and if X'BX is positive defi= CY which carries Z'/lZinto

Xiyi + \2y2 + \\B-A\ =0.


By Theorem
(i)

^nTn

^'^'^

^'B^

into yi + y| +

+ y^, where Xi, Ag, .,

are the roots of

VII there exists an orthogonal transformation

GV

which carries X'BX into

V'(G'BG)V
jjii.jj.^

jji^vf + fJ.2V2 +

+ Ait;^

where

/x are the characteristic roots (all positive) of B.


l/i//!^)-

Let
(ii)

= diag(l/\/7H, X/sfJT^

Then V =
1

HW
2

carries

(i)

into

W
for the real quadratic form
it

(H G

BGH)W
G'AGH)W
=

+ wt

Now

(H

there exists an orthogonal transformation

KY

which

carries

into

Y'(K'h'g'AGHK)Y
where Xi.Xg
formation

X^y2 + X^y^ +

...

+ X^y^
there exists a real non-singular trans...

X are the characteristic roots ot H G AGH. Thus, CY = GHKY which carries X'AX into X^y^ + X^y^ +
Y'(K'h'g'BGHK)Y
Y (K~ IK)Y
-'i

+ Xy^

and X'BX into


'n

-'2

Since for

all

values of

X,

K'h'g'(\B-A)GHK
X

XK'h'g'BGHK - K'h'g'AGHK

= =

diag(X,X

X)

- diag(Xi,X2

X)

diag(X-Xi,

X-X2

X-X)

it

follows that Xi, Xg

are the roots of |XB-/l|

=0

5.

Prom Problem

3,

the linear transformation

\/^pr

l/vA3"

l/sfQ

1//6"

(GH)W
-l/\f2 \/2\fl

l/VT
IATs

-2/'/6"

1/^r
1/2x^3"

1/v^
1/(

1/3n/2

1/3-/2

-1/:
1/f

_-l/2>/3
7

1/3\A2

-2
10

carries

X'BX =

X'

2
1

-2
7

into

ir'ZIF.

-2

The same transformation

carries

1/3

X'AX

into

tr'

0^

CHAP.

21]

SIMILARITY TO A DIAGONAL MATRIX

167

Since this is a diagonal matrix, the transformation

W
=

KY

of Problem 4 is the identity transformation

= lY.

Thus, the real linear transformation

CY

{GH)Y

carries the positive definite quadratic form


It

X'BX
that

and the quadratic form X' AX into ri + y| + 7l \kB-A\ = 36(3\-l)(2X-l)^.

into ^y^ + iy| + |y|.

is left as an excersise to

show

6.

Prove:

Every non-singular real matrix A can be written as symmetric and P is orthogonal.


exists an orthogonal matrix

A = CP

where C

is positive definite

Since A is non-singular, AA' is positive definite symmetric by Theorem X, Chapter Q"'' with each k^) = B AA' Q = diag(A;i, ij Q such that

17.
A:^

Then there
>
0.

Define

Bi

diag(\/^,

VT^

\fk~^)

and

= QB-^Q''^. =

Now C

is positive definite

symmetric and

C^
Define

QB^Q-^QB^Q-^
=

QB^Q-^
= I

QBQand

AA'
is orthogonal.

C'^A.

Then

PP'

C'^AA'C'^^ C ^C'^C^
is orthogonal as required.

Thus A

CP

with C

positive definite symmetric and

7.

Prove:

The

characteristic roots of an Hermitian matrix are real.

that

Let \j be a characteristic root of the Hermitian matrix H. Then there exists a non-zero vector X^ such Now X'^HX^ = XjX'j^Xiis real and different from zero and so also is the conjugate transHXj^ = A jX^
.

pose

X\HX;j^ =

\^X\X^.

Thus, X^=X.j andX^isreal.

8.

Prove:

If X: is an invariant vector corresponding to a characteristic root A,- of a normal matrix A, then Z^ is an invariant vector of A corresponding to the characteristic root X^.

Since A is normal,

(XI-A)(XI-A)'

(KI-A)(XI-A')
XXI - XA'

XXI - \ a'
a' A
0;

XA

AA
-A)

-XA+
-A)X^=

(XI -A)' (XI

so that XI

-A

is normal. =

By hypothesis, BX;
=

= (X^I

then

(SX^)'(SA'i)

X-B'-BXi

X'iB.B'X^

(W X ^)' {W X ^)

and

B'X^

(X^/-I')Zi

Thus, X: is an invariant vector of A' corresponding to the characteristic root X

^.

9.

Prove:

An

re-square matrix
is normal.

is unitarily similar to a diagonal matrix if

and only

if

A is normal.

Suppose A

By

Theorem vni. Chapter 20, there exists a unitary matrix U such that
^1
6i2

bin

As

bin

U AU
....

Xfi-i

bn-i. n

B'b

By Theorem XV, B is normal so that B' B = BB is XiXi while the corresponding element of BB

Now
is

the element in the first row and first column of

AjAi + 612^12 + bigbis +

...

bmbm

Since these elements are equal and since each b^j b-^j 2 0, we conclude that each b^j = 0. Continuing we conclude that every b^; of B with the corresponding elements in the second row and second column, A). Conversely, let A be diagonal; then A is normal is zero. Thus, B = diag(Ai,A2

168

SIMILARITY TO A DIAGONAL MATRIX

[CHAP. 21

10. Prove:

If

is normal, the invariant vectors corresponding to distinct characteristic roots are

orthogonal.

Let Ai,

A*!

AXi=\iXi, AX^
and, since

and Xj, ^2 be distinct characteristic roots and associated invariant vectors of A. Then = \^X^ and, by_Problem 8. jI'A'i =Xi^i,_I'A'2 = AaATa. Now ^2/lA'i = Ai^a^i and,

taking the conjugate transpose,

X[a' X^

= X^X'-^X^.

But

X[a' X^

\^X[X2.

Thus, Xi^^A's = Xa^i-^s

Xx iXo.,

XiX2=

as required.

11.

Consider the conic

x^ -

12xix.2.

- 4.xi = 40 =

or

(i)

X'AX

X
[-:
::]

40

referred to rectangular coordinate axes

OX^ and OX2.

The characteristic equation

of

A is

A-1
|A/-^|
=

6 =

(A-5)(A + 8)
[3, -2]'

For the characteristic roots Ai = 5 and A2 = -8, take


invariant vectors.

and
"

[2, 3]'

respectively as associated

Now

form the orthogonal matrix

tsVTs '__
|

2/V/T3I
'^lll

whose columns

are the two

-2/v/T3 vectors after normalization.


/v/T3

3/vT3j
to
o"!

The transformation
1

PY

reduces

(i)

-2/\flf\
3/VT3.

-6~]

3/x/T3

13"! Vis'

fs
-

'VTs

2/\/l3

-\y 3/\/l3.
13 J

y'\
Lo

-Sj

5yl

^-y|

40

The conic

is an hyperbola. is the familiar rotation of

Aside from the procedure, this

axes

in plane analytic

elimination of the cross-product term in the equation of a conic.

geometry to effect the Note that by Theorem VII the result is

known as soon as the characteristic roots are found.

12.

One problem
to

of solid analytic geometry is that of reducing, by translation and rotation of axes,

the equation of a quadric surface to simplest form.

The main tasks

are to locate the center and

determine the principal directions,

i.e.,

the directions of the axes after rotation.

Without

at-

tempting to justify the steps, equation of a central quadric.


Consider the surface

we show here

the role of two matrices in such a reduction of the

Zx^ + 2xy + 2xz + iyz - 2x - 14y + 2z "


3
1

9
1

and the symmetric matrices

"
1

3
1

-1

2 2

-7
1

1
1

2
2

and

=
1

-1

-7

-9

formed respectively from the terms of degree two and from

all the terms.

The

characteristic equation of

is

A-3
1A/-.11

-1

-1

-1

A
-2

-2 A
:

-1 The characteristic roots and associated


Ai =
1,

unit invariant vectors are


= 4,

fi =

~.

-;=|

A2

V2

-2,

V3

^N-TfJ

CHAP.

21]

SIMILARITY TO A DIAGONAL MATRIX

169

Using only the elementary row transformations Hj(k) and H^:(k), where
r-1

4,

3
1
1

-1
-7
--VJ

-4

Bi

2 2

1
1

1 1

ft]
y +

-1

-7

-9

-4
'3x +

Considering B^ as the augmented matrix of the system of equations


\^

X X + 2y

0-1=0 +22-7=0 +1=0

we

find

from Di the solution x = -1, y =

0,

z =

or

C(-l,

0, 4).

Prom

D^,

we have

d = -4.
0, 4).

The rank
equation is

of

is 3

and the rank of B is

4; the

quadrlc has as center C(-l,

XtX'^ + X^y'^ + XsZ^ + d

111
+

The required reduced

4F
4.

2Z

- 4

The equations

of translation are

x = x'
di,

1,

y = y',

z = z'

The

principal directions are

1-2. i^s-

Denote by E the inverse of

\_v\.v.2,v^.

The equations of the

rotation of axes to the principal directions are


i/\A3"
\_X

-i/i/s"

-1/vT
i/vT

Y Z\- E

[X Y Z]

2/vT

i/vT
-i/x/T

i/VT

SUPPLEMENTARY PROBLEMS
13.

For each of the following real symmetric matrices A find an orthogonal matrix nal and has as diagonal elements the characteristic roots of A.
,

P such

P that /""^.^P is diago


"4
(e)

2
(a) 2

-1
.

2
(b)
_1

()

2
(c)

-4
2

21

"3

2
2

2"

-1
4

r
-1
4_

3
2_

-4
_ 2

-2
-1_

id)

2
_2

-1

-1

2_

-2

4_

_1
2/3 1/3

-1
2/3 1/3

i/\/^
Ans.
(a)
1

1/VT
(b)

l/\f2

\/sf&

l/v/
l/v/s"
i/a/s"
(c)

-2/vT

-2/3
1/3

2/3 2/3

i/VT

-i/v^T
2/3
(d)

-1/VT
1/3

1/\A6"

-2/3

2/3
1/3
(e)

i/x/T
2/^/6'

1/vT

i/vT
-l/v/T

-2/3

2/3
--2/3

-1/ 3
14.

2/3

yV 6

-1/\A2

i/VT

Xi are the roots

Find a linear transformation which reduces X'BX to X' AX to Xtyl-^X^y^ + Asy^, where yl-^y'^-^y'l and of \XB - A\ =0, given

-2
{a)

l\

[7 -2
1

10

-2
7J

5=03
\\

r2

fj
(6)
-4

7 =

-4
1

-4
-8
1_
,

2 5 4

2 4 5

-4
_-4

2 2

-2
l/\/T

2J

-8

1/3VT
-2/3x^2"
1/31/2"

1/3 1/3 1/3


(6)

2/3 2/3 1/3

2/3
1/3

l/3x/l0

/4ns.

(a)

2/3^/10
2/3\/l0

-\/\[2

-2/3

170

SIMILARITY TO A DIAGONAL MATRIX

[CHAP.

21

15.

Prove Theorem IV.


Hint.
'^i

If

/'"^^P = diag(Ai,
'^1

A.1

Xi,
'^ ^

X^ + in

'^r-i-2

^ri)'

"^S"

P"\Ai/ -/1)P

= diag(0,

0,

-^r + i'

~'^r+2

^1 ~^v)
2 to

''^"'^

r.

16.

Modify the proof in Problem

prove Theorem XI.

17.

Prove Theorems XII, XIH, and XIX.

18. Identify
(o) (h)

each locus:
24xi:>;2

20x2-

27%^
=

=
"i'

369,

(c)
(''^

\Q?,

x\ - 312%ia:>, + 17 %|
= 8

900,

3x^+ 2%2

+ 3%2

*i + 2^:1x2 + Xg

19.

Let A be real and skew-symmetric.


(a) (6) (c)

Prove:

Each characteristic
7

root of

is either zero or

a pure imaginary.

+ ^ is non-singular,
(/

/ -.4 is

non-singular.

B =

+ /iri(/-4) is orthogonal.

(See Problem 35, Chapter 13.)


A~'^

20.

Prove:

If

Is

normal and non-singular so also is

21.

Prove:

If

is normal, then

is similar to

22. Prove:

square matrix A is normal

if

and only

if it

can be expressed as

H + iK, where H

and

are

commu-

tative Hermitian matrices.

23. If

is n-square with characteristic roots

Xi, A,2

A.,

then

is normal if imd only if the characteristic

roots of
Hint.

AA
for
J

are XiAi,
C/"^<4(7 =
T*

X2A2
T
= [. ],

^n^rv
where
C/

_
is unitary

_
tr(-4/l')

Write

and T is triangular.

Now

tr(rT') =

requires

ty =
24.

Prove:

If

is non-singular, then

AA'

is positive definite Hermitian.

Restate the theorem when A

is real

and non-singular.
25.

Prove:

If

A and B are n-square and normal and

if

A and S' commute, then

^B

and

BA

are normal.

26. Let the characteristic function of the n-square matrix


<^(X)

A be

(X-x^f^iX-x^f'^ ...a-'Kfs
P such
that

and suppose there exists a non-singular matrix


(/)

P'^ AP
B.,

diag(Ai/.ri' '^2-'r2
0, ...,

-^sV^)
0,/
'i
,

Define by

(j

= 1, 2

s) the

n-square matrix diag(0,


of (/) and define
=

0) obtained

by replacing A. by

and Xj,

r a

i).

by

in the right

member
T? E,
.

DR. , PBiP~^.

= 1,2

s)

Show
(a) (6) (c)
(d)

that
~-

P'^AP
A

AiBi

A282+

...

A5B,

= Ail + A22 + +'^3^5 Every ^ is idempotent.

E^Ej The

=0

for

^j.
=
/

(e)
(/) (g)

1 + 2 + + E3

rank of ^ is the multiplicity of the characteristic root A^.


= 0,
(J

(Xil-A)Ei

= 1,2

s)

(h)

IfpM
Hint.

is any polynomial in x. then

p(A)

p(Ai)i + p(A2)2 +

+ P(^s^^s-

Establish

^^

Aii + A22 +

xIe

Aii + A22 +

+ X^E^.

CHAP.

21]

SIMILARITY TO A DIAGONAL MATRIX

171

(i)

Each E^
Hint.

is a polynomial in A.

Define

/(A)

(A

-Xi)(A-X2)

(A -X^)

and

/^(A)

/(A)/(A-A^),

(i

= l,2

s).

Then

(/)

A
If

matrix
If

fi

commutes with A
with

if
it

Hint.
(k) (I)

B commutes

and only if it commutes with every ^ commutes with every polynomial in A.

A A

is normal, then each Ei is Hermitian.


is non-singular, then

U
If

A-^
(m)

Al%
=

A-2%

...

+ A-^fig

is positive definite Hermitian, then


/f

A^'^

,1/2

vTii

VA22

...

VAs^s
it

is positive definite Hermitian.


(n)

Equation

(6) is called the spectral

decomposition of A.

Show

that

is unique.

27. (o)

Obtain the spectral decomposition


"

24

-20
24

lO"

4/9
=

-4/9
4/9

2/9"

5/9

4/9
5/9

-2/9'

-20
_

-10
9_

49

-4/9
2/9

-2/9
1/9

+ 4

4/9

2/9

10

-10
"

-2/9

-2/9

2/9

8/9

29

20 29 10

-lO"
10
44_

(b)

Obtain

A-^

1
--

20

196

-10
38/9
(c)

-20/9
38/9

10/9'

Obtain

//^

-20/9
_
1

-10/9
23/9_

0/9

-10/ 9

28.

Prove: If A is normal and commutes with B, then A' and


Hint.

B commute.

Use Problem
If

26

(/).

29.

Prove:

is non-singular then there exists a unitary matrix

U and a positive

definite Hermitian matrix

such that
Hint.

A= HU
H
by H^ = AA'

Define
If

and

f/

= H~'^A.

30.

Prove: Prove:

is non-singular, then

is

normal

if

and only

if

and U of Problem 29 commute.

31.

The square matrix A

is similar to a diagonal matrix if

and only

if

there exists a positive definite

Hermitian matrix
32.
33.

such that

H~^AH

is normal.

Prove: Prove:

real symmetric (Hermitian) matrix is idempotent if and only if its characteristic roots are O's real

and

I's.

If .4 is

symmetric (Hermitian) and idempotent, then


I

r.

= tiA.

34.

Let A be normal, B =
Prove:
(a)

+ A be non-singular, and C = B^^B'.


(6)

A and

(S')"^ commute,

is unitary.

35.

Prove: If

tf

is Hermitian, then {I + iHT'^{I

~iH)

is unitary.

36. If

is n-square, the set of

numbers X' AX where


of

is

2i

unit vector is called the field of values ot A.

Prove:

(a)

The characteristic roots

(6)

Every diagonal element of of values of A.


If If

A are in its field of values. A and every diagonal element

of

U'^AU, where U
real.

is unitary, is in the field

(c)
(d)

A
A

is real

symmetric (Hermitian), every element in its field of values is

values least and Ais the greatest characteristic root of A.


its field of

is real

symmetric (Hermitian),

is the set of reals

Ai i A < A, where Ai

is the

chapter 22

Polynomials Over a Field


POLYNOMIAL DOMAIN OVER
to be

F.

commutative with

itself

Let X denote an abstract symbol (indeterminate) which is assumed and with the elements of a field F. The expression
n
n-i

(22.1)

f(X)

cirik

On-^X

a^X +

p a^X

where the
If

a^ are in
oj;

F
,

is called

a polynomial in \ over F.
,

(22.1) is called the zero polynomial and we write f(\) = 0. If a^ ^ of degree n and a is called its leading coefficient. The polynomial /(A) (22.1) is said to be = said to be of degree zero; the degree of the zero polynomial is not defined. Oq^ oq ^ is

every

If

a =

in (22.1), the

polynomial is called raonic.

polynomials in A which contain, apart from terms with zero coefficients, the same terms are said to be equal.

Two
The

totality of polynomials (22.1) is called a polynomial

domain F[\] over F.

SUM AND PRODUCT. Regarding


f(X) + g(\)
f(X) is of degree
is of

the individual polynomials of

F[X\ as elements

of a

number sys-

tem, the polynomial domain has most but not

all of the properties of a field.

For example

= g(X) + f(X)

and
n,

f(X)-g(X) = g(X)-f(X)

If

and g(X) is of degree


TO

(i)

^(X) + g(A)

degree

when m.>n,

of degree at most

m when m=n, and

of degree

re

when m<n.
(ii)

/(A)-g(A)
If

is of

degree

m + ra.
0,

/(A)

?^

while

f(X)-g(X) =

then

g(X) =

If

g(A) ^

and h(X)-g(X) = h(X)-g(X),

then

A(A) =

A:(A)

QUOTIENTS.

In
I.

Problem
If

1,

we prove
are polynomials in F[A], then there exist unique polynomials
is either the

/(A) and g(X) ^

and r(A) in F[A], where r(A) that of g(A), such that


/((A)

zero polynomial or is of degree less than

(22.2)

/(A)

h{X)-g{X) +

r(A)
r(A) = 0, g{X) is said

Here, r(A) is called the remainder in the division of /(A) by g(A). to divide /(A) while g(A) and h{X) are called factors of /(A).

If

172

CHAP.

22]

POLYNOMIALS OVER A FIELD

173

Let

/(A)

= h(X)-g(X).

When

g(A.) is

of degree zero, that is,

the factorization is called trivial.

non-constant polynomial over

when g(A,) = c, a constant, F is called irreducible over

if its

only factorization is trivial.


Over the rational field A, -3 is irreducible; over the real field Over the real field (and hence over the rational field) A^+4 field it is factorable as (\+2i)(\-2i)

Example

1,

it

is factorable

is irreducible;

as (X+\/3)(X-^j3). over the complex

THE REMAINDER THEOREM.


(22.3)

Let f(X) be any polynomial and g(X) =


/(A)

X-

a.

Then

(22.2)

becomes

=
r,

h(X)-(X-a) +

where

is free of A.

By

(22.3),

f(a) =

and we have
until a remainder free of

n.
is f(a).

When

/(A) is divided by

A- a

A is obtained, that remainder


f(a) =

m. A

polynomial /(A) has

X-

a as a factor

if

and only

if

0.

GREATEST COMMON
f(X) and g(A).

DIVISOR.

If

h(X) divides both f(X)

and

g(A), it is called a

common
if

divisor of

A
(i)
(ii)

polynomial d(X) is called the greatest common divisor of

/"(A)

and g(X)

d(X) is monic, d{X) is a

common

divisor of /(A) and g(X),

(Hi)

every common divisor of /(A) and g(X) is a divisor of d(X).


In

Problem

2,

we prove
not both the zero polynomial, they have

IV. If /(A) and g(X) are polynomials in F[X],

a unique greatest

common

divisor d(X) and there exist polynomials h(X) and ^(A) in F[X]

such that
(22-4)
d(X)

h(X)-f(X) +

k(X)

g(X)

See also Problem

3.

When the only common divisors


is

of

/"(A)

and g(X) are constants, their greatest common divisor

d(X) =

1.

Example?. The greatest common divisor

of /(A)

(A^

+4)(A^+3A+
i/(A)

5)

and g(A) =

(A^

-1)(A^ + 3A+5)

is

A +3A+5. and

(22.4) is
A.=

3A +

- lg(A)
5

We have also
V.

(1-A)-/(A) + (X+4)-g(X)

0.

This illustrates

not

1,

If the greatest common divisor of /(A) of degree re>0 and g(X) of degree m>0 is there exist non-zero polynomials a(X) of degree <m and 6(A) of degree <n such that

a (A) -/(A) +

b(X)- g(X)

and conversely.

See Problem

4.

RELATIVELY PRIME POLYNOMIALS. Two


common
divisor is
i.

polynomials are called relatively prime

if

their greatest

174

POLYNOMIALS OVER A FIELD

[CHAP. 22

VI. If g(A) is irreducible in

F[X] and /(A)


/(A).

is

any polynomial of F[X], then either g(X)

divides /(A) or g(X) is relatively prime to


VII.
VIII.
If

g(A) is irreducible but divides

/(A)

h(X), it divides at least


if

one of

/(A)

and

h(X).

If /"(A)

and

g:(A)

are relatively prime and

each divides

A(A),

soalsodoes /(A)-g(A).

UNIQUE FACTORIZATION.
IX.

In

Problems, we prove

Every non-zero polynomial f(X) of F[A] can be written as


/(A)

(22.5)

9i(A)

?2(A)

^^(A)

where

is a

constant and the qi(X) are monic irreducible polynomials of F[X]-

SOLVED PROBLEMS
1.

Prove:
in F[X],

If

/(A) and g(X)^0 are polynomials in F[X], there exist unique polynomials h(X) and r(X) where r(A) is either the zero polynomial or is of degree less than that of g(X), such that
/(A)

(i)
Let

h(X)-g(X) +

r(X)

n
/(A)
= o^jA

n-i + %1-iA. +

+ a^A + Oq

and
g(A)
=

h^X
if

i^-iA

6iA +

6o.

^m then

?^

Clearly, the theorem is true

/(A) =

or if

n<m.

Suppose that
=

n>m;
c^-iA

/(A)

7^ A

g(A)

/i(A)

cpX

Co

is either the zero polynomial or is of degree less than that of /(A).


If /i(A)

or is of degree less than that of g(A),

we have proved

the theorem with h(X) =

~X

and

r(X) = /i(A).

Otherwise, we form

/(A)

^A"-\(A) - ;^A^-\(A)

/,(A)

f^X) =0 or is of degree less than that of g(A), we have proved the theorem. Otherwise, we repeat Since in each step, the degree of the remainder (assumed ^ 0) is reduced, we eventually reach a remainder r(A) = 4(A) which is either the zero polynomial or is of degree less than that of g(A).
Again,
if

the process.

To prove uniqueness, suppose


/(A)
=

h(X)-g(X)

+ r(X)

and

/(A)

'c(A)-g(A)

s(A)

where the degrees of

r(A)

and s(\) are less than that of g(A).


h{X)-giX) + r(X)
=

Then
+ s(X)
s(X)

k(X)-g(X)

and
[k(X) -h(X)]g(X)
=
A:(A)

r(X)

Now r(A)-s(A)

is of

degree less than

while, unless
=
0,

h(X) = 0.

[k(X)

- A(A)]g(A)

is of degree equal

to or greater than m.

Thus, k{X) - h(X)

r(A)

s(A) =

so that k{X) = h{X) and r(A) = s(A).

Then both

h(X) and r(A) are unique.

CHAP.

22]

POLYNOMIALS OVER A FIELD

175

2.

Prove:

If f(X) and g(A) are polynomials in F[X], not both zero, they have a unique greatest divisor d(\) and there exist polynomials A(A) and k(\) in F such that

common

(a)

da)
If,

h{X)-f(X) +
is the

ka)-g(\)

say,
1

/(A) =

0,
.

then d(X) =

bin,

g(X)

where i^

leading coefficient of g(k) and

we have

(a) with

A(A) =

and k(X) = 6

Suppose next that the degree of g(A)


(*)

is not greater

than that of /(A).


ri(A)

By Theorem

I,

we have

/(A.)

?i(A)-g(A) +
If r^(X)

where
h(X) =
If (ii)

r^^(X)

=0
7^

or is of =

degree less than that of g(A).

= 0,

then d(X) =

b'^^giX)

and we have

(a)

with

and k(X)
r^(A)
0,

6^

we have
g(A)
=

g2(A)-ri(A)
ri(A).
If r^(X)

+ r^iX)
= 0,

where

/^(A)

or is of

degree less than that of


HiX)
=

we have from

( i

/(A)

?i(A)-g(A)

and from
If

it

obtain (a) by dividing by the leading coefficient of ri(A).

r^(X) 4 0,

we have
'i(A)

(i")

93(A)
r^(X').

TsCA) + rsCA)
If rg(A) = 0,

where

rgCA) =

or is of

degree less than that of


= =

we have -

from

i)

and

(ii)

'2(A)

g(A)

?2(A)-'-i(A)

g(A)

?2(A)[/(A)

9i(A)-g(A)]

-92(A) -/(A)

[1 + 9i(A)-92(A)]g(A)

and from

it

obtain (a) by dividing by the leading coefficient of r2(A),


0,

Continuing the process under the assumption that each new remainder is different from
general,
(v)

we have,

in

'i(A)

9i+2(A)-'-i+i(A)

ri+2(A)

moreover, the process must conclude with


(V)

's-2(A)

9s(A)-'-s-i(A)

r^cA),

&(A)?^0

and
(^^

's-i(A)

9s+i(A)-'-s(A)

By

(vi), rjCA)

divides rs-i(A), and by


's-3(A)

(v).

also divides rs_2(A).

Prom

(iv).

we have

9s-i(A)-'s-2(A)

r5_i(A)
(vi),

so that rs(A) divides r^-gCA).

Thus, by retracing the steps leading to


c,

we conclude

that rs(A) divides

both /(A) and g(A).

If

the leading coefficient of rs(A) is


= =

then

rf(A)

= e~^'s(A).

Prom(l)

ri(A)

/(A)

91(A)

g(A)

/!i(A)

-/(A) + /ci(A)

g(A)
=

and substituting
A2(A)-/(A)

in (ii)

'2(A)

-52(A). /(A)

+
.

[l + 9i(A)-92(A)]g(A)

+ i2(A)-g(A)

Prom

(iii),

/3(A)

t^(K)

qa(>^)

r^(X)

Substituting for r^(X) and r2(A),

we have
92(A) 93(A)]g(A)

'3(A)

= =

[1 + 92(A) 93(A)]/(A)

[-9i(A)

93(A)

9i(A)

h3(X)-f(X)
finally,

43(A) g(A)

Continuing,

we obtain

rs(A)

hs(X)'f(X)

+
=

ksiX)-g(.X)

Then

d(X)

c-\^(X)

c-i/5(A)-/(A) + c-iA:s(A)-g(A)

h(X)-f(X) + k(X)-g(X)

as required.

The proof

that d(X) is unique is left as an excercise.

176

POLYNOMIALS OVER A FIELD

[CHAP. 22

3.

Find the greatest common divisor d(X) of


/(A)

3A^ + 1)C + llA + 6

and
III.

g(\)

)C

2)i-)^-X+2

and express d(\)

in the form of

Theorem

We
(i)
(li)
(iil)

find = =

/(A)
g(A)

(3A+l)g(A) +

(A'

+ 4A^+6A+4)
(A^

(A-2)(A^ +
=

4A^ +

6A+4) +

+ 7A+10)

A^+4A2+6A + 4
A^

(A-3)(A^ + 7A+10) + (17A + 34)

and
(Iv)

7A + 10

(^A + p^)(17A+34)
J-(17A+34)
=

The greatest common


Prom
(iii),

divisor is

A +

17A

+34
from

(A

+ 4A^ + 6A + 4)

(A

3)(A^ + 7A + 10)

Substituting for A^ + 7A + 10

(ii)

17A

+34

(A^

+ 4>f + 6A + 4)

(A

- 3)[g(A) - (A- 2Xi^ +


(A

4;? +

6A+4)]

(A^

- 5A +

7)(A^ + 4>f + 6A + 4) -

- 3)g(A)

and

for

+ 4A^ + 6A + 4 from

17A

+34
A +
2

(A^

- 5A + 7)/(A) + (-3A +

14A^

- 17A - 4)g(A)

Then
=

i(A^ - 5A +

7) -/(A)

i(-3A

+ 14A^ - 17A

4)-g(A)

4.

Prove:

If

the greatest

common

divisor of /(A) of degree

re>

there exist non-zero polynomials o(A) of degree


(a)

< m and

b(X) of degree < n

and g(X) of degree m > such that

is not 1,

a(X)-f(X) +

b(X)-g(X)

and conversely.
Let the greatest common divisor of /(A) and g(A) be d(X) ^
/(A)
=
rf(A)-/i(A)
1
;

then
=

and

g(A)

d(X)-g^(X)

where

/i(A) is of

degree

<n

and gi(A) is of degree


gi(A)-/(A)
=

<m. Now
=

gi(A)-rf(A)-/i(A)

g(A)-/i(A)

and
gi(A)-/(A)

[-/i(A)-g(A)]
(a).

Thus, taking a(A)

= gi(A)

and 6(A) = -/i(A),

we have

Conversely, suppose /(A) and g(A) are relatively prime and polynomials h(X) and ^(A) such that
A(A)-/(A) +
k(X)-g(X)

(a) holds.

Then by Theorem IV

there exist

Then, using

(a),

a(X)

= =

a(X)-h(X)-f(X)

+
+

a(A)-A:(A)-g(A)

-b(X)-h(X)-g(X)
if (a)

a(A)-i(A)-g(A)

and g(A) divides

a(A).

But this is impossible; hence,

holds, /(A) and g(A) cannot be relatively prime.

CHAP.

22]

POLYNOMIALS OVER A FIELD

177

5.

Prove: Every non-zero polynomial /(A) in F[\] can be written as


/(A)

c-q^(X).q^a)

...

qr(\)

where

,^

is a

constant and the q^(\) are monic irreducible polynomials in F[A].

Write (i)
/(A)
is the leading coefficient of /(A).

a-/i(A)
is irreducible,

where a
theorem.
(")
If g(A)

If /^(A)

then (i) satisfies the conditions of the

Otherwise, there is a factorization


/(A)

n-g(A)-A(A)
Otherwise, further factor-

and

/!(A)

are irreducible, then

(ii)

satisfies the conditions of the theorem.

ization leads to a set of monic irreducible factors.

To

prove uniqueness, suppose that

"n
are two factorizations with
Pi(A) which,
Pi(A).

9i(A)

?2(A)

. .

9r(A)

and

a p^(X) PaCA)

p^CA)

r<s.

Since 91(A) divides

Pi(A)-p2(A)
p^(\).

by a change
92(A) divides

in numbering,

may be taken as
...

Then

... ps(A), it must divide some one of the Since Pi(A) is monic and irreducible, ?i(A) =

Eventually,

we have
r

9i(A) = Pi(A) for

ity is impossible,

= s

and, after a repetition of the argument above, 92(A) = P2(A). r and Pr-^i(A) Pr+2(A) ... p^cA) = 1. Since the latter equaland uniqueness is established.
Ps(A)
i

P2(A)-p3(A)

= 1,2

SUPPLEMENTARY PROBLEMS
6.

Give an example

in

which the degree

of /(A) + g(A) is less than the

degree of either /(A) or g(A).

7.

Prove Theorem
Prove:

III.

8.

If /(A)

divides g(A) and h{\).

it

divides

g(A) + h(\).

9.

Find a necessary and sufficient condition that the two non-zero polynomials /(A) and g(A) each other.
For each of the following, express the greatest common divisor
()
(6)

in

F[X] divide

10.

in the form of

Theorem IV

/(A)
/(A)

= = = =

2A^-A=+2A2-6A-4,
A^A"

g(A) g(A)
1,

= =

A*-A=-A2+2A-2
A=
=

3A=

- 11A+

- 2A" - 2A -

3
1

(c)
(d)

/(A) /(A)
(a)

2A^ + 5A^+ 4A=

3A^ -

4A='

+ A^

- A^ - A + - 5A + 6,
1)/(A) +

g(A) g(A)
=

A% 2A%
-h

2A +

A= + 2A

Ans.

A=-2

- ^ (Ato

i (2A=+ l)g(A)
lo

(6)

A-

-l_(A+4)/(A) + l-(A^+5A+5)g(A)

('^)

^+1
^

^(A+4)/(A)-^ ^(-2A=^-9A2_2A + 9)g(A)


+

^"^^

jL(5A+2)/(A)

^^{-l5X^+4.A>?~55X+ib)sa)

11.

Prove Theorem VI.


Hint.

Let d{\) be the greatest common divisor of /(A) and g(A)


h{X) is a constant.

then

g(A) = d{X)-h(X)

and either d(X)

or

178

POLYNOMIALS OVER A FIELD

[CHAP. 22

12.

Prove Theorems VII and


Prove:

VIII.

13.

If

/(A) is relatively prime to g(A)

and divides g(X)-a(X),

it

divides a( A).

14.

The least common multiple of /(A.) and


and is of minimum degree.
(a)

g(A) is a monic polynomial which is a multiple of both /(A) and g(A),

Find the greatest common divisor and the least common multiple of
=

/(A)

= =

A^g.c.d. g.c.d.

1,

g(A)

A=l.c.m,

(b)

/(A)
(a)

(A-l)(A+lf(A+2).
= =

g(A)
=

(A+l)(A+2f(A-3)
1)

Ans.

A-l;

(A^- l)(A^ + A+
=

(b)

(A+i)(A + 2);

l.c.m.

(A- l)(A+ if (A + 2f (A-

3)

15.

Given 4

show

2 ij 12 ;
(o) (b)
(f)(k)

A^

SA^

- 9A =

and

(f)(A)

A -

sA!^

- 9A -

51

m(A)

= 0.

when m(A)

A^

- 4A -

16.

What property of a

field is not satisfied

by a polynomial domain?
/(c) = 0.

17.

The scalar
and only
if

c is

called a root of the polynomial /(A) if

Prove: The scalar e is a root of /(A)

if

A-

c is a factor of /(A).

18.

Suppose

/(A) =

(A-c)^g(A).

(a)
if

Show

that c is a root of multiplicity


if c

k-l

of /'(A),

(b)

Show

that c is a

root of multiplicity

k>

of /(A)

and only

is a root of both /(A) and /'(A).

19.

Take Show
Hint:

/(A) and g(A), not both 0, in F[\] with greatest that if D(A) is the greatest

common

divisor d(X). Let

be any field containing F.

common

divisor of /(A) and g(A) considered in K[A], then


/(A) = s(X)-D(X).

D (A)
=

= d(X).

Let

d(X) = h(\)- f(\) + k(X)- g(\).


is normal it

g(A) = t(X)-D(X).

and D(X)

c{X)-d(X).

20.

Prove:

An n-square matrix A

'

can be expressed as a polynomial


+
...

a^A
in
/I.

+ a^^^A

+ aiA + oqI

chapter 23
Lambda
Matrices

DEFINITIONS. Let F[X] be a polynomial domain consisting


in F.

of all polynomials in A with coefficients

non-zero

mxn

matrix over F[Aj

oii(A)
(23.1)

ai2(A)
a22(A.)

...
...

ai(A) CsnCA)

^(A)

[ij (A)]

asiCA)

Omi(A)
is called a A-matrix.

a^^iX.)

...

aj^n(>^)

Let p be the maximum degree in A of the polynomials a^jiX) of (23.1). written as a matrix polynomial of degree p in A,
(23.2)

Then A(X) can be

^(A)

Ap\^

sp-l A.^X^-^

...

A,\

+ Ao

where the ^^ are


Example
1.

mxn
'^(A)

matrices over F.

A^+A
.

A* + 2A= + 3A^ + 5]
A^

A=

- 4

- 3A=

J
1

A^ +
u

-J

A +

-4
L

is a A-matrix or matrix polynomial of degree four.

zero.

The

A(X) is ra-square, it is called singular or non-singular according as \A(X)\ is oris not Further. A(X) is called proper or improper according as A^ is non-singular or singular matrix polynomial of Example 1 is non-singular and improper.
If

OPERATIONS WITH A-MATRICES.


over F(X)
(23.3)

Consider

the

two

n-square

A-matrices

or

matrix polynomials

^(A)

A^X^

.P-'
p -1

+ Ai^X + Aq

and
(23.4)

B(X)

S^_,

a"^

'

...

SiA + So

(j

The matrices (23.3) and (23.4) are said to be equal, ^(A) = 5(A), provided = Bp=a andAVI,' ^ ^ = 0,1,2 p).

The sum A(X)


two A-matrices.

+ S(A) is a A-matrix C(A) obtained by adding corresponding elements of the

The product /1(A) B(A) is a A-matrix or matrix polynomial of degree at most p+q. If either ^(A) or B(A) is non-singular, the degree of ^(A) 5(A) and also of 5(A) /1(A) is exactly p + 9. The equality (23.3) is not disturbed when A is replaced throughout by any scalar k of F For example, putting A = A in (23.3) yields

A(k)

Apk^

+ Ap_.^k^~^ +

...

A^k ^ Ao

179

180

LAMBDA MATRICES

[CHAP. 23

However, when \

is replaced by an re-square matrix C,

two results can be obtained due

to the

fact that, in general, two ra-square matrices do not commute.

We

define

(23.5)

Ag(C)

A^C^
. ^^ C^A^

A^_,C^ p-i
A^ ^p

...

+ A,_C + Ao

and
(23.6)

4 (J)
2.

. C^ + .

+ ^^^i +

^o

called respectively the right and left functional values of A(\).


A' Jx^

Example

Let

X XmI
X''

o1.

To
[l

i1.
oj

To
[-2

il

2
4_

(X)

\X-2
Then
j,nd

+ 2j

P [O
4J

and
2j

=
_3

ij

[lo

15

Ajf(C)
[o ij [3

'

[1

oJ

[3

4)

'

[-2

14
2J

26

12
27
1.

Al(C)
[17

See Problem

DIVISION.

In

Problem
I.

2,

we prove
if

If

A(\) and B{X) are matrix polynomials (23.3) and (23.4) and

Bq is non-singular,

then there exist unique matrix


/?2(A) are either zero or of

polynomials

Qi_(X), RxiX);

Q2(X), R2(X), where /?i(A) and

degree less than that of 6(A), such that


=
(?a(A)-S(A) + Ri(A)

(23.7)

.4(A)

and
(23.8)
If

A(X)

S(A)-(?2(A) + R2(X)
if

/?i(A)=0,
/1(A).

S(A) is called a right divisor of i(A);

/?2(A) = 0,

6(A) is called a left di-

visor of

jV
Example
3. If

A"^

+ A -

A% A% A +
2A''

21

(A)

L
'4(A)

2A^

-A
2

and
+

X%
(A)

'

2A

a^aJ
Q-l(X)-B(X)

'1

then

Ta^-i
=
|_

A-iirA^+i

ii
A'^+aJ

r 2A
^

2A+3
+

Ri(X)

2A

JL

j_-5A

-2A_

and

A(X)
1_

A^ + aJLA-I

ij

B{k)-Q2{X)

Beie, B (A) is a left divisor of A (A)

See Problem

3.

A
(23.9)

matrix polynomial of the form

6(A)

bqX"-!^ + 5,_,A'?"'-/ +
B(A) =

...

+ b,X

+ bol^

i(A)-/
ra-square ma-

is called scalar.
trix

scalar matrix polynomial

fc(A)-/

commutes with every

polynomial.
If in

(23.7) and (23.8),

6(A) = b(X)

I.

then

(23.10)

A(X)

Q^(X)- B(X) + Rt(X)


rA^ +

B(X)- Q^(X) +

R^X)

2A

A + ll
and

Example

4.

Let

(A)

B(A) = (A + 2)/2.

Then

LA^^-I

2A+1J

CHAP.

23]

LAMBDA MATRICES

181

rA + 2

If

ll

To

-ll

If

/?i(A)
II.

in (23.10), then

^(A) = 6(A)- /

(?i(A)

and we have

matrix polynomial i(A) = [oy (A)] of degree n is divisible by a scalar matrix polynomial S(A) = 6(A) / if and only if every oy (A) is divisible by 6(A).

THE REMAINDER THEOREM.


matrix over F.
(23.11)

Let A(X) be the A-matrix of (23.3) and let B =[6^,-] be an re-square Since Xl - B is non-singular, we may write
A(\)

(2i(A)-(A/-S) +

/?!

and
(23.12)

A(\)
A.
It

(A/-fi)-(?2(A) + R2

where R^ and Kg are free of

can be shown

m.

If

the matrix polynomial A(X) of (23.3) is divided by \I

-B, where B =[b^-]


''

is

ra-

re, until

remainders ^1 and R2, free of


=
Aj^(B)

A, are obtained,

then

=
=

A^B^
B^Ap

Ap_^B^~'
B^~'a^_^
\I - B
=

...

A,B

+ Ao

and

a,
Examples. Let

Aj^(B)

...

+ BA, + "^

^
The

^(A)

."^M
A='

and

r~^
|_-3
flO

LA-2
[a +

+ 2j
3

A-4J
isl 26j
=
'^'

IfA-l
4JL-3

[4
and
'^'^

A+

-2I A-4J

"

[14

U
2,

a'-JH'

a!4J
j?2

[17

27]

(^'-B^Q^i^^-^^
Theorem
III.

Prom Example

R^ = Ag(B) and

= A^(B) in accordance with

When ^(A)

is a scalar matrix polynomial

'4(A)

/(A)-/

apIX^ + ap.jx''^ +

...

+ ai/A + Oq/

the remainders in (23.11) and (23.12) are identical so that

^1
and we have

^2

apB^

ap_^B^~^

...

+ oifi + aol

IV. If a scalar matrix polynomial /(A)-/ is divided by


free of A, is obtained, then

A/-B

until a remainder/?.

= f(B).

As

a consequence,
V.

we have

scalar matrix polynomial /(A)-/ is divisible by

KI^- B

if

and only

if

f(B) =

0.

CAYLEY-HAMILTON THEOREM.
trix

Consider the re-square matrix A = [o,-,-] having characteristic maijj Xl - A and characteristic equation 0(A) = \Xl - A\ = 0. By (6.2)

(A/-i)-adj(A/-i)

(f>(X)-I

182

LAMBDA MATRICES

[CHAP. 23

Then ^(A)

/ is divisible

by XI - A and, by Theorem V,
[oj,-]

(^(A) =

0.

Thus,
<fi(X)

Vl. Every square matrix A =

satisfies its characteristic equation

0.

Example

6.

The characteristic equation

of

is

V.i
32
31

A -

7>i

+ llX - 5 =

0.

Now

62

31 31

63

31

62

32

and
32
31 31

62 63
62

3l"

12
13
12

6
6
7.

"2

2 3
2

r
1

o"
1

31
32_

- 7

6 6

11

_1

1_

See Problem

4.

SOLVED PROBLEMS
1.

For the matrix

A(X)

[,
A+
1

compute

-4

ij

p(C) and 4r(C) ^

when

C=
L
2J

^'<'^'

i o]C

'2
'

"3

o][o

'

c 3

and

2.

Prove:

there ^(A) and B(A) are the A-matrices (23.3) and (23.4) and if Bq is non-singular, then are either and R^iX) exist unique polynomial matrices Q-,(X), i(A); QsO^). R2M. where Ri(A) zero or of degree less than that of 6(A), such that
If

(i)

^(A)

QAX)-B(X) +

Ri(X)

and
(ii)

^(A)
If

6(A) (?2(A) + R^iX)


fii(A) = A(X).

p <

q,

then

(I)

holds with (3i(A) =

and

Suppose that p iq; then


=

4(A) - ApB-^B(X)X^''^

C(A)

where C(A)
If

is either zero or of

degree at most p q,

1.

C(A) is zero or of degree less than


(3i(A)

we have

(i)

with

ApB'q'-^''^

and

R^iX) =

C(A)

CHAP.

23]

LAMBDA MATRICES

183

If

C(X) =

C^X +

...

where

s>

q.

form

A(\)
If

- ApB-^''B(\)\P-1 q,

C^S-1B(A)X^-'?
(i)

D(X)

D(X) is either zero or of degree less than

we have

with

Qi(\)
otherwise,

Aj^B-^XP'^l .C^B'^^X'-^ ApB-'XP'^ + C^B^^X^-'f

and

Ri(X) = 0(A)
...

we continue
(i).

the process.

Since this results in a sequence of matrix polynomials C(A), D(A),

of decreasing degrees,

we

ultimately reach a matrix polynomial which is either zero or of degree less than g

and we have

To

obtain

(II),

begin with
^(A)

- B(X)B-^''ApXP~1
left

This derivation together with the proof of uniqueness will be

as an exercise. See Problem

1,

Chapter 22.

rA^ +
3.

Given

2A=-l
1

A'-A-l]
and
S(A)
A= A"

^(A)
|_ L

A + A' A" A^ +

L-A^ + 2

A^

- A

find matrices Q-l(X), Ri(X);


(a)

(?2(A),

R2(X) such that


(b)

A(X)

= Qi(X)-B(X) + Ri(X).

A(X)

= S(A)-(?2(A) + R^iX)

as in Problem

2.

We have

and

Here,

So

-ii

and
ij

S2

-,
=

Tl [i

ii

L-1
(a)

2J

We compute
^(A)

-^4S;^5(A)A^

3 =
1

1'

-2
A +

1]

10
-10
3]
2

fo

-l"!

^ A +

f-l

-1'
C(A)

11
D{X)

C(A)

- C3S2^S(A)A

2
= 3

A2 +
1

A +

-6

and
D(A) - D^B^^BCX)

J
3 5
1

11 L
P
5]

P-i

-1I
= _

-6
=

5"]

["-13

-2
3J

^ +
[_

-9
'1

-6A-13 -2A-9
A= +

5A + 3 3A + 5

fli(A)

Then

(?i(A)

(A^X^ + Cs A

+O2 }B^-^

--

A +

p
fA^

q
+

P
e]

^1

4A

+ 4
4

A^ +

sA +
5

2A+

3A +

(b)

We compute
^(A) - B(A)S2^'44A^ E(X)

(A) - 5(A) B2

^sA

and
F(X) - B(X)B2

F^

ftsCA)

184

LAMBDA MATRICES

[CHAP. 23

Then

QziX)

B2^(A^>? + 3 A +

F2)

[A^+4A
A^ +

+ 4
9

2X +

2I

6A +

3A + 5J

1
1

2
1
1

4.

Given

3 2

use the fact that A satisfies

its characteristic

equation to compute A"

and A

also, since

is non-singular, to

compute A
-2
-1

and A'

A-1
A/-^|
Then
-3 -2

-1

A-1
-3

A^-3A^-7A-11

A-1
5"

"8

8 7 8

"l

1
1

2"

"1

0"
1 1

42
=

31

29
31

3A +1 A + 11/

8
_13

8
8_

3 2

1
1_

11

45 53

39

45

42

42

31

29"

'8 + 7
8

8 7
8

5'

"1

1 1

2
1
1_

193
=

160
177

144
160

^*

3/ +7/
=

UA

45
_53

39 45

31
42_

8 8

3
_2

224
272

13

224

193

From

11/

-1 A -3-4

+A

we have
'1
0' '1

1 1

2"

8 7
8

5 8
8_

-J-7/

34 +/II
11
_0

1
1_

3 2

1
1_

8
_13

-2
-1

-1
5

-3
-1

11
7

-2
2 5

-1"
5

"1

0"

1 1

2
1 1

i{_7^-i -3I

+ A\
121

-3
-1

- 33

1
1

11

3 2

-2
29'

-8
40
121

-24
-1
40

-24
-8

-27

5.

Let Ai, As,

,^n
x:.

be the characteristic roots of an re-square matrix A and let h(x) be a polynomial

of degree p in

Prove that

\h(A)\

= h(\)- h(X2)

h(X^).

We have
(i)

\XI-A\

(A-Ai)(A-Ao)...(A-A^)

Let
(ii)

h(x)

c(,si- x)(s2-x) ...(Sp - x)

Then
h(A)
=

c(sil-A)(S2l-A)...(spI-A)

4
CHAP.
23]

LAMBDA MATRICES

185

and
|A(^)|
=

c'P\siI-A\-\s2l-A\ ...\sp!-A\

!c(si-Ai)(si-A2)

...

(si-A)i
...

Jc(S2-Ai)(S2-A.2)

(S2-X)!

...

{c(Sp -\t)(,Sp -As)

...

(Sp

-X)!

jc(Sl-Ai)(S2-Xl) ... (S^- Xl)i {c(si-A2)(S2-A2) ... (Sp-\2)\


=

...

ic(si-A)(s2-A)

...

(s^-A)!

h(\i_)h(\2) ...h(\^)

using

(ii).

SUPPLEMENTARY PROBLEMS
6.

Given

A(\) =

Pr^ [a^+i
[_A^

^1
A-iJ

sna

B(A)=r^^
[a +
1

^^^^1 a

compute:

_r2\2+2A
(a)
-4(A)

A^+2A1
2A ij

+ B(A) + A + 2

(6)

A(\) - B(A)

r2A
^

-A^l

[a^-a
rA* +

-ij
A* + 3A= + 3A=1 -, A o
A-'

2A=+A=+A
o
1

(c)

^(A)-S(A) =

L ^ +2A^ (rf)

A'

2A^J

B(A)M(A)

r2A'' +
1_

3A'+A^+A
+ 3A^ + 3A

2A^

-a]

2A='

2\^

7.

Given

/1(A)

compute:

Ag(C)

B;p(C)
5
5

-2
-1

[-:
3
<?i?(C)

1
3"]

4;f(C)-B^(C)
[l7

Bn(C)-4n(C) i?^
-7J

^;?(C)

= 9

= 3

-3

-3I

^i(C)

^i(C)-B^(C)

B^(C)-A^(C)

E
[3

:]

[::;]

3-

--M
if

where P(A) = 4(A)- B(A) and Q(X) = B(A)M(A).

8.

If /4(A)

A-matrix,

and B(A) are proper n-square A-matrices of respective degrees p and q, and show that the degree of the triple product in any order is at least p+q.

C(A) is any non-zero

186

LAMBDA MATRICES

[CHAP. 23

9.

For each pair of matrices A(X) and B(X), find matrices

(?i(A), /?i(A);

Q^X), Rd^) satisfying (23.7) and (23.8).

|> + 2X
(a)
.4(X)

A
,,
,

La%i
f
-A.2

,,

"

'

1
I
.

Fa
BiX)

a]

a^-aJ
+
A.

''''-'I

-aJ

-A^ + 2A + 2l
1

FA-I

-ll

(b)

AiX)

U.2A-1
A^-2A+1
A'*

''''-

J'
+ A^ +

[o
5A^+2A
4A^ +

x4
+ 4
1

A'^ 1

7A-2
+ 2

(c)

.4(A)

3A''

3A 2

3A^+ 2A

6A+

2A - A +

A^ + 2A''

A + A^ +

8A -

A^+1
B(X)

3A-1
A+
1

2A

A^

A-2

2A

A""

3AnA"-l
(d)

A"-l
A* + A^ +
2

A^-A
A 1

A(X)

A=-A^+l
A^ + A

B(X)
2

A^-2A-1 A+1
A

A^
A''

A +

+ A

A^-2A
A^ + A 1

A+
a1

2A* + A -

A-2
.

[\+i
Ans.
(a)

r 2a
'^^''-'-

a-i1
-A + 2j'

fo [l

o]
ij

Q
X

^=^'^^

ij'

''^'''+2

f-A
(b)

-A-l]
.,(A)^0;

f-A-l

-A +

ll

f-

3
1

0,(A)=
\^_^^^

,,(A)=

_J,
A^+3

|^_^

J,
-fjA-3

..(A).
|^_ 1

-A+1
(c)

-A+7

-16A + 14
fti(A)

-5A + 2

(?i(A)

A^-1

3A + 5
A
A"
2

-3A+2

-21A + 4

-2A+3
loA +
3

A-5
18A-7

2A-3
A-l
<?2(A)

A-6

5A-7

A^ +
1

,2

A
2

/?2(A) =
1

A+

3A^ + 6A + 31
(d)

-3A^-5A-16
A=^

3A'^-7A
-A^ +
2A'^

+ 8

Qi(X)

A -

3
1

- A -

4A -7
1

-2A -

- 2A -

81A +
Ri(X)

46
1

-12A 15A -

16
9

-85A 12A -

23
5 2

4A -

-9A -

-7A
+ 31

17A -

3A^+5A
<?2(A)

-A^-A-4
A"
3

2A^-4A

+ 3

A - 14

-2A + 6A - 6

-3A 7lA +
fl2(A)

2A - 2A -A +
ll

46

-12A llA +

8 6 4

-26A - 30 -15A - 30

4A - 4
16A - 16

2A +

10.

Verify in Problem 9(b) that

Ri(A) = .4^(0) and

figCA) = -4^(0)

where B(A)=A/-C.

CHAP.

23]

LAMBDA MATRICES

187

11.

Given

[\^

3X+
^2 A^-3A
.

1
''*

A-2
.4(A) =

C(X)

=1fx

A +

2
^

A 1 A A+lJ

+ 2j

[A - 3 ^A

(a)

compute

B(A)- C(A)

(b)

find (3(A) and fi(A) of degree at

most one such that


+
l]

4(A) = Q(X)- B(\) + R(\).

Fa^'+SA^-VAL^- 5A=+llA 12.

A^'+SA^+sA

[A + 4

A +
A -

si

r-9 A +
S(A)

-A - gl

10

A='-A^-3A+2j
in

A-6

ij

|_13A-6

9A +

10J

Given

compute as

Problem 4

10
'1
,

A^

|_i2

^^1 i7j

[I
13.

!;]

Prove:
Hint.

If

A and B

are similar matrices and g(A) is any scalar polynomial, then g(A) and g(fi) are similar.

Show
If

first that

and B
S^

are similar for any positive integer k.

14.

Prove:

fi

= diag(Si,

B^) and g(A)

is

any scalar polynomial, then

g(B)
15.

diag(g(Si), g(B2)

g(B))

Hint.
16.

Prove Theorem ni. Verify: A/ - B divides A(\)-Ajf(B).


= 0.

The matrix C is called a root of the scalar matrix polynomial B(A) of (23.9) if B(C) C is a root of B(A) if and only if the characteristic matrix of C divides B(A).
Prove: If Ai, Aj A are the characteristic roots of A and characteristic roots of f(A) are /(Ai), /(A2) /(A)Hint.
if

Prove: The matrix

17.

f(A) is any scalar polynomial in A, then the

Write

\- f(x)

c(xi- x)(x2=

x) ...(x^

- x)

so that

\\I

Nowuse

|*j/-4|

(xj-Ai)(::i-A2)...(*^-A)

and

- f(A)\ = c'^\xj - A\ \x2l - A\ ...\xsl -A\. c(xi -Xj) (x2-\j) ... (x^ -\j) = \- f(\j).
1

-1
3
1

18.

Find the characteristic roots of

f(A) =

A'^

-2A+3,

given

2
1

19.

Obtain the theorem of Problem 5 as a corollary of Problem 17.


Prove:
If

20.

is an invariant vector of

A of Problem

17, then

is an invariant vector of f(A)

21.

Let A(t)
A(t)

= [a^,-()]

where the

a^j{t) are real

polynomials

in the real variable

t.

Take

and differentiate the last member as


nition

if it

were a polynomial Sfnomial with wit! constant coefficients to suggest the defi,

22.

Derive formulas
(a)
-r-\A(t)

for:

+ B{t)\;

(b)

-T-\cA(t)\.

where

dt

c is

a constant or c =[ciA;

(c)

^{A(t)dt

B(t)\;

(d)

dt

^A

'it).

n
Hint.

use

For (c). write A{t)-A-\t) = /.

A(t)- B(t) = C(t) = [ci;.(t)]


'

and differentiate

cv,-()

k=i ^^

a,-i,(0 6i,,-().

For

(d),

"^^

chapter 24
Smith Normal Form

BY AN ELEMENTARY TRANSFORMATION
(7)

on a A.-matrix A(X) over F[\] is meant:


ff|-;

The interchange

of the ith and /th tow, denoted by


.

the interchange of the ith and /th

column, denoted by K^j


(2)

The multiplication

of the ith row by a non-zero constant k, denoted by H^(k);

the multiplication of the ith column by a non-zero constant k, denoted by K^ik).


(3)

The addition

to the ith

row of the product of

/(X),

any polynomial of F[\], and the /th row,

denoted by ^^(/(X));
the addition to the ith column of the product of /(X) and the ;th column, denoted by K^j (/(X)).

These

are the elementary transformations of Chapter 5 except that in (3) the word scalar has

been replaced by polynomial. An elementary transformation and the elementary matrix obtained by performing the elementary transformation on / will again be denoted by the same symbol. Also, a row transformation on ^(X) is effected by multiplying it on the left by the appropriate H and a column transformation is effected by multiplying ^(X) on the right by the appropriate K.
Paralleling Chapter
I.

5,

we have
is an elementary

Every elementary matrix in F[\] has an inverse which in turn

ma-

trix in F[\].
II.

If

\A(\)\ = k

0,

with k in F, A(\) is a product of elementary matrices.

III.

The rank

of a X-matrix is invariant under elementary transformations.

Two

re-square X-matrices i(X) and B(X) with elements in F[X] are called equivalent proff 2
'

vided there exist P(\) = H^


(24.1)

^1 and Q(\) = K^- K^


=

...

K^ such

that

B(X)

P(\)-A(X)-Qa)

Thus,
IV.

Equivalent

mxn

X-matrices have the same rank.

THE CANONICAL SET.


V.

In

Problems

and

2,

we prove

Let A(\) and B(X) be equivalent matrices of rank r; then the greatest common divisor of all s-square minors of A(\), s r, is also the greatest common divisor of all ssquare minors of B(\).
In

Problem
VI.

3,

we prove
r

Every X-matrix A(X) of rank

can be reduced by elementary transformations

to the

Smith normal form

188

CHAP.

24]

SMITH NORMAL FORM

189

A (A)
/2(A)

(24.2)

N(\)

...

0_

where each

/^(A) is

monic and
of rank

f^(\) divides f^^^ (A),


r

(i

1, 2, ...

1).

When a A-matrix A(\)


all

has been reduced

to (24.2), the greatest

common

divisor of

s-square minors of A(\), s ^ r, is the greatest common divisor of all s-square minors of A^(A) by Theorem V. Since in N(\) each f^(X) divides /^^j(A), the greatest common divisor of all ssquare minors of N(\) and thus of A(\) is
(24.3)

g,(A)

/i(\)-f2(A)-...-4(A),
to

(s

= l,2,

.r)

Suppose A(\) has been reduced


/V(A)

diag(/-i(A), /2(A)

/^(A), 0,

and to
/Vi(A)

diag(Ai(A). AsCA)

h^iX),

O)

By

(24.3),

gs(A)

/i(A)-/2(A)-...-/s(A)

Ai(A)-A2(A)-...-As(A)
so that

Now
(24.4)

gi(A) =/i(A) =Ai(A),

g2(A) =/i(A)-/2(A) =Ai(A)-A2(A)


1,

U\)
r)

h^W,

in gen-

eral, if

we

define

go(A) =

then

s:s(^)/s-i(^)

4(^)

h^(\).

(s

1, 2

and we have
VII.

The matrix N{X)

of (24.2) is uniquely determined by the given matrix /1(A).


for

Thus, the Smith normal matrices are a canonical set


A +
2

equivalence over F[A].


A.

A +

+ 3

Example

1.

Consider

.4(A)

A'+2A^+A
A^ +

A^+A'^ + A
A^ +

2A%3A'^+A
3A^ +

3A +

2A +

6A

+ 3

It

is readily found that the greatest

common divisor

of the one-row minors (elements) of

A(X) is

gi(A) =

1,

the greatest
= A^

common
=

divisor of the two-row minors of A(X) is g2(A)

=X,

and g3(A)
/i(A)

= i\A(X)\

+X^

Then, by (24.4),
/2(A)

gi(A)

1,

g2(A)/gi(A)

A,

/3(A)

g3(A)/g2(A)

A^+A

and the Smith normal form of A(X) is

10
A'(A)

A^+A
For another reduction, see Problem
4.

INVARIANT FACTORS. The

polynomials

f^(X),f-2{X)

/V(A)

in the diagonal of the Smith normal


1,

form of A(X) are called invariant factors of ^(A). If 4(A) = fk(X) = 1 and each is called a trivial invariant factor.

k r,

then

/i(A) = /2(A) =

...

As

a consequence of Theorem VII,

we have

190

SMITH NORMAL FORM

[CHAP. 24

VIII.

Two

re-square X-matrices over F[X] are equivalent over F[X\ if and only if they

have the same invariant factors.

ELEMENTARY
(24.5)

DIVISORS. Let A(X) be an re-square X-matrix over F[X] and

let its invariant factors

be expressed as
f^(\)

{p,(X)\'^i^\p^(\)fi^...\PsMi^^^.(i = i.2

r)

where pi(A), p2(X), ., PgCX) are distinct monic, irreducible polynomials of F[X\. q^j may be zero and the corresponding factor may be suppressed; however, since r-l;/ = l,2, .s). /i+i(A), qi+t,j^gij. (f = 1.2

Some
fj

of the

(A)

divides

The
of A(X).

factors

!p-(X)S'^*i

which appear

in (24.5) are called

elementary divisors over F[X]

Example

2.

Suppose a 10-square A-matrlx A(X) over the rational

field

has the Smith normal form


I

10

10
(A.-1)(A^+1)

'

(A-1)(A^ + 1)^A
(A-l)^(X" + l)^A^(A"-3)

The rank

is 5.

The invariant
/i(A)
=
1,

factors are
/2(A)
=
1,

/3(A) =

(A-1)(A^+1),

U(k)

(\-l)(X^+lfX.
are

/5(A)

(A-l)''(A.''+lf A^(A''-3)

The elementary divisors

(X-l)^

A-1,

A-1.

(A^'+l)'',

(A^+l)^,

(A'^+l),

A^

A,

A""

- 3 each ele-

Note that the elementary divisors are not necessarily


mentary divisor appears as often as
it

distinct;

in the listing

appears in the invariant factors.

Example

3.

(a)

Over the

real field the invariant factors of A(X) of

Example

2 are

unchanged but the

ele-

mentary divisors are


(A-l)"^,

A-1,

A-1,

(A^lf,

(A^'

l)''

(A^'

l),

X',

A,

X-\fz,

X+x/l

since
(6)

A^-3

can be factored.
field the invariant factors remain

Over the complex


are

unchanged but the elementary divisors

(A-1)^,

A-1,
(X-if,

A-1,
X-i,

{X + if,
A^,
A,

(X + if,

X + i.

(X-if.

X-\/l,

X+\/l

The

Invariant factors of a A-matrix determine its rank and its elementary divisors;

conversely, the rank and elementary divisors determine the invariant factors.

Example

4.

Let the elementary divisors of the 6-square A-matrix A(X) of rank


A=,
A"",

be

A,

(A-l)^

(A-^)^

A-l,

(A+l)^

A+

Find the invariant factors and write the Smith canonical form.

To form

/5(A), form the

lowest common multiple of the elementary divisors,


/5(A)
=
a" (A-1)'' (A

i.e.,

if

CHAP.

24]

SMITH NORMAL FORM

191

To form

f^i\),

form the lowest

common multiple

remove the elementary divisors used of those remaining, i.e.,


UiX.)

in /sCX) from the original list

and

A^(X-lf(A+l)
1.

Repeating, /sCX) =

A(X-l).
is

Now

the elementary divisors are exhausted; then /sCX) = /i(A) =

The Smith canonical form


1

\(XN(\)

1)

A^(A-lf (A + 1)
A=(A--lf(X+lf

also are the elementary divisors.

Since the invariant factors of a A-matrix are invariant under elementary transformations, so Thus,
IX. Two re-square X-matrices over F[\] are equivalent over F[X] have the same rank and the same elementary divisors.
if

and only

if they

SOLVED PROBLEMS
1.

Prove:

If P(\) is a product of elementary matrices, then the greatest common divisor of all ssquare minors of P(\)- A(X) is also the greatest common divisor of all s-square minors of A(\).
It

is

necessary only

to

consider P(A)-.4(A) where P(A) is each of the three types of elementary matrices H.

Let R(\) be an s-square minor of .4(A) and let S(A) be the s-square minor of P(X)- A(\) having the same position as R(X). Consider P(A) = H^j its effect on ^(A) is either (i) to leave R(X) unchanged, (u) to interchange two rows of R(X). or (iii) to interchange a row of R(X) with a row not in R(X). In the case of (i), S(X) = R(X); in the case of (ii), S(A) = -R(X); in the case of (Iii), S(X) is except possibly for sign another s-square minor of ^(A).
;

Consider P(X)

= H^ (k);

then either S(A) = R(X)


Its effect

or S(X) = kR(X).

(i) to leave R{X) unchanged, (U) to increase one of the rows of R(X) by /(A) times another row of R(X). or (iii) to increase one of the rows of R(X) by /(A) times a row not of R(X). In the case of (i) and (li), S(X) = R(Xy, in the case of (iii),

Finally, consider

P(A) = H^j (/(A)).

on ^(A) is either

S(X)

R(X) f(X)-T(X)

where 7(A) is an s-square minor of

.4(A).

Thus, any s-square minor of P(X)' A(X) is a linear combination of s-square minors of /4(A). If g(A) is the greatest common divisor of all s-square minors of .4(A) and gi(A) is the greatest common divisor of all s-square minors of P(A)--4(A), then g(A) divides gi(A). Let B(X) = P(XyA(X).

Now

.4(A) =

P"^(A)-

fi(A)

and P~'^(X) is a product of elementary matrices.

Thus, gi(A) divides g(A) and

gi(A) = g(A).

2.

Prove:

If P(X) and Q(X) are products of elementary matrices, then the greatest common divisor of all s-square minors of P(A) -^(A) -QiX) is also the greatest common divisor of all s-square minors otA(X).

Let B(A) = P(A)M(A) and C(A) = B(X) Q(X). Since C'(A) = Q'(X)- b'(X) and Q'(X) is a product of elementary matrices, the greatest common divisor of all s-square minors of C'(A) is the greatest common divisor of all s-square minors of s'(A). But the greatest common divisor of all s-square minors of C'(X) is the great

est

common divisor of all s-square minors of C(A) and the same is true common divisor of all s-square minors of C(A) = P(X)- A(X)- Q(.X) is
square minors of ^(A).

for B'(A)

and B(A). Thus, the greatest

the greatest

common

divisor of all s-

192

SMITH NORMAL FORM

[CHAP. 24

3.

Prove:

Every A-matrix A(\) =

[a^j(\)]

of rank

can be reduced by elementary transformations to

the Smith normal form


fi(X)

/2(A)

N(X)

L(A)

where each

fi(\) is

monic and

f^(X) divides /^^j^(A),

(i

1, 2,

...,r-

1).

The theorem is true for A(\) = 0. Suppose A(X) ^ 0; then there is an element a::(K) of minimum J By means of a transformation of type 2, this element may be made monic and, by the proper interchanges of rows and of columns, can be brought into the (l,l)-position in thematrix to become the new aii(A).
7^

degree.

(a)

Suppose a 11 (A) divides every other element of ^(A). duced to


/i(A)
(i)

Then by transformations

of type

3,

4(A) can be re-

B(A)

where /i(A) = a 11(A).


(6)

Suppose that an(A) does not divide every element of A(X).


which is not divisible by

Let

ai--(A)

be an element in the

first

row

an (A).

By Theorem

I,

Chapter 23, we can write

ay (A)
where
type
ri^-(A) is

g(A)an(A) + ry(A)
Prom the
/th

of degree less than that of aii(A).

column subtract the product of q{X) and

the first column so that the element in the first row and /th column is
2,

now

ri,-(A).

By a transformation
it

of

replace this element by one which is monic and, by an interchange of columns bring

into the
(i).

(l,l)-position as the

new oii(A).
row and the

If

now an(A) divides every element

of A{X),

we proceed

to obtain

Otherwise, after a finite number of repetitions of the above procedure, we obtain a matrix in which every

element
If

in the first

first

column is divisible by the element occupying the (l,l)-position.

this element divides every element of A(X),

we proceed

to obtain

(i).

Otherwise, suppose

a,-,-(A)
"J

is

not divisible by

aii(X) - qix(X)- a^^iX) and aij(A) = gijXA)- aii(A). From the ith row subtract the product of gii(A) and the first row. This replaces aii(A) by and aij (A) by aij{X) - qii(X) a^jiX).

On (A).

Let

Now

add the

jth

row

to the first.

This leaves oii(A) unchanged but replaces aij(A) by


^ij(X) +
it

aij (A)

- qii(X)- aij(X) + aj(\)

gij(A)U- ?ii(A)!aii(A)

Since this is not divisible by aii(A), we divide

by aii(A) and as before obtain a new replacement (the

remainder) for aii(A). This procedure is continued so long as the monic polynomial last selected as aii(A) does not divide every element of the matrix. After a finite number of steps we must obtain an a 11(A) which

does divide every element and then reach


Next,

(i).

we

treat B(A) in the

same manner and reach


i(A)

/2(A)

C(A)
Ultimately,

we have the Smith normal

form.

Since /i(A) is a divisor of every element of B(A) and /2(A) is the greatest common divisor of the elements
of S(A), /i(A) divides /2(A).

Similarly,

it

is found that

each /^(A) divides fi^^iX).

CHAP.

24]

SMITH NORMAL FORM

193

4.

Reduce
X + ^(A)
A='

A+
+A
2

A + 3
+ A
1

+ 2A= +

A +

a""

2A''

+ 3A^

+A

A^

3A +

X" + 2

A +

a""

+ 6a + 3

to its
It

Smith normal form.


is not

necessary to follow the procedure of Problem 3 here. The element /i(A) of the Smith normal common divisor of the elements of A(\); clearly this is 1. We proceed at once to obtain such an element in the (l,l)-position and then obtain (1) of Problem 3. After subtracting the second column
form is the greatest

from the

first,

we obtain
1

A+
a"
1

A+
2A +
3A^ +
1

"

A + A

A +

'4(A)

A^

A% A
2A +

3A%A
6A
+ 3

A%A
2A^ + 2A.

A+
1

A^ +

A + A

2A + 2A_

Lo

S(A)J

Now

the greatest

common

divisor of the elements of S(A) is A.

Then

A+A
2A^ + 2A and this is the required form. 2A^ + 2A
A" + A

5.

Reduce
A -4(A)
1

A + 2
A''

A""

+ A

A^

2A
:

A=- 2A
to its

- 3A +

A= + A -

Smith normal form.


find
1

We

AA=

A +

AA

A +

2"

^(A)

"l
^\^

AA

A^-3A
"

+ 2

A=+2A A^+A-3_
"l

A A +
1 1

A+1
1

-A -

-V-

-1

-A -1

A +
-A'

A+1
^23(1); Hs2(A +
l),

A+1

A+

-A
l),

A(A+i)
.Ksi(-A -2);

using the elementary transformations ^i2(-l);

^2i(-A), ^3i(-A +

2);

/f2i(-A +

HqsC-I);

H^i-iy. K32(-A-l), Xs(-l)-

SUPPLEMENTARY PROBLEMS
6.
7.

Show

that

H.jK^j

H^{k)K^a/k)

H^j{f{\))

Kj^{-f(,X))

/.

Prove:

An n-square A-matrix A(\)

is a product of elementary matrices if

and only

if |.4(X)| is

a non-zero

constant.

194

SMITH NORMAL FORM

[CHAP. 24

8.

Prove:

An n-square A-matrix ^(A) may be reduced

to / by

elementary transformations

if

and only if\A(X)\

is a non-zero constant.

9.

Prove:

A-matrix .4(A) over F[AJ has an inverse with elements in F[\]

if

and only

if /1(A) is

a product of

elementary matrices.

10.

Obtain matrices P(\) and Q(k) such that

P(\)- A(\)- Q(X) =

and then obtain

A(Xr^
given

Q(X)-P(X)

A+
-4(A)

A+

A+2
A+2

A+1

-A-1
-A^'-A +
A^ + 2
l

Hint.

See Problem

6,

Chapter

5.

Ans

A-1
-A

A^ +

2A-1

-A^ - 3 A - 2

A+

11.

Reduce each of the following


X
(a)

to its Smith

normal form:
1
'-\>

A
>?

A%A
_2A''

+ 2A

A-1 A^-1
2A^ - 3A + 2

A
A^

A
A= +

\^-2A
A
1

>+
(b)

2A

-A^A
1
'Xy

A-1
_ A^

A^
A^

A^- 2A+
2A=
- A=

A+

l_

A=

I
1

A+1

2A-2
+
1

A1

A^ A"
'-Xj

00
1
a""

A% A
(c)
a""

2A''

- 2A +

a''2A''
a''

2A
A + 4
A""

- A -

3\^

-7A

+ 4

-2A=
A=

- A

A^A""
A^ + 2
A^ +
(d)

2A''-2A''
1

- 2A^
1

A^1 1
'-\j

A +
1

A" + A
\^ + l

A^ +

A""

+ A -

A^ +

A
1

A +

A"

A"1

A^

+A

A"
A=
A= +

A%
+ 3
A^ +

A -

A^

A-1
1
a""

A%A^
+
1

A-^

A= +
2

A^

i_

'a=^

3A

4A 2

A= + 3

A-2
(e)

A-1
4A+3
A^ +

A+

A-2
3A + 2
1

3\ +
A^ +

2\ + 2

2A

6A

+ 4

A%
'-VJ

6A 1

A ^+ 2A + 3J
D

"a^
if)

\^

-2\

A+

A=( A-

ifOV+l I

12.

Obtain the elementary divisors over the rational


matrices of Problem 11.

field, the real field,

and the complex field

for

each of the

CHAP.

24]

SMITH NORMAL FORM

195

13.

The following polynomials


real field.
(a)
(b)

are non-trivial invariant factors of a matrix.

Find

its

elementary divisors in the

X^-X. X=-A^ A^-2A^


A+1, X^X,
1,

+ X^

(X^-lf. (X^-lf
X''

(c) (d)

X" + X,
A^ + A,
(a)
(b) (c) (d)
X^,

A'
A''

+ 2X'' A,

2A^+

A''

X"^

X.

+ 2A +
A,

A''+A''+2A'' + 2A +

A''

+A

Ans.

A^,

(A

-if, A-1,

A-1
(A

A+1, A+1,
A, A, A, A,

(A+lf, (A+lf, A-1,


+
l,

-if,

(A

-if

A^
A.

A^'

(A^'+lf,
l,

A-1
(A^

A,

A^ +

(A^+lf,

+ lf.

A+1
whose rank
is six.

14.

The following polynomials


factors?
(a)
(6)
A, A,

are the elementary divisors of a matrix

What are

its invariant

A+
A,

1,

A+2, A +

3,

A+

(c) (d)

(A
A^,

-if,

(A
A,

-if,

(A

-if, A-1, (A+lf


(A + 2f,

A^

A=,

(A1,
1,

1)2,

A-1
A(A + l)(A + 2)(A+3)(A + 4)

A^

(A + 2f,

(A+2f

.4ns.

(a)
(b)

1,
1,

1,
1,

1,

A,

A,

A^cA-l),

A^A-lf
+ 2f,
A''(A

(c)

1,
1,

1, 1,

A-1, (A1,

if, (A- if, (A- if (A+lf


A''(A

(d)

A(A + 2f,

2f

15.

Solve the system of ordinary linear differential equations

!Dxx
(0 + 2)^1

+ (D + i)x2

- (D-l)xs
(D + l)x2
+

= = =

(D+ 2)X2
t

e*

where

xj, x^,

xs are

unknown

real functions of a real variable

and

at

4.

Hint.

In matrix notation, the

system is

AX
Now
the polynomials in

D D+2

D+1 D+1

Xl

'o'

-D + 1 D+2

X2
3_

e\

computing form similar to


:i2(-l),
.ffi(-l),

combine as do the polynomials in A of a A-matrix; hence, beginning with a that of Problem 6, Chapter 5, and using in order the elementary transformations:
of
/I

K2i(D +

l),

H2i(-D-2). ff3i(D+l), K^siD). H^si-i). K^ih. K^gi^D


k(D +
l)

+1).

Hs^i-kD),

Hs(2). Ks(l/5)

obtain

-1

^(5D2+12D + 7)
^-^(SD^ + TD)

PAQ

5D + 6

-4

.4

-1

-jD
\D

A'l

-5D2-8D-2 -D 4D + 2
the Smith normal form of A.

i(5D2+7D + 2)

d=+|d + | 5 5

Use the
get
yi = 0,

linear transformation

QY

to carry

AX

into

AQY
ys
=

and from

PAQY

N-J

PH

y2=-4e*
X
=

(D2+|D

+ i)y3

6e*-l

and

K^e"^^^ + K^e-^ + \e^ ~ ^ 3


4

Finally, use
i
=

QY

to obtain the required solution

3Cie-**/5 + it _ |,

^2

12Cie-'^*/5 + Cje"* - i,

Xs

-2Cie-**/s+ ie*+ i

chapter 25
The Minimum Polynomial of a Matrix
THE CHARACTERISTIC MATRIX \I-A
I.

of an -square matrix A over F is a non-singular A-matrix having invariant factors and elementary divisors. Using (24.4) it is easy to show
If

is a
1,

diagonal matrix, the elementary divisors of \I- D are its diagonal elements.

In

Problem
II.

we prove
A and B over F are similar over F if and only if their charsame invariant factors or the same rank and the same elemen-

Two

re-square matrices

acteristic matrices have the


tary divisors in F[X.].

Prom Theorems
HI.

and

II,

we have
A over F
is similar to a diagonal matrix if and only if

An

re-square matrix

\I- A

has linear elementary divisors in F[\].

SIMILARITY INVARIANTS. The

invariant factors of \I -

are called similarity Invariants of A.


is the

Let P(X) and Q(\) be non-singular matrices such that P(X) (\1-A)-Q(\) mal form
diag(fi(X), 4(X),
...,

Smith nor-

j^a))

Now

\P(\).(XI-A).Q(\)\
Since <^(X) and
IV.
/^(A) are

\P(X)\.\Qa)\4,a)
|

=
=
1

A(A)-/2(A)-.../na).

monic,

P(A)

1 |

(?(A)

and we have
is the

iant factors of

The characteristic polynomial of an re-square matrix A XI- A or of the similarity invariants of A.

product of the invar-

THE MINIMUM POLYNOMIAL. By

the Cay ley-Hamilton Theorem (Chapter 23), every re-square matrix A satisfies its characteristic equation <^(A) = of degree re. That monic polynomial m(X) of minimum degree such that m(A) = is caUed the minimum polynomial of A and m(X) = is called the minimum equation of A. (m(X) is also called the minimum function oi A.)
5.^

The most elementary procedure for finding the minimum polynomial of i following routine:
(I)
(II)

involves the

If

If
If

4 = A ^

oq/,

then m{X) =

X-ao:

al for all a but

i^ = a^A +

aj

then

m(A) =

A^

- a^X - uq
,

(III)

A^ ^ aA + hi

for all a

and b but
're(A)

A''

=
a^)?

a^A"^

+ a^A + a^I

then

A^

- a^X - Oq

and so on.

122
Example
1.

Find the minimum polynomial of ^

2 2

2
1

196

CHAP.

25]

THE MINIMUM POLYNOMIAL OF A MATRIX

197

Clearly A

oqI

is

impossible.
8 9

Set

'9
8
8

8
8 9 =

'122"
i
2
2
1

'100"
+
oo
1 1

2
1

Using the

first

two elements of the

first

row of each matrix, we have

9 =
|

Oj^

+ ao_

then

8 =

2ai

a^= i and oq
A^ = iA +
51

= 5.

After (and not before) checking for every element of A^, we conclude that and the required minimum polynomial is X^ - 4X - 5.

In

Problem
V.
if
If

2,

we prove
if

is

and only
3,

any re-square matrix over F and /(A) is any polynomial over F, then f(A) = the minimum polynomial m{\) of A divides /(A).

In

Problem
VI.

we prove
ra-square matrix

fn(X) of

The minimum polynomial m(\) of an A which has the highest degree.


f^(X), f^(k)
(fi(\)

is that similarity invariant

Since the similarity invariants

of

Vn. The characteristic polynomial A and certain monic factors of m(X).

of

fn-iM all divide A is the product

^(A), of the

we have
minimum polynomial

and
Vin. The characteristic matrix of an ra-square matrix A has distinct linear elementary if m(A), the minimum polynomial of A, has only distinct linear factors.

divisors if and only

NON-DEROGATORY MATRICES. An
IX.

re-square matrix

A whose
if

characteristic polynomial and

minimum

polynomial are identical is called non-derogatoiy;

otherwise, derogatory.

We have
just one non-trivial

An

re-square matrix

is

non-derogatory

and only

if

A has

similarity invariant.
It

is also

easy

to

show
/rei(A)

X.

If

Si and Sg have minimum polynomials


ro(A) of the direct

and m^iX) respectively, the minimum

polynomial and m2(A).

sum D = diag (61,55)

is the least

common

multiple of ^^(A)

This result may be extended to the direct sum of


XI.
let

matrices.
in

Let gi(X),g2(X),

..gnCA)

be distinct, monic, irreducible polynomials


\XI - Aj\ = \g-(X)\ "i
fii
(;

F[A] and

Aj be a non-derogatory matrix such that

= 1,2,
'^

...,m).

Then

B = diag(^i, ^2
acteristic and

^n)

has

^(A)

fgi(A)!

ig2(A)P-ig^(A)!

,02

as both char-

minimum polynomial.

COMP ANION MATRIX.


(25.1)

Let A be non-derogatory with non-trivial similarity invariant

g(X)

^(A)

a" +

a-iA"'^

a^X +

Oq

We

define as the companion matrix of g(X),


C(g)

(25-2)

= [-a],

if

g(X)

X+a

and for

re

>

198

THE MINIMUM POLYNOMIAL OF A MATRIX

[CHAP. 25

1 1

(25.3)

C(g)
1 1

Oq

-%

-e

-'^n-3

-an-2

-Gj

In

Problem
XII.

4,

we prove
its character-

istic

The companion matrix C(g) of a polynomial g(\) has g(\) as both and minimum polynomial.

(Some authors prefer to define C(g) as the transpose Both forms will be used here.)

of the matrix given in (25.3).

See Problem
It

5.

is

easy to show
XIII.
If /4 is

non-derogatory with non-trivial similarity invariant f^(X) = {k-af,


"a
1
...

then

O'

...

(25.4)

/ =

[o],

if

ra

1,

and

=
... ...

if

ra

>

has

f^{\)

as its characteristic and minimum polynomial.

SOLVED PROBLEMS
1.

Prove: Two ra-square matrices A and S over F are similar over F if and only if their characteristic matrices have the same invariant factors or the same elementary divisors in F[X\-

equivalent.

Suppose A and B are similar. From (i) of Problem 1, Chapter 20. it follows that Xl - A and XI -B are Then by Theorems VHI and IX of Chapter 24, they have the same invariant factors and the same
Conversely,
let Xl - A and Xl - B have the same invariant factors or elementary divisors. Chapter 24 there exist non-singular A-matrices P(X) and Q{X) such that

elementary divisors.

Then by

Theorem

VIII,

P{X)-{XI-A)-Q{X)
or

Xl - B

(i)

P{X)-{XI-A)

(Xl-B)-Q

(X)

Let
(ii)

P{X)
Q(X)

= = =

(X/-S).Si(A) + Ri S2(X)-(A/-S) +
S3(A)-(A/-.4) +
/?2

(iii)

(iv)

Q'\X)

/?3

where R^, Rq, and R3 are free of

A.

Substituting in

i ),

we have
(A/-B)-S3(A)-(A/-4)
=

(Xl-B)-S^(X)-(Xl-A)
or
(V)

+ Rj^(Xl-A)

(XI-B)Rs

{Xl-B){S^{X)-Sa{X)\{Xl-A)

(A/-B)Rg

R^{XI-A)

CHAP.

25]

THE MINIMUM POLYNOMIAL OF A MATRIX

199

Then
(vi)

S^(X)

Sg(A) =

and

(Xl-B)Rg
left

R^(\I-A)
member
is of degree at

since otherwise the


one.

member

of (v) Is of degree at least two while the right

most

Using
I

(ill), (Iv),

and

(vi)

= =
=

Q(^)-Q~\\)

Q(\)\Ss(\)-(Xl-A) + Rs\
Q(X.)-S3(X).(X!-A) +
Q(X)
Ss(X)

\S2(X)-(XI-B) + R^lRg

= =
or
(vii)

(XI

- A) + S2(X)-(Xl-B)Rs + R^Rs

Q(X)-Ss(X)-(Xl-A) + S2(X)-R^-(XI-A) + R^Rg

- R^Rs
and
/ =

\Q{X) Sg(X) ^ S^iX)

RxUXl - A)
(vii) is of

Now

Q{X)-Sq(X) + S^(X)R-^ =
right

R2R3

since otherwise the left member of

degree zero

in

X while the

member

is of

degree at least one.


=

Thus, Rg = R^
=

and, from (vi)

XI - B
Since A.B.R^, and R^ are free of A. to be proved.

/?i(A/-^)/?2

ARi2 - R^AR^
=

/Ji =

R^: then XI - B

XI - R'^AR^ and A and B are similar, as was

2.

Prove:
only
if

If

is

any ra-square matrix over

F and

f(X) is any polynomial in F[X]. then f(A) =


/(A).

if

and

the minimum polynomial m(X) of


the division algorithm. Chapter 22,
f(X)
=

A divides

By

q(X)-n(X) +

r(A)

and then
f(A)
=

q(A)-n(A) + r(A)

r(A)

Suppose f(A) = 0; then r(A) = 0. Now if r(A) ji 0, its degree is less than that of m(X), contrary to the hypothesis that m(A) is the minimum polynomial of ^. Thus, r(A) = and m(A) divides /(A).
Conversely, suppose /(A) = ?(A)-m(A).

Then f(A)

= q(A)-m(,A) = 0.

3.

Prove: The minimum polynomial m(X) of an ra-square matrix A which has the highest degree.
Let
gri-i(X)

is that similarity invariant fn(X) of

denote the greatest common divisor of the

(n

- l)-square minors

of XI

-A. Then

\XI-A\
and

cf,(X)

g-i(A)-/(A)

adj(A/-/l)

g_i(A)-S(A)
is 1.

where the greatest common divisor of the elements of B(X)

Now

(A/-^).adj(A/-^)

(f)(X)-I

sothat
=

(X/-A)-gn_^(X).B(X)
or

gn-i(A)-/(A)-/

(i)

(A/-^).S(A)

fn(X)-I
0.

Then XI

-A

is a divisor of f^(X)-I

and by Theorem V, Chapter 23, f^(A) =

200

THE MINIMUM POLYNOMIAL OF A MATRIX

[CHAP. 25

By Theorem
(ii)

V, m(\) divides

fn(X-)-

Suppose
fn(\)
=

g(X.)-m(\)

Since m(A) =

0,

XI

-A

is a divisor of

m(X)

/,

say

m(\)-l

(\I-A).C(\)

Then, using (i) and

(ii).

(\I-A)-B(\)
and

fn(X.)-l

9(A)-m(A)-/

9(A)

(A/ - ^). C(A)

5(A)

?(A)

C(A)
and, by

Now

9(A) divides every element of iS(A); hence ?(A)


/n(A)

=1

(ii),

m(A)

as was to be proved.

4.

Prove: The companion matrix C(g) of a polynomial g(\) has g(A) as both minimum polynomial.

its

characteristic and

The characteristic matrix

of (25.3) is

-1
A -1

^1

"2

an-2
2

A + a^_j
third column,
,n-l
.

the first column add A times the second column, column to obtain

To

A times the

times the last

-1

A
G(A)

A
g(A)

-1

Oi

a^

a_2

A+a^-l

Since

|G(A)| = g(A),

in G(X) is 1, the greatest

the characteristic polynomial of C(g) is g(A). Since the minor of the element g(A) common divisor of all (n-l)-square minors of G(\) is 1. Thus, C(g) is non -derogis g(A).

atory and its

minimum polynomial

5.

The companion matrix

of

g(\)

A^ + 2A

A=

+ 6A -

is
5

10

10
10
1

or, if

preferred.

10 0-6 10 1

5-61-20

10-2
1

CHAP.

25]

THE MINIMUM POLYNOMIAL OF A MATRIX

201

SUPPLEMENTARY PROBLEMS
6.

Write the companion matrix of each of the following polynomials:


(a)
X''

+ \^

- 2\ -

(d)
(e) (/)

X""

2X!'

- X^

+ 2X

(b)
(c)

(X^-i)(X+2)

]?()? + I)

(X-lf
1

(A+2)(A^-2A^ + 4A-8)
1

'o
1

o"
1

Ans.

(a)
1

(b)

(c)
1
--3

2
1

-1

8
o"

4
1

-2

1 1 1

o'

(d)
1

(e)
1

(/)
1

-2

0-10
A=
[a^]
for

16

7.

Prove: Every 2-square matrix

which (011-022)^ +
g(X)
).

4012021/0

is non-derogatory.

8.

Reduce G(X) of Problem

4 to diag (1

9.

For each of the following matrices ^. (i) find the characteristic and minimum polynomial and
non-trivial invariant factors and the elementary divisors in the rational field.
1
1 1

(ii) list

the

2
(c)
1
1

2
1

1
1

2
2 2

(a)

2
3

(b)

id)

2 2

2
1

(e)

-2 -1 -3
1"

_1

'211
4
if)

2-3
(g)

1-3
ih)

-6 -2 -3 -2 -3 -1 -1 -2
(a)

-1 -6 -3 -6 -3 -3 -4 -3
2

-5 -2

4-6 3 s" 3-2 2 4-3 4-1 -6 4-2 4 0-4 -1 0-2 1 2


1

Ans.

<^(A)

m(X)
m(,\.)

=
=

(A-l)(A-2)(A-3);
X^;
if. = e.d
i.f.

i.f.

(A-l)(A-2)(A-3);

e.d. (A-1), (A-2).

(A-3)

(b) <^( A)

=
= =

= A=

(c)

(^(A)

(A-if(A-2);
(A-1) (A-2)
;

A-1, (A-i)(A-2)

m(X)
(/)(A)

e.d.

A-1, A-1, A-2


A+1, (A+i)(A-5)

id)

(A+ihA-5)
(A+l)(A-5)
A=

i.f.

m(A)
,^(A)
ie)

= =

e.d.

A+i, A+l, A-5

- 4A=

i.f.

A,

A''-4A

m(A) = A^ - 4A
c^(A)
= =

e.d. A, A,

AA+l,

A(A-H)^(A-1)

i.f.

A^-A

if)

m(A)
(/)(A)

A(A^-l)
A2(A+1)2
;

e.d. A,
i.f.

A+l, A+l, A-1

A,

A(A+l)2

is)

m(A) =
oS(A)

A(A+1)2

e.d. A. A,

(A+D^

(A-2) (A""-

A-

2)'';

i.f.

(h)

m(X)

A^-A-2
VIII.

e.d.

A-2, A'^-A-2, A^-A-2 A-2. A-2, A-2, A+l, A+l

10.

Prove Theorems VII and Prove Theorem X.


Hint.

11.

m(D)

= diag(m(Bi), m(B^)

requires

m(Si) = miB^) =

thus, mi(A) and

m^X)

divide m(X).

202

THE MINIMUM POLYNOMIAL OF A MATRIX

[CHAP. 25

12.

Prove Theorem XI.

13. If

A Show

is

n-square and if k is the least positive integer such that A = 0, .4 is culled nilpotent of index that A is nilpotent of index k if and only if its characteristic roots are all zero.

A:.

14.

Prove: (a) The characteristic roots of an n-square idempotent matrix A are either
(b)

or

1.

The rank

of

is the

number of characteristic roots which are

1.

15.

Prove: Let

A.B.C.D

be ra-square matrices over


that

F
it

with C and

non-singular.

There exist non-singular


and
S(A.)

matrices

P and Q such
in

PCQ

= A.

PDQ

and only

if

R(X) = \C - A

XD-B

have the

same invariant factors


Hint.

or the

same elementary divisors.


1,

Follow the proof

Problem

noting that similarity is replaced by equivalence.


then A~^ is expressible

16.

Prove:

If

the minimum polynomial m(X) of a non-singular matrix A is of degree

s,

as a scalar polynomial of degree

slmA.
A
of

17.

Use the minimum polynomial

to find the inverse of the matrix

Problem

9(h).

18.

Prove: Every linear factor X-A.J of 0(A) is a factor of m(A).


Hint.
r

^ 0.

The theorem follows from Theorem VII or assume the contrary and Then {A -\^I)q(A) + rl = and A -X^l has an inverse.

w:rite

m(A) = {\-X^)q{X) +

r,

19.

Use

-4

[o

^ to

show

that the

minimum polynomial

is not the product of the distinct factors of 0(A).

;]

20.

Prove:

If

g(\) is any scalar polynomial in A, then g(^) is singular if and only if the greatest

common

divi-

sor of g(A) and m(A), the


Hint,
(i)

minimum polynomial
i-

oi A, is d(k) ^ 1.

(ii)

Suppose d{\) Suppose d(A)

1 1

and use Theorem V, Chapter 22. and use Theorem IV, Chapter 22.
expressible as a polynomial in A of

21. Infer from

Problem 20 that when

g(.4) is

non-singular, then [g(/l)]

is

degree less than that of m(\).

22.

Prove:

If

the

set of all scalar polynomials in

minimum polynomial m(A) of A over F is irreducible in F\X\ and is of degree A with coefficients in F of degree < s constitutes a field.

s in A,

then the

23.

Let A and S be square matrices and denote by m(A) and (A) respectively the minimum polynomials oi and BA. Prove:
(a) m(A) = n(A)
(&) m(A)

AB

when not both


at

.A

and B are singular.

and n(A) differ


=

most by a factor A when both A and B are singular.


=

Hint.

B.m(^S)./l

(B,4).m(B.4)

and

A-n{BA)-B
m >
=

(AB)-n(AB)

0.

24.

Let A be of dimension

mxn

and S be of dimension nxm,

n,

and denote by 0(A) and 0(A) respectively


(/f(A).

the characteristic polynomials of

AB

and BA.

Show 0(A)

25.

Let

X.I

be an invariant vector associated with a simple characteristic root of A.


Xj^ is

Prove:

If

A and B com-

mute, then

an invariant vector of B.
of

26. If the matrices

A and B commute, state a theorem concerning the invariant vectors

B when A has only

simple characteristic roots.

chapter 26
Canonical Forms Under Similarity

THE PROBLEM.

In

Chapter 25

matrices A and
In this chapter,

it was shown that the characteristic matrices of two similar n-square R'^AR over F have the same invariant factors and the same elementary divisors. we establish representatives of the set of all matrices R'^AR which are (i) sim-

ple in structure and

(ii)

put into view either the invariant factors or the elementary divisors.

These matrices,
ical matrix

four in number, are called canonical forms of A.

They correspond
r

to the

canon-

'[^

o]

introduced earlier for '

all

mxn

matrices of rank

under equivalence.

THE RATIONAL CANONICAL FORM.

Let A be an ra-square matrix over F and suppose first that its characteristic matrix has just one non-trivial invariant factor /(\). The companion matrix C(f ) of /(A) was shown in Chapter 25 to be similar to A. We define it to be the rational canonical form S of all matrices similar to A

Suppose next that the Smith normal form of \I - A


<26-l)

is
...,

diag

(1, 1

1,

f.(X), f.^^iX),

f^(X))
1

with the non-trivial invariant factor

f^(\.)

of degree s^,

(i

= j.j +

n).

We define as the

ra-

tional canonical form of all matrices similar to (26-2)

A
C(/))
is similar to

S
that

di^s {C(fj), C(fJ ^^)

To show
D^ = diag

A and S have the same similarity invariants we note that C(f-)


l,/i(A))

(1, 1

and, thus, S is similar to diag(D^-

.^^

D).
to

By a sequence

of

interchanges of two rows and the same two columns, we have S similar
diag (1,1

l.fja), fj^^(\)

f^iX))

We have proved
I.

Every square matrix A is similar to the direct sum (26.2) of the companion matrices

of the non-trivial invariant factors of XI - A.

Example

1.

Let the non-trivial similarity invariants of A over the rational field be


/8(A)
=

X+1.

fg(X)

A%L

/io(A)

A^ +

2A +

Then
1

1 1 1

C(fa)

= [-1],

C(f9)

C(fio)

-10

10
1

-10
203

-2

204

CANONICAL FORMS UNDER SIMILARITY

[CHAP. 26

and
-1

1
1

-1
1

diag(C(/g),C(/9),C(/io))
1 1

1 1

-1
Is the required form of

-2

Theorem

I.

Note.
immaterial.

The

order in which the companion matrices are arranged along the diagonal is

Also

-1 -1
1
1

-1
1 1 1

-2
1
1

using the transpose of each of the companion matrices above is an alternate form.

A SECOND CANONICAL FORM.

Let the characteristic matrix of A have as non-trivial invariant tors the polynomials f^(X) of (26.1). Suppose that the elementary divisors are powers of t tinct irreducible polynomials in F[X]: pi(A,), paCX), ..-, PjCA). Let
(26.3)
fi(\)

facdis-

ipi(X)l^"!p2(A)!^2^..{p,(X)!'^is(pi(A)i'"''iP2(A)i'"''...iPi(

(i

= i,i+'^

n)

where not every factor need appear since some of the ^'s may be zero.
C{p?^^) of any factor present has
C{{^) is similar to

The companion matrix


hence,

\])^(X)\ ^^ as the only non-trivial similarity invariant;

diag (C(p^?"

).

Cipl-'i

Cip^^ti))

We have
Every square matrix A over F is similar to the direct sum of the companion matriII. ces of the elementary divisors over F of XI A.
Example
2.

For the matrix A of Example


(A.+ l)^,

1,
.

the elementary divisors over the rational field are

A+1, X+1,

A^-A,+

l,

(X

-A+1)

Their respective companion matrices are


1 1

[-1],

[-1],

[-:

.;]

[.:

\
1

-3

and the canonical form of Theorem

II is

CHAP.

26]

CANONICAL FORMS UNDER SIMILARITY

205

-1

-1
1

-1 -2
1

-1

1 1 1
1

-1

-3

THE JACOBSON CANONICAL FORM.

Let A be the matrix of the section above with the elementary


If

divisors of its characteristic matrix expressed as powers of irreducible polynomials in F\_\].

Consider an elementary divisor {p(X)!^.


build

q =

1,

use C(p), the companion matrix;

if

q>l,

C(p)

M
C(p)

..

(26.4)

Cq(P)

=
C(p)

M
C(p)

where

is a matrix of the

and zeroes elsewhere.

same order as C(p) having the element 1 in the lower left hand corner The matrix C^^Cp) of (26.4), with the understanding that Ci(p) = C(p) is
Note that
in (26.4), there is a

called the hypercompanion matrix of \p(X)]'^.


I's just

continuous line of

above the diagonal.


C'(p) is used, the hypercompanion matrix of jp(A)!''is

When the alternate companion matrix


C'ip)

N
Ca(P)

C'ip)

C'ip)

C'ip)

C'ip)

where A' is a matrix of the same order as C'ip) having the element 1 in the upper right hand corner and zeroes elsewhere. In this form there is a continuous line of I's just below the diagonal.
Examples. Let
IpiX.)]'^ = (\^

2\-

1)'^.

Then
1 1

C(p) =

M,

M=y

and

-2

1 1

-2

1 1 1

CqiP)

-2

1 1
1

-2

206

CANONICAL FORMS UNDER SIMILARITY

[CHAP. 26

In

Problem

1, it

is

shown
to

that Cq(p) has {pCA)}*^ as its only non-trivial similarity invariant.

Thus, Cq(p) is similar

C(p^) and

may be substituted

for it in the

canonical form of Theorem

II.

We have
III.

Every square matrix A over F

is similar to the direct

sum of the hypercompanion

matrices of the elementary divisors over

of

\I - A.
1,

Example

4.

For the matrix A of Example

2,

the hypercompanion matrices of the elementary divisors A +

A+1 and A -A+1


r-1

are their companion matrices, the hypercompanion matrix of


1

(A+l)^is

-11
and that of (A^-A + 1)^ is

10
1

Thus, the canonical form of Theorem

0-11
nils
-1

-1
-1
1

-1
1

-1

1 1

-1

1
1

-1

The use

of the term "rational" in connection with the canonical form of


It

Theorem

is

some-

what misleading.

was used

originally to indicate that in obtaining the canonical form only

rational operations in the field of the elements of

also of the canonical forms (introduced later) of


fusion, the canonical form of

Theorem

III

is

A are necessary. But this is, of course, true Theorems n and III. To further add to the consometimes called the rational canonical form.

THE CLASSICAL CANONICAL FORM.

Let the elementary divisors of the characteristic matrix of A be powers of linear polynomials. The canonical form of Theorem III is then the direct sum of hypercompanion matrices of the form
i
1 1

..
.

(26.5)

C^(P)
..
. .

1
<^.,

corresponding to the elementary divisor

!p(A)!

(A-a^)^
III is

For an example, see Problem

2.

This special case of the canonical form of Theorem


canonical form.
IV.

[Note that C^(p) of (26.5) is of the type / of (25.4).

known as the Jordan We have ]

or classical

into linear polynomials.

be the field in which the characteristic polynomial of a matrix A factors Then A is similar over ^to the direct sum of hypercompanion matrices of the form (26.5), each matrix corresponding to an elementary divisor (X-a^)'^.
5.

Let

Example

Let the elementary divisors over the complex field of \I

-A

be:

A-i, X + i, (X-i)

(X+i)

CHAP.

26]

CANONICAL FORMS UNDER SIMILARITY

207

The classical canonical form


1

of

is

-1
i

1
i

-i

-i

From Theorem IV follows


V.

An

re-square matrix

is similar to a diagonal matrix if

and only

if

the elementary

divisors oi XI -

are linear polynomials, that is, if and only if the is the product of distinct linear polynomials.

minimum polynomial of
See Problems 24.

A REDUCTION TO RATIONAL CANONICAL FORM.


it

In concluding this discussion of canonical forms, that a reduction of any re-square matrix to its rational canonical form can be made, at least theoretically, without having prior knowledge of the invariant factors of \I - A. A somewhat different treatment of this can be found in Dickson, L. E., Modern Algebraic

will be

shown

Benj.H. Sanborn, 1926. Some improvement on purely computational aspects Browne, E. T., American Mathematical Monthly, vol. 48 (1940).
ries,

Theoin

is

made

We
in to

shall need the following definitions:

If A is an re-square matrix and X is an re-vector over F and if g(X) is the monic polynomial F[\] of minimum degree such that g(A)-X = 0. then with respect to A the vector X is said belong to g(X).
If,

with respect to A

the vector

belongs

to g(X) of

degree

p,

the linearly independent

vectors

X,AX.A''X
2

/4^

are called a chain having

as its leader.

-6 -3 -2
= X.

Example

6.

Let A

1 1

The vectors
Then
2

= [l, 0, oj' and

AX

= [2,

1, l]'

are linearly independent

while

(A

-I)X=0
Y;
thus,

and

A"

[1,0, -ij',
If

AY

=[-1,0,1]'

(A+I)Y=0

belongs to the polynomial A,^-l For y = and F belongs to the polynomial A+ 1.

m(\) is the minimum polynomial of an re-square matrix A, then m(A)- X = for every reThus, there can be no chain of length greater than the degree of m(A). For the matrix of Example 6, the minimum polynomial is A^ - 1.
vector X.

non-singular matrix
(26-6)

Let S be the rational canonical form of the re-square matrix A over F. R over F such that

Then, there exists a

R'^AR

diag(C.,C.^^

C)

where, for convenience, Q/^) in (26.2) has been replaced by C^. companion matrix of the invariant factor
/iCA)

We
A
+

shall

assume

that

C,-

the

A^i

c^ ^_X'i-

c,.

has the form


-^tl
1
1

-^i2
.

Ci
.

-^i3

208

CANONICAL FORMS UNDER SIMILARITY

[CHAP. 26

Prom
(26.7)

(26.6),

we have

AR

RS

Rdiag(Cj,Cj^^

C)
so that R^

Let R be separated into column blocks have the same number of columns.

R-,R-^
(26.7),

R^

and C^, (i=/,/ +

n)

From
=

AR
and

A[R..R.^^

RJ

IRj.Rj,,

/?Jdiag(C,,..C^.^^

C)

iRjCj.Rj^^C.^^
(i=j,j +
l

R,,CJ

AR^ = R^Ci.
Denote the
si

n)

column vectors of Ri by Ri^, R^s.


[Rii, Ri2,

Ris- and form the product


t

RiCi
Since

, Risi\Ci
=

st
^__

[Ri2,Ri3, ,Risj,> -

Rik^ik]
=

ARi we have
(26.8)

A[Ri^,Ri^, -.Risi]

[ARi^.ARi2,. ..ARisil

RiCi

Ri2 =

ARn,

Ris - ARi2 = A Rii,

Risi = A^'-'Ri.

and
(26.9)

-|>i.%
-i =
K

^Ris,

Substituting into (26.9) from (26.8),

we obtain
=

c^kA^-'Ri^
l

A'iRu
+ cirDRir =

or

(26.10)

(A^^ + cis.A^'i'^
definition of C^ above, (26.10)

...

a^A
=

Prom the
(26.11)

may be

written as

fi(A)-Ri^

Let Rit be denoted by X, so that (26.11) becomes fj(A)-X: =0; then, since X-,AXj, A ^ X^ are linearly independent, the vector X^ belongs to the invariant factor f^iX). Thus, the column vectors of fi^ consist of the vectors of the chain having X^, belonging to /^(X),
Xj^

as leader.

To summarize:
chains

the n linearly independent columns of R, satisfying (26.2), consist of re-/+l

Xi,AXi
whose leaders belong
satisfy the condition

A^^"'Xi
i
^ s. n

(i=j,i +

n)

to the respective invariant factors

/ (A),

fj.AX.)

/^(X)

and whose lengths

<

s,-

s- ^^ +1
J

...

We have
VI.

Por a given re-SQuare matrix A over F:


(i)

let let

X^ be

the leader of a chain

of

maximum length
of

for all

ra-

vectors over F\

(it)

X^_^ be the leader

of a chain

is linearly independent of the

maximum length (any member of which preceding members and those of for all n^_^
)

vectors over
(iii)

F which

are linearly independent of the vectors of ;

let

X^^^he

is linearly

the leader of a chain -n-2 ^ maximium length (any member of which independent of the preceding members and those of and fi'^.i)

for all n- vectors over

F which

are linearly independent of the vectors of

and_i;
and so on.

Then,

for

CHAP.

26]

CANONICAL FORMS UNDER SIMILARITY

209

AR

is the rational canonical form of A.


1
1 1

Example

7.

Let

1 1

2
3

Take ;f=[l,0,o]'; then ^,

4X

= [l,

1, l]',

^^A: =

[3, 5. 6].' are line-

arly

Independent while
/gCX) =

belongs to

^;f = [14,25,30]'= 54^X- a:. m(X) = A - 5A^ + 1 = <jS(A). Taking


1

Thus,

{A-bA^+I)X

= Q

and

A'

R
we
find
1

[a;.

4A',4=A']

15
1

-3

3
5

14 25 30

6-5
-1
1

AR

[AX.A'^X^a'^X]

1
1

0-1
and

R ^AR

=
1

Here A is non-derogatory with minimum polynomial m(A) irreducible over the rational Every 3-vector over this field belongs to m(X). (see Problem 11), and leads a chain
three.

field.

of length
S.

The matrix R having

the vectors of any chain as column vectors is such that

R'^AR =

Example

8.

Let

Tak

A"

= [l,-l,o]';

then ^ A' =

AT

and

AT

belongs to

A-

1.

NowA-1

cannot be the minimum polynomial m(A) of A. 11). and could be a similarity invariant of A.
Next, take

It is,

however, a divisor of m(A), (see Problem

F=[l,0,0]',

The vectors

Y, -4? = [2,

1, 2]',

^^^ =[ 11,

8, s]'

are linearly
to

independent while

A^Y

= [54,43,46]'

= 5A'^Y +

SAY -77.

Thus, Y belongs

m(A) =

A - 5A - 3A +

7 = <^(A).

The polynomial

A-

is not a similarity invariant;

in fact,

unless

the first choice of vector belongs to a polynomial which could reasonably be the function, it should be considered a false start. The reader may verify that

minimum

R'^AR
11

when

[Y,AY,A'^Y]

See Problems

5-6.

SOLVED PROBLEMS
Prove:

The matrix C^ip)


common

of (26.4) has !p(A)!'? as its only non-trivial similarity invariant.

Let C^(p) be of order


so that the greatest
tors of

s.

The minor of the element in the last row and first column of A/ - C (p) is 1 divisor of all (s-l)-square minors of A/ - C (p) is 1. Then the invanant facl.f^(\).
=

A/ - C^(p) are 1,1


0(A)

But /^(A) =
=

ip(A)!'?

since
=

|A/-C^(p)|

\XI-C(pf

jp(A)!9

210

CANONICAL FORMS UNDER SIMILARITY

[CHAP. 26

2.

The canonical form


divisor being

(a) is that of

X^+4X +6A^ +4X +


1

1.

Theorems I and II, the non-trivial invariant factor and elementary The canonical form of Theorem III is (b).
1 1 1

-1
(b)
1

(a)

-1

-1

-4

-6

-4

-1

The canonical form

(a) is that of

Theorem
2,

I,

the invariant factors being \ +


2,

and the elementary divisors being X + Theorems II and III is (b).


2
1

X+

X+

2,

X-

2,

X-

2,

X+

3.

2,\ -4, A'+3A -4A-12 The canonical form of both

-2

4
(a) 1 1
(b)

-2
2

12

-3

-3

4.

The canonical form


X+
2,

(a) is that of
,

X+

2,

(X'^

+2X-lf

(X^

Theorem III. Over the rational field the elementary divisors +2X- if and the invariant factors are
1)^
(X +

are

(X + 2)(X^ + 2X -

2)(X^+2X-1)^
II

The canonical form

of

Theorem

is (b)

and that of Theorem

is (c).

-2

-2
1 1
-

-2

1
1
(a)
--2

-2

1
1

-2

1
1

-2

1 1 1

2
(b)

-10

--6
1 1
1
ID

1 1
1

11

12

17

-14

--21

-8

CHAP.

26]

CANONICAL FORMS UNDER SIMILARITY

211

-200000000000
-2
1 1

-1
(c)

-2

-4
1 1 1 1

1
1

-6

-9

-6

-2

-1

-6
2

-2
1

-1
5.

Let

A =

-2
1

-1 -1 -1

-1
1

Take X
2
3
1 1

-2

0_

Then

AX
-4

[-2

1, 1, 1. 1, 1]',

A^X

= [1,0,-1,0,0

-ir.

A''

=[-3.1.1,1.

1,2]'

are linearly inde-

pendent while m(\) = A* - 2X^ +

A"

= [l, 0, -2, 0, 0, -2]' = 2^=A:

-Z;

belongs to

A*-2A^ + 1

We

tentatively

assume

and write Xg for X.

The vector

AY
Xs

- [-1,0,

1,

Y belongs
for Y.

Y = [o, 0, 0, 1, 0, O]' is linearly independent of the members of the chain led by -1, 1,0]' is linearly independent of Y and the members 2,, = Y of the chain. Now A^Y
Since the two polynomials complete the set of non-trivial invariant factors

Xg and
so that
write

to

A -1.

When
-1
1

we

-2

-3
1

"o 1

[Xs.AXs.Xg.AXe.A^Xe.A^'Xfi
1

-1
u

-1
1

R~'^AR 1 1

-1
1

1
1

-1
the rational canonical form 'of ^.

Note.

The vector Z =
1,

[o, 1, 0, 0, 0. Oj' is linearly

and

AZ

- [3, 0, -2,

-2,

o]' is

linearly independent of

1-1,1,0,0,0,1]'= -AXg + A Xg + Z; then (A'' ~ 1)(Z - AXg) = and W=Z-AXe = [2.0 -1 -1 ll -i]' belongs to A - 1. Using this as X^. we may form another R with which to obtain the rational canonical' form.

independent of the members of the chain led by Xg Z and the members of the chain However A'^Z =

-2
1

-1
3

-1
1

-1
1

6.

Let

A =

-1 -1

-4 -4
-2
=

-2
-1

-1

Take X = [1,0,0,0,0]'.

-2

-2
Then
r

-2

-2
^^^=[1,1,-1,-1,0]' are linearly independent while a'^X^ 3. We tentatively assume this to be the minimum

^.^5

[-2,1,-1,-1,-2]',

L-1,

-2, -2,0J - 2A polynomial m(A) and label


2,

X-3X
X

and

belongs to A - 2A^ +

as X^.

212

CANONICAL FORMS UNDER SIMILARITY

[CHAP. 26

When, in A, the fourth column is subtracted from the first, we have [-1,0,0,1,0]'; hence, if y = Again, when the fourth column of A is subtracted from -1, o]', AY = -Y and Y belongs to X+ 1. the third, we have [o, 0, -1, 1, O]'; hence, if Z = [O, 0, 1, -1, O]', AZ = -Z find Z belongs to A+ 1. Since y, Z, and the members of the chain led by X^ are linearly independent, we label Y as X^ and Z as X^. When
[l, 0, 0,

11-2
R
=

-1

11
[Xs.X^.X^.AXs,.A%]
=
1

0-100
R~ AR
=

0-1-1
-1

0-3

-1

-1

-1

10

0-2
the rational canonical form of A.

12

SUPPLEMENTARY PROBLEMS
7.

For each of the matrices (a)-{h) of Problem


over the rational field,

9,

Chapter 25, write the canonical matrix of Theorems

I,

n, in

Can any
1

of these matrices be

changed by enlarging the number field?

Partial Ans.

(a)

I,

II,

ni,

diag(l,2,

3)

-11
"o

6
1
o"

(b)

I, II,

m.
_0

1
0_

"o
(e)
I,

o'

1
_0

n, in.

-1
1
(/)
I.

-1
n, ni.

-1

1
o'

1
(g)

n.
1

m.
-1
-2.

-1

-1
'2

-1

(h)

I,

10 10

n. in,

diag(2, 2,2, -1, -1)

8.

Under what conditions will (a) the canonical forms of Theorems I and n be identical? (b) the canonical forms of Theorems n and in be identical? (c) the canonical form of Theorem H be diagonal?

CHAP.

26]

CANONICAL FORMS UNDER SIMILARITY

213

"o
9.

o" 1

Identify the canonical form


_0

Check with the answer

to

Problem

8(6).

0_

10.

Let the non-singular matrix A have non-trivial invariant factors


Write the canonical forms of
that of

(a) X + 1 Theorems I, n,

A,

(X + i)^

(b)

X^+

1,

III

over the rational field and

Theorem
(a)

IV.

Ans.

-1
1
1

-1

10
1 1 1 1

_0
-1

-10

-2

0_

-1
1

-1
n.

-2
1

-1

1
1

1
1
.

-1

-3

2
6"

-1

-1 -1
1

-1
1

in,

-1

1 1
--1

1 1
-

-1
o"

1_

-1

-1 -1
1

-1

a
IV,

where a,^ = ^(li\/3).

a
/S /3

214

CANONICAL FORMS UNDER SIMILARITY

[CHAP. 26

11.

Prove: If with respect to an n-square matrix A, the vector polynomial m(A) of A.


Hint.

belongs to g(X) then g(X) divides the minimum


r (X).

Suppose the contrary and consider

m(A)

A (A)

"

g (A) +

12.

In

Example

6,

show

that

X,AX. and Y

are linearly independent and then reduce

to its rational canonical

form.

13.

In
(a)

Problem

6:

Take 7 Take

= [o,

1, 0, 0, o]',

linearly independent of the chain led by ^5, and obtain

X4

y_

(3/1

_2/)

A^g

belonging to A+1.
(i>)

= [O, 0,

1, 0, O]',
1.

linearly independent of

X^ and the chain

led by X5, and obtain

X^ = Z

-X^

belonging to A +
(c)

Compute R~

AR

using the vectors X3 and X^ of (b) and

(a) to build R.

14.

For each of the matrices A of Problem 9(a)-(h), Chapter


ical form of

25, find

R such

that

R' AR

is the rational canon-

15.

Solve the system of linear differential equations


dxi
dt

2Xx
4^1

+ +

X2

+ +

Xg

X4 +

dx2
dt

2x2
2a:2

3At3

dxs
dt

6*1

- 3*3 t.

2xi^

dX4
dt
3ai

X2

Xq

2x4.

where the x^ are unknown functions of the


TT.

real variable

Hint.

i.

i Let

V ^

T U, . .3. -J
r

J define

"^^

dxi

dxQ

dxn

dxA'

111
3

|^^,

-jf. -jf,

^J

and rewrite the system as

dX_
(i)

4 -6 -3

dt

-2 -1

-3
-1

-2 -2
-

AX

Since the non-singular linear transformation

RY

carries

(i)

into

dY
dt

R ^ARY +

R^H

choose R so that R~
with

AR

is the rational canonical form of A.

is leader of the chain

X-i

= Ei, AX^.A'^X-i

while

The elementary 4-vector 1 belonging to A - A Now 4 yields X2 = E^- Xi + 2AXi belonging to A + 1.

12-1
[X.L,AXi,A^Xi,X2]
0'
t

4-2 8 0-6 4 -12 0-3 2-5


t

dY^
dt

1
1

Y +
-1

yi + 73

y2

-74

Then
C^+it'^
Cge* + Cge-* t

2Ci +

C2e*+

3(Cs + C4)e"*+

^-2t+l'
4 + 2
6t

-Ci +

-t_l,2^ J. Cse"Cse^Sr
46"*

and

RY

2C1 + 2C2e*+ 2(3C3 + 4C4)e-* + ^ -4C1 - 2C2e*- 2(5C3+6C4)e-* - 2^+

- 4

-2C1 -

Cae*-

5(C3+C4)e"* -

2+3{-2_

INDEX
Absolute value of a complex number, 110 Addition
of matrices,
2,

Characteristic roots (cont.)


of Hermitian matrices, 164 of inverse A, 155 of real orthogonal matrices, 155 of real skew-symmetric matrices, 170 of real symmetric matrices, 163
of unitary matrices, 155 Characteristic vectors {see Invariant vectors) Classical canonical form, 206

of vectors, 67

Adjoint of a square matrix


definition of, 49

determinant of, 49 inverse from, 55 rank of, 50 Algebraic complement, 24 Anti-commutative matrices, 11 Associative laws for
addition of matrices, 2 fields, 64 multiplication of matrices, 2 Augmented matrix, 75

Closed, 85
Coefficient matrix, 75

Cofactor, 23 Cogredient transformation, 127

Column
space of a matrix, 93 transformation, 39 Commutative law for addition of matrices, 2 fields, 64 multiplication of matrices, 3 Commutative matrices, 11 Companion matrix, 197 Complementary minors, 24 Complex numbers, 12, 110 Conformable matrices
for addition, 2

Basis

change

of,

95

of a vector space, 86

orthonormal, 102, 111 Bilinear form(s) canonical form of, 126 definition of, 125 equivalent, 126 factorization of, 128

rank of, 125 reduction of, 126


Canonical form
classical (Jordan), 206

for multiplication, 3 Congruent matrices, 115

Conjugate of a complex number, 12


of a matrix,

Jacobson, 205 of bilinear form, 126 of Hermitian form, 146 of matrix, 41, 42 of quadratic form, 133 rational, 203 row equivalent, 40 Canonical set under congruence, 116, 117 under equivalence, 43, 189 under similarity, 203 Cayley-Hamilton Theorem, 181 Chain of vectors, 207 Characteristic equation, 149 polynomial, 149 Characteristic roots definition of, 149 of adj A, 151 of a diagonal matrix, 155 of a direct sum, 155

12
13

of a product, of a sum, 13

Conjugate transpose, 13 Conjunctive matrices, 117 Contragredient transformation, 127 Coordinates of a vector, 88 Cramer's rule, 77
Decomposition of a matrix into Hermitian and skew-Hermitian parts, 13 symmetric and skew-symmetric parts, 12 Degree of a matrix polynomial, 179
of a (scalar) polynomial, 172

Dependent
forms, 69 matrices, 73 polynomials, 73 vectors, 68

Derogatory matrix, 197

215

216
Determinant
definition of, 20

INDEX
Hermitian form canonical form
definite,
of,

146

derivative of, 33

expansion of along first row and column, 33 along a row (column), 23 by Laplace method, 33 multiplication by scalar, 22 of conjugate of a matrix, 30 of conjugate transpose of a matrix, 30 of elementary transformation matrix, 42 of non-singular matrix, 39 of product of matrices, 33
of singular matrix, 39 of transpose of a matrix, 21

147 index of, 147 rank of, 146


semi-definite, 147

signature

of,

147 146

Hermitian forms
equivalence
of,

Hermitian matrix, 13, 117, 164 Hypercompanion matrix, 205


Idempotent matrix, 11 Identity matrix, 10

Image
1

Diagonal elements of a square matrix,


matrix, 10, 156

of a vector, 94 of a vector space, 95

Diagonable matrices, 157


Diagonalization by orthogonal transformation, 163 by unitary transformation, 164 Dimension of a vector space, 86
Direct sum, 13 Distributive law for fields, 64 matrices, 3 Divisors of zero, 19 Dot product, 100

Index of an Hermitian form, 147 of a real quadratic form, 133 Inner product, 100, 110
Intersection space, 87

Eigenvalue, 149 Eigenvector, 149

Elementary
matrices, 41 n-vectors, 88 transformations, 39

Invariant vector(s) definition of, 149 of a diagonal matrix, 156 of an Hermitian matrix, 164 of a normal matrix, 164 of a real symmetric matrix, 163 of similar matrices, 156 Inverse of a (an) diagonal matrix, 55 direct sum, 55 elementary transformation, 39 matrix, 11, 55 product of matrices, 11 symmetric matrix, 58 Involutory matrix, 11

Equality of
matrices, 2 matrix polynomials, 179 (scalar) polynomials, 172

Jacobson canonical form, 205 Jordan (classical) canonical form, 206


Kronecker's reduction, 136

Equations, linear equivalent systems solution of, 75

of,

75

Lagrange's reduction, 132

system of homogeneous, 78 system of non-homogeneous, 77 Equivalence relation, 9 Equivalent


bilinear forms, 126

Lambda

Hermitian forms, 146


matrices, 40, 188 quadratic forms, 131, 133, 134 systems of linear equations, 76

matrix, 179 Laplace's expansion, 33 Latent roots (vectors), 149 Leader of a chain, 207 Leading principal minors, 135 Left divisor, 180 Left inverse, 63

Linear combination of vectors, 68 Linear dependence (independence) of forms, 70


of matrices, 73 of vectors, 68

Factorization into elementary matrices, 43, 188 Field, 64 Field of values, 171
First minor, 22

Lower triangular matrix, 10


Matrices congruent, 115
equal, 2

Gramian, 103, 111 Gram-Schmidt process,


Greatest

102, 111

equivalent, 40

common

divisor, 173

over a

field,

65

INDEX
Matrices (cont.) product of, 3
scalar multiple of, 2 similar, 95, 156 square, 1

217
Null space, 87
Nullity, 87

w-vector, 85

sum
Matrix

of, 2

Order of a matrix, Orthogonal


congruence, 163 equivalence, 163 matrix, 108

definition of, 1

derogatory, 197 diagonable, 157 diagonal, 10

elementary row (column), 41 elementary transformation of, 39 Hermitian, 13, 117, 164
idempotent, 11
inverse of, 11, 55 lambda, 179
nilpotent, 11

similarity, 157, 163 transformation, 103 vectors, 100, 110

Orthonormal

basis,

102,

111

Partitioning of matrices, 4 Periodic matrix, 11

nonderogatory, 197 non-singular, 39 normal, 164

Permutation matrix, 99 Polynomial domain, 172


matrix, 179 monic, 172 scalar, 172 scalar matrix, 180
Positive definite (semi-definite)

normal form
nullity of, 87

of,

41

of a bilinear form, 125 of an Hermitian form, 146 of a quadratic form, 131

order of, 1 orthogonal, 103, 163


periodic, 11

Hermitian forms, 147 matrices, 134, 147 quadratic forms, 134 Principal minor
definition of, 134

permutation, 99 polynomial, 179


positive definite (semi-definite), 134, 147

leading, 135

Product of matrices
adjoint of, 50 conjugate of, 13 determinant of, 33

rank

of,

39

scalar,

10 singular, 39

skew-Hermitian, 13, 118 skew-symmetric, 12, 117 symmetric, 12, 115, 163
triangular, 10, 157 unitary, 112, 164

rank

inverse of, 11 of, 43


of, 12

transpose

Quadratic form
canonical form of, 133, 134 definition of, 131 factorization of, 138 rank of, 131 reduction of

Matrix Polynoniial(s)
definition of, 179 degree of, 179 product of, 179 proper (improper), 179 scalar, 180 singular (non-singular), 179 sum of, 179 Minimum polynomial, 196

Multiplication
in partitioned form, 4 of matrices, 3

Kronecker, 136 Lagi-ange, 132 regular, 135 Quadratic form, real definite, 134 index of, 133 semi-definite, 134 signature of, 133

Quadratic forms
equivalence

Negative
definite form (matrix), 134, 147 of a matrix, 2 semi-definite form (matrix), 134, 147 Nilpotent matrix, 11

of, 131, 133,

134

Rank
of adjoint, 50 of bilinear form, 125 of Hermitian form, 146 of matrix, 39 of product, 43 of quadratic form, 131 of sum, 48

Non-derogatory matrix, 197 Non-singular matrix, 39

Normal form of a matrix, 41 Normal matrix, 164

218
Right divisor, 180 Right inverse, 63 Root
of polynomial, 178 of scalar matrix polynomial, 187

INDEX
Symmetric matrix
deiinition of, 12
(cont.)

invariant vectors of, 163 System(s) of Equations, 75


Trace, 1

Row
equivalent matrices, 40 space of a matrix, 93 transformation, 39

Transformation
elementary, 39 linear, 94 orthogonal, 103 singular, 95 unitary, 112

Scalar matrix, 10 matrix polynomial, 180 multiple of a matrix, 2 polynomial, 172 product of two vectors (see inner product) Schwarz Inequality, 101, 110 Secular equation (see characteristic equation) Signature of Hermitian form, 147 of Hermitian matrix, 118 of real quadratic form, 133 of real symmetric matrix, 116 Similar matrices, 95, 196 Similarity invariants, 196 Singular matrix, 39 Skew-Hermitian matrix, 13, 118 Skew-symmetric matrix, 12, 117 Smith normal form, 188

Transpose
of a matrix, 11 of a product, 12 of a sum, 11

Triangular inequality, 101, 110 Triangular matrix, 10, 157

Unit vector, 101 Unitary


matrix, 112 similarity, 157 transformation, 112 Upper triangular matrix, 10
Vector(s)

Span, 85
Spectral decomposition, 170 Spur (see Trace) Sub-matrix, 24 Sum of matrices, 2 vector spaces, 87
Sylvester's

belonging to a polynomial, 207 coordinates of, 88 definition of, 67 inner product of, 100 invariant, 149 length of, 100, 110 normalized, 102 orthogonal, 100 vector product of, 109

law

Vector space basis of, 86


definition of, 85

of inertia, 133 of nullity, 88

Symmetric matrix
characteristic roots of, 163

dimension of, 86 over the complex field, 110 over the real field, 100

Index of Symbols

Symbol

Page
1 1 1

Symbol
E^, (vector)

Page
88
100, 110
100, 110 103, 111

Hj
[^i;]

X-Y; X Y
\\n

S
^k
A-'A':

10
11

zxy
Q
p
s

109 115 116

A'

/
A

11

I;
A'-

12 13

116
131

A*- A^
det

\A\;

20

h
A. A,-

146

\^ij\

22
im i^ 23

149
149 170
172

^ii'i2
ii, is, ....

<^(A)

a,.
r

23
39
39

E^, (matrix)

/(A)

h-^ij
Hi(k), K^ik)

F[A]
^(A)
AjfiC),

172
179 180
189

39 39

H^.{k). K.j(k)
-V.

A^ (C)

40 43

NiX)
fi(^)

N
adj

189 196 198 198

49

m(A)
C(g)
J

F
X. X^

64
67

yn(f)
V^tiF)

85

S Cq(p)

203

86
87

205

^A

219

You might also like